Está en la página 1de 401

COLECCIÓN CIENCIAS BÁSICAS

PROBLEMAS D E ÁLGEBRA,
GEOMETRÍA Y TRIGONOMETRÍA
L
A. NlETUSHIL

TALA^
JCCION: GUILLERMO GARCÍA TA LAVE I
fERA
:

ti

STITUTO POLITÉCNICO NACIONAL


COLECCION

CIENCIAS BAsICAS
PROBLEMAS DEA.LGEBRA, GEOMETRlA YTRIGONOMETRlA

A. Nietushil
DIRECTORIO IPN DIRECTORIO UTP

DR. JOSE ENRIQUE VILLA RIVERA M. EN C. GERARDO MARCELINO LARA OROZCO


Director General Rector

DR. EFREN PARADA ARIAS M. EN C. LUIS RICARDO MORALES JUAREZ


Secreta rio General Secrctario Academico

ORA. YOLOXOCHITL BUSTAMANTE DIEZ QFB. MARIA ONEIDA ROSADO GARCIA


Secretaria Academica Secretaria de Vinculacion

L1C. I'CO. JAVIER TREVINO MUSALEM


DR. JOSE MADRID FLORES
Abogado General
Secretario de Extension e Integraci6n Social
CPo MONICA CANDIA DE LA ROSA
DR. LUIS HUMBERTO FABILA CASTILLO
Contralora Interna
Secretario de Investigacion y Posgrado
CPo TOMAs E. SILVA LIMON
DR. HECTOR MARTINEZ CASTUERA Director de Administraci6n y Finanzas
Secretario de Servicios Educativos

DR. MARIO ALBERTO RODRiGUEZ CASAS DIRECTORES DE CARRERA:


Secretario de Administracion
CPo JOSE DONATO RODRiGUEZ ROMERO
Lie. LUIS ANTONIO RIOS CARDENAS Administracion
Secretario Tecnico
ING. MARCOS ESPINOSA MARTiNEZ
ING. LUIS EDUARDO ZEDILLO PONCE DE LEON Electricidad y Electronica Industrial
Secretario Ejecutivo de la Comision de Operacion
y Fomento de Actividades Acadell1icas ING. EDGAR EDUARDO PASTOR PEREZ
Mantcnimiento Industrial
ING. JESUS ORTIZ GUTIERREZ
Secretario Ejecutivo del Patronato de Obras e Instalaciones M. EN C. RAMON BELTRAN MARTINEZ
Mecatronica
MTRO. FERNANDO SARINANA MARQUEZ
Director de XE-IPN TV Canal I I ING. JOSE BELARMINO BUENO MONEDA
Procesos de Produccion
Lie. LUIS ALBERTO CORTES ORTIZ
Abogado General LlC. LEONEL TORRES LEZAMA
Tecnologias de la Informacion y Comunicaci6n
Lie. ARTURO SALCIDO BELTRAN
ING. POMPILIO MARTiNEZ VALENCIA
Director de Publicaciones
Encargado de Tecnologia Ambiental
COLECCION

CIENCIAS BAsICAS
PROBLEMAS DE ALGEBRA, GEO~TRiA YTRIGONO~TRiA

A. Nietushil

Compilaci6n y Traducci6n. Guillermo Garcia Talavera

Instituto Politecnico Nacional


-Mexico--
Coleccion Ciencias Basicas
Problemas de Algebra, Geometria y Trigonometria
A. Nietushil
Traductor: Guillermo Garcia Talavera

Primera Edicion: 2008

D.R. © 2008 Instituto Politecnico Nacional


Luis Enrique Erro sin Unidad Profesional "Adolfo Lopez Mateos"
Col. Zacatenco, 07738, Mexico D.E

Direccion de Publicaciones
Tresguerras 27, Col. Centro Historico
06040, Mexico D.E
http:/www.publicaciones.ipn.mx

ISBN Coleccion: 978-970-36-0468-5


ISBN: 978-970-36-0469-2
FIPN: 2007-505

Impreso en Mexico/Printed ill Mexico


PRESENTACION

El presente libro contiene 658 problemas, minuciosamente


resueltos, en el dominio de las matematicas elementales: Algebra,
Geometria (plana y del espacio) y Trigonometria. Constituye,
por tanto, un recurso valioso para la preparaci6n de aspirantes a
ingresar a escuelas de educaci6n media superior y superior.
Se trata de la traducci6n de un texto originalmente escrito en
lengua rosa, destinado a reforzar el instrumental materruitico basico
de los alurnnos de instituciones denominadas Tejnikum, cuyos
curricula corresponden, en sus etapas iniciales, a los de los
CECyT del IPN y, en las finales, a los de las UT mexicanas.
Esta coedici6n IPN-UTP, hani posible poner al alcance
econ6mico de los alurnnos de estas instituciones este valioso
aporte bibliognifico, que los ayudani a la comprensi6n de los
materiales matematicos que se exponen en cursos mas avanzados:
GeometriaAnalitica y Calculo Diferencial e Integral.
Esta es una de las mas valiosas traducciones, entre mas de
cien que ha publicado el ilustre maestro Guillermo Garcia
Talavera.
ALGEBRA 9
,
ALGEBRA Tambien forman una progresion aritmetica.

2. Los numeros positivos aI' a 2, ••• , an forman una progresion


1. Progresiones aritmetica y geometrica aritmetica. Demostrar que
_ ~l~ __ + 1 + + 1 = n-l
Observaciones preliminares JQ1 +,Ja2 ,Ja2 + JQ1 .. . ~ an-I +.ra;; JQ1 +.ra;; .
Si an es el tennino enesimo, d-Ia diferencia y Sn-la suma de los 3. Demostrar que si los numeros a" a 2, ••• , an no son iguales a
n primeros terminos de una progresion aritmetka, entonces cero y forman una progresion aritmetica, entonces
(1)
S = (a\+a n ) = 12a\+d(n - l) ln (2)
n 2 2
4. Demostrar que toda sucesion de numeros a" a 2, ••• , an' que
Si u, es el termino enesimo, q ---el denominador y Sn-Ia para cualquier n ~ 3 satisfacen la condicion
suma de los n primeros terminos de una progresion geometrica,
entonces
n-l
= u1q un (3) forma una progresion aritmetica.
S = unq-ul = uI(qn_l) (4)
n q-l q-l 5. Mostrar que para toda progresion aritmetica aI' a 2, a 3 • • • , an'
Si, por fin, S es la suma de una progresion geometrica tienen lugar las igualdades
decreciente infmita (iqi< I), entonces a 1 - 2a 2 +a 3=0,
a 1 - 3a 2 + 3a 3-a 4 = 0,
S=~ (5) a1 - 4a 2 + 6a 3 -4a 4 + a 6 = 0,
l-q

1. Demostrar que si los numeros positivos a, b, c forman una


progresion aritmetica, los nlimeros y que, en general, para cualquier n>2 tenemos
1 1 1
.Jb+JC' JC+.Ja' .Ja+.Jb
10 PROBLEMAS DE MATEMATICAS ELEMENTALES

Indicacion. En este y en el siguiente problema es conveniente 10. La sucesion de nllineros 1,4, 10, 19, ... posce ~ propiedad de
hacer uso de la identidad facil de comprobar que la diferencia de dos numeros vecinos forman una progresion
aritmetica. Hallar el termino enesimo y la suma de los n primeros
terminos de esta sucesion de nllineros.
6. Demostrar que para cualquier progresion aritmetica a" ... ,
11. Hagamos una tabla 1,
an' an+I , ••• siendo n ~ 3 tiene lugar la igualdad
2,3,4
a 12 - (~ ) a; + ... + (- 1Y (~ ~; + 1 = O. 3, 4, 5, 6, 7
4,5,6, 7, 8, 9, 10
7. Demostrar que si los numeros klog x, mlog x, nlog x (x "* 1)
forman una progresion aritmetica, entonces
Demostrar que la suma de los terminos de cada fila es igual al
n2 = (1m )kIOgm. cuadrado de un numero impar.
8. Hallar una progresion aritmetica en la que la relacion entre la 12. En la progresion geometrica aI' a 2, a3 , ••• se conocen los
suma de los n primeros terminos y la suma de los 1m siguientes terminos a m+n = A, a m-n = B. Hallar a m y an (A*- 0).
no depende de n.
13. Supongamos que Sn es la suma de los n primeros terminos
9. Los numerosxI,x2, •• • ,xn forman una progresion aritmetica. de una progresion geometrica, Sn "* 0, q"* 0. Demostrar que
Hallar esta progresion, si

XI + X 2 + ... + Xn = a,xI2 + X 22 + ... + Xn3 = b3 .


14. Conociendo la suma Sn de los n primeros terminos de una
Indicacion. En este y en el siguiente problema es conveniente progresion geometrica y la suma de las reciprocas de estos
hacer uso de la igualdad terminos, hallar el producto Pn de los n primeros terminos de la
progresion.
12 + 22 + 32 + ... + n 2 = n (n+I)(2n+l)
6.
15. Hallar la suma
1+2x+3x2 + 4x 3 + ... +(n+lh n .
ALGEBRA 11

16. Hallar la suma Demostrar que xI' x 2 ' ••• , x n' ••• son terminos sucesivos de
una progresion geometrica.
1+11+111+ ... +111 ... 1, lndicaciones. Se puede aplicar el metodo de induccion
completa.
si e1 ultimo sumando es un nillnero de n cifras.
22. Se conocen una progresi6n aritmetica con el termino comtIn
17. HaHar la suma
an y una progresion geometrica con el termino comtIn bn, con la
particularidad de que para todos los numeros naturales de n,
. al = bl , a 2 = b2 , a l "* a 2 Y an > O. Demostrar que para n>2,
18. Hallar la suma a n<b.n
I + 3 5 + + 2n-1
"2 22+ 23 ... ~.
23. Demostrar que si para una progresion aritmetica aI' a 2, ... ,
19. Demostrar que los nillneros 49, 4489, 444889, ... , obtenidos an' ... yuna progresion aritmetica bl' b2, ••• , bn, •• • se cumplen
colocando el nillnero 48 en medio del nillnero anterior, son los las desigualdades
cuadrados de numeros enteros.

20. Demostrar que se puede hallar una progresion geometrica


decreciente existe tal numero a, que el a log an - bn no depende de n.
1,q,q 2 ,... ,q n ,... ,

cada termino del cual difiere de la suma de todos los terminos 2. Ecuaciones algebraicas y sistemas de ecuaciones
que Ie siguen en el factor constante kprevisto. l,Para que valores
de k es po sible el problema? Observaciones preliminares
AI resolver los sistemas de ecuaciones propuestos mas abajo; el
21. Una sucesion infinita de nillnerosxl'x2,x3, •• •,xn'··. (XI*" 0) sistema inicial se reduce, mediante ciertas simplificaciones, a un
para cualquier n ~ 3 satisface la condicion sistema equivalente, todas las resoluciones del cual 0 son
conocidas, 0 pueden ser halladas por los procedimientos
( XI2 +X22 + ••. +Xn2 _ 1 )(X 22 +X32 + ••. +Xn3) = conocidos. En ciertos casos es necesario pasar a sistemas que
se satisfacen de antemano con todas las resoluciones del sistema
= (X IX 2 + X 2X 3 + ... + Xn_IX n Y.
12 PROBLEMAS DE MATEMATICAS ELEMENTALES

inicial, sin embargo, hablando en general, no solamente con estas. 26. Hallar las soluciones reales del sistema de ecuaciones
En estos casos, los conjuntos de val ores determinados de las
incognitas deben comprobarse con ayuda de la sustitucion en el
sistema inicial.
En problemas aislados se usan las formulas de Viele que
enlazan los coeficientes de la ecuacion de tercer grado con la condicion de que a es real y no es igual a O.

x 3 + px 2 + qx + r = 0 (I)
27. Resolver el sistema de ecuaciones
Con sus rakes xI' x 2' x 3• Estas formulas tienen el aspecto:
"'-+
2
y
y2
x
= 12' }
1+1=1
x y 3"
Las formulas (2) se hallan igualando los coeficientes de las X 28. Resolver el sistema de ecuaciones
de iguaks potencias en la identidad

x 3 + px 2 + qx + r = (x - XI XX - x 2 Xx - X3)' x 2 + x 2y2 + y2 = 9l,}


x 2 _ xy + y2 = 7.
24. Hallar todas las soluciones reales del sistema de ecuaciones
29. Resolver el sistema de ecuaciones

x 3_ y3 = 19{x- y),}
x3 + i = 7 (x + y).
25. Resolver el sistema de ecuaciones
30. Hallar todas las soluciones reales del sistema de ecuaciones
x 2 + xy + / = 4,} 2{x + y) = 5XY,}
x+xy+ y = 2.
S(x3 + i)= 65.
ALGEBRA 13

31. Hallar las soluciones reales del sistema de ecuaciones Indicacion. Supongase que xy = u, x+y = u.

36. Resolver el sistema de ecuaciones

(x 2 + l )f == 6,1
32. Hallartodas las soluciones reales del sistema de ecuaciones
(x 2 - l )~ == I.J
x + y == I,} 37. Resolver el sistema de ecuaciones
x4+l==7.
33. Resolver el sistema de ecuaciones 4 4 b 2 2
1
x-? + y 2 == axy,
X + Y == x Y .J

x+y==1,[ 38. Resolver la ecuaci6n (descomponiendo el primer miembro


x5 +/ == 31.f en factores)

34. Hallar las soluciones reales del sistema de ecuaciones (-x+a)2 +(x_a)2
- - -(a+ x2_a
-ah) - -2- -0
x +6 \x-h b xl _62 .

X4 + y4 -x 2 i == 13,1 39. Resolver la ecuaci6n


2
x - y + 2xy == 1.J
~

que satisfacen la condicion

ry~O.
40. Resolver el sistema de ecuaciones

35. Resolver el sistema de ecuaciones X+Y+·x}'--==a+~


xy x+y a'
l
x-y xy _ b 1 r
- +x-y
- - +-.1
(x 2 + 1:&2 + 1)== 10,1 xy h )

(x + y Xxy - 1) = 3.J
14 PROBLEMAS DE MATEMAnCAS ELEMENTALES

41. Hallartodas las solucionesde laecuaci6n tiene la unica soluci6n real y hallar esta soluci6n.
{x-4,5)4 +(x-5,5)2 = I.
47. Demostrarque para cualquier (en general compleja) soluci6n
42. Resolver el sistema de ecuaciones del sistema
2 2 1)
I x - II + I y - 5 1= I,} X + Y +xy+-=a'I
xy

y =: 5+ I x-II .
) . x 2 + y2 + x 2 y2 - --f-z
x y
- 2 = b2 .
la suma x 2 + yes real para cualesquiera valores reales de a y b
43. Aclarar para que val ores reales de x e y se cumple la igualdad si a*" O.

5x 2 + 5i + 8xy + 2y - 2x + 2 =: O· 48. Resolver el sistema de ecuaciones

44. Hallar todos los valores reales de x e y que satisfacen la ax + by + ez = a + b + e,}


ecuaci6n bx + ey + az = a + b + e,
x 2 + 4xcos(xy)+ 4 =: O. ex + ay + bz = a + b + e.
45. Hallar las soluciones reales del sistema
suponiendo que a, bye son reales y que a + b + c *" o.
x+ y+: =:2,}
49. Resolver el sistema de ecuaciones
2xy-z- = 4.

46. Determinar para que valor de a el sistema ax+ y+ z = 1,1


x+ay+z = a, ~
X2 + y 2 = z, }
x + y + az = a 2 .J
x + y + z = a.)
50. LQue condici6n deben satisfacer los numeros ai' a z y a3
*) Se llama magnitud absoluta del numero x (se designa con ~ ' ) a un
para que el sistema
numero no negativo que se dete~ina de las condiciones siguientes:
I
,X
1= )lX,"
x, SI X < 0;
x 2 O.
ALGEBRA 15

(I + al)x + Y + z = I,} XI +X2 +X3 =0,1


x + (1 + a2 )y + Z = 1,
x + Y{l + a3 )z = 1. x~ + ~~ + ~~ ~ 0, l
tenga soluci6n y ademas unica? X99 + XIOO + XI : 0, I
XIOO + XI + X2 - 0,)
51. Resolver el sistema de ecuaciones entonces

ax+by+cz+dt = P'I
-bx+ay+ dz -ct = q, 54. Resolver el sistema de ecuaciones
-cx-dy +az +bt = r,
- dx + cy - bz + at = s, x:+ xy + xz - X = 2'1
y +xy+ yz- y=4,
donde a, b, c y d satisfacen la condici6n:
z2 +xz + yz-z = 6. J

55. Resolverel sistema de ecuaciones


52. Resolver el sistema de ecuaciones
x+y-z=7,1
XI +2x2 + 3X3 +4X4 + ... +nxn :a]'l X2 +y 2 -z 2 =37, )
nx] + x 2 + 2x3 + 3X4 + ... + (n-l)xn - a 2" 3 3 3
x+y-z=1.
(n ~I )x, + ""2 + ~J +2~4 ++(n ~ 2 )x~~aJ' r 56. Resolver el sistema de ecuaciones

2x] + 3x2 + 4X3 + 5X4 + ... + lXn = an'j


x+y
xyz
xyz _ 6
= ,
21
y+z - 5"

53. Demostrar que si xy: 3


z+x = '2'
16 PROBLEMAS DE MATEMAnCAS ELEMENTALES

57. Resolverel sistema de ecuaciones suponiendo que a, k, I Y m son numeros reales y que I2-+P+
+m2>O.
u2 + v 2 + W = 2,}
v2 + w 2 +u = 2, 61. Hallar las soluciones reales del sistema de ecuaciones

w 2 + u 2 +v = 2.
X+y +z =6,1
58. Resolver el sistema de ecuaciones I
X2 + y2 + Z2 = 14,
xz + yz = (xy + 1tJ
X- +xy + y = 1,
?
?
X- +XZ+z = 4,
2}
2
62. Resolver el sistema de ecuaciones
? 2
Y + yz+z = 7.
x 2 +xy+xz + yz = a'l}

I
59. Resolver el sistema de ecuaciones y2 + xy + xz + yz = b,
+ xy + xz + yz = c,
~=a
Z2
Xl I'

XIX:~.. XIJ = Q2'


suponiendo que abc:f::. o.
. ... ~ ......... .
x,x, . X,,_,
Xl
_
- an'
j 63. Resolver el sistema de ecuaciones

suponiendo que abc = O.


suponiendo que los numeros aI' ... , anY Xl' ... , xn son positivos.

64. Hallar las soluciones reales del sistema de ecuaciones


60. Resolver el sistema de ecuaciones

(X+ y+z)(ax+ y+z)=e,l


i +Z3 =2a(yz+zx+xy ),1

(x + y + z) (x + ay + z) = [2, ~ Z3 + x 3 = 2b (yz + zx + xy), I


(x + y + z) (x + y + az) = m2 , J x 3 + i = 2c(yz +zx +xy).j
ALGEBRA 17

65. Resolver el sistema de ecuaciones 69. Resolver el sistema de ecuaciones

y + 2x + z = a{x + y)(z + x),l y+ y + z = a,1


z+2y+x=b(y+z)(x+ y),(. ?
x- + y 2
+ z- = a 2 ,).I
?

x+2z+ y=c(z+x)(y+z)J x 3 +y 3 +z 3 =a.


3
,
-'
I
66. Resolver el sistema de ecuaciones 70. Sean (x,y, z) las soluciones del sistema de ecuaciones:

x+ y+z =9,} x+ y+z =a,


~+~+~=1 x 2 + y2 +Z2 =b 2 ,
x v z '

xy + xz + yz = 27. 1+1+1=1.
x y : c

67. Resolver el sistema de ecuaciones Hallar la suma


3 3 3
x+ y+z=a,1 X + Y +z

xy+ yz +xZ ~ a:.l 71. Resolver el sistema de ecuaciones


xyz = a .J

68. Demostrar que la soluci6n (mica de sistema de ecuaciones

2x+ Y + z = 0'1 72. Resolver el sistema de ecuaciones


yz+zx+xy- =0,i I
xy+ Z2 = o.J

es la soluci6n x = y = z = 0.
18 PROBLEMAS DE MATEMATICAS ELEMENTALES

,
73. Resolver el sistema de ecuaciones I + x + y + z == 0, I
xy + yz + zx == 47,1 a + bx + cy + dz == OJ
x 2 + y2 == Z2, ~ 2 ') ')
a +b-x+c-y+d z==O.)
2 I
(z - x) (z - y) = 2.j entonces la multiplicaci6n xyz es positiva.

74. Hallar todas las soluciones reales del sistema de ecuaciones En las ecuaciones propuestas a continuaci6n, en el caso de
raices cuyas potencias sean pares, se examinan s6lo aquellos
valores de las inc6gnitas para los cuales la expresi6n contenida
bajo la raiz no es negativa; tomando, al mismo tiempo, solamente
los valores no negativos de la raiz. En el caso de rakes impares,
la expresi6n contenida bajo la raiz puede ser cualquier numero
real (en este caso el signo de la rafz coincide con el signo de la
expresi6n subradical).

75. Hallartodas las soluciones reales del sistema de ecuaciones 77. Resolver la ecuaci6n

78. Resolver la ecuaci6n


m!(
Ijl-x
)2 m l-( -)2 m j 2
- 'l/ l-x ="I-x.

79. Resolver la ecuaci6n

80. Resolver la ecuaci6n


76. Demostrar que si a, b, c y d son por pares numeros reales
desiguales y x,yy z son las soluciones del sistema de ecuaciones fx---;~Jx - -J~ - -l~ == l, r -;:.
2 V x+ ,/ x
ALGEBRA 19

81. Resolver la ecuaci6n 86. Resolver el sistema de ecuaciones

~ ~ 7
IX~X +'\/x+7 = - - - .
~ ~x+1 ~x+l

82. Hallar todas las rakes reales de la ecuaci6n

87. Resolver el sistema de ecuaciones

/Y+I +2\j~
'\ x-y y+1
=3 1
'~
83. Resolver la ecuaci6n
x+xy+y=7.1 )

~x -4a+ 16 = 2~x - 2a + 4 --E.


88. Hallartodas las soluciones reales del sistema de ecuaciones
LPara cwiles valores reales de a tendra soluci6n la ecuaci6n?
x+ y - -,,/I'~ . -
x- y - I~}
-
x- y ,
84. resolver el sistema de ecuaciones
xy = IS.
-v'I 1-16y 2 - '.i'"1-16x~~ =2 ( x+ y, )t 89. Resolver el sistema de ecuaciones
2 ~ 1
X + y~ + 4xy = s.J

85. Resolver el sistema de ecuaciones Y+ 2~ x~12 y + I = x 2 + 17 f


3 12 '
x -2 = /IX
-+ --'- I- - -Y
x - y ="27('/'2 312)}
~, X Y - 'V xy , 8y 3 'v3y'4 2x·J

if;-VY=3.
20 PROBLEMAS DE MATEMATICAS ELEMENTALES

90. Resolver el sistema de ecuaciones 95. Resolver el sistema de ecuaciones

X+~X2 _y2
-~'-;==
2 ~2
x_~x2 _y2
+ - 1- -
2 -
17}
2 = -4 '
x-Jx - y~y = a(-Jx -~),} (a>O,b > O).
x- x -y X+ , x -y 1 ??
x- + xy+ y = b-
x(x+ y)+ -Jx 2 +xy+4 =52.

91. Resolver el sistema de eeuaeiones 3. Desigualdades algebraicas

Observaciones preliminares
l + ~3l-2x+3 = tx+ 5,} Expongamos algunas desigualdades utilizadas en la resoluei6n
3x-2y=5. de los problemas que se proponen mas abajo.
Para cualesquiera a y b reales
92. Hallar las soluciones reales del sistema de eeuaeiones
(I)

La desigualdad (1) es una deduce ion de la evidente desigualdad


°.
(a ± b)2 ~ EI signa de igualdad en (1) tiene lugar solo en el
easo en que lal=lbl.
93. Resolver eI sistema de ecuaeiones Si ab > 0, dividiendo las dos partes de la desigualdad (1)
entre ab se tendra:
a b
(x-YNY=~,l -+ - ~2.
b a
(2)

(x+ y}f; =3-jyJ


Si u~O, v~O, entonees, haciendo en (1) u = a2, v = b2 ,
obtendremos
94. Resolver el sistema de eeuaeiones
u+v ,
-- ~~uv. (3)
2
x
~ +y - ~
,- -
y = x- a,t
(a > O). En las desigualdades (2) y (3) el signa de igualdad tiene lugar
/22 ' 222
\Ix +y +ljx -y =a J solamente cuando a = b y u = V, eorrespondientemente.
ALGEBRA 2i

Sefialemos, ademas, ciertas propiedades del trinomio Para todos los demas valores de x diferentes de XI y x 2' el
cuadrado trinomio po see el mismo signa que a.
7
Y = ax- + bx + c, (4) Asi pues, el trinomio siempre conserva el signo del coeji-
cienle del termino de mayor polencia, excepto en el caso en
que en adelante se emplean en toda una serie de problemas. que sus raices son reales y
De la representaci6n del trinomio (4) par la f6rmula Xl ~ X ~ x2 •

b)2 b 2 - 4ac 96. Hallar todos los valores reales de r para los cuales el polinomio
y=a ( x+ - - (5)
20 4a
se deduce que en el caso cuando el discriminante del trinomio
es positivo para todos los valores reales dex.

(en este caso las rakes del trinomio no son reales), el trinomio 97. Demostrar que la expresi6n
adquiere para todos los valores dex, val ores de un mismo signo,
que coincide con el signa del coeficiente a del termino de mayor
3l2-.-
(? v~ '\ ( y)
+"-- 1-8 -"- + - + 10
2 2) ). X
potencia. )1 X

En el caso en que D = 0 el trinomio conserva tambien el signo no es negativa para cualesquiera X e y reales y no iguales a cero.
constante, haciendose igual a cero para el valor unico x =- ;0 .
98. (,Para cuales valores de a se satisface el sistema de
Por fin, cuando D > 0 (en este caso, las rakes x I y x 2 del
desigualdades
trinomio son reales y diferentes), de la descomposici6n

cualesquiera que sean los valores de x?


se deduce que solamente con la condici6n de que
99. Demostrar que para cualesquiera valores reales de los
numeros a, b, c y des valida la desigualdad

el trinomio adquiere valores de signo contrario al de a.


22 PROBLEMAS DE MATEMATICAS ELEMENTALES

100. Hallar todos los valores de a para los cuales el sistema 105. Demostrar que siendo n> 2
rrusmo
x2 + l + 2x ~ I,}
x-y+a=O 106. Demostrar que con tres segmentos de longitud a> 0, b > 0
y c > 0 se puede construir un triangulo solamente cuando
tiene soluci6n (mica. Hallar las soluciones correspondientes.
? 2 ?
pa- + qb > pqc-
101. Hallar los pares de los numeros enterosxe y que satisfacen
al sistema de desigualdades para cualesquiera numeros p y q enlazados en la relaci6n

y-i x 2 - 1
2x I + > a,} p+q=l.
y+ i x-ll< 2. 107. Demostrar que para cualesquiera valores reales de x, y y z
es valida la desigualdad
102. Demostrar que para cualquier valor entero de n > 1 es
valida la desigualdad
1 1 I 1 108. Demostrar que para cualesquiera valores reales dex eyes
- - + -- + ... +-- >-. valida la desigualdad
n+ 1 n+ 2 2n 2
? 2
103. Demostrar que para cualquier valor entero y positivo de m x- + 2xy + 3 y + 2x + 6 y + 4 2 1.
es valida la desigualdad
109. Demostrar que con la condici6n de que sea 2x+4y=1 se
1 1 1
- + - - + ... +
m +1 m +2
(
m + 2m + 1
»l. cumple la desigualdad

104. Demostrar que para cualquier valor entero positivo de n 11 O. (,CuaIes condiciones debera satisfacer el numero d> 0 para
que siendo R 2 r > 0 sea valida la desigualdad
I lin-I
'2" + -3 2 +"'+ -n 2 <-.
n
2
0< ,-1 +R_~~ <
2
I?
?

2dR _.
ALGEBRA 23
-----~--- - - -- --

111. Demostrar la desigualdad 117. Demostrar que


Ixlal +xZa2 + ... +xnan !:<;;~{XI2 +x~ + ... +x~)+
(a, b, c son numeros positivos) +1(a 2 +a~ + ... +a~),
j

112. Demostrar que si a, by c son numeros de igual signa ya < b don de XI' x 2' ••• , x n' ai' a 2, .•. , an y E son numeros reales
<c, entonces arbitrarios y ademas E> O.

118. z,Para cuales valores reales de X se cumple la desigualdad

113. Demostrar que siendo ai' a z' a 3, .. . , an numeros positivos


y a l ap3·· .an=l, entonces
X
119. Demostrar que para todos los valores positivos de x e y y
los valores enteros y positivos de my n (n ::::0: m) se cumple la
114. Demostrar que si a + b = 1, entonces desiguaJdad

a 4 +b 4 :2:i·
115. Demostrar que el polinomio 120. Demostrar la desigualdad

! I r-
'y a+ -y a+ ... +-Ja < - -2-
1+ .J4a+l
,a>O.
es positivo para todos los valores dex.

116. Demostrar que si ~I<l, para cualquier valor entero de n ::::0: 2 121. Demostrar la desigualdad
se cumple la desigualdad
2-
--
J2 ~2
-- I
+ +
I
~2 + ... + J2 ~ >-
1
2- ~ 2+ --v 2+ ... +~2
4
24 PROBLEMAS DE MATEMAnCAS ELEMENTALES

con la condicion de que el numerador de la parte izquierda de la En esta formula, N es cualquier numero positive y a una base
desigualdad contiene n signos radicales y el denominador contiene arbitraria, al mismo tiempo a > 0, a -::j. 1.
n - I signos radicales. A continuacion, al resolver toda una serie de problemas, para
pasar de los logaritmos con base a a los logaritmos con base b y
122. Demostrar que para cualesquiera numeros reales ai' a2, viceversa, se usa la formula siguiente
... , anY bl' b2 ,
•.. , bn, que satisfagan las relaciones
logN
(II N b
og = (2)
a~ + a~ + ... + a~ = I,} h loga

2
b1+ b22+···+ bn=l.
2 (esta formula se demuestra por logaritrnacion de la identidad (1)
tomando como base b). De la formula (1), siendo N=b, en
es valida la desigualdad particular, se deduce:

<1 10gb = -b - I- .
(3)
loga
123. Demostrar que si los numeros XI ' x 2' .•• , xn son positivos
124. Resolver laecuacion
y satisfacen la relacion
210gx 21 log x V;; log x. log x.
(1 (I

entonces 210g2 a
hloga
125. Resolver laecuacion
x x
4. Ecuaciones logaritmicas y exponenciales, x log 2· T6 log 2=64 log 2.
identidades y desigualdades
126. Resolver laecuacion
Observaciones preliminares
De acuerdo con la definicion dellogaritmo del numero Neon
base a tendremos: 127. Resolver la ecuacion
a u log N = N. (1)
3Xlog(i )+3[og2 X = I.
ALGEBRA 25

128. Demostrar que la ecuaci6n 133. Resolver la ecuaci6n

2x log(~ )210g2 x+ 210g4 X = 1 1+ a log(p - x) = 2j -qlog 4


(p>q>O).
a log(x - q) p-q log(x + q)
tiene una sola raiz que satisface a la desigualdad x > I. Hallar
estaraiz. 134. Resolver la ecuaci6n

129. Resolver la ecuaci6n


-i5 log XI) x log 5-J 5 + -i5 log 515 = -16.
,,'.[; log a u, 1- 135. Resolver laecuaci6n
, + "loga· "log2x=O.
d\oga
130.;.,A cuales condiciones deberan satisfacer los numeros a y b
para que la ecuaci6n
136. Resolver la ecuaci6n
1+hlog(210ga - xy 10gb = ~
logx
tenga por 10 menos una soluci6n? Hallartodas las soluciones de
esta ecuaci6n. GCuantas ralces tiene esta ecuaci6n para un valor determinado
de n?
131. Resolver laecuaci6n*l
137. Resolver la ecuaci6n
I· I

FIog~+xlog~ + ,j (j 10g~~+xlogV¥ = a. senx log 2·sen 2 x loga + 1 = O.

132. Resolver la ecuaci6n 138. Resolver el sistema de ecuaciones

log(~ + 1) =3
logVx-40 .
*) Aquf y a cotinuaci6n las rafces se comprenden en el sentido menciona-
do en la pagina 24.
26 PROBLEMAS DE MATEMATICAS ELEMENTALES

139. Resolver el sistema de ecuaciones 144. Resolver el sistema deecuaciones

XO = yb'1
Cloa.,!.= .Clogx 1210gx(_I_+210gy1=2\OgX,l
to Y C logy
~ Xlog2 J

2\ogx . 3\og (x + y) =3 3 \ogx.J


(a '" b, ab '" O.)J
140. resolver e\ sistema de ecuaciones 145. Resolverel sistema de ecuaciones

61ogx+38logy = 7,1 x2logy~ log 2 ~ yjY ~- xlog 2),}


v 12 log2-.J21 ogx = 1.
X' = 5 .j yO

141. Resolver e\ sistema de ecuaciones


146. Resolver el sistema de ecuaciones
yxylogx= X2, 2 4 4 ~
5 }
logx+ logy+ logz = 2, I
410g y-Y log (y - 3x) = 1.
31ogy+9Iogz+91ogx = 2'J
142. Resolver el sistema de ecuaciones 4Iogz+16\ogx+16 1ogy = 2.

1 aXb Y =ab,1 147. Resolverel sistema de ecuaciones


2° logx=b logy· ""1ogb.J
, log ( y - x ) + ~~ log y1 = -2, ~I
05

143. Resolver el sistema de ecuaciones ~


x- + y-
~
= 25.)I
3( iOgX-~IOgy) = 10,}
2y2 148. Resolver la ecuaci6n

xy = 81.
ALGEBRA 27

149. Resolver el sistema de ecuaciones 153. Resolverel sistema de ecuaciones

suponiendo que x > 0, y > 0 y pq > O.


suponiendo que x > 0 y y > o.
154. Resolver el sistema de ecuaciones
150. Resolver el sistema de ecuaciones

a 2x + a 2y == 2b,} (a> 0).


a X + Y == c
suponiendo que x > 0, y > 0, p > 0, q> O.
GA cmiles condiciones debenin satisfacer bye para que el
sistema tenga soluci6n? 155. Demostrar que

151. Hallar las soluciones del sistema de ecuaciones


si a2 + b2 == c2 y a> 0, b> 0, c> O.

156. Simplificar la expresi6n

(hloga-alogb) + (b±IOga_a' 10gb) + ...


suponiendo que x > 0, y > 0 y n > 0.
... + (
b-j;;
loga-
02'1 ]2
10gb .
152. Resolver el sistema de ecuaciones

(3x + yy-y== 9,}


157. Simplificar la expresi6n a '0',:,':'" ,admitiendo que todos los
x y ,---
- ~324 ==18x 2 +12xy+2/. logaritmos han sido tornados con la misma base b.
28 PROBLEMAS DE MATEMAnCAS ELEMENTALES

158. Se conoce: alogb = A, qlogb = By un numero entero 163. Resolver la desigualdad


n > O. Calcular el Clog b, donde c es igual al producto de n
terrninos de una progresi6n geometrica con eJ primer terrnino a y
el denominador q.
164. Resolver la desigualdad
159. Demostrar que si para cierto valor positivo de N t= 1, para °logx+Olog(x + 1)<alog(2x + 6) (a> I).
los tres numeros positivos a, bye se cum pie la relaci6n
165. Resolver la desigualdad
UlogN GlogN_blogN
ClogN blogN-ClogN'
3 log (x 2 -5x+6)<O.
entonces, b es el valor medio proporcional entre aye y la relaci6n 166. Resolver la desigualdad
se cumple para cualesquiera val ores positivos de N t= 1 .
1 1
-~-- - ' ) - -- < I.
160. Demostrar la identidad -Iogx -logx-I

167. Resolver la desigualdad


GlogN blogN+hlogN ClogN+clogN UlogN =

UlogN hlogN ClogN


abc[ogN

168. (,Para cuales val ores reales de x y a se cumple la


161. Demostrar la identidad desigualdad
a log X_I aI b 2log x+"' log 2. + 2 cos a :s; O?
b - + og .
a logx
169. Resolver la desigualdad
162. Resolver la desigualdad

1 ,
'Iogx+ologx> I.
ALGEBRA 29

5. Combinatoria y binomio de Newton


(5)
Observaciones preliminares
Del desarrollo (4) se deducen las igualdades
EI numero de variaciones de orden m con n elementos se
determina haciendo uso de la formula

VJn) = n{n - l) ...(n- m+ 1). (1)


EI numero de permutaciones con n elementos se determina
por la formula
P(n) = 1 . 2·3 ... n = nt (2) 170. Hallar m y n si se conoce que

Para las combinaciones de orden m de n elementos es valida


laformula
( 11 +1
l ll1+1
\j (n 1J ' (n+ 1J
' l
+
Ilt ' 111-1
= 5'5 3
"

c (n)= n(n-1Xn-2) ... (n-m+l) = lm(n)


m l' 2 ' 3, .. m P(m) (3) 171. Hallar el coeficiente de x 8 en el desarrollo

Se cumple la igualdad

172. Hallar el coeficiente de xm en el desarrollo segun las


potencias de x de la expresion
Para los valores enteros y positivos de n y cualesquierax y a
es valido el desarrollo

( (n) (n)
x+a ) n =x n + 1 ax n-\ + 2 a 2 x 11 - 2 + ... Examinar los casos cuando m < k, m ~ k.

... + n Ja
(n-2 n-2 x2 + ( n Ja n- \ x+a;
n
(4) 173. En el desarrollo lxE + ~ ) ", el coeficiente binominal del
x
n-I
tercer termino es mayor que el coeficiente del segundo termino en
44 unidades. Hallar el tennino que no contienex.
el termino comun de este desarrollo es igual a
30 PROBLEMAS DE MATEMATKAS ELEMENTALES

174. Hallar en el desarrollo 179. Demostrar que si en el desarrollo de x(l+xY cad a


coeficiente se divide entre el exponente de lax a la cual pertenece
este coeficiente, entonces la suma de los cocientes obtenidos
sera igual a
el sumando que no contienex.
n+l
175. (,Para cueil valor de k el termino Tk+ 1 del desarrollo por la
180. Demostrar que para un valor entero de n > 0
f6rmula del binomio de Newton

(? }(I -x r- + 2( ~ )x2(1- x r-
1 2
+ ...
sera al mismo tiempo mayor que los terminos precedente y
consecuente de este desarrollo?
... + k( Z)xk (1 - xr- k+ ... + n( ~ )xn = nx.
176. Hallar la condici6n para la cual el desarrollo (l +aY (n es un 181. Hallar el numero de metodos en que puede ser dividida una
numero entero y positivo) seg(ln las potencias de a -::j::. 0 contiene baraja de 36 cartas por la mitad de manera tal, que en cada una
dos sumandos consecutivos iguales. (,Puede contener el desarrollo de las mitades entren dos ases.
tres sumandos consecutivos iguales?
182. (,Cuantos numeros telef6nicos se pueden fonnar de cinco
177. Hallar el numero de terminos diferentes, no semejantes entre cifras de manera tal, que en cada numero tornado por separado
sf, del desarrollo todas las cifras sean diferentes?

183. Se dan 2n elementos. Se examinan todas las agrupaciones


obtenidos despues de la potenciaci6n. posibles de estos elementos en pares, considerando al mismo
tiempo que las agrupaciones que difieren s610 por el orden de
178. Supongamos que PI' P 2 , "'P n son numeros simples
los elementos en los pares y por el orden de la disposici6n de los
diferentes. (,Cuantos divisores po see el numero q = PIP2 ... Pn' pares coinciden. (,Cuantas agrupaciones diterentes existen?
inciuyendo 1 y q?
ALGEBRA 31

184. Hallar el numero de permutaciones con n elementos, en las 6. Planteamiento de ecuaciones


cuales dos elementos a y b no son inmediatos.
189.AI muitiplicardos numeros, uno de los cuales es mayor que
185. En una loteria se sortean 8 objetos. El primero que se acerca el otro en 10 unidades, el escolar cometio un error disminuyendo
ala uma saca 5 billetes. Hallar el numero de metodos en que en 4 la cifra de las decenas en el producto. Al dividir (para
puede sacarlos, de modo que: I) dos de ell os sean premiados; comprobar el resultado) el producto obtenido entre el menor de
2) por 10 menos dos de ellos sean premiados. En la uma hay 50 los factores obtuvo en el cociente 39 y en el resto 22. Hallar los
billetes. factores.

186. En una de dos rectas paralelas se han elegido m puntos, en 190. Dos ciclistas partieron al mismo tiempo del puntoA hacia el
la otra, n puntos. Cada uno de los m puntos de la primera recta punto B con velocidades diferentes pero constantes. Al alcanzar
esta unido por med io de una Ifnea recta con cada uno de los n el punto B volvieron inmed iatamente hacia atras. EI primer ciclista
puntos de la segunda recta. Hallar cuantas veces se cruzan todos dej6 atds al segundo y 10 encontr6 en el camino de regreso a la
los segmentos que unen los puntos, si se sabe que en ningun distancia de a km del punto B. Luego, despues de alcanzar el
punto se cruzan mas de dos segmentos al mismo tiempo. puntoA y de volver hacia el punto B, encuentra el segundo ciclista
despues de recorrer una k-esima parte de la distancia de A a B.
187. n rectas paraielas de un plano se cruzan por una serie de m Hallar la distancia entre A y B.
rectas paralelas. LCuantos paralelogramos pueden ser separados
en Ia red obtenida? 191. Dos autom6viles partieron al mismo tiempo de un mismo
punto en una misma direcci6n. La velocidad del primer automovil
188. Cierto alfabeto se compone de seis letras que con el fin de es de 50 kmlh y la del segundo, de 40 km/h. Despues de media
transmitirlas portelegrafo se codificaron de la siguiente manera: hora, del mismo punto y en la misma direcci6n parte un tercer
automovil que alcanza al primero 1,5 h mas tarde que al segundo.
" ,." ,.,. Hallar la velocidad del tercer autom6vil.

Al transmitir una palabra no se hicieron los intervalos que 192. De los puntosA y B parten al mismo tiempo al encuentro uno
separan una Ietra de la otra, de modo que result6 llna cadena del otro un transeunte y un ciclista. Despues de su encuentro, el
continua de puntos y rayas con 12 signos. LDe cuantas maneras tranSelll1te continua su camino hacia B, mientras que el ciclista,
se puede leer la palabra transmitida?
32 PROBLEMAS DE MATEMATICAS ELEMENTALES

vuelve atnls y se dirige tambien hacia B. EI transeunte, que partio km de A, 'horas despues del primer encuentro. Hallar la distaneia
deA , llegaaB' h mas tarde quee\ cic\ista. GCuanto tiempo paso entre A y B Yla veloeidad de ambos motociei istas.
hasta el encuentro del transeunte con el cic\ista si se sabe que la
velocidad del transeunte es k veces menor que la del cic\ista? 196. Un avion vuela de A a Ben Ifnea recta. Al eabo de cierto
tiempo, a eausa del viento contrario el avion disminuye su velocidad
193. Un cartero que se dirige sin pararse del puntoA al punto C a hasta v kmlh, como resultado de 10 eual tarda ') minutos. Durante
traves del punto B, pasa el camino deA aB con una velocidad de su segundo vuelo, el avion, por la misma causa, disminuye su
3,5 kmIh y el de B a C con la velocidad de 4 kmIh. Para conseguir velocidad hasta la mismamagnitud, pero adkm milS lejos deA
regresar de C aA en el mismo tiempo por el mismo camino, debe que en el primer vue 10 y tarda t2 minutos. Hallar la velocidad inieial
hacer 3,75 km por hora en el curso de todo el trayecto. Sin embargo, delavion.
aillegar, en el camino de regreso, al punto B con la velocidad indicada,
se detiene en este punto 14 min y para conseguir regresar al punto A 197. De dos aleaciones con diferente poreentaje de cobre que
en el tiempo indicado debe pasar de B a A con 4 km por hora. pesan m kgfy n kgfse cortan dos pedazos de igual peso. EI
Hallar la distancia que hay entre A y B Yentre B y C. pedazo eortado de la primera aleacion se funde con el resto de
la segunda y eJ pedazo cortado de la segunda aleaeion se funde
194. EI camino deA aB, de 11,5 kmde longitud, va al principio eon el resto de la primera, despues de 10 eual el poreentaje de
cuesta arriba, despues por un lugar llano y luego cuesta abajo. cobre en ambas aleaciones se hace igual. GCu<into pesa eada
Un peaton que se dirige de A a B, pasa todo el camino en 2 uno de los pedazos cortados?
horas 54 minutos y en el camino de regreso pierde 3 horas 6
minutos. La velocidad de marcha es la siguiente: cuesta arriba 3 198. Se tienen dos pedazos de aleacion de plata con cobre. Uno de
km/h, por ellugar llano 4 km/h y cuesta abajo 5 km/h. GQue ellos eontiene p% y el otro q% de eobre. GEn que proporcion se
extension ocupa el camino llano? deben tomar las aleaeiones del primero y segundo pedazos para
obtener Wla nueva aleaei6n que contenga,nlo de cobre? GPara euales
195. Para los ensayos de motocicletas de diferentes tipos, dos relaciones entre p, q y rei problema es posible y cmil sera el peso
motociclistas parten al mismo tiempo del puntoA aB y del punto maximo de la nueva aleaei6n si el primer pedazo pesa P gf y el
BaA. la velocidad de los dos motociclistas es constante y al segundo Q gf?
lIegar al punto [mal vuelven inmediatamente hacia atras. La primera
vez se encuentran a la distancia de p km de B y la segunda a q 199. Los obrerosA y B trabajan el mismo numero de dias. SiA
hubiese trabajado un dia menos y B 7 dias menos, entonees A
ALGEBR.... 33

habria ganado 72 rub. y B, 64 rub. 80 kop. * Si al contrario, A hasta la altura de b m sobre el nivel del agua en el recipiente.
hubiese trabajado 7 dias menos y B un dia menos, B habria Calcular la altura del agua en el tubo, si se sabe que la altura de
ganado 32 rub. 40 kop. mas queA. LCuanto gano cada uno en la columna de Iiquido en el barometro de agua a presion
realidad? atrnosferica es igual a em.

200. Dos cuerpos se mueven por una circunferencia en 203. Un tuba cilindrico en el que se desplaza un piston esta
direcciones opuestas. El primero se mueve uniformemente con sumergido en una taza con mercurio. En el tubo, el mercurio se
una velocidad lineal v y el segundo tiene un movimiento encuentraa 12 cm mas alto que su nivel en la taza, y la columna de
uniformemente acelerado con una aceleracion lineal a. En el aire sobreel mercurio (hastael piston) es igual a 291 cm. EI piston
instante inicial ambos cuerpos se encontraban en un mismo punto desciende 6 cm. (, Cua! sera eneste caso la altura de la columna de
A y la velocidad del segundo era nula. (,Al cabo de cuanto tiempo mercurio, si la presion exterior del aire equivale a 76 cm de Hg?
se encontraran por primera vez estos cuerpos si el segundo
encuentro sera de nuevo en el puntoA? 204. En cierto instante el reloj marca 2 minutos menos de 10
debido, aunque va adelantado. Si marcase 3 minutos menos de
201. Una piscina se Ilena de agua con ayuda de dos grifos. Al 10 que debe marcar, pero se adelantara al dfa en ~ minuto mas de
principio, el primer grifo permanecio abierto una tercera parte del 10 que se adelanta, entonces marcaria la hora exacta un dia antes
tiempo requerido para lIenar la piscina, valiendose solamente del que 10 marca. LEn cuantos minutos al dfa se adelanta este reloj?
segundo grifo. Luego, al contrario, el segundo grifo permanecio
abierto una tercera parte del tiempo necesario para Ilenar la piscina 205. Dos depositantes depositaron en la caja de ahorros iguales
haciendo uso solo del primer grifo. Despues de esto se Ilene una +it cantidades. EI primero retiro su deposito al cabo de m meses y
parte de la piscina. Calcular el tiempo necesario para llenar la piscina recibio p rub. EI segundo, al retirar su deposito pasados los n
haciendo uso de cadagrifo por separado, si manteniendo abiertos meses recibio q rub. (,Cuanto dinero deposito cada uno y que
ambos grifos a la vez la piscina se !lena en 3 horas 36 minutos. porcentaje paga la caja de ahorros?

202. Un tubo cilindrico con piston se encuentra sumergido en un 206. Dos puntos se desplazan uniformemente y en una misma
recipiente con agua; entre el piston y el agua hay una columna de direccion por una circunferencia de radio R. Uno de ellos hace
aire de h mala presion atmosferica. Luego, el piston se eleva una vuelta completa en t s, mas rapido que el otro. EI tiempo
entre dos encuentros consecutivos de los puntos es igual T. Hallar
* EI rublo (rub. ) contiene 100 kopeks (kop). N . del T. las velocidades de estos puntos.
34 PROBLEMAS DE MATEMATICAS ELEMENTALES

207. En un frasco hay una soluci6n de sal decocina. Del frasco se el tiempo perdido, conduce el tren el resto del camino a una
vierte a una probeta ~ parte de la soluci6n y se concentra por velocidad que supera en un 20% la veJocidad del tren antes de
evaporaci6n hasta que el porcentaje de sal en la probeta aumente su parada. De resultas, el tren lleg6 a B con un retraso de I h. Al
el doble. Luego, la soluci6n concentrada se vierte de nuevo al dia siguiente el tren que se dirigia deA aB por el mismo horario
frasco. Como resultado, el contenido de sal en el frasco aumenta se detuvo por la misma causa alSO km mas !ejos de A que el
en un p por ciento. Determinar el porcentaje de sal inicial. primer tren. Despues de 2 horas de parada, este tambien aument6
su velocidad en un 20% en comparaci6n con la inicial, pero
208. Dos recipientes iguales de 30 I de capacidad cada uno. consigui6 recuperar solamente media hora y tard6 a B 1 h 30
Contienen en total 30 I de alcohol. EI primer recipiente se llena min. Hallar la distancia entre A y B.
hasta los bordes can agua, y con la mezcla obtenida se rellena
adicionalmente el segundo recipiente. Luego, del segundo 211. EI embarcadero A se encuentra a la distancia de a km rio
recipiente se echan al primero 121 de la nueva mezcla. (,Cwlnto abajo del embarcadero B. Un bote autom6vil hace e! viaje deA
alcohol habia al principio en cada recipiente, si al final en el a B y de vuelta (sin detenerse en B) en T horas. Hallar la velocidad
segundo hay 21 de alcohol menos que en el primero? del bote en agua muerta y la velocidad de la corriente, si se sabe
que en uno de los viajes, al regresar de BaA, el bote sufri6 una
209. Tres turistasA, By C pasan un embalse de almacenamiento averia a b km deA, a causa de 10 cual se detuvo To h y disminuy6
de s km de ancho: A a nado con una velocidad de v kmlh, mientras en dos veces su velocidad ulterior, como resultado de 10 cual el
que By C, con auxilio de un bote autom6vil cuya velocidad es VI recorrido de B aA requiri6 el mismotiempo que el deA aBo
km/h. Pasado cierto tiempo desde el inicio del paso, C decide
vencer a nado el resto de la distancia (nada con la misma velocidad 212. Un dep6sito de 425 m3 de capacidad se llen6 de agua con
que A). B, mientras tanto, vue Ive hacia atnis para coger a A. A ayuda de dos grifos. EI primer grifo permaneci6 abierto 5 horas
sube al bote y continua su camino junto can B. Los tres turistas mas que el segundo. Si el primer grifo hubiese estado abierto el
lIegan a la orilla opuesta al mismo tiempo. Hallar el tiempo que tiempo que en realidad estuvo abierto el segundo y el segundo,
consumi6 el paso. el tiempo que verdaderamente permaneci6 abierto el primero,
entonces, del primer grifo hubiera fluido dos veces menos agua
210. Un tren parte de la estaci6nA en direcci6n haciaB a las 13 que del segundo. Si se abren los dos grifos al mismo tiempo el
h 00 min. A las 19 h 00 min tuvo que detenerse debido a la dep6sito se !lena al cabo de 17 horas.
obstrucci6n de la via por acumulaci6n de nieve. Despues de 2 Tomando en consideraci6n todas las condiciones sefialadas,
horas se consigui6 Jimpiar Ja via y eJ maquinista, para recllperar determinar el tiempo que permaneci6 abierto el segundo grifo.
ALGEBRA 35

213. Segun el horario un tren debe pasar el trecho AB de 20 km atras, se desplaza en movimiento uniformemente acelerado yen
con una velocidad constante. La primera vez el tren pasala mitad el primer segundo pasa 25 m, mientras que en el segundo siguiente
del camino a dicha velocidad y se detuvo 3 min. Para lIegar a recorre t m mas. EI otro cuerpo se encuentra en movimiento
tiempo aBel tren tuvo que aumentar su velocidad en la segunda uniformemente retardado y en el primer segundo pasa 30 m,
mitad del trecho en 10 km por hora. La segunda vez el tren
mientras que en el segundo siguiente hace ~ m menos. GDentro
estuvo parado a Ia mitad del camino durante 5 min. GA que
de cuantos segundos el primer cuerpo alcanzara al segundo?
velocidad tuvo que recorrer el resto del trecho para llegar a B
segun el grafico?
217. Una lancha pasa rio abajo una distancia de 10 km y despues
214. Dos aviones despegan al mismo tiempo de los puntosA y B rio arriba 6 km. La velocidad de la corriente es igual a 1 kmlh.
al encuentro uno del otro y se encuentran a la distancia de a km GEntre cuMes limites debera encontrarse la velocidad propia de
de la mitad deAB. Si el primer avian hubiese despegado b horas la lancha para que todo el viaje Ie ocupe de 3 a 4 horas?
mas tarde que el segundo, entonces se habrfan encontrado en la
mitad deAB. Si, al contrario, el segundo avian hubiese despegado 218. Las capacidades de tres recipientes cubicosA, Bye son
b horas despues de despegar el primero, estos se habrfan entre sf como 1:8:27 y los volumenes del agua que ellos contienen
encontrado a una cuarta parte de la distancia hastaB. Hallar la como 1:2:3. Despues del transvase de A a B y de B a C, en los
distancia entre A y B y la velocidad de los aviones. tres recipientes se obtuvo una capa de agua de igual profundidad.
Luego, de CaB se transvasan 128 fly despues de esto de B a
215. Del embarcadero A partieron al mismo tiempo rio abajo un A una cantidad tal, que la profundidad del agua en A resulta dos
veces mayor que en B. Al mismo tiempo resulto que enA hay
bote y una balsa. EI bote, despues de pasar 96 km rio abajo,
100 I de agua menos que en el momenta inicial. GCuanta agua
volvio hacia atnis y regreso a A al cabo de 14 horas. Hallar la
habfa al principio en cada recipiente?
velocidad del bote en agua muerta y la velocidad de la corriente, si
se sabe que en su camino de regreso el bote encontro a la balsa a 219. Hallar un numero de cuatro cifras por las condiciones
la distancia de 24 km deA. siguientes: la suma de los cuadrados de las dos cifras extremas
es igual a 13; la suma de los cuadrados de las cifras del medio es
216. Dos cuerpos iniciaron su movimiento al mismo tiempo yen iguala 85. Si del numero buscado se resta 1089, se obtiene un
una misma direccion a partir de dos puntos, la distancia entre los numero que se escribe con las mismas cifras, pero en orden
cllales es igllal a 20 m. Uno de elios, el que se encuentra mas contrario.
36 PROBLEMAS DE MATEMATICAS ELEMENTALES

220. Dos puntos se desplazan por una circunferencia de I m de trozo entre A y B fuera la misma que en la seccion de B a C,
longitud con las velocidades v y w < v. l,Pasado cuanto tiempo entonces todo el camino deA a C ocuparia 5,5 horas. l,Cwinto
despues del inicio del movimiento sucederan los encuentros tiempo se necesitaria, en este caso, para recorrer el camino de C
consecutivos de los puntos, si estos se mueven en una misma a A rio arriba?
direccion y el primero comenzo su movimiento t segundos antes
que el segundo, retrasandose al principio a m del segundo en 224. Un vasa contiene una solucion de un p% de acido. De este
sentido del movimiento (0 < l)? se vierten 0 lyse anade la misma cantidad de solucion de q% de
acido (q <p). Lliego, despues del mezclado, esta operacion se
221. Una aleacion de dos metales de P kgf de peso pierde en el repite k-I veces, como resliitado de 10 cllal se obtiene una
aguaA kgf. Un pedazo de uno de los metales que componen la solucion de r% de concentracion. Hallar la capacidad del vaso.
aleacion, de P kgf de peso, pierde en el agua B kgf de peso, y
del otro, C kgf. Hallar el peso de los metales que componen la 225. En una caj a de ahorros que paga un p% anual se depositan
aleacion y estudiar la posibilidad de la solucion del problema en A rublos. A final de cada ano el depositario retiraB rublos. l,Dentro
dependencia de las magnitudes P, A, By C. de cuantos afios, al retirar la suma correspondiente, el resto sera
tres veces mas que el dep6sito inicial? l,Para cllales condiciones
222. Unas balsas partieron del puntoA hacia la desembocadura el problema tiene soluci6n?
del rio a favor de la corriente. En la desembocadura del rio un
barco las tomo a remolque y pasados 17 t dias desde el momenta 226. En una parcela forestal, el acrecimiento anual de madera es
en que salieron del puntoA las condujo porun lago al punto B. igual a un p%. Cada invierno se asierra cierta cantidad x de
l,Cuanto tiempo remolco el barco a las balsas por ellago desde madera. l, Cual debera ser x para que dentro de n afios la cantidad
la desembocadura del rio hasta el punto B, si se sabe que el de madera en la parcela aumente q veces, si la cantidad inicial de
barco hacfa(sin remolque) el viaje deA aBen 61 horasy deB madera es igual a o?
aA en 79 horas, y que la velocidad durante el remolque es dos
veces men or? 227. Se tienen n recipientes cilfndricos igllales. El primero se
llena por completo de alcohol y los demas hasta la mitad de una
223. En el trozo de A a B de un rio, la corriente es tan debil que mezcla de alcohol con agua, con la particularidad de que la
se puede despreciar; en la seccion de B a C la corriente ya es concentraci6n de alcohol en cada recipiente es k veces menor
bastante fuerte. Una lancha salva la distancia deA a C rio abajo que en el anterior. Con el contenido del primer recipiente se lIen6
en 6 horas, de C aA, rio arriba, en 7 horas. Si la corriente en el hasta los bordes el segundo, despues con el contenido del
ALGEBRA 37

segundo, el tercero y as! sucesivamente hasta el ultimo. Hallar la 232. Se dan dos series de numeros:
concentraci6n de alcohol obtenida en el ultimo recipiente.

228. Se examina una fracci6n (la relaci6n entre dos numeros


enteros), cuyo denominador es menor en una unidad que el
cuadrado del numerados. Si ai'iadimos dos unidades al numerador suponiendo que sea S/1 = b l + b1 + ... + bn , demostrar que
y al denominador el valor de la fracci6n sera mayor que Si del ±.
numeradory del denominador se restan tres unidades, la fracci6n
sigue siendo positiva, pero sera menor que TrJ. Hallar esta alb l +a2b2 + ... +anb" =(al -a1)SI +(a2 -a3 )S2 + ...
fracci6n. ... +(an_1 -an)S,,_1+a"S".

233. Demostrar que de la igualdad


7. Problemas diferentes ~ ~ 1
a- + b- + e- = be + ae + ab,
Transformaciones algebraicas
donde a, bye son numeros reales, se deduce que a = b = e.
229. Calcular la suma
234. Demostrar que si a 3 + b 3 + e 3 = 3abe, entonces, 0 bien
~I_+ 1 2 + ... + (n+ k'-1\ (n- k)'
n(n+l) (n+I)(n+)
a 2 +b 2 +c 2 =bc+ca+ab, 0 a+b+c=O.
230. Simplificar laexpresi6n
235. Demostrar que si
(x + a Xx 2 + a 2 ) ... (X 2n - 1 + a 2n - 1 ). 2 2 2
a l + a 2 + ... + an =p2,
231. Simplificar la expresi6n ?? ?
b l- + bi. + ... + b; = q~ ,
1

alb l + a 2 b2 + .. , + anbn == pq
38 PROBLEMAS DE MATEMATICAS ELEMENTALES

y pq =t- 0, entonces a l = 2b l , a o = 2b o' ••• , a = 2b , donde 240. Supongamos que sean Xl y x 2 las rakes de la ecuaci6n
f.
Iv = (Todas las magnitudes se suponen reale~). n

236. Se conace que la secuencia de los numeros ai' a2, a3 , •.. para
cualquiera que sea n, satisface a la relaci6n S in resolver la ecuaci6n, expresar por medio de sus coeficientes
las cantidades:
1 1 4 22 4
1) 2+-2; 2) XI +XI x 2 +X 2 ·
xI x2
Expresar an por medio de ai' a2 y n.
241.l,Cwiles condiciones deberan satisfacer los coeficientes reales
237. La sucesi6n de los numeros ai' a2, a3, ••• , an' ... para n > 2 ai' b l , a z' b2 , a3 y bJ para que la expresi6n
satisface ala relaci6n

sea el cuadrado de un polinomio de primer grado respecto de X


donde a y ~ (a =t- ~) son numeros conocidos. Expresar an por con coeficientes reales?
medio de a, ~,al' a2 .
242. Demostrar que las rakes de la ecuaci6n cuadniticax2 + px +
Teorema de Bezout, propiedades de las raices de los +q = 0 con coeficientes reales son negativas 0 tienen una parte
polinomios real negativa en el unico caso cuando p > 0, q> O.

°
238. Las rakes XI y x 2 de la ecuaci6n Xl - 3ax + a 2 = son tales
que xf + xi = 1,75. Determinar a.
243. Demostrar que si ambas rakes de la ecuaci6n

X2 + px+ q = 0
239. Se tiene la ecuaci6n X2 + px + q = O. Componer una son positivas, entonces las rakes de la ecuaci6n
ecuaci6n cuadnitica, cuyas rakes sean
qi + (p - 2rq) y + I - pr = °
senin tambien positivas para todos los valores de r ;::: O. Aclarar
si es justa esta afirmaci6n siendo r < O.
ALGEBRA 39

244. Hallar todos los valores reales de p para los cuales las rakes 248. Demostrar que para cualesquiera valores reales de a,p y
de fa ecuaci6n q, las ralces de la ecuaci6n
(p - 3 )X2 - 2 px + 6 p = 0 1
- -+ - - =-0
x- p x-q a-
son reales y positivas. son reales.

245. Para cualquier valor positivo de Ie todas las rakes de la 249. Demostrar que la ecuaci6n cuadnitica
ecuaci6n
ax2 + bx + c + Ie = 0 a 2 x 2 + (b 2 + a 2 - c 2 )x + b2 = 0

son reales y positivas. Demostrar que en este caso a = 0 (se no puede tener ralces reales si a + b > c y la-hi < c.
supone que los coeficientes a, by c son reales).
250. Se conoce que Xl' x 2 y X 3 son las ralces de la ecuaci6n
246. Demostrar que ambas ralces de la ecuaci6n
x3 - 2X2 + X + 1 = O.
Xl +X + I=0
Componer una nueva ecuaci6n, cuyas rakes sean los numeros
satisfacen a la ecuaci6n

251. Se conoce que XI' x 2 YX3 son las ralces de la ecuaci6n

donde m, n y p son numeros enteros cuafesquiera. X


3
-X
2
-1=0

247. EI sistema de ecuaciones Componer una nueva ecuaci6n, cuyas rakes sean los numeros

a(x2 -l- i)+ X + Y - Ie = 0,1


X-Y+Ie=oJ 252. Expresar ef termino independiente c de la ecuaci6n cubica

tiene soluciones reales para cualquier valor de Ie. Demostrar


que a = 0
40 PROBLEMAS DE MATEMATICAS ELEMENTALES

por medio de los coeficientes a y b, conociendo que las rakes 256. Todas las rakes del polinomio
de la ecuaci6n forman una progresi6n aritmetica.
p(x)=x 3 + px+q
253. Supongamos que todas las rakes de cierta ecuaci6n
con coeficientes rea!es y q ::/; 0 son reales. Demostrar que P < o.
x 3 + px 2 + qx + r = 0
257. Demostrar que la ecuaci6n
sean positivas. ~A cwil condici6n suplementaria debenin satisfacer
sus coeficientes p, q y r para que de los segmentos, cuyas
longitudes son iguales aestas rakes, se pueda construir un
triangulo? donde ay b son reales y b > 0, tiene solamente una raizpositiva.

lndicacion. Estudiar la expresi6n 258. Hallar todos los val ores reales de a y b para los cuales las
ecuaClones
, 2
XO + ax + 18 = 0,
254. Las ecuaciones
X
3
+ Plx+ql =0,
x 3 + bx + 12 = °
tienen dos rakes comunes y determinar estas rakes.
x 3 + P2 X + q2 =0 259. Demostrar que
(PI ::/; P2' ql ::/; q2) tienen una raiz comun. Hallar esta raiz y las 31 r- 3/ r;:;
demas rakes de ambas ecuaciones. ",120+14"' 2 + 1/ 20-14\1'2 =4.
260. Supongamos que a, bye sean por pares numeros no iguales
255. Hallar todos los val ores de 'A para los cuales las ecuaciones entre sl.
Demostrar que la expresi6n
"'--'< 3 -x 2 -x-/dl
( ) =0

'Ax2-x-('A+l)=0
no es igual a cero.
tienen una raiz com un y hallar esta raiz.
ALG EBRA 41

261. Descomponer en factores la expresi6n que cierta afirmaci6n es justa para cualquier numero real n, es
suficiente que: a) esta afirmaci6n esjusta para n = 1; b) si esta
( x+y+z ) 3 - x3 - y3- z 3 ;
afirmaci6n es justa para cualquier numero real n, sera tambien
justa para el numero consecuente n+ 1.
262. Demostrar que si tres numeros reales a, bye esta enlazados
por la relaci6n
266. Demostrar que
1 1 1
-+ - + - = - -- - = n(n+I)(n+2)
abc a+b+c 1+ 3 -r' 6 + 10 + ... + (n -I2 )n + n(n+l)
2 6'

entonces, obligatoriamente dos cualesqu iera de estos numeros


267. Demostrar que
son iguales en valor absoluto y tienen signos contrarios.
12 +-')2,,2 2 _ n(n+I)(2n+l)
+-' + ... +n - 6 .
263. Determinar para cuales valores complejos de p y q el binomio
X4 - 1 se divide entre el trinomio cuadrado x 2 + +px +q. 268. Demostrar que
1 1 1 n(n+3)
264. l,Para cuales valores de a y n el polinomio 1·23 + 2·3-4 + ... + n(n+l)(n+2) = 4\n+I)(n+2)'
xn _ax n- 1 +ax-l 269. Demostrar la f6rmula de Moivre
es divisible entre (x - 1)2?
(cos<p + isen<p r= cosn<p + isen n<p.

265. Al dividir el polinomio p(x) entre x - a, el resto es A, al 270. Demostrar que para cualquier valor entero y positive de n,
dividirlo entre x - b, el resto es B, y si se divide entre x - c, el lacantidad
resto es C. Hallar el polinomio que se obtiene en el resto de la
an an _b n
= ~ ,
don de 1+,/5
a=-~- ,
b =--
1+.Js
divisi6n de p(x) entre (x - a) (x - b) (x - c), admitiendo que ~5 L 2 '
entre los numeros a, bye no hay iguales.
es un numero entero y positivo.
Metodo de inducci6n matematica
271. Demostrar que si los numeros reales aI' a 2, ••• , an' ...
satisfacen a la condici6n - 1 < ai S;; 0, i = 1,2 ... . , entonces para
En los problemas propuestos a continuaci6n es conveniente usar
cualquier valor de n se cum pie la desigualdad
el metodo de inducci6n matematica completa. Para demostrar
42 PROBLEMAS DE MATEMATICAS ELEMENTALES

Este se alcanza siendo


_ b
Xo - 2a' (4)
272. La enesima potencia generalizada de cualquier nUmero
De fonna amiloga se examina el problema sobre el valor
a, designada por el simbolo (a)n' para los valores enteros y
nuiximo del trinomio, en el caso en que a < 0
no negativos de n se determina de la siguiente manera: si
n=O, (a)n=l, siendo n>O, (a)n = a(a - n + 1). Demostrar 273. Dos ferrocarriles rectos AA ' YBB' son perpendiculares
que para la potencia generalizada de la swna de dos nUmeros entre si y se cruzan en el punto C. Las distancias AC y BC son
es valida de la formula de binomio de Newton respectivamente iguales a a y b. De los puntos A y B parten al
mismo tiempo dos trenes en direccion hacia C con las velocidades
VI y V1 respectivamente. GAl cabo de cuanto tiempo despues de
su partida la distancia entre los trenes sera minima? GA que es
Valores maximos y minimos igual esta distancia minima?

Para hallar el valor minimo del trinomio cuadrado 274. Los puntosAy B se encuentran en la via principal y rectilinea
que va del Oeste al Este. El punto B se encuentra a 9 km mas al
Y =ax 2 +bx+c (1) Este que A. Un coche parte del punto A hacia el Este, y se
desplaza uniformemente con una velocidad de 40 klnlh. AI mismo
en el caso en que a> 0, el trinomio se representa en la forma tiempo, una motocicleta sale del punto B en la misma direcci6n
y = a(x + ~)2
2a
_ &2-4aO'.
4a
(2) con una aceleraci6n con stante igual a 32 kJnIh2 • Hallar la distancia
maxima entre el coche y la motocicleta en el curso de las dos
Puesto que el primer sumando en el miembro derecho no es primeras horas de movimiento.
negativo sea cual fuere el valor de x y el segundo sumando no
depende de x, el trinomio adquiere su valor minimo con la Indicaci6n. Es util trazar la grafica de la distancia entre el
condici6n de que el primer sumando es igual a cero. coche y la motocicleta en funci6n del tiempo.
Asi pues, eI valor minimo del trinomio es igual a
275. Hallar el valor maximo de la expresi6n
& 2 _ 4ac
Yo = - - 4-a- (3)
210g4 x + l2210g2 x·210g ·~,
x
ALGEBRA 43

suponiendo que x varia entre 1 y 64. 280. Hallar el numero complejo z que satisface a las igualdades
::"':121 _ 5 ' :-41-1
276. Hallar el valor maximo de la funci6n I :- 8i i -"3 Y I :-81- .
y =_ x_· (a > 0, b > 0). 281. Hallar el producto de la multipl icaci6n
ax 2+h

277. Hallarel valorminimo de laexpresi6n

siendo x 2: 0. 282. Entre los numeros complejos z que satisfacen a la condici6n

278. Hallar el valor minimo de la funci6n


Iz - 25i ! ~ 15,

hallar el numero con menor argumento. Trazar el dibujo.


I
<p(x) = x - a I+Ix - b I+ Ix - c I+!x - d I'
donde a < b < c < d son numeros reales fijos y x adquiere 283.;.,A cmil condici6n debeni satisfacer el numero complejo
valores reales arbitrarios. a + bi para que se Ie pueda presentar en la forma
a+bi= J-ix
J+1X'
Indicacion. Es c6modo llevar a cabo los razonamientos
sefialando los numeros a, b, c y den el eje numerico. donde x es un numero real?

Numeros complejos 284. ;.,Cua! valor maximo puede adquirir el m6dulo del numero
complejozsi
279. Hallar todos los valores de z que satisfacen la igualdad i z+~ I =I?
285. A traves del punto A se han trazado n rayos bajo angulos
iguales a ";' . En uno de estos rayos, a la distancia d de A se ha
(Izl es el m6dulo del numero complejo z). tornado un punto B, del cual se baja una perpendicular al rayo
vecino. De la base de esta perpendicular se baja de nuevo una
perpendicular al rayo siguiente y as! sucesivamente hasta 10 infmito.
44 PROBLEMAS DE MATEMATICAS ELEMENTALES

Hallar la longitud L de la linea quebrada que se enreda alrededor GEOMETRIA


deA, obtenida de este modo, y aclarar como variaraL al aumentar
el numero n, en particular, al aumentarlo ilimitadamente.

286. Un numero de seis cifras empieza por la izquierda con la


cifra I. Si se pasa esta cifra del primer lugar al ultimo sin alterar el
A. PLANIMETRiA
orden de las demas cinco cifras, se obtiene un nuevo numero
Observaciones preliminares
tres veces mayor que el inicial. Hallar el numero inicial.
Senalemos las siguientes relaciones entre los elementos de un
triangulo de lados a, bye y los angulos opuestos A, Bye
287. Demostrar que si el numero realp = abc (a, bye son cifras
correspond ientemente.
de las clases correspondientes) es divisible entre 37, entonces
los numeros q = beay r=eab tambien son divisibles entre 37.
1. Teorema de los senos:
288. Demostrar que la suma de los cubos de tres numeros enteros
a _ b _ e _ 2R
sen A - sen B - sen C - ,
sucesivos es divisibles entre 9.
donde R es el radio del circulo inscrito.
289. Demostrar que la suma
2. Teorema de los cosenos:

es divisible entre 3 para cualquier valor entero y positivo de n.


Para calcular el area S de un triangulo, ademas de la formula
290. 120 bolas iguales se han colocado compactamente en forma
de una piramide triangular regular. (,Cuantas bolas hay en la base?

291. En un cajon se han metido k cajones. En cada uno de estos Donde acs uno de los lados del triangulo y ha es la altura bajada
a este lado, a continuacion se emplean las siguientes formulas:
kcajones, 0 bien se han metido kcajones 0 no se hametido ni uno
Formula de Heron
y as! sucesivamente. Hallar la cantidad de cajones vacios si m
cajones resultaron llenos.
S ="j p(p - a)(p - b)(p - c).
GEOMETRiAIPlanimetria 45

donde p = a+i+ c es el semiperfmetro; 297. A traves de cierto punto tornado dentro del triangulo, se
han trazado tres rectas paralelas respectivamente a sus lados.
S = 1absenC;S = I'p. Estas rectas dividen el area del triangulo en seis partes, tres de
las cuales son triangulos con areas iguales a SI' S2 YS3' Hallar el
donde r es el radio del cfrculo inscritoen el triangulo y pes el
area del triangulo dada.
semiperimetro.
298. Los lados bye de un triangulo son conocidos. Hallar el
1. Problemas de calculo tercer lado x, si se sabe que es igual a la altura bajada a este
lado. (,Para cuales relaciones entre bye existe este triangulo?
292. En el triangulo ABC, el angulo A es dos veces mayor que el
B. Por los lados conocidos bye hallar ellado a. 299. En el trianguloABC se han trazado las alturasAA I, BBI y
CC I' cuyas bases se han unido entre sf. Determinar la relaci6n
293. Los catetos de un triangulo rectangulo son iguales a b y c. entre el area del trianguloAIBjC j yel area del trianguloABC, si
Hallar la longitud de la bisectriz del angulo recto. se conocen los angulos del trianguloABC.

294. Hallar el tercer lado de un triangulo, si se conocen dos de 300. En el trianguloABC, a traves del punto de intersecci6n de
sus lados a y b, y se sabe que las medianas correspondientes a las bisectrices de los angulos B y C, se ha trazado una recta A1N
estos lados se cruzan formando un angulo recto. (,Para cuales paralela aBC hasta su intersecci6n con los ladosAB y AC en los
condiciones existe este triangulo? puntos My N respectivamente. Hallar la dependencia entre los
segmentos A1N, BM y CN.
295. El angulo del vertice de un triangulo, cuyos lados laterales
son iguales a a y b (a < b), esta dividido en tres partes iguales Examinar los casos:
por rectas, cuyos segmentos dentro del triangulo son entre Sl
como m:n (m < n). Hallar las longitudes de estos segrnentos. 1) las dos bisectrices son interiores;
2) las dos bisectrices son exteriores;
296. Intersectar el triangulo dado ABC con una recta DE paralela 3) una de las bisectrices es interior y la otra exterior.
al lado BC, de modo que el area del triangulo BDE sea igual a la 4) (,Cuando coincidiran los puntosMy N?
magnitud dada IC-. (,Para cuales relaciones entre IC- y el area del
triangulo ABC el problema es soluble y cuantas soluciones tiene?
46 PROBLEMAS DE MATEMATICAS ELEMENTALES

301. Dentro de un triangulo regular se ha tornado un punto 308. Se conocen las bases superior e inferior a y b de un trapecio.
arbitrario P, desde el cual se han bajado las perpendiculares PD, Hallar la longitud del segmento que une las mitades de las
P E y PF a los lados BC, CA y AB respectivamente. Hallar diagonales del trapecio.
PD+PE+PF
309. Cada uno de los vertices de un paralelogramo se ha unido
BD+CE+AF
con las mitades de los dos lados opuestos. LQue parte del area
302. Hallar la relacion entre el area del triangulo ABC y el area del paralelogramo compone el area de la figura limitada por las
de otro triangulo, cuyos lados son iguales a las medianas del Ifneas trazadas?
triangulo ABC.
310. Los puntos P, Q, R y S son respectivamente las mitades de
303. En un triangulo con lados a, b y c se ha inscrito una los lados AB, BC, CD y DA del paralelogramo ABCD. Hallar el
semicircunferencia, cuyo diametro se encuentra sobre ellado c. area de la figura limitada por las rectas AQ, BR, CS y DP, si se
Hallar el radio de esta semicircunferencia. conoce que el area del paralelogramo es igual a a-.

304. Hallar los angulos de un triangulo recrnngulo, si se sabe que 311. Se conocen las cuerdasde dos arcos de unacircunferencia
el radio de la ci rcunferencia circunscrita a este triangulo es al de radio R. Hallar la cuerda del arco igual a la suma de los arcos
radio de la circunferencia inscrita como 5 :2. dados 0 a su diferencia.

305. A un rectangulo dado circunscribir un nuevo rectangulo, el area 312. La distancia entre los centros de dos circunferencias que se
delcual seaigual anr. (,ParacuaJ valordemessolubleel problema? cruzan, de radios Ry r, es igual ad. Hallar el area de su parte comun.

306. Sobre ellado AB del rectanguloABCD hallar un punto E, 313. Tres circunferencias de radios r, r l y R tienen contacto
desde el cuallos ladosAD y DC se yean bajo angulos iguales. exterior de dos en dos. Hallar la longitud de la cuerda cortada
(,Para cual relacion entre los lados del rectangulo es soluble el por la tercera circunferencia de la tangente intema com un de las
problema? dos prirneras circunferencias.

307. Hallar el area de un trapecio isosceles, si su altura es igual a 314. Dos circunferencias de radios Ry r(R > r) tienen contacto
h y su lade lateral se ve desde el centro de la circunferencia interior. Hallar el radio de una tercera circunferencia que hace
circunscrita bajo el angulo u . contacto con las dos primeras y con su diametro comun.
GEoMETRiAlPlanimetrfa 47

315. Tres circunferencias iguales que tienen contacto entre SI de 320. Dentro de un angulo agudo se inscriben circunferencias que
dos en dos, hacen contacto exterior con un circulo de radio r. hacen contacto una con otra. Demostrar que los radios de estas
Hallar las areas de los tres triangulos curvilineos formados por circunferencias forman una progresion geometrica. Hallar la
dichas circunferencias. dependencia entre eI denominador de la progresion y la magnitud
del angulo agudo.
316. Con el segmento de longitud 2a + 2b Y sus partes de
longitudes 2a y 2b tornados como diametros, se han construido 321. En el punto A de un plano Pesta ubicado un manantial de
semicircunferencias que se encuentran a un mismo lado del luz. Sobre el plano se ha colocado un espejo semies-ferico de
segmento. Hallar el radio de la circunfe-rencia que hace contacto radio 1, cuya superficie especular interior esta dirigida hacia eI
con las tres circunferencias construidas. plano de tal modo que el eje de simetria del espejo es
perpendicular al plano P en el punto A. Determinar la distancia
317. Se tienen dos rectas paralelas y un punto A entre elias. desde eI espejo basta el plano y el radio del circulo iluminado en
Hallar los lados del triangulo rectangulo, el vertice del angulo el plano P, si se conoce que el angulo minimo entre los rayos
recto del cual se encuentra en el punto A y los vertices de los reflejados por el espejo y el plano P es igual alSO.
angulos agudos descansan en las rectas paralelas dadas, si se
conoce que el area del triangulo es igual a una magnitud dadaE. 322. Los centros de cuatro circunferencias de radio r estan
dispuestos en los vertices de un cuadrado cuyo lado es a. Hallar
318. Dentro de un poligono regular de n lados iguales aa se han el area S de la parte comun de las cuatro circunferencias que se
inscrito n circunferencias igualesde modo que cada una de estas encuentran dentro del cuadrado.
hace contacto con dos lados contiguos del poligono y con otras
dos circunferencias. Hallar el area de la «estrella» formada en el 323. Las diagonales de un trapecio dividen a este en cuatro
centro del poligono. triangulos. Hallar el area del trapecio, si las areas de los triangulos
adyacentes a las bases son iguales a SI y S2'
319. Por uno de los puntos C del arcoAB de una circunferencia
se han trazado dos rectas arbitrarias que cortan la cuerda AB en 324. Expresar las diagonales de un cuadrilatero inscrito por medio
los puntos D y Ey ala circunferencia, en los puntos Fy G. 2,Para de sus lados. Obtener el teorema de Ptolomeo: en todo
cual posicion del punto C en la cuerda AB, al cuadrilatero DEGF cuadrilatero inscriptible el producto de las diagonales es igual a
se Ie puede circunscribir una circunferencia? la sum a de los productos de los lados opuestos.
48 PROBLEMAS DE MATEMATICAS ELEMENTALES

2. Problemas de construccion 330. Trazar un triangulo por los puntos de intersecci6n de las
prolongaciones de la bisectriz, la mediana y la altura que parten
325. Se conocen dos circunferencias de diferentes radios, una fuera de un mismo vertice, con el circulo de la circunferencia circunscrita
de la otra, y un punto A en una de elias. Trazar una tercera at triangulo.
circunferencia que haya contacto con las dos dadas y que pase
porel puntoA. Examinar los distintoscasos posibles de disposici6n 331. Tomando los vertices de un triangulo como centros,
del punto A en la circunferencia dada. circunscribir circunferencias de modo que hagan contacto de dos
en dos. Examinar los casos de contacto exterior y los casos de
326. Se conocen una circunferencia, una recta y un punto A sobre contacto interior.
esta recta. Trazar una nueva circunferencia que haga contacto
con la circunferenciay la recta dadas y que pase por el puntoA. 332. Inscribir el triangulo ABC en una circunferencia dada, si se
Examinar detalladamente las soluciones que tiene el problema en conocen el vertice A, la direcci6n de la altura hA y el punto de
distintos casos. intersecci6n de la altura hB con la circunferencia.

327. Se tienen una recta, una circunferencia y un puntoA en esta 333. Cortar un trapecio con una recta paralela a la base, de
circunferencia. Trazar una nueva circunferencia que haga contacto modo que el segmento de esta recta dentro del trapecio se divida
con la recta y la circunferencia dadas y que pase por el puntoA. por las diagonales en tres partes iguales.
Analizar detalladamente las soluciones que tiene el problema en
cada caso determinado. 334. Construir un cuadrado, si se conocen uno de sus vertices y
dos puntos ubicados en los dos lados, 0 sus continuaciones, que
328. Construir un triangulo recmngulo, si se conocen su h ipotenusa no pasan por el vertice dado.
c y la altura h bajada ala hipotenusa. Hallar la longitud de los
catetos y aclarar para cual relaci6n entre h y c es soluble el 335. A traves de un punto M que se encuentra sobre la baseAC
problema. del trianguloABC, trazar una rectaMN, que separe del triangulo
una parte, cuya area sea igual a -} del area de todo el triangulo.
329. Se conocen las longitudes de los ladosAB, BC, CD y DA de (,Cuantas soluciones tiene el problema?
cierto cuadrilatero plano. Trazar este cuadrilatero, si se sa be que
la diagonal AC divide al anguloA por la mitad. 336. En un triangulo dado inscribir, haciendo uso de un compas
y una regia, un rectangulo con una de sus diagonales dadas.
GEOMETRiAlPlanimetria 49

337. A una circunferencia dada circunscribir un triangulo, si se 343. Demostrar que si P, Q, R son respectivamente los puntos
con ace uno de sus angulos y ellado opuesto a este angulo. Hallar de intersecci6n de los lados BC, CA y AB (0 sus prolongaciones)
la condici6n de solubilidad de este problema. del trianguloABC con cierta recta, entonces

338. Se tienen una recta CD y dos puntosA y B no pertenecientes PB QC RA =1.


a esta recta. Hallar en la recta dada un punto M de modo que PC QA RB

LAMC = 2LBMD. 344. En el triangulo rectangulo ABC el cateto AC es tres veces


mayor que AB. EI cateto AC esta dividido por los puntos K y F
3. Problemas de demostracion en tres partes iguales. Demostrar que

LAKB + LAFB + LACB = ~.


339. Demostrar que la mediana de todo triangulo es menor que
la semisuma de los lados que la comprenden y mayor que la 345. Supongamos que sean a y bios catetos de un triangulo
diferencia entre esta semisuma y la mitad del tercer lado. rectangulo, c su hipotenusa y h la altura bajada desde eJ vertice
del angulo recto a la hipotenusa. Demostrar que el triangulo con
340. Demostrar que en todo triangulo ABC la distancia desde el los lados h, c + h y a + b es rectangulo.
centro de la circunferencia circunscrita a este triangulo hasta su
lado BC es dos veces menor que la distancia del punto de 346. En un triangulo is6sceles de base a y lado b, el angu 10 del
intersecci6n de las alturas al verticeA. vertice es igual a 20°. Demostrar que a 3 + b 3 = 3ab 2 .

341. Demostrar que la suma de las distancias desde un punto 347. Demostrar que el angulo de un triangulo sera agudo, recto
cualquiera, tornado dentro de un triangulo regular, hasta los lados u obtuso, segun que ellado opuesto sea menor, igual 0 mayor
de este triangulo es una magnitud constante, que no depende de que el doble de la mediana correspondiente.
la posici6n del punto.
348. En el triangulo is6sceles ABC el angulo del vert ice B es
342. Demostrar que en todo triangulo, al mayor lado Ie igual a 20°. En los lados AB y BC se han tornado respectivamente
corresponde menor bisectriz. los puntos Q y P de modo que LACQ = 60° y LCAP = 50° .
Demostrar que LAPQ = 80° .
50 PROBLEMAS DE MATEMATICAS ELEMENTALES

349. Demostrar que si entre los lados a, b y c de un triangulo 354. Sobre los ladosAB y AC del trianguloABCse han trazado,
existe la dependencia a 2 = b 2 + bc, entonces los angulosA y B en sentidos opuestos, dos segmentos BD = CE. Demostrar que
opuestos a los lados a y b satisfacen a la igualdad LA = 2L B. el segmento DE se divide por ellado BC en proporcion inversa
ala proporcion de los lados AB y AC.
350. EI trianguloAOB ha sido girado en su plano 90° alrededor
del vertice 0, como resultado de 10 cual el vert ice A ha pasado al 355. Demostrar que en todo triangulo la bisectriz se encuentra
punto Aj y el vert ice B, al B j • Demostrar que en el triangulo entre la mediana y la altura trazadas desde el mismo vertice.
OAB I la medianadeiladoABj es laalturadel ilOAIB (demanera
analoga, la mediana dellado A jB en el triangulo OA jB es la altura 356. Demostrar que una recta simetrica con la mediana respecto
del Ll OABI ). ala bisectriz del angulo interno de un triangulo, divide allado
opuesto en partes proporcionales a los cuadrados de los lados
351. Demostrar que la suma de los productos de las alturas de contiguos.
un triangulo acuffingulo por los segmentos de estas comprendidos
entre el ortocentro y el vertice es igual a la semisuma de los 357. Sobre los lados de un triimgulo ABC se han tornado los
cuadrados de los lados. Generalizar esta proposicion para el caso puntos P, Q Y R de modo que tres rectas AP, BQ y CR se
de un triangulo obtusangulo. intersectan en un mismo punto. Demostrar que

352. Supongamos que las longitudes a, by c de los lados de un AR·BP·CQ = RB·PC·QA.


triangulo satisfacen a las desigualdades a < b < c, formando una
358. Demostrar que en todo triangulo, la relacion entre los radios
progresion aritrnetica. Demostrar que ac = 6Rr, donde R es el radio
R y r de las circunferencias circunscrita e inscrita y la distancia I
de la circunferencia circunscrita al triangulo y r, el radio de la
entre los centros de estas circunferencias, es
circunferencia inscrita en el mismo.

353. Demostrar que el cuadrado de la bisectriz, trazada desde


el vert ice de un triangulo arbitrario, es igual al producto de los 359. Demostrar que en todo triangulo, la relacion entre el radio
lados laterales menos el producto de los segmentos de la base. de la circunferencia circunscrita a este triangu 10 y el radio de la
Aclarar el sentido de la igualdad indicada para el caso de un circunferencia inscrita en el mismo no supera a 1.
triangulo isosceles.
GEOMETRiAlPlanimetria 51

360. Demostrar que para todo triangulo rectangulo es valida la


desigualdad AF + BE + CN = 1.
0,4 < ~ < 0,5, AB BC CA

donde res el radio de la circunferencia circunscrita y h, la altura 364. En un triangulo se ha inscrito un cuadrado de modo que
bajada ala hipotenusa. uno de sus lados se encuentra sobre ellado mayor del triangulo.
Demostrar que la desigualdad Er < x < 2r, don de xes la
361. Demostrar que en todo triangulo acutangulo longitud dellado del cuadrado y r es el radio de la circunferencia
inscrita en dicho triangulo.

365. Demostrar que los pWltoS medios de los lados de un triangulo,


donde ka' kb y kc son las perpendiculares bajadas del centro de las bases de las alturas y los puntos medios de los segmentos de
la circunferencia circllnscrita, a los lados correspondientes; ry R las alturas, comprendidos entre cada vert ice y el punto de
son los radios de las circllnferencias inscrita y circunscrita. intersecci6n de las alturas, representan nueve puntos que se
encuentran en una misma circunferencia. Demostrar, al mismo
Indicacion. Los miembros izquierdo y derecho de la igualdad tiempo, que el centro de esta circunferencia es el punto medio del
buscada pueden ser expresados por medio de los lados y los segmento que lme el punto de intersecci6n de las alturas del triangulo
angulos del triangulo. dado con el centro de la circunferencia circunscrita y que su radio
es igual a la mitad del radio de la circunferencia circunscrita.
362. Los verticesA, By C de un triangulo estan lInidos los puntos
,
A "B, y C dispuestos arbitrariamente sobre los lados opuestos 366. En un triangulo, desde las bases de cada altura se han bajado
(excepto en los vertices). Demostrar que los puntos medios de los perpendiculares a los otros dos lados. Demostrar que: I) las
segmentosAA" BB, y CC, no se encuentran en unamisma recta. bases de estas perpendiculares son los vertices de un hexagono,
tres lados del cual son paralelos a los lagos del triangulo; 2) a
363. A traves de un punto arbitrario 0, tornado dentro del triangulo este hexagono se Ie puede circunscribir una circunferencia.
ABC, se han trazado las rectas DE, FK Y MN paralelas
respectivamente a AB, A C y BC Los puntos F y M se encuentran 367. Demostrar que en todo triangulo rectangulo la suma de los
sobre elladoAB; Ey K, sobreBCy Ny D, sobreAC Demostrar catetos es igual ala suma de los diametros de las circunferencias
que inscrita y circunscrita.
52 PROBLEMAS DE MATEMAnCAS ELEMENTALES

368. Demostrar que en el triangulo rectangulo, la bisectriz del 373. Cierto punto 0 de un plano se ha unido con los vertices del
angulo recto divide en dos partes iguales al angulo formado por paralelogramoABCD. Demostrarqueel area de! trianguloAOC
la mediana y la aitura bajada ala hipotenusa. es igual a la suma 0 a la diferencia de dos de los triangulos
adyacentes formados por dos de las rectas OA, OB, OC y OD y
369. Dos triangulosABCy AIBIC[ estan dispuestos simetrica- ellado correspondiente del paralelogramo. Examinar los casos
mente uno al otro respecto al centro de la circunferencia comun cuando el punto 0 se encuentra dentro y fuera del paralelogramo.
de radio r, circunscrita a estos triangulos. Demostrar que el
producto de las areas de los triangulosABCy AIBIC[ y los seis 374. En el trapecioABCD la suma de los angulos de la base AD
triangulos obtenidos de la intersecci6n de sus lados es igual a r16. es igual a ~. Demostrar que el segmento que une los puntos
medios de las bases es igual a la semidiferencia de las bases.
370. Demostrar que la diferencia entre la suma de los cuadrados
de las distancias, des-de un punto arbitrario M de un plano hast a 375. Demostrar que la suma de los cuadrados de las diagonales
dos vertices opuestos del paralelogramo ABCD y la suma de los de un trapecio es igual ala suma de los cuadrados de los lados
cuadrados de las distancias desde el mismo punto, hasta los otros laterales con el doble del producto de las bases.
dos vertices, es una magnitud constante.
376. Demostrar que la recta que une los puntos medios de los
371. Sobre los lados del triangulo ABC se han construido los lados paralelos de un trapecio pas a por el punto de intersecci6n
triangulos equilaterosABC I , BCA I y CAB I, no superpuestos al de las diagonales.
~ ABC. Demostrar que las rectasAAI' BBI y CC I se intersectan
en un punto. 377. Demostrar que si el segmento que une los puntos medios
de los lados opuestos de un cuadrilatero es igual a la semisuma
372. Sobre los lados AB, AC y BC del triangulo ABC, tornados de los otros dos lados, el cuadrilatero es un trapecio.
como bases, se han construido tres triangulos is6sceles semejantes
ABP, ACQ YBCR, los dos primeros fuera del triangulo dado y el 378. Demostrar que si las diagonales de dos cuadrilateros son
tercero hacia el mismo lado que este. Demostrar queAPRQes respectivamente iguales y se intersectan formando angulos iguales,
un paralelogramo (0 que los puntos A, P, Ry Q pertenecen a una los cuadriliiteros son equidimensionales.
misma recta).
379. Demostrar que por 10 menos una de las bases de las
perpendiculares bajadas desde un punto tornado arbitrariamente
GEoMETRiAlPlanimetria 53

dentro de un polfgono convexo a los lados de este, se encuentra 385. Tres circunferencias iguales se cruzan en un punto. EI
en el mismo lado, y no en su prolongaci6n. segundo punto de intersecci6n de dos cualesquiera de estas
circunferencias y el centro de la tercera determinan a la recta que
380. Demostrar que las bisectrices de los angulos internos de un pas a por dichos puntos. Demostrar que las tres rectas que se
paralelogramo, en su intersecci6n forman un rectangulo, cuyas obtienen se intersectan en un punto.
diagonales son iguales a la diferencia de los lados adyacentes del
.parale\ogramo. 386. Dos circunferencias tienen contacto interno en el puntoA .
EI segmento AB esel diametro de la circunferencia mayor. La
381. Demostrar que las rectas que unen sucesivamente los centros cuerda BK de la circunferencia mayor hace contacto con la
de los cuadrados construidos sobre los lados de un paralelogramo circunferencia menor en el punto C. Demostrar que AC es la
y que son contiguos a este por fuera, forman tambien un cuadrado. bisectrizdel trianguloABK.

382. Demostrar que si en un cuadrilatero arbitrario ABCD se 387. En el sector de una circunferencia de radio R se ha inscrito
trazan las bisectrices internas, los cuatro puntos de intersecci6n una circunferencia de radio r. La cuerda del sector es igual a 2a.
de las bisectrices de los angulosA y C con las bisectrices de los Demostrar que
angulos B y D se encuentran sobre una circunferencia. III
- =-- +- .
r R a
383. A una circunferencia se Ie han trazado dos tangentes.
388. A una circunferencia se Ie han trazado dos tan gentes que
Demostrar que la longitud de la perpendicular bajada desde un
cortan en los puntos A y B a una recta que pas a por el centro de
punto arbitrario de la circunferencia a la cuerda que une los puntos
la circunferencia, formando con esta recta angulos iguales.
de contacto de estas tangentes con la circunferencia, es la media
Demostrar que cualquier tangente (m6vil) corta en las tangentes
proporcional entre las longitudes de las perpendiculares trazadas
(fijas) dadas unos segmentosACy BD, el producto de los cuales
desde el mismo punto a las propias tan gentes.
es constante.
384. Demostrar que las bases de las perpendiculares bajadas
389. Demostrar que la suma de los cuadrados de las longitudes
desde un punto arbitrario de una circunferencia a los lados de un
de dos cuerdas de una circunferencia, perpendiculares entre sf y
triangulo inscrito en esta ultima, se encuentran en una misma recta.
que se cruzan, es mayor que el cuadrado del diametro de la
circunferencia, y que la suma de los cuadrados de los segrnentos,
54 PROBLEMAS DE MATEMAnCAS ELEMENTALES

en los cuales el punta de interseccion divide a las cuerdas, es 394. Demostrar que si dos puntos se encuentran fuera de una
igual al cuadrado del diametro. circunferencia y la recta que los une no corta a esta circunferencia,
entonces la distancia entre estos dos puntos es mayor que la
390. Demostrar que si se divide la cuerda de una circunferencia diferencia de las longitudes de las tangentes a la circunferencia,
en tres partes iguales y los extremos de la cuerda y los puntas de trazadas desde los puntas dados, y menor que su suma.
division se unen can el centro de la circunferencia, entonces, el Demostrar, ademas, que una de estas desigualdades no se
angulo central correspondiente se dividini en tres partes, una de cumplira si la recta corta a la circunferencia.
las cuales es mayor que las otras dos.
395. A traves del punto medio C de una cuerda arbitraria AB de
391. Demostrar que si de los extremos del diametro de una una circunferencia, se han trazado dos cuerdas KL y MN (K YM
circunferencia se trazan dos cuerdas que se cruzan, la suma de se encuentran a un mismo lado de AB), Q es el punto de
los productos de cada cuerda por su segmento desde el extremo interseccion de AB y KN, P es el punta de interseccion de AB y
del diametro hasta el punto de interseccion es una magnitud ML. Demostrar que QC = CPo
constante.
396. Una circunferencia ha sido dividida arbitrariamente en cuatro
392. Desde cada uno de dos puntos de una recta se han trazado partes, y los puntas medios de los arcos obtenidos se han unido
dos tangentes a una circunferencia. En los angulos formados con con segmentos de rectas. Demostrar que entre estos segmentos
sus vertices en estos puntos se han inscrito circunferencias de dos senin perpendiculares entre Sl.
iguales radios. Demostrar que la linea de los centros de estas
circunferencias es paralela a la recta dada. 397. Demostrar que para cualquier linea quebrada cerrada en el
plano, cuyos segmentos no se cruzan entre Sl, existe una
393. Demostrar que si el diametro de una semicircunferencia se
divide en dos partes arbitrarias y a cada una de estas se la
circunferencia cuyo radio es igual a * del perimetro de la linea
quebrada y fuera de la cual no hay ning(In punta de la quebrada.
circunscribe una semicircunferencia dentro de la
semicircunferencia dada, entonces, el area encerrada entre las
398. (,Puede ser regular un triangulo, las distancias desde los
tres semicircunferencias sera igual al area de la circunferencia
vertices del cual hasta dos rectas perpendiculares entre Sl se
cuyo diametro es igual a la longitud de la perpendicular levantada
expresan con numeros enteros?
dentro de la semicircunferencia dada desde el punto de division
de su diametro.
GEoMETRiAlPlanimetria 55

399. En dos puntos A y B de una recta, por un mismo lado de 404. Demostrar que si una circunferencia tiene contacto interior
esta, se han levantado dos perpendiculares AA I = a y BB I = b. con tres lados de uncuadrilatero y corta al cuarto lado, entonces
Demostrar que conservando sin variac ion las magnitudes ay b, el la suma de este ultimo lado y ellado opuesto es mayor que la
punto de interseccion de las rectasAB I y AlB se encontrara a una suma de los otros dos lados del cuadrilatero.
misma distancia de la rectaAB independientemente de la posicion
de los puntos A y B. 405. Demostrar que si una circunferencia tiene contacto interior
con tres lados de un cuadrilatero, el cuarto lado del cual no corta
400. En el angulo recto con el vertice A se ha inscrito una ala circunferencia, la suma del cuarto lado y del opuesto es menor
circunferencia; By C son los puntos de contacto. Demostrar que que la suma de los otros dos lados del cuadrilatero.
si a dicha circunferencia se Ie traza una tangente que corte a los
lados AB y AC en los puntos My N, entonces, ella cortara en 406. Se tienen dos semicircunferencias iguales que hacen contacto
estos lados los segmentos ME y NC, la suma de las longitudes de una con otra de modo que sus diametros se encuentran en una
loscualesesmayorque 1(AB + AC),ymenorque HAB + AC). misma recta. Trazamos a estas una tangente comun e inscribirnos
una circunferencia que haga contacto con esta tangente y las dos
401. Una circunferenciade radio igual a la altura de cierto triangulo semicircunferencias dadas; luego, inscribimos una segunda
isosceles, rueda por la base de este triangulo. Demostrar que la circunferencia que haga contacto con la primera y las dos dadas;
magnitud del arco cortado en esta circunferencia por los lados a continuacion, trazamos una tercera circunferencia que haga
laterales del triangulo permanece constante. LSera justa esta contacto con la segunda y las dos dadas y asi sucesivamente
proposici6n para un triangulo no isosceles? hasta 10 infinito. Utilizando esta construccion, demostrar que,
cuando n aumenta inconmensurablemente, la suma de los
402. Demostrar que las diagonales de un cuadrilatero inscrito en quebrados
una circunferencia son entre sl como la sum a de los productos
de los lados concurrentes en los extremos de las diagonales.

403. Demostrar que la sum a de los cuadrados de las distancias tiende a la unidad, es decir, que
desde un punto cualquiera de una circunferencia hasta los vertices
,'2 + 2'3 + ... + n(~+1) = 1.
de un triangulo regular inscrito en esta circunferencia, es una
magnitud con stante, que no depende de la posici6n del punto en
la circunferencia.
56 PROBLEMAS DE MATEMATICAS ELEMENTALES

407. En el punto A, que se encuentra ala distancia a del centro 411. Hallar ellugar geometrico de los puntos para los cuales la
de una mesa redonda de biHar de radio R, hay una bola elastica, suma de las distancias hasta dos rectas dadas m y I es igual a la
cuyas dimensiones pueden ser despreciadas.lA cual punto B longitud a de un segmento dado. Examinar los casos de rectas
del borde del billar hay que dirigir la bola para que, despues de que se cruzan y paralelas.
rebotar dos veces del borde, vue Iva al punto A?
412. Hallar ellugar geometrico de los puntos, para los cuales la
408. De un punto A, dispuesto dentro de un angulo con espejos diferencia de las distancias hasta dos rectas dadas m y I es igual
como lados, sale un rayo de luz. Demostrar que el numero de a un segmento de longitud dada. Examinar los casos de rectas
reflexiones que experimenta este rayo por parte de los lados del que se cruzan y paralelas.
angulo, siempre es fmito. Hallarel nUmero de reflexiones, si el angulo
dado es igual a u, y el rayo forma can uno de los lados un angulo 413. En un plano se tienen dos segmentosABy CD. Hallar el
~. Aclarar las condiciones para las cuales este rayo pasa de nuevo lugar geometrico de los puntos M, que poseen la propiedad de
porel puntoA. que la suma de las areas de los triangulos AMB y CMD es igual
a cierta constante a2•

4. Lugar geometrico de los puntos 414. Se tienen una circunferenciaK y su cuerdaAB. Se examinan
todos los triangulos inscritos en esta circunferencia, cuya base es
409. En una circunferencia se tienen dos puntas fijosA y B Yun la cuerda dada. En cad a triangulo se toma el punto de intersecci6n
punta m6vil M En la prolongaci6n del segmentoAMfuerade la de las alturas. Hallar ellugar geometrico de estos puntos.
circunferencia se traza el segmento A1N = MB. Hallar ellugar
geometrico de los puntas N. 415. Dentro de una circunferencia dada se ha fijado un punto A
que no coincide con el centro. A traves de este punto A se ha
410. Se tienen dos rectas paralelas y un punta 0 entre ellas. A trazado una cuerda arbitraria y en susextremos, tangentes a la
traves de este punta se traza una secante arbitraria que carta a circunferencia que se cruzan en el punto M. Hallar el lugar
las rectas paralelas en los puntas A y A'. Hallar ellugar geometrico geometrico de los puntos M
de los extremos de la perpendicular a la secante, levantada del
punto A " cuya longitud es igual a OA. 416. Demostrar que ellugar geometrico de los puntos M, las
distancias desde los cuales hasta dos puntos dados A y B se
encuentran en la relaci6n
GEoMETRlAlPlanimetria 57

punto S. Demostrar que todas las rectas RS, obtenidascomo


resultado de esta construcci6n, se cruzan en un punto pertene-
es una circunferencia con centro en la rectaAB. ciente a la circunferencia dada.

Expresar el diametro de esta circunferencia por medio de la


longitud a del segmentoAB. Examinartambien el caso en que 5. Determinacion de los valores maxim os y minimos
E=l.
q
421. Se tienen dos rectas paralelas y un puntoA entre elias, que
417. Se tienen un segmento AB y un punto C sobre este. Cada se encuentra a la distancia a de una de las rectas y a la distancia
par de circunferencias, una de las cuales pas a por los puntosA y b de la otra. El punto A sirve de vert ice de los angulos rectos de
C, y la otra, por los puntos C y B, tiene, ademas de C, un punto unos triangulos rectangulos, los otros dos vertices de los cuales
comun mas D. Hallar ellugar geometrico de los puntos D. se encuentran en cada una de las rectas paralelas. l,Cual de los
triangulos tiene menor area?
418. Los lados de un polfgono deformable permanecen
respectivamente paralelos a las distancias dadas, mientras que 422. Se conoce un triangulo rectlingulo, uno de los angulos agudos
todos los vertices, excepto uno de ellos, se deslizan por las rectas del cual es igual a u. HaHar la relaci6n entre los radios de las
dadas. Hallar ellugar geometrico de las posiciones del ultimo circunferencias circunscrita e inscrita y determinar para cual valor
vertice. de u esta relaci6n sera minima.

419. Se conocen una circunferenciaK de radio r y su cuerdaAB 423. De un rectangulo de lados a y b se ha cortado un triangulo,
de longitud 2a. Supongamos que sea CD una cuerda m6vil de la cuyos catetos son iguales a a l y bl. l,C6mo debe cortarse la
misma circunferencia, de longitud constante 2b. Hallar ellugar parte restante para obtener un rectangulo de maxima area, cuyos
geometrico de los puntos de intersecci6n de las rectas AC y BD. lados sean paralelos a los lados del rectangulo dado?

420. A traves de un punto P que se encuentra sobre una 424. Sobre uno de los lados de un angulo agudo se han elegido
circunferencia dada y un punto Qperteneciente a una recta dada, dos puntos A y B. Hallar en el otro lade del angulo un punto C
se traza una circunferencia arbitraria que cruza por segunda vez tal, que el anguloACB sea el maximo. Construir el punto C con
la circunferencia dada en el punto R, y a la recta dada, en el ayuda de un com pas y una regIa.
58 PROBLEMAS DE MATEMATICAS ELEMENTALES

425. Hallar en una recta dada I un punto tal, que la diferencia de B. ESTEREOMETRIA
las distancias desde esta recta hasta dos puntos conocidosA y B
que se encuentran a un mismo lado de la recta, sea la menor, as! Observaciones preliminares
como un punto tal, que esta diferencia sea la mayor. Expongamos una serie de formulas para el dlculo de los
voilimenes y las superficies de poliedros y cuerpos de rotacion,
426. A traves de un punto A que se encuentra dentro de un suponiendo que Ves el volumen del cuerpo, S/la superficie lateral,
angulo, se ha trazado una recta que corta de este angulo un S el area de su base y H la altura.
triangulo de minima area. Demostrar que el segmento de esta
recta, comprendido entre los lados del angulo, se divide en el Piramide: V = sf.
punto A en dos mitades.
Piramidetruncada: V = ~ (SI + S2 + ~SJS2)' donde SI y S2
son las areas de las bases superior e inferior de la piramide.
427. Demostrar que de todos los triangulos con un angulo com un
<p del vertice y una suma de los lados laterales igual a a + b, el Como (circular recto): V = JtR;H.
triangulo isosceles es el de menor base.
donde R es el radio de la base del eono:
428. Entre todos los triangulos de igual base e igual angulo del
vertice, hallar el triangulo de mayor perfmetro.
donde I es la generatriz.
429. Sobre la base BC de un triangulo ABC (0 sobre su
prolongacion) se ha tornado arbitrariamente un punto D y a los Cilindro (circular recto) : V = rr.R2 H, donde R es el radio de
triangulosACD y BCD se les han circunscrito circunferencias. la base;
Demostrar que la relacion de los radios de estas circunferencias Sf =2rr.RH.
es una magnitud con stante. Hallar la posicion del punto D, para
la cualla magnitud de estos radios sea minima. COl1otruncado: V = ~- (R12 + Ri + Rj R2 ),

430. De un triangulo dado cortar dos eircunferencias iguales de donde RI YR2 son los radios de las bases del cono;
radio maximo.
GEOMETJUAJEstereometria 59

donde I es la generatriz. 433. Hallar la superficie de una piramide regular de n caras y de


volumen V, si el radio de la circunferencia inscrita en su base es
Esfera: V = 1nR3 , S = 4nR2, igual al radio de la circunferencia circunscrita ala secci6n paralela
a la base y que se encuentra a una distancia h de esta.
donde R es el rad io de la esfera.

Sector esferico : V = 21t~2h , 434. Una piramide regular pentagonal SABCDE ha sido cortada
por un plano que pasa por los vertices A y C de la base y por los
donde R es el radio de la esfera y h la altura del segmento 0 capa puntos medios de las aristas DSy ES. Hallar el area de la secci6n,
esferica correspondiente. si q es la longitud dellado de la base de la piramide y b la longitud
de la arista lateral.
Segmento esferico (casquete esferico): V = t nhz (3R - h) ,)

Sl = 2nRh, 435. Una piramide regular triangular ha sido cortada por un plano
que pasa por uno de los vertices de la base y por los puntos
donde R es el radio de la esfera y h la altura del segmento. medios de dos de sus aristas laterales. Hallar la relaci6n entre la
superficie lateral de la piramide y el area de su base, si se conoce
que el plano secante es perpendicular ala cara lateral.
1. Problemas de calculo
436. De un prisma regular cuadrangular se ha cortado, por un
431. EI volumen de un prisma triangular regular es V, el angulo plano que pasa por la diagonal de la base inferior y uno de los
entre las diagonales de dos de sus caras, trazadas desde un mismo vertices de la base superior, una piramide cuya superficie total es
vertice, es igual a u. Hallar ellado de la base del prisma. S. Hallar la superficie total del prisma, si se conoce que el angulo
del vertice del triangulo, que se obtiene en la secci6n es igual a u.
432. Desde el vertice S de una piramide regular cuadrangular se
ha bajado a la base una perpendicular SB. Del punto medio 0 437. Calcular el volumen de una piramide regular triangular si se
del segmento SB se han bajado las perpendiculares OM de sabe que el angulo plano del vertice es igual a u y el radio dela
longitud h ala arista lateral y OK de longitud b a la cara lateral. circunferencia a la cara lateral es igual a r.
Hallar el volumen de la piramide.
438. Una piramide regular cuadrangular con ellado de la base
igual a a y el angulo diedro de la base igual a 2 u , se ha cortado
60 PROBLEMAS DE MATEMAnCAS ELEMENTALES

por un plano que divide alanguio diedro de la base por la mitad. pasa por el punto de intersecci6n de las diagonales de la base, y
Hallar el area de la secci6n. que los angulos diedros contiguos a los lados paralelos de la
base son entre sl como 2: 1.
439. Sobre el cielo raso de una sala de forma de un cuadrado de
lado a, se ha construido un techo de la siguiente manera: cada 443. En el planoP setiene el anguloBAC=60°. EI puntoSdista
par de vertices adyacentes del cuadrado, que forma el cielo raso 25 cm del vertice del angulo A, 7 cm dellado AB y 20 cm del
de la sala, esta unido por medio de rectas con el punto medio del lado AC. Hallar la distancia desde el punto Shasta el plano P.
lado opuesto; con cada uno de los cuatro triangulos obtenidos,
tomados como base, se ha construido una piramide, cuyo vertice 444. En una piramide hexagonal regular, cuyo angulo plano en el
se proyecta al punto medio dellado correspondiente del cuadrado. vertice es igual a a, se ha trazado una secci6n que pasa por la
Las partes de las caras de estas cuatro piramides, dispuestas por diagonal mayor de la base y que forma con el angulo diedro un
encimade las demas, forman el techo. Hallar el volumen del desvan angulo {3. Hallar la relaci6n entre el area de la secci6n y el area
(es decir, el espacio entre el cielo raso y el techo), si la altura de de la base.
cada pin'tmide es igual a h.
445. Los tres angulos pIanos de un angulo triedro son agudos.
440. Hallar el angulo diedro entre las caras laterales de una Uno de ellos es igual a a; los angulos diedros adyacentes a este
piramide regular triangular, si el angulo diedro formado por una angulo plano son iguales a 13 y y respectivamente. Hallar los
de las caras laterales y la base es igual a a . otros dos angulos pianos.

441. En una piramide regular triangular SABC, el angulo plano 446. Calcular el volumen de una piramide regular de altura h, si
del vertice es igual a a, y la distancia mas corta entre la arista se sabe que esta piramide tiene por base un poligono, la suma de
lateral y ellado opuesto de la base es igual ad. Hallar el volumen los angulos intemos del cual es igual a n7r, y la relaci6n entre la
de esta piramide. superficie lateral y el area de la base es igual a k.

442. Una piramide tiene por base un trapecio is6sceles, cuyos 447. Se examina un cubo cuya arista es Q. Por los extremos de
lados paralelos son iguales a a y b (a > b), y los segmentos cada tres aristas que parten del vert ice comun, se ha trazado un
desiguales de las diagonales forman un angulo <po Hallar el volumen plano. Hallar el volumen del cuerpo limitado por estos pianos.
de la piramide sabiendo que su altura trazada desde el vertice
GEOMETRiAlEstereometria 61

448. Por el centro de la base de una pinimide hexagonal regular lados de la base de la piramide y el punto medio de la arista
se ha hecho una secci6n paralelamente a la cara lateral. Hallar la lateral que se cruza con dicho lado, se ha trazado una secci6n.
relaci6nentre el area de la secci6n y el area de la cara lateral. Determinar la distancia desde el vertice de la piramide hasta el
plano de esta secci6n.
449. Por cada una de las aristas de un tetraedro se ha trazado un
plano paralelo a la arista opuesta. Hallar la relaci6n entre el 454. En un tetraedro regular SABC cuya arista es igual a a se
volumen del paralelepipedo obtenido y el volumen del tetraedro. han trazado tres pIanos, cada uno de los cuales pasa por uno de
los vertices de la base del tetraedro ABC y los puntos medias de
450. Sobre las caras de una pirimide cuadrangular regular, como dos aristas laterales. Hallar el volumen de la parte del tetraedro,
bases, se han construido tetraedros regulares. Hallar la distancia dispuesta sobre todos los pianos secantes.
entre los vertices externos de dos tetraedros adyacentes, si el
lado de la base de la piramide es igual a a. 455. La piramide SABCD tiene por base un rombo con las
diagonales AC = a y BD = b. La arista lateral SA es perpendicular
451. A traves de cierto punto de la diagonal de un cubo de arista al plano de la base e igual a q. Por el punta A y el punto medio K
a se ha trazado un plano perpendicular a esta diagonal. de la arista SC se ha trazado un plano perpendicular a la diagonal
1) Aclarar cual es la figura que se obtiene en la secci6n de este BD de la base. Determinar el area de la secci6n.
plano con las caras del cubo.
2) Hallar las longitudes de los segmentos que se obtienen en la 456. En un prisma cuadrangular regular se han trazado dos
secci6n del plano con las caras del cubo, en funci6n de la distancia secciones paralelas: una de ellas pasa por los puntas medias de
x del plano secante al centro de simetria 0 del cubo. los lados contiguos de la base y por el punto medio del eje, la
otra divide al eje en la relaci6n de 1:3. Conociendo que el area
452. Se exam ina la proyecci6n de un cubo de arista a sobre un de la primera secci6n es igual as, hallar el area de la segunda.
plano perpendicular a una de las diagonales del cubo. i., Cuantas
veces el area de la proyecci6n sera mayor que el area de la 457. Una piramide triangular ha sido seccianada por un plano en
secci6n del cubo por un plano que pasa por el punto medio de la dos poliedros. Hallar la relaci6n de los volumenes de estos
diagonal y que es perpendicular a esta ultima? poliedros, si se conoce que el plana secante divide a tres aristas
laterales, que concurren en uno de los vertices de la piramide, en
453. Ellado de la base de una piramide cuadrangular regular es las proporcianes 1:2, 1:2, 2: 1, contanda desde el vertice.
igual a a, la altura de la piramide es h. A traves de uno de los
62 PROBLEMAS DE MATEMATlCAS ELEMENTALES

458. Hallar el volumen de una pinimide triangular, si el area de las aristas del cua) son iguales entre S1. Hallar )a superficie total
sus caras son So' Sl' Sl' S3' Y los angulos diedros, adyacentes a del prisma.
la cara de area So' son iguales entre Sl.
464. Los cuatro lados de un trapecio equilatero hacen contacto
459. Par los puntas medias de dos aristas paralelas de un cuba, can un cilindro cuyo eje es perpendicular a los lados paralelos
que no se encuentran en una misma cara, se ha trazado una recta, del trapecio. Hallar el angulo formado por el plano del trapeeio y
alrededor de la cual el cuba ha sido girado 90°. Determinar el el eje del cilindro, conociendo que las bases del trapecio miden
volumen de la parte comun del cubo inicial y eI girado, conociendo a y b, y su altura es igual a h.
que la arista del cuba mide a.
465. En un prisma recto que tiene par base un triangulo rectangulo
460. A traves del vertice de un cono se ha trazado un plano que se ha inscrito una esfera. En el triangulo de la base del prisma, la
forma con la base del can a un angulo a. Este plano carta ala perpendicular de longitud h, bajada desde el vertice del angulo
base por la cuerdaAB de longitud a, que une los extremos del redo a la hipotenusa, forma con uno de los catetos un angulo a.
arco de la base del cono, correspondiente al angulo central [3. Hallarel volumen del prisma.
Hallar el volumen del cono.
466. En una piramide regular de n lados con ellado de la base
461. Un cono y un cilindro tienen una base comun, y el vertice igual a a y el arista lateral b se ha inscrito una esfera. Hallar el
del con a se encuentra en el centro de la otra base del cilindro. radio de esta esfera.
Hallar el valor del angulo formado par el eje del cono y su
generatriz, si se canace que las superficies totales del cilindro y 467. En una piramide triangular regular se ha inscrito una esfera.
del cono son entre Sl como 7:4. Hallar el angulo de inclinaei6n de la cara lateral de la piramide
respecto al plano de la base, conoeiendo que la relaei6n entre el
462. En un co no se ha inscrito un cilindro cuya altura es igual al volumen de la piramide y el volumen de la esfera es igual a 27-13
41f
.
radio de la base del cono. Hallar el angulo entre el eje del cono y
su generatriz, si se sabe que la superficie total del cilindro es al 468. A una esfera de radio r se Ie ha cireunserito una piramide
area de la base del cono como 3 :2. regular de n lad os, el ungulo diedro de la base de la eual es igual
a a. Hallar la relaci6n entre el volumen de la esfera y el volumen
463. En un cono cuya generatriz I forma con el plano de la base de la piramide.
un angulo a, se ha inscrito un prisma regular de n lados, todas
GEOMETRWEstcreometria 63

469. Hallar la relaci6n entre el volumen de una piriunide regular ha circunscrito otra esfera. Hallar la relaci6n entre los volumenes
de n lados y el volumen de la esfera inscrita en esta pinimide, de las dos esferas.
conociendo que las circunferencias circunscritas a la base y a las
caras laterales de la pinimide son iguales entre sf. 475. En una esfera se ha inscrito un tetraedro regular yen este
tetraedro, una nueva esfera. Hallar la relaci6n entre las superficies
470. Hallar la altura de una piramide cuadrangular regular, si se de las dos esferas.
conoce que el volumen de la esfera circunscrita a la piramide es
V, y que la perpendicular bajada del centro de la esfera a su cara 476. En un tetraedro regular se ha inscrito una esfera. En la esfera
lateral forma con la altura de la piramide un angulo a. se ha inscrito un nuevo tetraedro regular. Hallar la relaci6n entre
los volumenes de los dos tetraedros.
471. En una piramide que tiene por base un rombo con un angulo
agudo igual a a, se ha inscrito una esfera de radio R. Las caras 477. Se tienen dos esferas concentricas de radios r y R (R>r).
laterales de la piramide forman con el plano de la base un cingulo i,Para cual relaci6n entre R y r, dentro de la esfera mayor se
\jf. Hallar el volumen de la piramide. puede construir un tetraedro regu lar de modo que tres vertices
de su base se encuentren en la esfera mayor y tres de sus caras
472. Dos piramides regulares de n lados con iguales bases se laterales hagan contacto con la esfera menor?
han unido por estas bases. Hallar el radio de la esfera inscrita
dentro del poliedro obtenido, conociendo que ellado de la base· 478. Se han tomado dos vertices opuestos de un cuba y por los
comun de las piramides es igual a a y las alturas de estas ultimas puntos medios de seis aristas que no pasan por estos vertices se
son hy H. ha trazado un plano secante que divide al cuba en dos partes. En
cada una de estas partes se ha colocado una esfera que hace
473. Dos piramides regulares de n lados con iguales bases, pero contacto con tres caras del cubo y el plano secante. LCuantas
de diferentes alturas, han sido unidas por estas bases, y al poliedro veces el volumen de cada esfera sera menor que el volumen del
obtenido se Ie ha circunscrito una esfera de radio R. Hallar las cubo?
alturas de las piramides, conociendo que ellado de la base es
igual a a. i,Para cuat relaci6n entre a y R es soluble el problema? 479. Desde un punto de una esfera de radio R se han trazado
tres cuerdas iguales que forman un angulo a una con la otra.
474. En un prisma regular de n lados se ha inscrito una esfera Detemlinar la longitud de estas cuerdas.
que hace contacto con todas las caras del prisma. AI prisma se Ie
64 PROBLEMAS DE MATEMATICAS ELEMENTALES

480. En una pirfunide triangular SABC las aristas SA, SC y SB son 486. La razonentre la altura de un co no y el radio de la esfera
de dos en dos perpendiculares: AB = BC = a, BS = h. Hallar el circunscrita a este es igual a q. Hallar la relacion de los vohlmenes
radio de la esfera inscrita en la pirfunide. de estos cuerpos. (,Para cuales valores de q es soluble el
problema?
481. Hallar el angulo diedro <p entre la base y la cara lateral de
una pirfunide cuadrangular regular, conociendo que el radio de la 487. Hallar la razon del volumen de una esfera al volumen del
esfera circunscrita a la piramide es tres veces mayor que el radio co no recto circunscrito a esta esfera, si la superficie total del
de la esfera inscrita en la misma. cono es n veces mayor que la superficie de la esfera.

482. En una esfera de radio R se ha inscrito un tetraedro regular, 488. Calcular los radios de las bases de un cono truncado
todas las caras del cual han sido prolongadas hasta su interseccion circunscrito a una esferade radio R, conaciendo que la raz6n de
con la esfera. Las !ineas de interseccion de las caras del tetraedro la superficie total del cono truncado a la superficie de la esfera es
con la esfera cortan de su superficie cuatro triangulos esfericos y igual am.
varias figuras esfericas biangulares. Hallar la superficie de cada una
de estas figuras obtenidas. 489. En un cono se ha inscrito una esfera de radio r. Hallar el
volumen del cono, conociendo que eI plano que hace contacto
483. En un cono se ha inscrito una esfera. La superficie de la con la esfera y que es perpendicular a una de las generatrices del
esfera es a la superficie del cono como 4:3. Hallar el angulo del cono se encuentra ala distancia d del vert ice del cono.
vertice del cono.
490. En un cono, en el cual el angulo de la secci6n axial del
484. En un cono se ha inscrito una semiesfera, el circulo mayor vertice es igual a a, se ha inscrito una esfera de radio R. Hallar
de la cual descansa sobre la base del cono. Determinar el angulo el volumen de la parte del cono dispuesta por encima de la esfera.
del vert ice del cono, conociendo que la superficie del cono y la
superficie de la semiesfera son entre SI como 18:5. 491. Determinar los radios de dos esferas, que al cruzarse forman
una lente biconvexa, si se conace que el espesor de la lente es igual
485. En una esfera de radio R se ha inscrito un cono cuya superficie a 2a, su superficie total es Sy su diametro 2R.
lateral es k veces mayor que el area de su base. Hallar el volumen
del cono. 492. En un cono se ha inscrito una esfera. La razon de los
volumenes de estas dos figuras es k. Hallar la relaci6n entre los
GEOMETRiAlEstereometria 65

volumenes de los segmentos esfericos cortados de la esfera por area de la secci6n paralela a las secciones dadas y que divide
un plano que pas a por la linea de contacto de la esfera con el por la mitad la distancia entre estas ultimas.
cono.
497. Sobre un plano P descansan tres esferas iguales de radio R
493. En una esfera S de rad io R se han inscrito ocho esferas de que hacen contacto una con otra. Un cono circular recto esta
menor radio, cada una de las cuales hace contacto con las dos dispuesto de tal manera que el plano de su base coincide con P,
vecinas, y todas juntas tienen contacto con la esfera S por la y las esferas dadas hacen contacto con el cono y se encuentran
circunferencia de mayor diametro. Luego, en el espacio entre las fuera de el. Hallar el radio de la base del cono si su altura es igual
esferas se ha inscrito otra esfera S] que hace contacto con las aqR.
ocho esferas de menor radio y con la esfera S. Hallar el radio
p de esta ultima esfera. 498. Se tienen cuatro esferas iguales de radio R, cada una de las
cuales hace contacto con las otras tres. Una quinta esfera tiene
494. En una esfera S de radio R se han inscrito ocho esferas contacto exterior con cada una de las esferas dadas, y una sexta,
iguales, cada una de las cuales hace contacto con tres esferas contacto interior. Hallar la relaci6n entre el volumen de la sextaesfera
vecinas y la esfera S. Hallar el radio de las esferas inscritas, V6 y el volumen de la quinta VS.
conociendo que sus centros se encuentran en los vertices de un
cubo. 499. En un plano se encuentran tres esferas iguales de radio R,
cada una de las cuales hace contacto con otra de elias. Una
495. En una csfera se han inscrito dos conos iguales, cuyos ejes cuarta esfera hace contacto con cada una de las tres esferas
coinciden, y los vertices de los cuales se encuentran en los extremos dadas y con el plano. Hallar el radio de la cuarta esfera.
opuestos del diametro de la esfera. Hallar la relaci6n entre el
volumen de la parte comun de estos dos conos y el volumen de la 500. Sobre un plano reposan cuatro esferas iguales de radio R.
esfera, conociendo que la raz6n de la altura h del cono al radio R Tres de ellas hacen contacto entre sf de dos en dos, y la cuarta
de la esfera es igual a k. tiene contacto con dos de estas tres. Sobre estas esferas se
colocaron dos esferas iguales de menor diametro que hacen
496. Las areas de dos secciones paraleias de una esfera, contacto una con la otra y con tres de las esferas dadas. HaIlar la
dispuestas a un mismo lado respecto a su centro, son iguales as] relaci6n entre los radios de las esferas grande y pequefia.
y S2. La distancia entre estas secciones es iguai ad. Hallar ei
66 PROBLEMAS DE MATEMATICAS ELEMENTALES

2. Problemas de demostracion Demostrar que

501. Se tiene un conn truncado cuya superficie lateral es igual al


area de un circulo que tiene por radio a la generatriz del cono
506. Se examinan dos triangulosABC y A IBI C I que se encuentran
truncado. Demostrar que en el cono dado se puede inscribir una en pIanos no paralelos y cuyos lados son de dos en dos no
esfera. paralelos. Al mismo tiempo, las rectas que unen los vertices
correspondientes se intersectan en un punto O. Demostrar que
502. Se tiene un cono truncado cuya altura es media proporcional las prolongaciones de los lados correspondientes de los triangulos
entre los diametros de las bases. Demostrar que en el cono se
se cruzan de dos en dos y que los puntos de su intersecci6n se
puede inscribir una esfera.
encuentran en una misma recta.

503. Demostrar que al unir tres vertices de un tetraedro regular 507. Demostrar que los segmentos que unen los vertices de cierta
con el punto medio de la altura bajada desde el cuarto vertice, se piramide triangular con los centros de gravedad de las caras
obtienen tres rectas perpendiculares entre Sl de dos en dos. opuestas se cruzan en un punto y se dividen por este punto en la
relaci6n 1:3.
504. Sea R el radio de una esfera circunscrita a una piramide
cuadrangular regulary r, el radio de una esfera inscrita en esta 508. Demostrar que el area de cualquier secci6n triangular de
piramide. Demostrar que una piramide triangular arbitraria no es mayor que el area de una
j{ ? -J2 + 1. de sus caras.
I"

Indicacion. Expresar ~ en funci6n de tg ~,donde a es el 509. Una de las dos piramides triangulares con base comun, se
angulo diedro entre la base y la cara lateral. encuentra dentro de la otra. Demostrar que la sum a de los angulos
pIanos del vertice de la piramide interior es mayor que la suma
505. Desde el punto 0 de la base ABC de una piramide triangular de los angulos pIanos de la piramide exterior.
SABC, se han trazado las rectas OA " OB' Y OC', paralelas
respectivamente a las aristas SA, SB y SC, hasta su intersecci6n 510. Cuatro esferas cuyos centros no se encuentran en lin plano,
con las caras SBC, SCA YSAB respectivamente en los puntos hacen contacto entre si de dos en dos. Cada dos de estas esferas
A',B'yC'. deterrninan un plano perpendicular a SllS !ineas de centros y que
GEOMETRiAlEstereometria 67

tiene contacto con ambas esferas. Demostrar que los seis pianos 516. Demostrar que todo plano que pasar por los puntos medios
que se obtienen de tal modo tienen un punto eomun. de dos aristas opuestas de un tetraedro, divide a este en dos
partes equidimensionales.
511. Demostrar que si en una piramide triangular la suma de las
longitudes de cualquier par de aristas opuestas es la misma, los 517. Demostrar que si todos los angulos diedros de eierta
vertices de esta piramide son los centros de cuatro esferas que piramide triangular son iguales, entonces todas las aristas de esta
hacen contacto entre sf de dos en dos. piramide son tam bien iguales.

512. i.,A cual condici6n deberan satisfacer los radios de tres esferas 518. Sobre dos pianos paralelos se encuentran los segmentos
que tienen contacto entre sf de dos en dos, para que a estas AB y CD. Los extremos de estos segmentos son los vertices de
esferas se las pueda trazar un plano tangente com un? cierta piramide triangular. Demostrar que el volumen de la
pirimide no varia si estos segmentos se desplazan por estos pianos
513. Demostrar que sf un punto se desplaza por el plano de la paralelamente a sf mismos.
base de una piramide regular, permaneciendo dentro de esta base,
la suma de las distancias de este punto hasta las caras laterales 519. Demostrar que la recta que corta las dos caras de un angulo
es constante. diedro forma con ellas angulos iguales solamente cuando los
puntos de intersecci6n equidistan de la arista.
514. Demostrar que dos pianos trazados por los extremos de
las tres aristas de un paralelepfpedo, que parten de los extremos 520. En el espacio se examinan dos segmentosAB y CD que no
de la diagonal de este ultimo, cortan a esta diagonal en tres partes se encuentran en un plano. Supongamos que sea A1N el segmento
iguales. que une sus puntos medios. Demostrar que
AD+BC >MN
515. Demostrar que si un plano trazado por los extremos de tres 2
aristas de un paralelepfpedo, que parten de un mismo vertice,
(aqufAD, BCy A1N son las longitudes de los segmentos corres-
corta del paralelepfpedo un tetraedro regular, entonces el
pondientes).
paralelepipedo puede ser cortado por un plano de modo que en
la secci6n se obtenga un hexagono regular.
68 PROBLEMAS DE MATEMAnCAS ELEMENTALES

521. Demostrar que cualquier angulo plano de un cingulo tetraedro 528. Hallar ellugar geometrico de los pies de las perpendiculares
arbitrario es menor que la suma de los otros tres angulos pianos. bajadas desde un punto dado del espacio a las rectas que se
encuentran en un plano dado y que se cruzan en un punto.
522. Demostrar que cualquier angulo tetraedro convexo puede
ser cortado por un plano de manera tal, que en la seccion se 529. Se tienen un plano P y dos puntos A y B fuera de este
obtenga un paralelogramo. plano. A traves de A y B se trazan todas las esferas posibles que
hagan contacto con el plano P. Hallar ellugar geometrico de los
523. Demostrar que si en una piramide triangulartodas las caras puntos de contacto.
son equidimensionales. entonces estas son iguales.
530. Un plano corta al angulo triedrico par el triangulo ABC.
Hallar el lugar geometrico de los centros de gravedad de los
3. Lugar geometrico de los puntos triangulos ABC con la condicion de que:

a) los vertices A y B son fijos;


524. Hallar ellugar geometrico de las proyecciones de un punto
b) el vertice A es fijo .
dado en el espacio sobre un plano que pasa por otro punto dado.

525. Hallar ellugar geometrico de los centros de las secciones


de una esfera por pianos que pasan por una recta dada l. Examinar 4. Valores maximos y minimos
los casos en que la recta cOrta ala esfera, es tangente a ella 0 no
tiene con ella puntos comunes. 531. Un cuba se corta par un plano que pasa par una de sus
diagonales. l,COIllO debera ser trazado este plano para que el
526. Hallar ellugar geollletrico de los centros de las secciones area de la seccion sea minima?
de una esfera por pianos qu~~ pasan por un punto dado C.
Examinar los casos en que el punto C se encuentra fuera de la 532. En una piramide triangular se hacen secciones paralelas a
esfera, en su superficie 0 dentro de ella. dos de sus aristas que no se cruzan. Hallar la seccion de mayor
area.
527. Hallar ellugar geometrico de los puntos desde los cuales se
pueden trazar ala esfera dada de radio R, tres tangentes que
formen con tres angulos pianos rectos un angulo tetraedrico.
TRlGONOMETRiA 69

x+v x-y
TRIGONOMETRIA cosx + cosy = 2cos 2 cosT' (11 )

x+y x-y
cosx -cos y = 2sen-2- sen T ' (12)

Observaciones preliminares 4. Multiplicacion de funciones trigonometricas


Expongamos algunas de las formulas que se encuentran en los
problemas propuestos a continuacion. 1
sen x sen y = icos(x - y) - cos(x + y)i '
(13)

1. Funciones trigonometricas de la suma y la diferencia de cosxcosy=ticos(x- y) + cos(x + y)i , (14)


dos angulos:
senxcosy = tlsen(x- y)+sen(x+ y)i, (15)
sen (x + y) = sen x cos y + cos x sen y. (1) ~
sen- x
l-cos2x
= - 2- ' (16)
sen(x - y) = sen xcos y - cos xsen y. (2) ~ l - cos 2x
(3) cos- X = - 2- ' (17)
cos(x + y) = cos x cos y - sen x sen y.
cos(x - y) = cos x cos y + sen ~ sen y. (4)
5. Expresion de sen x, cos x Y tg x en funcion de tg i .
2. Angulos doble y triple:
2tg ~
(5) senx =- -;- , (18)
sen 2x = 2sen x cos x, l+ tg 1
~ 2
cos 2x = cos~ x - sen x, (6) l - tg 2 ~
(7) cos X = 2 , (19)
sen 3x = 3sen x - 4sen 3 x, - -0-

l+tg"t
cos3x = 4cos 3 x-3cosx. (8) 2tg~
tgx =--;-. (20)
l-tg 1
3. Suma y diferencia de funciones trigonometricas:
x+~ x-y 6. Funciones trigonometricas inversas.
sen x + sen y = 2sen 2 cos-2-, (9)
x+y x-y
sen x - seny = 2cosTsen - 2- ' (10) a) Valores principales de las funciones trigonometricas inversas:
70 PROBLEMAS DE MATEMAnCAS ELEMENTALES

y = arcsen X , si X = sen y y - ~ :<: ; y :<: ; ~ , (21) 535. Demostrar que para todos los valores admisibles de x es
Y = arccos X, si X = cos y y 0:<::; y :<: ; n, (22) valida la f6rmula
y=arctgx, si x=tgy y -~<y<~, (23) tg x + tg2x - tg3x = tg xtg 2x tg 3x.
Y = arccotgx, si X = cotg y y 0< y < n. (24)
536. Demostrar que para todos los valores admisibles de xes
b) Funciones trigonometricas inversas de valuaci6n multiple: justa la igualdad

Arcsen x = (-lr arcsen x + nn, n = 0, ± 1, ± 2, ... (25) tg 3x = tg x tg(~ - x )tg(~ + x).


Arccosx = ± arccos x - 2nn, (26)
537. Demostrar la identidad
Arctg x = arctg x + nn, (27)
Arccotg x = arccotg x + nn. (28)
sen a + sen ~ + sen y - sen(a + ~ + y) =

Las f6rmulas (25}-{28) determinan el aspecto general de los = 4 sen a;~ sen ~ ;1 sen y;n .
angulos correspondientes al valor dado de la funci6n
trigonometrica. 538. Demostrar que si a + ~ + y = n, entonces

sen a + sen ~ + sen y = 4cos~ cos% cos~.


1. Transformaci6n de las expresiones que contienen 539. Demostrar que para un valor entero de n y a + ~ + y = J[ se
funciones trigonometricas cumple la identidad
sen 2na + sen 2nfJ + sen 2n y = (- 1)yn+14 sen na sen nfJ sen ny.
533. Demostrar la identidad

sen 6 X + cos 6 X =I- i sen 2 2x.


540. Demostrar que si cos( a + ~ )=0, entonces

534. Demostrar que sen(a + 2~) = sen a.

541. Demostrar que si 3 sen ~ =sen(2 a + ~), entonces


cos 2 a + cos 2 (a + ~)- 2 cosa cos ~cos(a +~) = sen 2 ~.
tg(a+~)=2tga
J'RIGONOMETRiA 71

para todos los valores admisibles de a y 13 . 546. Sea x + y + Z = ~ k.l,Para cuales valores enteros de k la
suma
542. Demostrar que si sen a = A sen (a +(3), tg Y tg z + tg z tg x + tg x tg Y
n.) sen p
tg ( a+f-' = cosP-A no depended de x, y y z?

para todos los valores admisibles de a y 13.


547. Hallar las relaciones algebraicas entre los angulos a, 13 y
y, si se conoce que
543. Demostrar que si los angulos a y 13 estan enlazados en la
relaci6n
tg a + tg 13 + tg Y = tg a tg 13 tg y.
senp ~
sen(2ap) m 548. Transfomlar en una multiplicaci6n la expresi6n
entonces se cumple la igualdad
cotg 2 2x-tg 2 x-8cos4xcotg4x.

l+~ 549. Transformar en una multiplicaci6n la expresi6n


~ I-tga·tgp
m-n
m+n
sen 2 a + sen 2 13 + sen 2 y + 2 cos a cos 13cosy - 2.
544. Demostrar que si cos x . cos y . cos Z 7= 0, entonces es v31ida
laf6rmula 550. Calcular, sin emplear las tablas, la expresi6n

cos(x+y+z)=
= cosxcos ycosz(l- tgxtgy - tgytgz - tg ztgx). 551. Demostrar que
cos'S -cos s = 2'
7l 271 I

545. Demostrar que si a, 13, y son los {mgulos de un triangulo,


entonces se cum pie la igualdad 552. Demostrar que

tg ~2 + tg2
tg [l2 1'. +2
[l tg2 tg ~
tg .I 2 = 1. cos ~71
7
+ cos .'!~
7
+ cos.£7t
7
= 21. .
72 PROBLEMAS DE MATEMATICAS ELEMENTALES

553. CaJcu\ar, sin hacer uso de las tablas, la expresi6n 560. Resolver la ecuaci6n
4 1t 4 3lt 4 5lt 4 7lt
sen T6 + sen T6 + sen ]"6 + sen ]"6. 2sen 17x +.J3 cos5x + sen 5x = O.
554. Demostrar que 561. Resolver la ecuaci6n

tg 20° tg 40° tg 80° = .J3. sen 2 x( tg x + I) = 3 sen x(cos x - sen x) + 3.

562. Resolver la ecuaci6n


2. Ecuaciones trigonometricas y sistemas de
3 '
ecuaciones sen x + cos) x = 1- ~ sen 2x.
563. Resolver la ecuaci6n
A. ECUACIONES TRIGONOMETRIC AS

555. Resolver la ecuaci6n


_1,- _ _ 12- _ _ 1__ _ 1_, _ _ _ 1 ____ 1_ _ = -3.
sen" x cos x tg 2 x colg - x sec 2 x cosec 2 x

sen 3 X cos X
556. Resolver laecuaci6n
- sen x COS 3 X = *. 564. Resolver la ecuaci6n

~gx = I + sen 2x.


1+lgx
565. Resolver la ecuaci6n
557. Resolver la ecuaci6n
~ (sen 4 x + cos 4 x) = sen 2 x cos 2 X + sen x cos x.
1 + senx + cos x + sen2x + cos 2x = O.
566. Resolver laecuaci6n
558. Resolver 1a ecuaci6n
(1 + k) cos x cos(2x - a) = (I + k cos 2x) cos(x - a).
1 + sen x + cos 3x = cos x + sen 2x + cos 2x.
567. Resolver laecuaci6n
559. Resolver la ecuaci6n
sen ax sen bx = sen ex sen dx,
(sen 2x + ,,/3 cos2x») - 5 = cos (~- 2x).
TRlGONOMETRiA 73

donde a, b, c y d son los terminos positivos sucesivos de una sen 2x - 12( sen x - cos x) + 12 = O.
progresi6n aritmetica.
575. Resolver la ecuaci6n
568. Resolver la ecuaci6n sec 2 ~
1+ 2 cosec x = -T'
2 + cos x = 2 tg 1.
576. Resolver la ecuaci6n
569. Resolver la ecuaci6n
cotg 2 x = l+senx.
cotg x - 2 sen 2x = 1. l+cosx

577. Resolver la ecuaci6n


570. Hallar tg x de la ecuaci6n

2 cos x cos(f3 - x) = cos 13.


2 tg3x - 3 tg2x = tg 2 2x tg 3x.
578. Resolver la ecuaci6n
571. Hallar cos <jJ , si
2 cotg 2x - 3 cotg 3x = tg 2x.
sen a + sene <jJ - a) + sen(2<jJ + a) = sene <jJ + a) + sen(2<jJ - a)
y el angulo <jJ se encuentra en el tercer cuadrante. 579. Resolver la ecuaci6n

572. Hallar cotgx de la ecuaci6n 6 tg x + 5 cotg 3x = tg 2x.

cos 2 (a + x) + cos 2 (a - x) = a, 580. Resolver la ecuaci6n

donde 0 < a < 2. Analizar para cuMes valores de a tiene soluci6n sen 5 X - cos 6 X = ~I~ - ~I- .
cosx senx
el problema.
581. Resolver la ecuaci6n
573. Hallar el valor de tg t, si sen a + cos a = ;; y el angulo
a se encuentra en los Ifmites entre 0 y 45°. tg (x - B.)tg x tg (x + B.) = 4cos 2 x .
4 4 toE-cotgE
"2 2

574. Resolver la ecuaci6n 582. LPara cuales valores de a tiene soluci6n la ecuaci6n
sen 2 x-senxcosx-2cos 2 x=a?
74 PROBLEMAS DE MATEMATICAS ELEMENTALES

Hallar esta soluci6n. 590. Resolver laecuaci6n

583. Hallar todos los valores de a, para los cuales es soluble la 8sen 6 x + 3cos2x + 2cos4x + 1 = O.
ecuaci6n 591. Resolver la ecuaci6n
sen 4 x - 2 cos-') x + a 2 = 0.
cos 3x cos 3 x + sen -,x
,.,
sen 3 x = 0.
584. Resolver la ecuaci6n
592. Resolver la ecuaci6n
x
cos n..£
31
cos 2rr..£
31
cos 4rr 3xl cos 8n :1
J
cos 16rr 3I = 321 . 8 8 17
sen x + cos x = 32 .
585. Resolver la ecuaci6n
593. Resolver laecuaci6n
cos 7x - sen 5x = -J3(cos 5x - sen 7x).
sen 10 x + cos 10 x 29
= 16 cos 4 2 x.
586. Resolver la ecuaci6n
594. Resolver la ecuaci6n
2 - (7 + sen 2x)sen 2, x + (7 + sen 2x)sen4 x = O.
" )3 .
sen 3 x + sen 3 2x + sen 3 3x - (sen x + sen 2x + sen -'x
587. Hallar sen x y cos x, SI
595. Resolver la ecuaci6n
a cos x + b sen x = c.
- x + cos 2n x = 1,
sen ')n
. Para cua!
G condici6n respecto a, bye es soluble el problema?
donde n es un numero entero positivo.
588. Resolver la ecuaci6n
596. Resolver la ecuaci6n
asenx +b = acosx+h.(a 2 ":2b 2 ).
h cos x+a b sen x+a
senC~ + 3;)=2sen(~Z -1)·
589. Resolver laecuaci6n
597. Resolver la ecuaci6n
32 cos 6 x - cos 6x = 1.
TRlGONOMETRiA 75
sen x = cosec x + sen y,
(cos4x-cos2x)2 =sen3x+5. cosx = sec x + cos y.
598. Resolver la ecuaci6n . 603. Resolver el sistema de ecuaciones

(sen x + cos x}fi = tg x + cotg x. sen 3 x = 1sen y,l


599. Demostrar que la ecuaci6n cos 3 x = -lL cos y.1
)

604. Resolver el sistema de ecuaciones


(sen x + -fj cosx )sen 4x = 2

no tiene soluci6n. tg x + tg Y = I,}


cos x cos y = --k .
-v2
600. Determinar en que limites se puede variar el parametro A
para que la ecuaci6n
605. Resolver el sistema de ecuaciones
sec x + cosec x = A sen x sen y = ~
4-.,)2 '}

tenga una raiz x que satisfaga ala desigualdad 0 < x < ~. tgxtgy - 3'
_I

606. Resolver el sistema de ecuaciones


B. SISTEMAS TRIGONOMETRlCOS
x+ y = <p,}
601. Hallar todas las soluciones del sistema de ecuaciones
cos x cos y = a.
sen(x + y) = 0,[
sen(x-y)=O.r GPara cwiles valores de a es soluble el sistema?
J

que satisfagan las condiciones: 0 :::; x :::; n, 0 :::; y :::; n. 607. Hallar todos los valores de a, para los cuales es soluble el
sistema de ecuaciones
602. Resolver el sistema de ecuaciones
sen xcos2y = a 2 + I,}
cosxsen 2y = a.
76 PROBLEMAS DE MATEMAnCAS ELEMENTALES

GPara cuales valores de a son posibles las soluciones halladas?


Resolver este sistema.
612. Resolver el sistema de ecuaciones
608. Resolver el sistema de ecuaciones
tgx + _L == 2sen(y + ~4),1
cos(x - 2y) = acos 8 y,l tgx r
tgy+ t;y ==2sen(x-1)'J
sen(x-2y)=acos 3 yJ

613. Eliminar x e y del sistema de ecuaciones


GPara cuales val ores de a es soluble el sistema?
asen 2 x+bcos 2 X=I'l
609. Hallar cos(x + y), si x e y satisfacen al sistema de ecuaciones
a cos 2 y + b sen 2 y = I,
sen x+ sen y = a,} atgx = btgy'J
cos x + cos y =b
y admitiendo que el sistema es soluble y que a*- b.

614. Expresar cos a y sen ~ en funci6n de A y B con la condici6n


610. GPara cuales valores de a es soluble el sistema de de que
sena == A sen (J, == B tg (J.
ecuaClOnes
x- y = a,l tg a

2(cos 2x + cos 2y) = 1 + 4 cos 2 (x - y)? j 615. Resolver el sistema de ecuaciones

Hallar las soluciones de este sistema.


= tg 3
tgx y,}
sen x = cos 2y.
611. Hallar todas las soluciones del sistema 616. Resolverel sistema de ecuaciones
8 cosx cos ycos(x - y) + 1 = 0,1 sen x + sen y = sen(x + y),
x + y = a.) Ixi + Ivi = I.
i I '" I
TRIGONOMETRiA 77

617. Resolver el sistema de ecuaciones 623. Demostrar que para a < 1/32, la ecuaci6n
3 3
sen(y -3x) = 2sen 3 x,l (arcsen x) + (arccosxf
0

= an
cos(y - 3x) = 2 cos 3 x.J no tiene ralces.

618. Aclarar a cwiles condiciones debenin satisfacer los numeros 624. Demostrar la f6nnula
a, bye, para que el sistema de ecuaciones
arccos x = {arcsen~l-x2, siO~x~l;
sen x + sen y = 2a,l rr-arcsen~l-x2, si -1~x~O.

cos x + cos y = 2b, ~ 625. Demostrar las f6rmulas


tgxtgy = c J
arcsen( -x) = - arcsen x, arccos( - x) = n - arccos x.
tenga por 10 menos una soluci6n.
626. Demostrar que si - ~ + 21m ::::: x ~~ + 21m, entonces
arcsen( sen x) = x - 21m.

3. Funciones trigonornetricas inversas 627. Demostrar que si 0 < x < 1 y

619. Calcular el arccos {sen(- ~)J . a = 2arctg II~x,


x
f3 = arcsen l+x
I-X; , entonces a + f3 = 1[,

620. Calcular el arcsen (cos 3; n) . 628. Hallar la relaci6n entre


621. Demostrar que
arcsen cos arcsen x y arccos sen arccos x.
arctg 13 + arctg 15 + arctg 17 + arctg 81 = 1!4'

622. Demostrar la f6nnula 4. DesiguaJdades trigonornetricas

arcsen x + arccos x =~. 629. Resolver la desigualdad sen x > cos 2 X.


78 PROBLEMAS DE MATEMATICAS ELEMENTALES

637. Demostrar la validez de la desigualdad


630. LPara cmlles valores dex se cumple la desigualdad
(I - tg 2 x Xl - 3 tg 2 x Xl- tg 2x tg 3x) > 0
, ?
4sen~ x+3tgx-2sec- x>O?
para todos los valores de x, para los cuales la parte izquierda
tiene sentido.
631. Resolver la desigualdad sen x sen 2x < sen 3x sen 4x, si
O<x < n/2.
638. Demostrar la validez de la desigualdad
632. Resolver la desigualdad {cotg 2 x-I )(3cotg 2 x -1 )(cotg3x tg 2x -1):<::: -1

sen 2 x- -I para todos los valores de x, para los cuales la parte izquierda
4 >0
Jj - (sen x + cos x) . tiene sentido.

633. Hallar todos los valores de x, mayores de cero, pero 639. Suponiendo que sea tg e = ntg <p (n > 0), demostrar que
menores de 2 7t, para los cuales se cumple la desigualdad

cos x - sen x - cos 2x > O.


640. Demostrar la desigualdad
634. Resolver la desigualdad
senx-J +1 > 2-senx
t c!. > ~gx-2 senx- 2 2 - 3-senx
g2 tgx+2'
LPara cu<iles valores de x esta desigualdad se transforma en
635. Resolver la desigualdad igualdad?
cos 3 X cos 3x - sen 3 x sen 3x > i.
641. Demostrar que para 0 :<: : <p :<: : n 12 se cumple la desigual-
636. Demostrar para 0 < <p < n 12 la desigualdad dad
cos sen cp > sen cos cp.
cotg ~ > 1 + cotg <po
TRlGONOMETRiA 79

642. Supongamos que sea nun numero entero positivo, mayor 647. Demostrar que si tg a = ~ y sen ~ = Jo '
entonces
que 1, Yque el angulo a satisface a la desigualdad a + 2~ = 45° (a y ~ son angulos del primer cuadrante).

648. Demostrar que


Demostrar, empleando el metoda de induccion completa, que senx+tgx
Y == cosx+cotgx
entonces
tgna > n tga. adquiere valores positivos para todos los valores admisibles
de x.
643. Supongamos que sea 0 < a 1 < a 2 < ... < an < n12.
Demostrar que entonces 649. Demostrar que la igualdad
tga <senal+··+sena n t
1 cos a 1+. .+COSU n < g an' sen a sen 2a sen 3a =1
644. Demostrar que si A, Bye son los angulos de un triangulo, no es valida para ninguno de los val ores de a.
entonces
C < 1
sen 2A senB
2 sen 2 - 8" . 650. Expresar sen 5x en funcion de sen x, y con ayuda de la
formula obtenida calcular, sin hacer uso de las tab las, el valor de
645. Demostrar que para 0 <x < n 14 es justa la desigualdad
sen 36°.
cosx 8
1 > . 651. Hallar los valores maximo y minimo de la funcion
sen - x( cos x - sen x)

5. Problemas diferentes 652. Hallar los valores maximo y minimo de la funcion

646. Calcular la funcion sen (2 arctg i - arctg 1;) . y = 2 sen 2x + 4 cos 2x + 6 sen x cos x.

653. (,Para cuales valores enteros de n la funcion

cosnxsen 1.n x
80 PROBLEMAS DE MATEMATICAS ELEMENTALES

tiene unperfodo igual a 3 1t ')? Indicacion. Emplear la formula de Moivre.

654. Demostrarque si la suma 658. Se exam ina la funcion

I(x) = Acosx + Bsen x,

parax=O y x = XI 7= 1m (kes un numero entera) seconvierte en donde A y B son ciertas constantes.


cera, entonces esta sum a es igual a cera paracualquiervalor dex.
Demostrar que si j(x) se hace igual a cera para dos valores
655. Demostrar que la funcion cos -E no es periodica (es decir, del argumento x I y x 2 tales, que
que no existe ningun numero constante T 7= 0, que para todos
los valores de X sea cos ~-; + T = cos -E ).
(k es un numero entero), entoncesj(x)es por identidad igual a
656. Demostrar la formula
cera.
sen nx sen (n+l)x
sen x + sen 2x + ... + sen nx = 2 2
sen"-2

Indicacion. Se puede emplear la formula de Moivre

(cos x + i sen x y = cos nx + i sen nx.


657. Calcu lar la suma
cos E cos 2lt cos nlt
444
-- + --~-+ ... +-- -.
2 2- 2n

*) La funci6nf(x) se llama peri6dica, si existe un numero T 7= 0 tal, que


para todos los val ores admisibles de x se cumple la igualdadf(x + 1) =f
(x). En este caso, el numero Tse llama perfodo de la funci6n.
ALGEBRAIResoluciones y Soluciones 81

ALGEBRA Asi pues, AI = A z' 10 que se exigia demostrar.


2. Si la diferencia d de la progresion dada es igual acero, la
validez de la formula es evidente. Por esta razon consideraremos
que d:j:. 0.
Designemos el miembro izquierdo de la igualdad supuesta por
RESOLUCIONES Y SOLUCIONES S. Liberando los denominadores de la irracionalidad,
obtendremos:
1. Progresiones aritmetica y geometrica

1. Segun la condicion del problema

b- a = c - b = dye - a = 2d. Puestoque segtin lacondicion del problemaa2-G 1 = =a3-G2=


=... = an-an- I = d, entonces, es evidente que
Supongamos que sea
A _ _1_ _ _1_
1- -Fc+.Ja Jb+-Fc
y Finalmente tendremos
A~
...
= ,-a +'\jh
--V
1 " _ ,- 1 I .
"";c+'Va

Demostremos que AI = A 2 • Si d = 0, entonces a = b = c YAI


= Ao = 0. Por esta razon consideraremos que d:j:. 0.
Libenindonos de la irracionalidad en los denominadores 10 que se exigia demostrar.
obtenemos:
A - -Fc--/i ~--Fc _ 2~--Fc-.Ja
3. Segun la condicion del problema
1- 2d + d - 2d
y
A - Jb-~ _ -Fc-~ _ 2Jb--Fc-.Ja
2 - d 2d - 2d
Si d = 0, entonces la iguaJdad propuesta es evidente.
°,
Considerando que d :j:. tendremos:
82 PROBLEMAS DE MATEMATlCAS ELEMENTALES

Reduciendo a un comun denominadory haciendo las simplifica-


an -al n-I ciones correspondientes, tendremos:
dalan alan

a 1 -an_1 =(n-2).(an_I -aJ,


10 que se exigia demostrar.
al - an_1 = (n - 2)· (a n- 2 - an-I).
4. Siendo n = 3 tenemos que ,,~) + (/ola = a I~ 3 . De aquf que
I I I I I -. ." 3 Por consiguiente, an_1- an = an- 2 - an_I, 10 que se exigia
al"2 - -':;1"3 = Ypor conslglllente, a3 - a 2 = a 2 - a l·
ala3 - "3a2
demostrar.
Por esta raz6n, es suficiente demostrar que para cualquier n ;::: 4
5. L1evaremos a cabo la demostraci6n por el metoda de induccion.
Senalemos que para n = 2 la igualdad tiene lugar, puesto que
Escribamos sucesivamente la igualdad indicada en la condici6n a 2 - a] = a3 - a 2 Y por consiguiente, a l - 2a 2 + a3 = o.
del problema, para los casos n - 2, n - 1 y n: Supongamos que la formula propuesta es valida para cierto valor
de n; con otras palabras, cualquiera que sea la progresion
(1) aritm et::icax l , x 2' •• • , xn+ I ' sera val ida la igualdad
(2)

(3)

Restando miembro a miembro de la igualdad (3) la igualdad


(2) y de la (2) la igualdad (I), obtenemos: Pasando a n + 1, hacemos uso de la identidad
ALGEBRA/Resoluciones y Soluciones 83

a]2 - (n +1 1) a2
2 + (n 2+ 1) a2 + ... + ( (n n+ 1) an2++
Entonces,
3 -1) n
l
a l + (n+l)
I a + (n+l)
2 2 a, + ...

+ (_l)n+l(:: ~)a~+2 = [a~ -(~ )ai + ... + (_l)n(:)a~+I]­


-[ai -(~)a; +... +(-I)n(~)a~+2]=O.
10 que se necesita para la demostraci6n.
Por suposici6n de la inducci6n, ambas expresiones que se Sefialemos que para la progresi6n aritmetica aI' a
2, ••• , an'
encuentran entre corchetes son iguales acero, puesto que tienen es justa tambien una f6rmula mas comun
an+I'

la forma (1). Por esta raz6n, la f6rmula propuesta es valida


tam bien para n + I. Con esto la afirmaci6n queda demostrada. ak (n)
J -
k (n) k
1 a2 + 2 a3 - ••. + (- l)n-I (n n_ 1Jan + k

6. Realizamos la demostraci6n por el metodo de inducci6n. + (-l)n(:)a!+1 =0.


Siendo n = 3, es facil comprobar directamente que
donde k ~ 1 es un numero entero.

Supongamos que se haya establecido que para cierto valor 7. Por la propiedad de los terminos de una progresi6n aritrnetica
de n y una progresi6n aritmetica arbitraria XI' X 2 , ... , X n+l tiene tenemos:
lugar la identidad 2 m log x=n log x+ k log x.

Xl2- (~)xi + ... + (-1) n(: )X;+l = O. De aqui (v ease la f6rmula(3) pagina 24)

~2~=_1_+_1_
Entonces, procediendo en el caso de n + I como en el problema
x logm x logn x logk
anterior,obtendremos:
y, por consigu iente,
2 = ~ogm + Xlogm
x x'
logn logk
84 PROBLEMAS DE MATE MATI CAS ELEMENTALES

Utilizando la formula (2) expuesta en la pag. 24, obtenemos:


(2a1 - 2a]kc -d + cdk) + n(d - cdk 2 - 2cdk) = 0.
2=n log m+ k log m. Puesto que esta igualdad tiene lugar para cualquier valor de n,
Escribamos esta igualdad de la siguiente manera: entonces
2a1 - 2a, kc - d + cdk = 0,
nlogn2=nIOgm+nIOg( /IOgm). d - cdk 2 - 2cdk = 0.
Por potenciacion, obtenemos que Dividiendo la segunda de estas dos igualdades entre d =t. 0,
tendremos:
_ I
2 k'ogm (2)
n =mn , C - k(k+2).
o bien
La primera de estas igualdades se puede escribir en la forma

con 10 cual el problema queda resuelto.

8. Supongamos que sea En virtud de (2), el segundo factor se diferencia de cero y, por
consiguiente, d = 2a] .
=c. (1) As! pues, en el caso en que d =t. 0, la igualdad (1) puede
tener lugar para todos los valores de n solamente cuando se
tiene la progresion
Designemos la razon aritmetica por d. Representa interes solo
°
el caso en que d =t. 0, puesto que siendo d = todos los terminos
de la progresion son iguales entre Sl y se cumple la igualdad (1).
a, 3a, 5a, ... (a =t. 0). (3)

Utilizando la formula para la suma de los terminos de una Ahora es facil comprobar directamente que la progresion (3)
progresion aritmetica, de la fonnula (1) obtenemos: satisface en reaJidad la condicion del problema. As! pues, la
progresion buscada es la (3).

9. Supongamos que sea dla razon aritmetica. Tenemos:


de donde, despues de la simplificacion y agrupacion de los
terminos, haHamos:
ALGEBRAIResoluciones y Soluciones 85

Sumando estas igualdades hallaremos que


b2 = XI" + (XI + d)2 + ... + [XI + (n -1)0']2 = n(n-I)
=nx12 +2xl d U+ 2 + ... +(n-l)]+ an =al + d - 2- '

+d 2 [i"+2 2 + ... +(n-IF]= Haciendo uso de esta f6nnula, obtenemos:


= nxl2 + n (n -I) XI d'T "(II -I)(2n-l) d2 n(n-1]d
6 Sn =a] +a2 + ... +an =a1n+ [2+
1.2 2-3
2 +"'+-2- .
yademas,
En el problema 266 se demuestra que

Eliminando de estas ecuaciones XI' despues de simples


transfonnaciones obtenemos: Por consiguiente,
d 2 n(n 2 -I) =b2_ ~
12 n .

Oeaqui Para el problema en cuesti6n d= 3, a l = 1. Por esta raz6n

an =1+1 n(n-l) y Sn =-rn(n 2 +1).


a continuaci6n XI se detennina en ambos casos por la f6nnula
11. En la enesima columna horizontal se encuentran los numeros
Xl =~la-n(~-I)dJ. n, n+ 1, ... , 3n-3, 311-2 (en total hay 2n- 1 numero). La sumade
estos nllmeros es igual a
As! pues, siendo n 2 b2 - a 2 1= 0, a las condiciones planteadas (n+3n-~)(2n-l) = (2n _1)2.
satisfacen dos progresiones.
12. Supongamos que sea q el denominador de la progresi6n;
10. Supongamos que la sucesi6n ai' a 2 , ••. , an po see tal entonces
propiedad que
86 PROBLEMAS DE MATEMAnCAS ELEMENTALES

De aqui q2n = 1y, por consiguiente, q= 211. Ahora y por consiguiente,


tenemos:

am = am-n qn = B(zn\jJj
rA)n =.JAB ,
2n-ni ...!!!.....
an = am+n q-m = A -"'B- )-"'= A ---,;;- B
(
Ifl

,,, • obtenemos

13. Tenemos pn = (~"


SII
)~.

15. Designemos la suma que se examina por Sn' Multipliquemos


cada sumando de esta suma por x y restemos de S" Ia magnitud
y obtenida; como resultado tendremos:

Deaqui Sumando la progresion geometrica que aqui entra, para x*-1


hallaremos:
I n~ I
(l-x)S n = ~ I-x
-(n+l)xn+ .
10 que se exigia demostrar.
Deaqui
S = l_xn+1 _ ( n+l)x n+1
(x *- 1).
14. Tenemos: n (l-x)2 I-x

Cuando x = 1 tenemos:

Sn = (n+l)(n+2).
2

16. Designemos Ia suma buscada por Sn' Transformemos los


sumandos de esta suma empleando la formula para la suma de
los terminos de una progresion geometrica
ALGEBRAIResoluciones y Soluciones 87

I+IO= ~ n 1
X~+X"'--------=---+X~+
n-I I n-2 I 2 I
... +X~+X~=
1
9 ' x-I x-I x-I x-I x-I

= X~I[x+x2+ ... +xn-n]=


1 + \0 + 100 = 10: -I.

x r xn -I ] x 2 (x n -1) nx
1 + \0 + 100 + ... \0 /I - I = 10 ~- I . =~lx~-n = (x-1)2 -~.

Puesto que, adem.ls, 1 = 1°9- 1 , sumando los segundos Para x = 1, la suma buseada es igual a n(~+I) ,eomo la suma
miembros de las igualdades, tendremos de los terminos de una progresi6n aritmetica.

2 +
18. Designemos la suma buscada por Sn' Entonees,
Sn =1(10+10
9 . ...
+lOn_n)=l(
9~
lO n+I- I O_ n ).
9

2Sn 3 5 5 2n-1
= 1+:;-+ - 2 +-3 +"'+ - -1 =
17. La suma buseada puede ser representada en la forma siguiente -1 (2 ~) 2(2 ,23) (2 2 - 5 )
n
(2. 2n-3)_
- +"2 +"2 + \22 -; 22 + ~23 + 23 + ... + ~2n-I + 2 1 -
11-

(X+X2 +X3 + ... +x n- 2 +x n- I +xn)+ 1-_L


2 3 n-3 n-I) = 1+ 2n- I + S _In-=l
+ ( X+x +X + ... +X +X + 1_1 n 2"'
2.
+(X+X2 +X3 + ... +X n- 2 )+
dedonde
S
n
= 3 _ 2n+3
2n .

+x, 19. EI aspeeto general de estos numeros es el siguiente:

de 10 eual es f.leil eonveneerse sumando las eolurnnas vertieales. n n-I n n


~~ ~ ~
AI sumar las eolumnas horizontales, hallamos que para x 1:- 1, la
44 .. .4 88 ... 89 = 4 ·11·· ·1 · 1On + 8 ·11...1 + 1.
suma buseada es igual a
88 PROBLEMAS DE lVlATEMATl{;AS ELEMENTALES

n
El numero ~ puede escribirse en forma de la suma de los
terminos de una progresi6n geometrica con denominador 10:
n
observamos que
~ = 1 + 10 + 10 2 + ... + 10 n - 1 = 10;-1.
De este modo, tenemos: 2C
dd ~
r ~ ..
e on d e (x2" - Xl x3 = 0, y por conslgUiente, XIx) = x 2. on
la condici6n de que Xl 1= 0, de aquf se deduce que los numeros
X 1,X2,X3 forman una progresi6n geometrica. Supongamos ahora
que la afirmaci6n propuesta se ha demostrado para el caso de
20. Por la condici6n del problema Iql< 1, y tenemos: una progresi6n compuesta de k(k ~ 3) terminos:

q n = k( q q . . . ) = kq n+1
n+! + n+2 + _1_
I-q' (1) (1)

De aquf 1- q = kq y por consiguiente, si el problema tiene y que sea q el correspondiente denominador de la progresi6n.
soluci6n, entonces Examinemos entonces la sucesi6n compuesta de k+ 1 terminos:

(2) (2)
Sin embargo, se ve facilmente, que si, a1 contrario, de la f6rmula
Escribamos la condici6n correspondiente
(2) se deduce que Iql<l, entonces la f6rmula (2) trae consigo la
igualdad (1), y la correspondiente progresi6n satisface a la
condici6n del problema. Asf pues, el problema es soluble para
cualquier valor de kque satisfaga la desigualdad Ik~l < 1i Esto
ultimo tiene lugar cuando k> 0 0 bien k < -2.
. Xl? + X 2 + ... + X ~ _ = a.
y hagamos para abreVlar 2 k 1 2 Observemos
21. Llevaremos a cabo lademostraci6n porel metoda de inducci6n que a 1= 0, puesto que XI 1= O. Por la suposici6n inductiva
completa. Examinemos al principio el caso de una progresi6n tenemos:
compuesta de tres terminos x l' x 2' x 3 • Abriendo los parentesis
en la f6nnula
ALGEsRAIResoluciones y Soluciones 89

Por esta razon, la igualdad (3) se puede vo Iver a escribir de la Nos es necesario demostrar que siendo n > 2
siguiente manera:
(2)
(a 2 + x;)(q 2 a 2 + XL1)= (qa 2 + XkXk+l).
Puesto que de la igualdad (1) tenemos que d= a(q-l), (2) es
Abriendo los parentesis y agrupando los terminos, establece- equivalente a la desigualdad
remosque
a(n -I)(q -I) < a(qn-I -1)

Puesto que a 1:- 0, paralelo a (4) obtenemos Xk+1 = qxk . Por Dividiendo ambas partes de esta desigualdad entre la magnitud
consiguiente, la sucesion xI' x 2' ••• , x k' X k+ 1 representa una positiva a( q -1), obtenemos
progresion geometrica con el mismo denominador q = :~ . n-2
n- 1 < 1+q+ ... +q .
Basandonos en 10 demostrado podemos afirmar que la suce- Ya que q > 1, esta desigualdad es justa. El problema esta
sion formada con los n primeros terminos de la sucesion dada, resuelto.
para cualquier valor real de n, es una progresion geometrica.
Por esta razon, tambien la sucesion infinita dada forma una 23. Seg(In la condicion del problema
progresion, 10 que se exigia demostrar.

22. Supongamos que sea a l = hI = a; entonces, seg(In la condicion


a l > 0, ~
QI
=q > ° y h2 - hI = d > 0,

de que a2 = h2 tenemos: donde, como ordinariamente, q es el denominador de la


a+d = aq. progresion geometrica, des la razon aritmetica. Aprovechando
el hecho de que an =a8n- 1 y que hn = hI + (n-l)d, obtenemos:
Aqui dy q son la razon y eI denominador de las progresiones
correspondientes. Sefialemos que por la condicion de que all > Ql ogan +hn = (n-l)(Qlogq -d)+Qlogal -hI'
0, para todos los val ores de n, la razon aritmetica d no sera
Para que la razon aritmetica examinada no dependa de n, es
negativa. Puesto que, ademas, a l 1:- a 2 , entonces d> 0.
necesario y suficiente que log q - d = 0. Al resolver esta
Q
Debido a esto, de la formula (1) se deduce que
ecuacion hallamos:
q=l+iL>l. (1)
Q
90 PROBLEMAS DE MATEMATICAS ELEMENTALES

Por consiguiente, el numero buscado ex existe y se determina 25. Transformemos las ecuaciones del sistema dado en la torma
por la f6rmula (I ).
(x+ y)2 -XY=4,~
(x+ y)+xy=2 .J
2. Ecuaciones algebraicas y sistemas de ecuaciones
Deaqui
24. Oespues de escribir el sistema dado en la forma

(x + y)(x 3- xy + y:)= I,} (1) y por consiguiente, x + y = 2 0 x + y = -3. Comparando cada


y(x+ y) = 2.) (2) una de estas ecuaciones con la segunda ecuaci6n del sistema
inicial, obtendremos los dos sistemas de eCllaciones siguientes:
dividimos miembro a miembro la primera ecuaci6n entre la
segunda. Libenindonos del denominador y reduciendo a x+ y =2} (1)
continuaci6n los terminos semejantes, obtenemos: xy=O

/ - 3xy+ 2x2 = O. (3)

Al resolver Ia ecuaci6n cuadrada (3) respecto a y, obtenemos y


x+ y = -3} (2)
xy=5
= x 0 y = 2x. Resolviendo a continuaci6n cada una de estas
ecuacionesjunto con la ecuaci6n (2), hallamos las soluciones EI sistema (1) tiene dos soluciones:
reales de los correspondientes sistemas. Estas son solamente dos:
Yl =0;
Y2 = 2.
EI sistema (2) tiene tambien dos soluciones:

Cada uno de estos pares de numeros satisface tam bien al X =_ l +z·.Ji!


3 2 2'
sistema inicial; esto puede comprobarse mediante la sllstituci6n
directa 0 analizando el metodo por el cual fueron obtenidos.
ALGEBRAIResoluciones y Soluciones 91

Es evidente que las solueiones de los sistemas indicados 27. Redueiendo las eeuaeiones dadas a un eomun denominador,
eomprenden eada una de las solueiones del sistema inieial. Por transformemos el sistema en la forma
medio de un simple amilisis no es difieil demostrar 10 eontrario.
Por otra parte, 10 ultimo es faeil de eomprobar empleando la (x+ y)[(x+ y? -3xy]= 12XY ,}
sustitueion direeta. Asi pues, el problema tiene euatro solueiones. 3(x+ y) = xy.

26. Transformemos las eeuaeiones del sistema dado en la forma Hagamos X + Y = u, xy = v. Colocando xy = v de la segunda
ecuacion en la primera, obtenemos:
(x+ y)[(x+ yf -3xy]= sa 3 ,}

xy{x + y) = a 3 , u(u 2 - 9u)= 36u.

despues de 10 eual hagamos x + y = u,.ry = v. Sustituyendo en la


°
Sefialamos que u 1= (en el caso contrario, de la segunda
eeuaeion tendriamos que.ry = 0, 10 que eontradiee a la eeuaei6n
primera eeuaeion x).{x +y) de la segunda, hallaremos que zi3 = 8d. inieial). Porestarazon, de laeeuaci6n(l) se deduce que, 0 u= 12,
Puesto que nos interesan solamente las solueiones reales, ten- o bien u=-3.
dremos que u = 2a. Ahora, de la segunda eeuaeion hallamos: En el primer easo (u = 12) obtenemos el sistema
v = tt
11
= la
2
2
x + y = 12,}
De este modo, obtenemos eI siguiente sistema respeeto ax e y: xy=36,
de donde x = y = 6
I 2
x+ y= 2a, xY=-Za.
En el segundo easo (u=-3) tenemos:
Resolviendo este sistemaobtenemos:

f2
- a 2+2
x+ y =-3,}
XI - ' xy = -9.
2 G
X =a~-=-
2 2' Este sistema tiene dos soluciones:
Estos numeros satisfaeen tambien al sistema inieial, el eual,
por eonsiguiente, tiene dos solueiones reales.
92 PROBLEMAS DE MATEMAnCAS ELEMENTALES

Las tres so[uciones halladas satisfacen tambien al sistema inicial.


As! pues, el sistema tiene s610 tres soluciones.
(X - y){x 2+ y2 + xy -19)= o,l
(x+ y)(x2 + i -XY-7)=0.]
28. Elevando la segunda ecuaci6n al cuadrado y restandola de la
primera,obtenemos: Conesto, el sistema inicia[ se reduce a los cuatro sistemas de
ecuaciones siguientes:
0)
De aqu!, en virtud de la segunda ecuaci6n del sistema dado, x- y= 0,1
x+ y = 0,] (2)
xy=3.
Colocando el valor de yen la segunda ecuaci6n del sistema,
obtenemos la ecuaci6n bicuadrada x 2 + y2 + xy -19: 0,1 x 2 + y2 + xy -19 =0,1
(3) x 2 + y2 _ xy -7 = O.J
(4)
x+y-O,)

El primer sistema tiene una sola soluci6n: X = 0, y = O. E[


De don de, Xl = -3, x2 = -3, X3 = 1, x 4 = -1 Y los val ores segundo tiene dos soluciones: X = ±-J7, y = ±--J7.El tercero
correspondientes de y son los siguientes J' son [os siguientes: tambien tiene dos so[uciones: x = ±09, y = +-Ji9. Pasando al
Yl = 1, Y2 = -1, Y3 = 3, Y 4 = -3. Por comprobaci6n nos con- cuarto sistema, observemos que sumando am bas ecuaciones y
vencemos de que [os cuatro pares de numeros son [as soluciones restando una de la otra, sustituimos este sistema por el siguiente
del sistema inicial. Por consiguiente, el sistema tiene cuatro sistema equivalente:
soluciones: xy = 6,1
Xl = 3, Yl = 1, x 2 = -3, Y2 = -1; X2 + y-J = 1"".). J
X3 =1, Y3 =3, x 4 =-1, Y4 =-3.
Este sistema tiene cuatro soluciones:
29. Transformemos el sistema en la forma x = ±2, y = ±3 Y x = ±3, Y = ±2.
ALGEBRAlResoluciones y Soluciones 93

As, pues, el sistema propuesto en el problema tiene nueve


soluciones:
Ambos pares de numeros satisfacen tambien la ecuaci6n inicial.
(0,0), (F7, F7), (-F7, -F7), (-Ji9, --Ji9), As! pues, el sistema tiene solamente dos soluciones reales.
(--Ji9,)i9), (2,3), (-2,-3), (3,2),(-3,-2). 31. Sumando ambas ecuaciones, y a continuaci6n restando de la
primera la segunda, obtenemos el siguiente sistema equivalente:
30. Transformemos las ecuaciones del sistema en la forma:

2{x + y) = 5xy, (x- y)~2 + y2 +xy)= 7,1


8{x+ y)[{x+ y)2 -3xy]=65. {x- y)xy = 2.J
Colocando x + y de la primera ecuaci6n en la segunda y Representando la primera ecuaci6n en la forma
haciendo xy = v, tendremos:
(x - y)3 +3xy{x- y)= 7.
25v 3 -12v 2 -13 = O. haHaremos que en virtud de la segunda ecuaci6n {x _ y)3 = 1.
Esta ecuaci6n, evidentemente, se satisface cuando v = I. Por cuanto nos interesan solamente las soluciones reales,
Dividiendo el miembro izquierdo entre v -I, obtenemos la entonces, x-y= 1. Teniendo esto en cuenta, haHamos facilmente
ecuaci6n quexy=2.
Resolviendo a continuaci6n el sistema
25v 2 -12v - 13 = O.
Esta ultima ecuaci6n no tiene rakes reales. As, pues, no queda
xy = 2.1
x- y = I,J
mas que una posibilidad: v = 1. Colocando este valor en la
primera ecuaci6n, obtenemos el sistema haHamos sus dos soluciones:

xy = I,}
_ 5
x+ y -"2'
94 PROBLEMAS DE MATEMAnCAS ELEMENTALES

No es dificil comprobar que los dos pares de numeros par de val ores de u y v debe ser omitido. EI segundo par nos da
satisfacen al sistema inicial. Por 10 tanto, el sistema tiene dos elsistema
soluciones reales. x 2 + y2 =3,}
xy=-l.
32. Transfonnamos la segunda ecuaci6n en la fonna

(x 2 + / ) - 2x2/ = 7. Este sistema tiene cuatro soluciones reales:

Suponiendo que sea x 2 + y = U Y xy = v, escribimos esta


(1+.J5 1-.J5) (1-.J5 1+.J5)
2' 2'\2' 2'
ecuaci6n de la siguiente manera: (-I+.J5 -1-.J5) (-I-.J5 -1+.J5)
2' 2' 2' 2'

Sin embargo, es facil comprobar que al sistema inicial Ie


Elevando al cuadrado la primera ecuaci6n del sistema, satisfacen solamente las dos primeras de estas. As! pues, el
obtenemos una condici6n mas para u y v: sistema propuesto en el problema tiene dos soluciones reales.
33. Elevando la primera ecuaci6n a la quinta potencia y restando
u + 2v = 1. del resultado obtenido la segunda ecuaci6n, despues de las
Eliminando a u de las dos ultimas ecuaciones, obtenemos: simplificaciones correspondientes, obtenemos que

(1)
V2 -2v-3 = O.
de donde De la primera ecuaci6n, despues de elevarla al cubo, se
desprende que x3 +1 = I - 3xy; en virtud de 10 cualla ecuaci6n
VI = 3, v2 = -I. (I) adquiere la fonna
Entonces, los val ores correspondientes de u serlin iguales a

ul =-5, u2 =3.
Resolviendo esta ecuaci6n, obtenemos
Puesto que u = x 2 + Y y a nosotros nos interesan solamente
las soluciones reales de la ecuaci6n inicial, entonces, el primer (XY)1 = 3, (xY)2 = -2.
ALGEBRAlResoluciones y Soluciones 95

Afiadiendo a estos resultados x + y = 1, hallaremos cuatro Entre todas las raices de esta ecuaci6n solamente dos son
pares de numeros: reales:
(1+I.JlI l-i.Jll). y
(2,-1); (-1,2); \ 2 ' 2 '
( l-i.Jlj l+i,m) Los val ores correspondientes de y, en virtud de (2), seran:
2 ' 2 •
YI =2 Y Y2 = -2.
Es facil comprobar que todas estas soluciones satisfacen al
sistema de ecuaciones inicial. Ambos pares de numeros (x,y) satisfacen tambien al sistema
inicial. Asf pues, el problema tiene dos soluciones:
34. Transformemos las ecuaciones del sistema a la forma
Xl =I YI = 2; x 2 = -1, Y2 = -2
(x 2 _ y2)' + x' y2 ~ ]3,} 35. Abriendo los parentesis en las ecuaciones del sistema y
x2 _ y2 + 2xy = 1. haciendo X + Y = u Yxy = v, escribimos el sistema en la forma

Colbcando x 2 - .0 de la segunda ecuaci6n en la primera, u 2 + v2 - 2v = 9,1


obtendremos:
uv -u = 3.)
r (1)
s(xyf -4xy -12 = 0.

Dedonde Si, ahora, multiplicamos ambos miembros de la segunda


(1) ecuaci6n por 2 y, a continuaci6n, al resultado obtenido Ie sumamos
y Ie restamos los miembros correspondientes de la primera
Puesto que nos interesan solamente las soluciones para las ecuaci6n, el sistema (1) se sustituini por el siguiente sistema
cuales xy 2 0, entonces, existe una sola posibilidad equivalente:
xy=2. (2)
(U+V)2 -2(U+V)=lS,1
Sustituyendo en la segunda ecuaci6n y par su valor hallado de (2)
(U-V)2 +2(u-v)=3.J
esta igualdad, obtendremos:
De la primera ecuaci6n del sistema (2) recibimos:
96 PROBLEMAS DE MATEMAnCAS ELEMENTALES

(u+v)] =5; (u+vh =-3.


(1. 1'i.Jls 1.. _i.Jls)
\2 2'2 2'
De la segunda ecuaci6n obtenemos:
(2,1), (1,2), (-3,0), (0, - 3),1, - 2), (-2,1).
(u-v)] =-3; (u-v), =1.
36. Sefialemos al principio, que por el sentido del problema x 7:-
e y 7:- 0. Multiplicando los miembros izquierdos y derechos de
°
De este modo, la determinaci6n de todas las soluciones del
sistema (2) se reduce a la resoluci6n de los cuatros sistemas las ecuaciones dadas, obtenemos:
siguientes:
(1)
U+V=5'} U + v = 5,}
(3) u - v = 1, (4)
u - v = -3, Multiplicando cada una de las ecuaciones por xy y, a
continuaci6n, sumandolas, tendremos:
U + v = -3'} u+v=-3,1
u-v = -3, (5) u-v=l.] (6) X4 - / +2x2/ = 7xy. (2)

Soluci6ndel sistema (3): HI = 1, VI =4; soluci6ndel sistema (4): u1 En virtud de (1) Y(2), ahora obtenemos:

soluci6ndel sistema (6): u4 =-1, v4 =-2.


°
= 3, v1 = 2; soluci6n del sistema (5): u3 = -3, V3 = y, por fin,
2X2 y2 - 7 xy + 6 = 0,
Para hallar todas las soluciones del sistema inicial tendremos que dedonde
resolver los siguientes cuatro sistemas de segundo orden, que se (xy)] = 2; (xy)z = 1. (3)
diferencian s610 por sus miembros derechos:
Asi pues, cualquier soluci6n del sistema inicial satisface a la
x+ Y = I'} x+ Y =3,l ecuaci6n (1) Y a una de las ecuaciones (3). Por esta raz6n, cada
(7) (8) una de las dos ecuaciones (3) se podria resolverconjuntamente
xy=4, xy = 2,j
con la ecuaci6n (1). Sin embargo, esto conduciria a una ecuaci6n
'I ,

x+ y=-.J,} x+v=-II
- ,> de octavo orden y complicaria la resoluci6n definitiva del problema.
(9) (10) En relaci6n con esto, sefialemos 10 siguiente. Si muItiplicamos de
xy =0, xy = -2.j
nuevo cada una de las ecuaciones del sistema inicial por xy y, a
Resolviendo estas ecuaciones hallaremos todas las soluciones continuaci6n, de la primera restamos la segunda, entonces,
del sistema inicial. Estas senin solamente ocho: obtendremos la ecuaci6n
ALGEBRAIResoluciones y Soluciones 97

X4 + y4 = 5xy, (4)
Deaqui,
ala cual satisface tambien cualquiersolucion del sistema inicial.
Examinemos dos posibilidades:
x- =4[27
) '\}4'
4m .
x 8 =-i '\}4'
I) Supongamos que en concordancia con (3)
los valores correspondientes de y son:
xy=2. (5)
Y7 = -iVI,
Entonces, en virtud de (4), X4 + y4 = 10. Resolviendo esta
Asi pues, entre los ocho pares de nillneros hallados se encuentra
ecuacion conjuntamente con (I), hallaremos:
cualquier solucion del sistema inicial. Es facil comprobar que los
ocho pares de numeros hallados satisfacen al sistema inicial. Por
consiguiente, se han hallado todas las soluciones del sistema.
y, por consiguiente,
37. Transfonnamos la segunda ecuacion en la forma

?2
En virtud de (5), los correspondientes valores de y seran: (x-? + y) "?
- 2x~y =bx 22
y .

YI = ~ = V2, Y2 = -V2, Y3 = -iV2, Y4 = iV2. Son ~osibles dos casos:


1) a - 2 - b *- o. Es facil de ver, que para esta condicion el
2) En el segundo caso
sistema tiene una sola solucion: x= O,y= O.
(6) 2) a 2 - 2 - b = O. Cumpliendo esta condicion, la segunda
ecuacion se obtiene como resultado de elevar al cuadrado ambos
La ecuacion (4) nos conduce, entonces, a la relacion miembros de la primera. Por eso, si x eyes un par de numeros
cualquiera que satisfacen ala primera ecuacion, entonces, este
X4 -r. y4 =li
12 par de numeros satisfara ala segunda. Por consiguiente, el sistema
tiene una infinidad de soluciones.
que junto con (I) da
38. Transformemos el miembro izquierdo de la ecuacion en la
forma
98 PROBLEMAS DE MATEMATICAS ElEMENTALES

x+a (x+a _!'- x-a)+ x-a (x-a _ k x-a) =0


x+b x.,.h h x-h x-b x-b a x+b . XI = 3 + 51, x 2 = 3 -51, X3 = 6, x 4 = -2.

por el factor - *'


Notando que las expresiones entre parentesis se diferencian
obtenemos:
40.Supongamosquesea
x+y x-y
( x+a _!'- x-a)(x+a _ k x-a)= 0 ----;y = u y ~ = v. (1)
x+b b x-b x+b a x+h

De aqui, con la condici6n de que a # b,obtenemos: Entonces, la ecuaci6n del sistema dado se puede escribir en
laforma

y hallamos cuatro soluciones de la ecuaci6n inicial:

Resolviendo cada una de estas ecuaciones, hallaremos:


I 0
-(a+b)±'J (a+b)- +4ab (2)
X 3,4 = 2
VI = b, v2 -- I
b· (3)
En el caso de que sea a = b, todos los valores dex satisfacen
ala ecuaci6n. Ahora, tenemos que resolver cuatro sistemas del tipo (1) en
cuyos miembros derechos se encLientran todas las combinaciones
39. Supongamos que sea -1- ~ = t; entonees, la eeuaei6n posibles de los valores de u y v, determinados por las f6nnulas
(2) y (3). Escribamos el sistema (I) en la forma siguiente:
se reduce a la forma
3t"2- 101 + 8 =0.
(4)
Deaqui
y De aqui obtenemos: 1 I 1
-; ="2 (u - v), l
Resolviendo a continuaci6n dos ecuaciones cuadradas respecto (5)
;=1(u+v).j
a x, hallaremos cuatro rakes de la ecuaci6n in iciaI:
ALGEBRAIResoluciones y Soluciones 99

De las formulas (5) se desprende que para que el sistema (4) 41. Es facil de verquelos numeros
sea resoluble y, por 10 tanto, tambien el sistema inicial, los nillneros
a y b deben obedecer a condiciones suplementarias, ademas de XI = 4,5 Y x2 = 5,5
la condicion de que ab 7: 0 que se desprende de la forma de la
satisfacen a la ecuacion dada. Por eso, el polinomio
ecuacion del sistema inicial. Sea que
(X - 4,5)4 + (x - 5,5)4 -I
(6)
es multiplo del producto (x - 4,5)(x - 5,5). Para realizar la
Entonces, colocando en los miembros derechos de las fonnulas
(5) los valores u = a, v = b, y, a continuacion, u = ~, v = i, division y reducir el problema a la resolucion de la ecuacion
cuadrada, es comodo representar el polinomio indicado en la fonna
hallaremos dos soluciones:

x - _ 2_ Y - _2_. X _ 2ah y 2ab [(x-4,5t -l]+(X-5.5)4.


1- a-b ' 1- a+h' 2 - h-a ' 2 = -a+b .
Descomponiendo la expresion contenida entre los corchetes
Supongamos que, a continuacion,
por la formula
(7)
a4 - I = (a - 1)(a + 1)(a 2 + 1) = (a - 1)(a 3 + a 2 + a + 1),
Colocando en los miembros derechos de las formulas (5) los
obtenemos la ecuaci6n
valores u= a, v=i y, a continuacion, u = -;, v = b, halJaremos
dos soluciones mas: (x - 5,5) {(x - 4,5)3 + (x - 4,5)2 + (x - 4,5) + 1 }+

--.1!2._ Y -~. x -~ y -~
x 3- +(x-5,5)4 =0 .
h-I'
lI 3- h+l' 4-1 - b'
a 4-1+ h a a

As! pues, si se cumplen las dos condiciones (6) y (7), el sistema Sacando el factor comun entre parentesis, tendremos:
tiene cuatro soluciones; si se incumple una de las condiciones, el
sistema tiene solo dos soluciones; si, por fin, se incumplen las (x - 5,5){(x - 4,5)3 + (x - 4,5)2 + (x - 4,5) + 1 +
dos condiciones (esto puede suceder solamente en el caso de
+[(x-4,5)-lP}=
que Ia I = Ib I = 1, el sistema no tiene soluciones.
(x - 5,5)(x - 4,5){2(x -4,5)2 - 2(x -4,5) +4}= O.
100 PROBLEMAS DE MATEMATICAS ELEMENTALES

Deaqui Sefialemos un metodo mas de resoluci6n. Disponiendo los


10±17
xI =5,5, x 2 = 4,5, X 3,4 =~. sumandosdel miembro izquierdo seglin las potenciasdecrecientes
de x, obtendremos una ecuaci6n cuadrada respecto de x:
42. De la segunda ecuaci6n del sistema hallam os que
y - 5 =I x-II
~ 0 y, por consiguiente, y ~ 5. Por esta raz6n, la
primera ecuaci6n puede escribirse en la fonna
Esta ecuaci6n, para los val ores reales de y tiene rakes reales
y-5=1-lx- 11· solamente cuando su discriminante no es negativo, es decir,

Sumando esta ecuaci6n a la segunda, obtendremos: (2)

2(y - 5) = 1. Despues de abrir los parentesis, esta desigualdad adquiere el


aspecto
Deaquf y =Jf.
I
Ahora, de la ~egunda ecuaci6n haHamos x-II = y, por 1
1. 1,
consiguiente, x -I = ± Por eso, XI = X2 = ~. EI sistema Esto tiltimo es po sible solamente cuando y = -1, y, entonces,
tiene dos soluciones: de la ecuaci6n (1) se desprende que x = 1.

44. Transformemos la ecuaci6n en la fomla


43. Por agrupaciones de los tenninos reducimos el miembro
izquierdo ala fonna
Ninguno de los dos sumandos es negativo, poreso
(2x+ y-I? +(x+2y+l/ =0.
x+2cos(xy)=O, cos 2 (xy) = 1
Por consiguiente,
De aquf, cos (xy) = ±l. En el primercaso tenemos el sistema
2x+y-I=, x+2y+I=0.
Dedonde cos{xy) = I, x + 2 cos(xy) = o.
x = 1, Y = -1.
Deaquf, x=-2 e y=k:rt, donde k=O,±I,±2, .. .
ALGEsRAlResoluciones y Soluciones 101

En el segundo caso, Colocando este valor de z en la segunda ecuaci6n, obtenemos:

COs(xY)=-1 x+2cos(xy)=O

De aqui, x = 2 e y = ~ (2m + 1), donde m = 0, ± 1, ± 2, ... Esta ecuaci6n es equivalente a la ecuaci6n


Asi pues, la ecuaci6n tiene dos series infinitas de diferentes
soluciones reales, con la particularidad de que el valor de x en {x+ 1Y +(y +1)2 = a+1· (2)
cada serie es el mismo. 1
Si ahora a + < 0, entonces, laecuaci6n (2) no tiene
soluciones reales, puesto que siendo x e y reales, en el miembro
45. Eliminando z del sistema dado, tendremos:
izquierdo seencuentra un numero no negativo. Si a + > 0, 1
entonces, laecuaci6n (2) y junto con esta tOOo el sistema, tiene,
2xy-(2-x- y? =4 evidentemente, mas de una soluci6n.
o bien Por consiguiente, la unica soluci6n real es posibIe solamente
x2 - 4x + 4 + y2 - 4y + 4 = o. 1
cuando a + = 0. En este caso, la ecuaci6n (2) adquiere la forma

es decir, {x + 1Y+ (y + 1Y= °


-1,
ytiene la unica soluci6n real: x = y = -1· De aqui, hallando
Para los numeros realesx e y esto es posible solamente cuando z de la ecuaci6n (I) deducimos que eI sistema dado tiene la unica
x = 2 ey= 2. soluci6n real solamente cuando a = -1 a saber:
De la primera ecuaci6n del sistema hallamos z = -2. El sistema x=_l y=_l z=l
2' 2' 2.
tiene s610 una soluci6n real:
Segunda soluci6n. Se ve facilmente, que si el sistema dado
x = 2, Y = 2, Z = -2.
tiene cierta soluci6n x = x o, y = Yo, z = zo, entonces, este
46. Primera soluci6n. Notemos que, por las magnitudes x e y sistema tiene tambien la siguiente soluci6n:
dadas, el valor de z se determina de la primera ecuaci6n de una x = Yo, Y = x o, z = Zo. Por eso, para que el sistema tenga una
sola manera: sola soluci6n es necesario que x = y. Con esta condici6n, el
(1) sistema dado adquiere la forma
102 PROBLEMAS DE MATEMATICAS ELEMENTALES

Deaqui
2X2 = z} 2 2 Q2+b 2

=~.
Xo +Yo =~.
2x+z
Puesto que a y b son numeros reales, la confirmacion queda
Eliminando a z, obtenemos la ecuacion cuadrada respecto a demostrada.
x:
2X2 + 2x - a = O. 48. Es faei! de ver que el sistema tiene siempre la solucion
Para que tambien esta ecuacion tenga una sola solucion real,
x = 1, Y =1, Z = 1. (1)
en necesario y suficiente que el discriminante de la ecuacion sea
igual a cero: Es tambien evidente, que en el easo de que
D=2 2 -4.2(-a)=4(1+2a)=O. (2)
a=b=c
t
De aqui, a = - y el valor correspondiente de x = - t. En
las tres ecuacionp.s toman la forma x + y + z = 3 Yel sistema tiene
resumidas cuentas, obtenemos el resultado anterior.
un numero infmito de soluciones.
47. Supongamos que sean Xo e Yo ciertas soluciones del sistema. Demostremos que si no se cumple la eondicion (2) es decir,
En virtud de la primera ecuacion que si no todos los tres numeros a, bye son iguales, entonces la
solueion (1) es la unica posible.
r(~xo2+ Yo2) -a]2 = xoYo
2 2+ - 2-'
I + 2. (1)
Sumando primero las tres ecuaeiones del sistema dado
.TO}! I obtenemos:
y de acuerdo con la segunda ecuaeion
(a+ b + c)(x + y + z)= 3(a+ b + c).

(2) Simplificando pora+ b +c, hallamos:

Abriendo en el miembro izquierdo de la ecuacion (l) los x+y+z=3 (3)


eorchetes y restandole la ecuacion (2), obtenemos:
De aqui, z = 3 - x - y. Colocando esta expresion de z en las
- 2a ( Xo2 + Yo2) + a 2 = -b 2 .
dos primeras ecuaciones del sistema tendremos:
ALGEBRAIResoluciones y Soluciones 103

49. Sumando todas las ecuaciones, obtenemos que


(a-c)x + (b -c)y = a+ b - 2C,}
(4)
(b - a)x + (c - a) y = -2a + b + c. (1)

Multiplicando la primera de estas ecuaciones por c - a, la


Si a * -2, entonces
2
segunda por c - b Ysumandolas, hallaremos: X + •V + x = l+a+a
a+2
.

[-(a-c)2 +(b -a)(c -b)]x = Resolviendo esta ecuaci6n junto con cada una de las
(5)
= (a + b-2c )(c -a)+(c -b )(-2a + b+c). eCllaciones del sistema inicial, sllponiendo que a * 1, hallaremos:

Por cuanto la ecuaci6n (5) se satisface siendo x= 1, el coeficiente x=-~


a+2 '
de x debe coincidir identicamente, seg(In a, bye, con el segundo
miembro de la ecuaci6n. Abriendo los parentesis en ambas S iendo a = -2 el sistema es incompatible (Ia ecuaci6n (1) es
expresiones, nos convencemos de que verdaderamente coinciden imposible cualesquiera que sean los valores dex,y, z). si a = 1 el
y son iguales a: sistema es indeterminado: tres numeros cualesqlliera que
satisfagan la condici6nx+ y+ z= 1 forman una soluci6n.
-~[2a2 -4ac+2c 2 -2bc+2b 2+2ac-2ab]=
50. Se ve f£lcilmente, que si dos de los tres numeros ai' a2, a3
=-t[(a-cJ +(b -cJ +(a-bJ]. son iguales a cera, entonces, e) sistema tiene una multitud infmita
de soluciones. En efecto, sllpongamos, por ejemplo, que a2=0 y
Por 10 tanto, si entre los numeros a, bye hay desiguales,
a3=0. Haciendo, en este caso x=0 y eligiendo a y y z tales que se
entonces, la ecuaci6n (5) se satisface solamente en el caso de
satisfaga la ecuaci6n X+z= 1, se satisfaran las tres ecuaciones del
que sea x = 1. Oespues de esto, de la ecuaci6n (4) se desprende
sistema.
con facilidad que y = 1, y de la relaci6n (3), que z = 1. Asf pues,
Por eso, al bllscar la condici6n de unicidad, podemos suponer
con la condici6n de que
desde el principio que dos numeras cualesquiera difieren de cera.
(a - c? + (b - C)2 + (a - b)2 *0 Supongamos, por ejemplo, que

el sistema tiene una sola soluci6n: (I)


x = 1, Y = 1, z = 1.
104 PROBLEMAS DE MATEMAncAs ELEMENTALES

Restando de la segunda ecuaci6n la primera y de la tercera la indicada no solamente es indispensable, sino tambien suficiente.
segunda, hallamos que a? = a,;y = a3z. De aqui, en virtud de
(1 ), se desprende que: 51. Multiplicamos la ecuaci6n por a, -b, -c, -d, respectivarnente,
y realizamos la suma. Hallamos que
y=~x, Z =~x. (2)
a2 Q3

(a 2 + b 2 + c 2 + d 2 )x = ap - bq - cr - ds
Colocando estas expresiones en la primera ecuaci6n, obte-
o ap - bq - cr - ds
nemos:
(3) x = a2 + b2 + c2 + d 2 .
Esta ecuaci6n es resoluble solamente con la condiei6n de que
la expresi6n entre parentesis difiere de cero. Analogamente hallamos que
Teniendo en cuenta (1), Ilegamos a la condici6n
y= ~+~-~+~ . 7=
~+~+M-~

a 2 +b 2 +c 2 +d 2 ' - a 2 +b 2 +c 2 +d 2 '
dp - cq + br + as
t= 2 ? ? 2·
Cumpliendo esta condiei6n, de (3) y (2) hallamos: a +b- +c- + d

(5) 52. Sumando todas las ecuaciones del sistema hallaremos:

Estos tres numeros forman la soluei6n del sistema y, ademas, _ 2(a1 + a2 + ... + aJ
como se desprende del metodo de su obtenci6n, unica. XI +x2 + ... +xn - n(n+ 1) (\)
Asi pues, la condici6n (4) es una condici6n imprescindible
para que el sistema tenga soluei6n y, ademas, unica. EI segundo miembro de esta ecuaci6n 10 designamos por A.
Es f£leil de comprobar, que si al principio hubieramos supuesto Restando de la primera ecuaci6n la segunda, obtenemos:
diferentes de cero a otro par de numeros aI' a3 0 aI' a2, entonees,
un razonamiento an£llogo nos conduciria de nuevo a la eondici6n
(4) y ala misma soluei6n (5). Puesto que, a continuaci6n, de la En virtud de (1) tenemos:
condici6n (4) se desprende que si aunque sea uno de los tres A-(a-a)
-
X 1- I 2
pares de numeros no es igual acero, entonces, la condici6n .
n
ALGEBRAIResoluciones y Soluciones 105

En general, para obtener xk (1 ::; k ::; n - 1) restamos de la Pero, seglin las igualdades iniciales, cada surna encerrada entre
k-esima ecuaci6n la (k+ 1)-esima. Analogamente a 10 interior parentesis es igual a cero. Por esta raz6n XIOO=O. De manera
hallaremos: A _ (_ analoga, pasando x lOO al primer lugar y presentando la igualdad
ak ak + 1)
xk = . (1) en la forma
n
Por fm, restando de la ultima ecuaci6n la primera, obtendremos: {XI 00 +X\ +XJ+(X3 +X4 +xs )+'"
x = A - (an - al ) ",+(X96 +X97 +X98 )+X99 =0,
n •
n
hallaremos que X98 =0. Colocando a continuaci6nx99 en el primer
Los valores hallados pueden ser agrupados en una sola f6rmula:
lugar y agrupando de nuevo los sumandos de tres en tres, nos
convenceremos de que X98 =0. etc. As! pues,
Xi = A-(a-a
1
)(
1+1 · 1 ::; i ::; n
)
(2)
n XI = x 2 = ... = x lOO = 0,
(aqui, por am-I debe entenderse aJ Mediante la sustituci6n
directa nos convencemos de que el conjunto de numeros (2) 10 que se exigfa demostrar.
satisface realmente a todas las ecuaciones del sistema. As! pues,
el sistema dado tiene una sola soluci6n. 54. Sumando todas las ecuaciones tendremos que

53. Sumando todas las igualdades y dividiendo el resultado (X+y+z)2-(x+y+z)-I2=O. (1)


obtenido entre tres, tendremos: Hagamos X + Y + z = I, entonces, de la ecuaci6n (I) halla-
remos:
(1)
(2)
EI miembro izquierdo de esta nueva ecuaci6n tiene cien
sumandos y puede ser presentado en la siguiente forma: Colocando la suma y + z = t - X en la primera ecuaci6n del
sistema inicial, hallaremos que
(XI + X2 + x3)+ (X4 + Xs + X6)+ .. ·
X2 + X(I - x) - x = 2,
... + (X97 + X98 + X99 )+ XIOO = 0. de donde,
2
X= - - . (3)
1-1
106 PROBLEMAS DE MATEMA T1CAS ELEMENTALES

De fa misma manera, la sustituci6n de x + z =t - y en la o


segunda y de x + y = t - z en la tercera ecuaci6n del sistema x3 + / = (7 + z Y- 3{6 + 7 z )(7 + z) == Z3 -18z + 217. (2)
inicial nos da 4
(4) Comparando (2) con la ultima ecuaci6n del sistema (1),
Y == [-1
y hallamos que z == 12. Pero, entonces
6
Z==--.
[-1
(5)
x+ y == 19,}
xy =90.
Colocando los dos vafores de t en las f6rmulas (3), (4) Y(5),
hallaremos las dos soluciones del sistema inicial
Resolviendo este sistema de dos ecuaciones, obtenemos:

Por sustituci6n, es flicil de comprobar que estos dos conjuntos


55. Escribamos el sistema en la forma siguiente: de numeros satisfacen tambien al sistema inicial. As! pues, el
sistema inicial tiene dos soluciones.
x+ y =7 + z,}
X
2 ? 3
+ y- = 7 -z~,
?
56. Dividimos la primera ecuaci6n entre la segunda y la tercera;
(1)
3 3 3 como resultado obtenemos
x+y==l+z.
y+z 5 z+x 4
==
Elevando la primera ecuaci6n af cuadrado y eliminando:xl +y x+y 3 ' x+y 3
con ayuda de fa segunda ecuaci6n, hallamos que
Multiplicando ambas ecuaciones por x + y, hallamos:
5x+ 2y -3z == o,l
de donde >
x+4y-3z==0.J
xy == 6 + 7z.

A continuaci6n, obtenemos que De estas ecuaciones se desprende que y==2x, z==3x. Colocan-
do de aquf el valor de y y z en la primera ecuaci6n del sistema
(7 + z y == x 3 + / + 3xy(x + y) inicial, hallaremos quex2==1. Como resultado obtenemos:
ALGEswResoluciones y Soluciones 107

u+ w-l =O,} u+w-l = O,}


Realizando la verificaci6n nos convencemos de que los dos v - w = 0, (4) v+w=O,
(5)
w-" + u~~ + v = 2,
.., ?
conjuntos de numeros satisfacen tarnbien al sistema inicial. w- +u- + v = 2.

57. Fijandonos en que la diferencia de dos ecuaciones del sistema En virtud de 10 dicho, es evidente que todas las soluciones de
propuesto se descompone en factores, formamos la diferencia estos cuatro sistemas, y solamente elIas, son al mismo tiempo las
entre la primera y la segunda ecuaciones yentre la primera y la soluciones del sistema inicial. Cada uno de estos cuatro sistemas
tercera. Las dos ecuaciones obtenidas de esta manera,junto con dados puede reducirse sin dificultad a una ecuaci6n cuadrada y
la tercera ecuaci6n del sistema inicial, compondran el siguiente tiene dos soluciones. Expongamos las soluciones
sistema: correspondientes (u, v, w) omitiendo los ca1culos. Soluciones
(u - w)(u + w-l)= O,} del sistema (2):
(v-w)(v+w-I)=O,
( -I+-Ji7 -1+-Ji7
(1)
w2 +u 2 +v=2.
-1+-Ji7J
4' 4 4 '
r.;:::,.\
( -I--Ji7 -1--Ji7
Es evidente, que cualquiera soluci6n del sistema inicial satisface -1-\117 1
al sistema (1).
Puesto que, y al contrario, todas las ecuaciones del sistema
4' 4 4 J
inicial pueden ser obtenidas sumando y restando las ecuaciones Soluciones del sistema (3):
del sistema (1), entonces, toda soluci6n del sistema (1) es al

-~).
mismo tiempo la soluci6n del sistema inicial y, por consiguiente, (
1 3
estos sistemas son equivalentes.
El sistema (1) se descompone en los cuatro sistemas siguientes:
(1,0,1),
l-2' 2'

Soluciones del sistema (4):


u-w=O} u-w = O,}
v-w = 0: (2) v+ W= 0,
~
w- +u +v
2
= 2, w2 + u2 + V = 2,
(3)
(0, I, 1),
(%' 2' -~).
108 PROBLEMAS DE MATEMATICAS ELEMENTALES

Soluciones del sistema (5): De {I) se desprende que los miembros derechos de la primera
y tercera ecuaciones del sistema (2) son iguales, es decir, que

(1,1,0), (-l' 1- 3xy = 7 - 3yz,


de don de 2
As! pues, el sistema inicial tiene en total ocho soluciones. Z-X=-. (3)
Y
58. Restando de la segunda ecuaci6n la primera, obtenemos: Puesto que de acuerdo con (1)

z+x=2y, (4)

dedonde entonces, resolviendo conjuntamente (3) y (4) hallamos:

(z - y)(x + y + z) = 3. 1 1
X=Y--, z = Y+ - .
Y Y
Restando de la tercera ecuaci6n la segunda, amilogamente
hallamos Colocando la expresi6n obtenida para x en la primera ecuaci6n
del sistema inicial tendremos que
(y-x)(x+ y+z)=3.
3/-4/ + 1 = 0,
De las dos ultimas ecuaciones se deduce que dedonde 1
Yl,2=±1, Y34=±-r=- '
z - y = y -x. , --J 3
(1)
Como resultado, hallamos cuatro conjuntos de l1umeros
A continuaci6n, escribimos el sistema inicial en la forma
siguiente: (0,1,2) (0, -1, -2);

(x- y)2 =1-3xy ,1


(x - z / = 4 - 3xz, (
(~, ~, -~}
-

(2)
(y - z)2 = 7 - 3 yz.J
(-,~, }j'~)'
ALGEBRA!Resoluciones y Soluciones 109

Mediante la verificaci6n nos convencemos de que tados estos Este sistema es resoluble unicamente con una condici6n
coftiuntos satisfacen al sistema inicial. complementaria

59. Multiplicando los primeros y segundo miembros de las (1)


ecuaciones entre sf, obtenemos:
Con esta condici6n, es evidente que se obtendn'i una cantidad
infinita de soluciones. A continuaci6n, podemos suponer que
dedonde a,.: l. (2)
(1)
Sumando todas las ecuaciones del sistema y haciendo, para
escribimos la k-esima ecuaci6n del sistema en la forma simplificar,
x + Y + z = t,
obtenemos:

De aqui, en virtud de (I), tenemos:


Puesto que segun la condici6n del problema el miembro

xk = iJ
I n-~ .. an (k = 1, 2, ... , )
n .
derecho es positivo, para a = -2 el sistema no tiene soluci6n.
ak Considerando que
Por medio de la comprobaci6n nos convencemos de que este a,.:-2. (3)
conjunto de numeros satisface al sistema inicial. Asf pues, el hallamos: Ie +/2 + m 2
problema tiene una sola soluci6n. t=±~ a+2 (4)

60. Notemos al principio que siendo a = I el sistema toma la Transformando a continuaci6n las ecuaciones del sistema en
forma laforma:
110 PROBLEMAS DE MATEMATICAS ELEMENTALES

de acuerdo con (4), hallamos de aqu! dos conjuntos de valores 1) Supongamos que sea
dex ey:
xy= 2. (3)

Eliminando x +y de la primera y tercera ecuaciones del sistema


inicial, obtenemos la siguiente ecuaci6n respectode z.

Z2 -6z+9 =0.
De aqu! se deduce que z(l)=3.
Entonces, la primera ecuaci6n da

Realizando la comprobaci6n establecemos que las dos temas x + Y = 3.


de numeros satisfacen al sistema inicial. As! pues, en el caso
general (a * 1, a * -2) el sistema tiene dos soluciones diferentes. Resolviendo esta ecuaci6njunto con la ecuaci6n (3), obte-
nemos:
61. Elevando la primera ecuaci6n al cuadrado y restando de la xiI) = 1, y}l) = 2,
relaci6n obtenida la segunda ecuaci6n, hallaremos: x~l) = 2, y~l) = 1.
xy+yz+zx= 11. (I)
2) Supongamos ahora que de acuerdo can (2)
En virtud de la tercera ecuaci6n, de aqu! se desprende que
xy = -5. (4)
(xyY +3xy-10 = O.
En este caso, de la primera y tercera ecuaciones obtenemos:
Resolviendo esta ecuaci6n, obtenemos: Z2 - 6z + 16 = O.
(xy) I =2, (xyL=-5. (2) Las raices de esta ecuaci6n son irreales y, por consiguiente,
las investigaciones relacionadas con la condici6n (4) pueden no
Examinemos dos posibilidades. Ilevarse a cabo.
ALGEBRAiResoluciones y Soluciones 111

ASl pues, solueiones reales pueden ser solamente las siguientes dedonde
temas de numeros (x,y, z): uvw = ± -Jabc. (3)

(l,2,3)y(2,1,3). Ahora, la determinaci6n de todas las solueiones del sistema


(2) no representa ninguna dificultad. Eligiendo en la f6rmula (3),
Mediante la verifieaei6n nos conveneemos de que ambas temas al principio, el signo mas y, a continuaei6n ei menos, establecemos
de numeros satisfacen al sistema inieial. ASl pues, han sido halladas que el sistema (2) tiene dos soluciont>s:
todas las solueiones reales del sistema.
(4)
62. Se ve faciimente que los primeros miembros de las eeuaciones
y
pueden ser deseompuestos en faetores, como resultado de 10
eual el sistema toma la forma -Mc --Jabc -Mc
u2 = c v2 =--b- ' w2 = a (5)
(x + y)(x + z) = a,} Queda resolver dos sistemas de eeuaciones, obtenidos al colo-
(x+ y)(y+z)=b, (1) car los val ores (4) Y (5) en los miembros dereehos de las
(x + z)(y + z) = c. ecuaciones

Hagamos para simplifiear x+ y =u,}


x+z = v,
(6)
x+ y=u, x+z=v, y+z=w. y+z = w.
Entonees
uv=a,} Sumando las eeuaeiones (6), obtenemos
uw=b, u+v+w
(2) x+y+z=-- -
vw=c. 2

Multiplicando todas las ecuaciones entre sl, hallaremos: De aqui, en virtud de (6), se desprende facilmente que

(UVW)2 = abc, u+v-w u-v+w -u+v+w


x=--2- ' y= - -2-' z= 2 (7)
112 PROBLEMAS DE MATEMAnCAS ELEMENTALES

As! pues, el sistema inicial tiene solamente dos soluciones que de donde db 2=0, 10 que contradice a la condicion del problema.
se determinan por las formulas (7}colocando en estas los valores As! pues, a131"* O. Por esta razon, el sistema (2) coincide
(4)y(5). exactamente con el sistema (2) del problema anterior. Por
consiguiente, este sistema tienedos soluciones:
63. Sumando todas las ecuaciones hallaremos:
~a131
~
la131
_-II_ - ,
XI - Z 1 -- l/a131 .,
Yl =-13-' a
a 2 + b2 + c 2 1
xy+xz+ yz= - - - -- (1)
2
x, =
--fa.I3Y , -~
- 1 Y2 = 13 '
En virtud de las ecuaciones del sistema, ahora obtenemos
facilmente que Es facil de comprobar que estos dos conjuntos de numeros
satisfacen tambien al sistema inicial. De este modo, las soluciones
(3) y (4) contienen todas I..''> soluciones del sistema.

64. Hagamos
(2)
xy + xz + yz = (3. (1)

Entonces, el sistema se puede escribir en la forma

Aquf hemos introducido, para mayor comodidad,


designaciones simplificadas para los quebrados obtenidos.
Notemos que si el sistema inicial tiene solucion, entonces, en (2)
nuestras condiciones los tres numeros a, 13 y 1 se diferencian
de cero. En efecto, supongamos, por ejemplo, que a = O.
Entonces 131 = xyz2 = O. Sumando la primera ecuacion del Sumando todas las ecuaciones de este sistema, hallaremos
sistema (2) con la segunda y la tercera, obtenemos: que

(3)
ALGEBRAIResoluciones y Soluciones 113

Restando consecutivamente de esta ecuaci6n las ecuaciones u = 0, v = 0, w = 0. (2)


del sistema (2), obtenemos:
Prestamos ademas atenci6n a que si u = 0, entonces, de la
x 3 = (b + c - a) t 3 , y3 = (c + a - b) t 3, z3 = (a + b - c) t 3 . °
primera ecuaci6n de (1) se desprende que v = y de la tercera
que w = 0. Por esta rawn, nos limitaremos a examinar los casos
Dedonde
enque
UVW7: 0.
x=Vb+c-a·t, y=Vc+a-b·t, z=Va+b-c·t.(4)

Colocando estas expresiones en la ecuaci6n (1), hallaremos


que, a II = 0, 0 bien

'2 == VU;+c - a)(c +a-b) +V(b +c -a)(a+b -c) +


+ V(c + a- b )(a + b - c)
Colocando estos valores de t en las f6rmulas (4), hallaremos
dos soluciones del sistema inicial. Este sistema tiene la misma forma que el sistema (6) en la
resoluci6n del problema 62. Empleando el mismo procedimiento,
65. Hagamos obtenemos:
a+b-c
x+y=u, x+z=v, y+z=w. u 2
Entonces, el sistema se escribe de la manera siguiente: a-b+c
v 2 (3)
u + v = auv,} _~ = - a + b + c .J
u+w=buw, w 2
(1)
v+w=cvw. De aqui se desprende que el sistema (1) puede tener una
soluci6n distinta de la soluci6n (2) solamente con una condici6n
Es evidente, que el sistema(l)tiene la siguiente soluci6n: complementaria, a saber:
114 PROBLEMAS DE MAiEMATIC AS ELEMENTALES

a + b - c =a =t- 0, a - b + c = [3 =t- O,} Asi pues, eI sistema inicial tiene solamente una soluci6n nula:
(4)
- a + b + c = y =t- 0. x = y = z = 0, y si se cumple lacondici6ncomplementaria(4)tiene
otra soluci6n mas que se detennina por las f6nnulas (8) y (4).
Si se cumple la condici6n (4), entonces, de las f6nnulas (3)
deducimos que 66. Por la forma de la segunda ecuaci6n deducimos que
2 2 2 x =t- 0, Y =t- 0, Y z =t- 0. Reduciendo el primer miembro de la
U= - V=- W = -. (5)
a' ~' y segunda ecuaci6n a un comun denominador, en virtud de la tercera
Para concluir la resoluci6n del problema nos queda resolver ecuaci6n,obtenemos:
dos sistemas: xyz = 27. (1)
2 Multiplicando a continuaci6n la tercera ecuaci6n por z, tomando
x+y=-,
a en consideraci6n (1), tendremos:
x+U = O,} 2
x+z = --
x+z = 0, ~' 27 + (x + y)z2 = 27 z.
(6)
(7) Y + z = 0, y+z =-.
2 Colocando aqui x + y 9 (<z de la primera ecuaci6n del sistema,
y J obtendremos:
Z3 - 9z 2 + 27z -27 = 0,
EI sistema (7) surge solamente al cumplir la condici6n (4). o
Cada uno de estos sistemas tiene una sola soluci6n; la soluci6n (z _3)3 = 0.
del sistema (6) es:
x = 0, y = 0, z = 0. Por eso, z = 3. Colocando este valor dez en la primera ecuaci6n
yen (I), hallaremos que x =3 e y = 3. Esto ultimo podia haber
yel sistema (7) tiene la soluci6n sido previsto, ya que todas las inc6gnitas entran en las ecuaciones
del sistema simetricamente. As! pues, si el sistema tiene soluci6n,
x =~ + ~ - ~
a ~ y'
y = ~ - ~ +~
a ~ y'1
I esta puede ser unicamente la temade numerosx= 3,y= 3 y z= 3.
Efectuando Ia verificaci6n nos convencemos de que estos numeros
z =-
I I
+ - - + -- . i
I I (8) fonnan efectivamente la soluci6n. As! pues, el sistema tiene Ia
a ~ Yj soluci6n (y, ademas, unica):
ALGEsRAIResoluciones y Soluciones 115

67. Colocando x + Y de la primera ecuaci6n en la segunda, De este modo, al sistema 10 satisfacen las seis soluciones
obtendremos: indicadas, que son las unicas que puede tener el sistema.
Este mismo resultado puede ser obtenido por una via mas
corta si se presta atenci6n en la relaci6n de las soluciones del
IntroducienJo de aqui -'Y en la tercera ecuaci6n, tendremos:
sistema en cuesti6n con las rakes de la ecuaci6n cubica

(1)
EI primer miembro se descompone facilmente en factores:
En efecto, de acuerdo con las f6rmulas de Viete (vease (2),
(z-a)(x-ai)(z+ai)=O. pagina 11) las tres rakes de la ecuaci6n (1)
Dedonde

Colocando z = a en la primera y segunda ecuaciones, obte- enumeradas en cualquier orden, forman la soluci6n del sistema
nemos el sistema en cuesti6n. De este modo, tenemos ya seis soluciones.
Demostremos que estas seis soluciones son las unicas que puede
X+ Y =0.2'1
tener el sistema. En efecto, supongamos que sean (Xl' Y l ' Zl)
xy = a .J cierta soluci6n del sistema. Examinemos la ecuaci6n de tercer
De aqui, x = ±ia,y = +ia, ademas, es facil de comprobar grado
que las dos temas de numeros (x,y, z):
(t - Xl) (t - Yl) (t - Zl) = 0, (2)
(ia, - ia, a) y (- ia, ia a)
cuyas rakes son los numerosxl'YI' z,. Abriendo los parentesis
Satisfacen al sistema inicia!. De manera analoga hallamos dos en la ecuaci6n (2) y haciendo uso de las igualdades
pares mas de soluciones que corresponden a los valores de Z2 y
Z3:
XlYI + YlZI + XlZI =a2,
(a, - ia, ia), (- ia, a, ia), (ia, a, - ia), (a, ia, - ia). X1YlZI = a3 .
116 PROBLEMAS DE MATEMATICAS ELEMENTALES

nos daremos cuenta de que las ecuaciones (2) y (1) coinciden. en virtud de la primera y segunda ecuaciones del sistema,
Por consiguiente, X 1'Y1 y Z1 son las rakes de la ecuaci6n (1),10 obtenemos
que se exigia demostrar. Esta observaci6n podia haber sido xy+xz+ yz=O. (2)
empleada al resolver el problema anterior.
Examinemos a continuaci6n la identidad que se obtiene al elevar
68. Colocando x de la primera ecuaci6n en la segunda, obtenemos: el trinomio al cuba:
\-, 3 1 :1 ? 2
(X+ Y + Z J = x + y + z· + 3x-y + 3x Z +
(3)
+3xy2 +6xyz+3xz 2 +3y2Z+3yz2 .
En virtud de la tercera ecuaci6n, de aqui se desprende que
EI segundo miembro de esta identidad se puede presentar en
3/ -xy = 0. (2) la forma siguiente:
x 3 + i +Z3 +3x(xy+xz+ yz)+
Por eso, 0 Y = 0, 0 bienx = 3y.
+3y(xy+ yz+xz)+3z 2(x+ y).
En el primer caso (y = 0), de acuerdo con (1), Z = 0, y, en
virtud de la primera ecuaci6n del sistema, x = 0. Por consiguiente, en virtud de las ecuaciones del sistema y de
En el segundo caso, colocando x de la igualdad x = 3yen la la igualdad (2), de la identidad (3) se desprende que
segunda ecuaci6n del sistema, obtenemos:
(4)
2y2 + 4yz = 0. (3)
En relaci6n con esto, examinemos dos casos:
Si ahora y = 0, obtenemos el primer caso ya examinado. Pero, 1) Si z = 0, entonces, de acuerdo con (2), xy = 0. Teniendo
si y = -2x, entonces, de la condici6n (1) se deduce que z = y, ° en cuenta, a continuaci6n, la primera ecuaci6n del sistema,

°
por consiguiente, y = y x = 0. La confirmaci6n queda
demostrada.
obtenemos dos conjuntos de valores:

Xl = a, Yl = 0, z\ = 0, (5)
Xl = 0, Y2 = a, z2 = 0. (6)
69. De la identidad
En este caso, es facil de ver que las f6rmulas (5) y (6)
(x + y + z)2 = X2 + y2 + z2 + 2(xy + xz + yz), (1) detenninan dos soluciones del sistema inicial.
ALGEsRAIResoluciones y Soluciones 117

2) Si x + y = 0, entonces, de la condici6n (2) obtenemos de Elevando la primera ecuaci6n al cuadrado y restandole la


nuevo xy = 0, y por consiguiente, x = 0, y = 0. De la primera segunda,obtenemos
ecuaci6n del sistema se deduce que z = a y obtenemos una
soluci6n mas del sistema inicial: xy + xz + yz = -1 \a
(2
- b2) . (3)
2
(7) A continuaci6n, escribamos la tercera ecuaci6n del sistema en
laforma
°
As! pues, con la condici6n de que a :f:. el sistema tiene tres
°
soluciones distintas y en el caso en que sea a = el sistema
xyz = c(xy + xz + yz). (4)

tendni una sola soluci6n nula. Teniendo en cuenta (3) y (4), de la identidad (2) hallamos
defmitivamente:
70. Examinemos la identidad 3 3 3 3 3 (2 b2) +-ca-
x+y+z=a--aa- 3 (2 b2) =
2 2
(x+ y+zY =x 3 + / +Z3 +3x 2y+3x 2 z+
(1) =a 3 +-3 (2
a -b 2\1
p.p-a ).
+ 3xy2 + 6xyz + 3xz 2 + 3y2 Z + 3yz2 . 2

Transformamos el segundo miembro de esta identidad a la 71. Abriendo los parentesis, escribamos la segunda ecuaci6n en
forma laforma
x 3 + / + Z3 + 3x(xy + xz + yz) + 3 y(xy + xz + yz) +
x2 + i + z2 + 3xy + 3xz + 3yz = 1,
+ 3z(xy + xz + yz)- 3xyz. obien
(x + y + z f + XY + xz + yz = 1.
De aqu! se desprende que la identidad (1) se puede escribir
as!: De aqu!, haciendo uso de la primera ecuaci6n del sistema,
y
(x + y + z = x + / + Z3 +
3
(2)
obtenemos:
+ 3(x + y + z )(xy + xz + yz)- 3xyz. xy+xz+ yz=-3 (1)
De la relaci6n (2) se ve que para determinar la suma Presentemos la tercera ecuaci6n del sistema en la forma
x 3 + y3 + z3 es suficiente expresar del sistema inicial
xy + xz + yz y xyz. x(xy+xz)+ y(yz+xy)+z(xz+ yz)=-6.
118 PROBLEMAS DE MATEMATICAS ELEMENTALES

72. Abriendo los parentesis en las tres ecuaciones notaremos


Entonees, tomando en eonsideraei6n a (1), tendremos que si a la suma de las dos primeras ecuaciones Ie restamos la
tercera, tendremos la ecuaci6n
x (3 + yz) + y (3 + xz) + z (3 + xy) = 6,
o bien (x - y + z f = a - b + c. (1)
x + y + z + xyz = 2,
es decir, Procediendo analogamente, hallaremos:
xyz =0.
f
(x + y + z = a + b - c, (2)
Reeibimos el siguiente sistema: (x + y + z)2 = b + c - a. (3)

x+y +z = 2,l No es diffcil convencerse de que tambien, al contrario, eJ


sistema inicial es resultado del sistema de ecuaciones (1), (2) Y
xy + xz + yz = -3, ~
(2) (3). En efecto, sumando, por ejempJo, las ecuaciones (2) y (3),
xyz = o.J obtendremos la segunda ecuaci6n del sistema inicial, etc. As! pues
el sistema inicial y el obtenido son equivalentes. Por esta raz6n,
De la ultima eeuaei6n de este sistema se desprende que, por es suficiente hallar todas las soluciones del sistema de ecuaciones
10 menos, una de las inc6gnitas es igual a cero. Sea que x = 0; (1), (2) y (3). Supongamos, para simplificar el problema, que
entonces,
(b + c - a = a\,
'\j ~'
a -b
b+C = \ Y ra + b - C = c\.
'\j
y +z = 2, yz = -3,
Entonces, el sistema de ecuaciones (1), (2), (3) es equivalente
de donde, 0 y = 3, z = «I, 0 y = «I, z = 3. De forma analoga se
°
estudian los easos en que y= y z= 0. De esta manera reeibimos
seis solueiones (x,y, z) del sistema (2):
a los ocho sistemas de primer grado siguientes:
x- y+z =±b\,
x+ y-z = ±c\.
(0,3,-1), (-1,0,3), (0,-1,3), (4)
-x+ y+z = ±a\
(3, -1,0), (3,0, -1), (-1,3,0).
Eligiendo en todos los segundos miembros el signa mas,
Es faeil de comprobar que todos estos conjuntos de numeros hallaremos facilmente la siguiente soluci6n unica del sistema
satisfacen tambien al sistema inicial. Asf pues, el problema tiene correspond iente:
seis soluciones.
ALGEBRAlResoluciones y Soluciones 119

Sumemos, a continuaci6n, la segunda ecuaci6n con la primera,


ambos miembros de la cuallos multiplicamos previamente por 2.
Como resultado obtendremos:
Realizando todas las posibles combinaciones con los signos
(3)
en los segundos miembros, hallaremos siete soluciones mas:

( - hi +_~ a l + ~ - hi + a l ) ( hi - ci a l - ci hi + a l I Examinemos ahora dos casos.


I) Supongamos aI principio que en la f6rmula (2) se ha elegido
2 ' 2' 2 ~ 2' 2' 2)'
el signa mas. Colocando entonces en (3) x+y de la ecuaci6n
hi + ci - a l + c i h, - a, ) ( - hi - ci al - c i - hi + a, )
(
2' 2' 2 ~ 2' 2' 2 x+y= 7, obtenemos que =2 -14z + 45 = O. Designando las raices
'
de esta ecuaci6n con z?) y z~I), hallaremos que z?) = 9 y
- hi - al 1( hi - C,
( - hi + ci - a l + ci - al - c, hi - a l )
2 ' 2 ' -2~-)l-2-' 2 '~' z~1) = 5. Siendo z = 9, de la ecuaci6n (I) se desprende que
xy = -16. Resolviendo esta ecuaci6n junto con x + y = 7,
( - hi - ci - a l - ci - hi - a i j\
\. 2 ' 2 ' 2 . hallaremos:

Es evidente, que con las ocllo soluciones indicadas se agotan


(I)
XI =
7 +-Jli3
YI
(I) _
-
7 --Jli3
todas las soluciones del sistema. 2 2
(1)_7-~lI3 (I) _ 7 +-J1i3
x2 - Y2 - .
73. Escribamos la tercera ecuaci6n del sistema en la forma 2 2
Si z = 5, entonces, de la ecuaci6n (I) se desprende que xy =
Z2 + xy - z(x + y) = 2. (1)
12. Resolviendo el sistema
Colocando aqui el valor de Z2 de la segunda ecuaci6n y el de
xy = 12,1
z(x+y) de la primera, obtendremos:
x+ y = 7,J
x2 + Y2 + xy - 47 + xy = 2, 0 (x + y )2 = 49.
obtenemos que x~J) = 4, y~l) = 3 Y x41) = 3, y~l) = 4.
De aqui
x+y=±7. (2)
120 PROBLEMAS DE MATEMATICAS ELEMENTALES

2) En el caso de que sea x + y = -7, procediendo anliloga-


mente, obtendremos la ecuaci6n z'2 + 14z + 45 = O. Sus rakes son
( 7+-Jli3
2 '
7- M
2 "
9) (7 - M2 '
7+-Jli3
2 '
9) ,
zf) = -9, z~2) = -5. Resolviendo, a continuaci6n, sucesiva-
mente los dos sistemas de ecuaciones .(4, 3, 5,
) (3, 4, 5), ( -7--Jli3
2 '
-7+M
2
)
'- 9 ,
xy = -16'}
( - 7 +2-Jli3 ' -7 - 2M _9) ,
(4) (_ 4, _ 3, - 5) , (- 3, - 4, - 5) .
x+y=-7, '
y
xy = 12'} (5) Mediante la verificaci6n nos convencemos de que todos estos
x+y=-7. temos de numeros son soluciones del sistema.
del sistema (4) hallaremos que
74. Supongamos que sea (x,y, z) la soluci6n real del sistema.
Examinemos la primera ecuaci6n del sistema. En virtud de la
(2) _ -7 -.Jil3 (2) - 7+ .Jil3 desigualdad (1) pag. xxxx tenemos:
xI - 3 ' YI = 2
y 2z
--2 sl.
1+ z
(2) _ - 7+ .Jil3 (2) - 7 -.Jli3
x2 - 2 ' Y2 = 2 Entonces, de la primera ecuaci6n se desprende que

y del sistema (5) xs z. (1)

X~2) = -4, yf) = -3, Analogamente, de la segunda y tercera ecuaciones del sistema
y obtenemos que
ysx, (2)
zsy. (3)

De nuestros razonamientos se deduce que s6lo pueden ser EI sistema de desigualdades (1) - (3) se satisface solamente
soluciones del sistema iniciallos siguientes ocho temos de nillneros en el caso en que
(x,y,z): X = y = z. (4)
ALGEBRAIResoluciones y Soluciones 121

Colocando z = x en la primera ecuacion, hallamos que Con la condicion (1), la igualdad aqui es posible solamente en
el caso cuando todas las incognitas son iguales a -fi. Puesto
que es faeil de comprobar que el eonjunto de numeros
De (4) hallamos definitivamente que el sistema tiene dos Xl = x2 = ... = xn = -fi satisface al sistema inicial, entonces, el

soluciones reales (0, 0, 0) y (1, 1, 1). sistema tiene solucion positiva y, ademas, solamente una.
Cambiando los signos en los valores de las incognitas,
75. Supongamos que sean Xl' X2' ••• , Xn las soluciones reales del obtendremos una solucion real mas
sistema. Los numerosxk(k= 1, ... , n), por 10 visto, debenin ser
de un mismo signo. Supongamos para mayor certeza, que todos
°
x k > (en caso contrario podriamos cambiar el signa en todas
las ecuaciones del sistema). Asi pues, con estas dos soluciones se agotan todas las solu-
ciones reales.
Demostremos que
76. Sean x,y, z las soluciones del sistema. Expresandox por su
Xk ~-fi, (k=I,2, ... ,n) (1) valor de la primera ecuacion y colocindolo en la segunda ytercera,
hallaremos:
En efecto, en virtud de la desigualdad (1) pagina 20.
(a-b)+(e -b) y +(d -b)z = 0,
)+ +
(a 2 _ b2 (e 2 - b2) Y (d 2 - b2) z = o.

De aqui, por medio de eomputos simples, hallamos:


En virtud de las ecuaciones del sistema, de aqui se desprende
la desigualdad (1 ). (a - b)(a -d) z=- (a-b)(a-e)
y=
Sumando ahora todas las ecuaciones del sistema, obtenemos: (e -b)(e -d)' (d -b)(d -e)"

222 Colocando los val ores hall ados dey y z en la primera igualdad,
Xl +X2 + ... +Xn = - +-+ ... + - . (2) obtenemos:
Xl X2 Xn (a-e)(a-d)
X=
(b-e)(b-d)"
122 PROBLEMAS DE MATEMA TlCAS ELEMENTALES

Por consiguiente, ,-----


II + x
Designando ~ 1- x por t, obtendremos la ecuaci6n
_ (a-b)2(a-c)2(a-d)2 0 /2- 1 =1,0 t 2 -t-l=0.
xyz- (b-c)2(C-d?(d-b)2 > .
1+-15 1--15
77. Si a::j:. 0, entonces x = a no es una raiz de la ecuaci6n. De aqu) II = - -- Y t2 =- - . Puesto que el
Dividiendo ambos miembros de la ecuaci6n entre ~ (a - x)3 , la segundo valor es ne~ativo, entonces?en virtud del acuerdo
sustituimos por la siguiente ecuaci6n equivalente: aceptado sobre las rakes, cuando el exponente m es par debemos
omitir t z. De este modo, en el caso cuando m es par, tenemos:

3/(a + x)2 + 4 = 53/ a+ x. l+x _(I+'\Jsim


~ a-x ')a-x --- - -- I

I-x \. 2 )

~a+x
-
y, por consiguiente,
Haciendo t = 3 1 - - , haIIamos que t,=4, tz=1. De aqu)
\ a-x
xI = ~~ a, x2 = O.
Si a = 0, entonces, la ecuaci6n inicial tiene una sola raizx = O.

78. Por sustituci6n nos convencemos de que x = I no es una Cuando m es impar la ecuaci6n tiene dos rakes:
raiz. Por eso, despues de dividir ambos miembros de la ecuaci6n
entre m~(I_ x)2 esta pasa a una ecuaci6n equivalente: (¥t -I
,
xI 2 = (l±:fi
- )m + 1

79. Hagamos la sustituci6n 12y - 5 = t ~ O. Como resultado


obtendremos
ALGEBRA.lResoluciones y Soluciones 123

De aqu! t + 1 + t + 3 = 14 y t = 5. Resolviendo la ecuaci6n 82. Elevando ambos miembros de la ecuaci6n al cubo,


obtendremos:
~2y-5=5.
hallamosquey= 15.
Deaqui
80. Multiplicando ambos miembros de la ecuaci6n por "Jx + Fx ,
obtendremos 2x + 3 ~ x 2 - 1( 'Jj x-I + 'Jj x + 1) = 2x3 . (I)

(I) yen raz6n de la ecuaci6n inicial


3~
Puesto que x > 0 (siendo x = 0 el segundo miembro de la 2x + 3 'V x-I x :?j 2
"Ir-

= 2x3 . (2)
ecuaci6n inicial pierde el sentido), entonces, la ecuaci6n (I) es
Despues de simples transformaciones obtenemos:
equivalente a la ecuaci6n

2Fx-l=2~. x ~J x 2 - 1 (3 Vi - 2 ~p ~1J2) = O.
Elevando ambos miembros al cuadrado nos convencemos de De aqu! hallamos todos los numeros que pueden servir de
que esta ecuaci6n tiene una sola raiz x = ~~, que satisface rakes de la ecuaci6n inicial. Tenemos directamente:
tambien a la ecuaci6n iniciaL

81. Multiplicando ambos miembros de la ecuaci6n por


-J x + I, hacemos x2 + 8x = t. Entonces, obtendremos la ecuaci6n Resolviendo a continuaci6n la ecuaci6n

Esta ecuaci6n tiene una sola ra!z t = 9. Resolviendo a hallaremos:


continuaci6n la ecuaci6n x2 + 8x - 9 = 0, hallaremos que XI = -9,
x, = I . En virtud de la condici6n adoptada respecto a los val ores
de las rakes, la ecuaci6n inicial queda satisfecha solamente siendo
x = 1.
124 PROBLEMAS DE MATEMATICAS ELEMENTALES

Puesto que se buscan solamente las rakes reales, entonces, En virtud de (4), el segundo miembra de (5) es iguaJ a cera;
por consiguiente, sin embargo, es facil de ver que a> 0, b > 0, c > 0, y por 10
tanto, la expresion entre Haves es positiva. As! pues, la igualdad
x 2 = 1+ 3.fj.
2 (3) queda demostrada. En anaioga forma se establece que Xs es
r-
OeaqUl' x4 = 1/ 1+ -3 -fj
2- ,
J 3 -fj
Xs =- ~ 1+ --2-'
tambien una raiz de laecuacion inicial.

Es facil comprobar, por medio de la sustitucion, que xl' X 2 y 83. Pasando -rx.
al primer miembro y elevando ambos miembros
X,son rakes de la ecuacion inicial. La verificacion directa de los de la ecuacion al cuadrado, obtenemos que
J

val ores de x 4 y Xs causa ciertas dificultades. Por eso, procedemos r- ,.-- - --


de la siguiente manera. -J X . --J X - 4a + 16 = X - 2a.
Hagamos Elevando, acontinuacion, ambos miembros de estaecuacion
2
al cuadrado hallaremos que x = G4 es la unica raiz posible de la
ecuacion. Colocandola en la ecuacion obtenemos que
y
c = ~2X4 ~a2 - 16a+64 = 2~a2 -8a+ 16 -j;2.
y demostremos que
a+b=c. (3) 0, en virtud de que los radicales son positivos,
Puesto que x 4 satisface a la ecuacion (2), tendremos que
(I)
(4)
Siendo a ~ 8 la igualdad (1) se cumple. Por consi~iente, con
y deberemos demostrar que de (4) se desprende (3). N otemos que 2
esta condicion, la rafz de la ecuacion inicial es x = (~ . Siendo
si en la ecuacion (4), en vez de c ponemos a + b, obtendremos una
4 ::; a < 8, la condicion (I) no se cumple, puesto que
identidad. Por consiguiente, par el teorema de Bezout, el palinomio
c 3 _ 3abc _ a 3 _ b 3 examinado respecto aces multiplo del
8 - a 7: 2(a - 4) - a.
binomio c «(a+b). Efectuando la division tendremos que
Siendo 0::; a < 4, la condicion (1) adquiere la forma
c3 _ 3abc _ a 3 _ b3 =
(5)
= [c - (a + b)]{ c2 + c(a + b) + a 2 - ab + b 2 } • 8-a=2(4-a) - a
ALGEBRAIResoluciones y Soluciones 125

y se cumple solamente siendo a = O. Por fin, siendo a < 0 la las soluciones de estos sistemas.
condici6n (1) se transforma en la identidad 8 - a = 2(4 - a)+ Q. Resolviendo el sistema (1 ), hallam os:
Por consiguiente, para Q ~8 Y Q::; 0 la ecuaci6n tiene la (mica
raiz (x + y) 2=.l_ 2xy =.l_.l =o.
5 5 5

x=-. a2
4
Por consiguiente, x + y = 0, y obtenemos dos soluciones del
sistema (1):
Para 0 < a < 8 la ecuaci6n no tiene rakes.
x - -t- t
84. Elevemos ambos miembros de la primera ecuaci6n y I-Fa' x2 = - Fa'
coloquemos en la ecuaci6n obtenida x 2+y de la segunda
ecuaci6n. Como resultado tendremos:
Transformando la primera ecuaci6n del sistema (2) a la forma
36xy -1 = ~ -
J

I; + 64xy + 256(xy) 2
. (x + y) 2 = 15
este ultimo se reduce a los dos sistemas siguientes:

3 ")
x+y~- ~,)
Elevando de nuevo ambos miembros de la ecuaci6n al
cuadrado, obtendremos una ecuaci6n cuadnitica respecto at =
x+ y=-~'J
.ry:
650t 2 - 85t + 2 = O. xy=-
65,
(2')
xy=- 65,
(2")

Resolviendo esta ecuaci6n hallaremos que tl = I~' t2 = ;5 .


Ahora, examinemos dos sistemas de ecuaciones: EI sistema (2') tiene dos soluciones:

x2 + i +4xy =i,1
(1) xy ~ :5 J (2) EI sistema (2' ') tambien tiene dos soluciones:
2 2
x - - -
Por 10 visto, todas las soluciones del sistema inicial entran en 5- -/ 65 ' Y6 = - -165'
126 PROBLEMAS DE MATEMATICAS ELEMENTALES

Como es faeil de eomprobar, solamente los eonjuntos de De aqui se desprende que t = 2 (omitimos la segunda raiz
numeros primero, segundo, tereero y sexto satisfaeen al sistema - ~ ). Resolviendo el sistema
inieial. Por 10 tanto, el sistema inieial tiene solo euatro solueiones.

85. Hagamos
f-
,--
IJ
IX
=2
'
}
x+ XY+: = 9.
v; =u, VY =v.
halJaremos sus dos solueiones:
Entonees, el sistema dado se eseribe en la forma siguiente:
XI = 4, YI = 1;
3
u -v
3 7( 2
=2 u l'-UV
2) 1
'~
que son tambien las solueiones del sistema inieial. Asi pues, el
U -v = 3.J sistema inieial tiene dos solueiones.

Transformaremos la primera eeuaei6n a la forma 87. Hagamos

(U - v f + 3uv = '2 uv. I


/y+l _ 0
- t> .
2 'V x- Y
Deaqui
uv = 18.
Entonees, la primera eeuaeion adquiere la forma
Resolviendo esta ultima ecuaeion junto con la segunda ecuaei6n
del sistema hallaremos que U I - 6, VI = 3; u2 = -3, v 2 = -6. 12 - 3t + 2 = o.
Volviendo al sistema inicial obtendremos sus dos solueiones:
De donde II = I, t2 = 2.
Examinemos ahora dos sistemas de eeuaeiones:
x1=216, Yl=27; x2=-27, Y2= - 216.

86. Por medio de la sustituei6n


primera eeuaeion a la forma
-H = t 2 0 transformemos la ry + 1
IJix-y
-- =1 , f
I
(I) (2)
X + xy + y = 7,)
ALGEBRAiResoluciones y Soluciones 127

EI sistema (1) tiene dos soluciones: x 2 _ y2 = 9}


= 15~
(2)
(- 5, -'-:'3), (3,1). xy
EI sistema (I) tiene dos soluciones reales:
EI sistema (2) tiene tambien dos soluciones:
Xl =5, Yl =3; x2 =-5, Y2 =-3.

EI sistema (2) tiene tambien dos soluciones reales:

Por consiguiente, el sistema inicial tiene cuatro soluciones _!.J98l-9.


Y3 - ~ 2 '
'~
88. Tomando en consideraci6n que !'\}981-9
Y4 = -~ 2
- - =, -1 -y~2
J Ix+y x- - y Sin embargo, no es dificil comprobar que s610 dos de los pares
Vx - y lx- yi
de numeros hallados satisfacen al sistema inicial, a saber:
y multiplicando la primera ecuaci6n por x»y, obtendremos: ( ,.
1~981-91
x2 - i - ~ x 2 - i - 12 = 0 siendo x- y >0 , ,l
(5 3) '_ /-v981 + 9
\V 2 '
_
~ 2 /
y
2 2 Jr--2- -
2
x - y + , x - y -12 =0 siendo x - y < O. As) pues, el sistema inicial tiene dos soluciones reales.

Deaquf 89. Hagamos


! 2
-~ x -12y + I = t.
Entonces, la primera ecuaci6n se puede escribir en la forma
En relaci6n con esto examinemos dos sistemas de ecuaciones:
x2 - i = 16,1
xy = 15,J (1)
128 PROBLEMAS DE MATEMAnCAS ELEMENTALES

De aquf t 1,2 = 4 Y obtenemos


Este sistema tienedos solueiones (5, 3, ~), que, como
i), (-
x2 -12y=15. (1) es f£leil de eomprobar, satisfaeen tambien al sistema inieial.

*
Notando en adelante que y 0, multipliquemos la segunda 90. Liberando la primera eeuaei6n de la irraeionalidad en los
ecuaci6n por ~~ , como resultado adquirini la forma denominadores,obtendremos:

( ~J2
2y
-2(~J ~ +(1 +3y
2y f'J"Y
4XJ = 0.
Deaqui Deaqui

(2)
5
y (~J 5
4 \y 2 4
Despues de elevarla al cuadrado obtenemos la ecuaci6n
Hagamos en la segunda eeuaci6n
3 ( X12 -16
yj (xJ
Y - 12 = 0, I 2
Ii x + xy + 4 = t, (1)

de la que hallamos que despues de 10 eual se puede eseribir en la forma siguiente

(~l =
2 (2 + (_ 56 = O.
3
De aquf I, = 7, 12 = -8. Ya que en (1) (z 0, omitimos la
El segundo valor, por 10 visto, no satisface ala eeuaei6n (2), segunda raiz. Como resultado obtenemos los dos sistemas de
por 10 tanto, podemos limitamos al examen del sistema eeuaciones siguientes:

x 2 -12y = I5,} x=45 Y'}


(2)
~=6. x 2 + xy - 45 = 0;
y
ALGEBRAIResoluciones y Soluciones 129

92. Hagamos
x=-"4 y,
5 } ~x2-6y+l =t;::::O.
(3)
X2 + xy - 45 = 0. Entonces, la primera ecuaci6n se escribira en la fomm

Las soluciones del sistema (2) son: (5,4), {- 5, - 4) Las del (2 -8t+16=0.
sistema (3) son: (15,-12), (-15,12). Las cuatro soluciones
satisfacen tambien al sistema inicial. De aqui t = 4 Ytenemos:

x 2 - 6y -15 = 0. (1)
91. Expresando x por su valor de la segunda ecuaci6n y
colocandolo en la primera, obtendremos Si ahora hacemos en la segunda ecuaci6n x 2y = U Y se toma
en consideraci6n (1), obtendremos la ecuaci6n
2 /1 2 4 1 2 2y + 5
y + 3y --y-- =---+5.
IJ 3 3 3 3 9u 2 -241u-13230 = 0,
de don de Uj = 54, u2 = _ 2~5 .
Haciendo aquf j9y2-34Y-l = t;:::: 0, obtenemos laecuaci6n
Obtenemos dos sistemas de ecuaciones
(2+ 3t - 18 =0.
Deaqui x 2 -6y-15 =0,1
(2) (3)
x 2Y=54,J
Puesto que segun la condici6n t no es negativa, tenemos Eliminando en el sistema (2)x 2 , obtendremos la ecuaci6n
solamente una ecuaci6n
9y 2 -4y-28 = 0. 2/+5y-18=0,
Resolviendo esta ecuaci6njunto con la segunda ecuaci6n del de donde Yj = 2, Y2 = -41·La segunda raiz se omite, puesto
sistema inicial, hallaremos sus dos soluciones: que, en virtud de la ecuaci6n x 2 y = 54, conduce a valores de x
17 14 irreales. Por consiguiente, el sistema (2) tiene dos soluciones
x1=3, y,=2; x2=27' Y2=--i· reales:
130 PROBLEMAS DE MATEMATICAS ElEMENTAlES

Multipliquemos a continuacion la primera ecuacion del sistema


(2) por v y la segunda por u y sumemoslas, como resultado
El sistema (3) se reduce a la ecuacion obtendremos:

54i +135y+245 = 0, u 4 - v 4 + 2u 2v 2 = -7 uv.


2
que no tiene soluciones reales. Asf pues, el sistema inicial tiene En virtud de (4) tenemos:
dos soluciones reales.
4(uv)2 -7uv+ 3 = 0. (5)
93. Hagamos
De aquf
-rx = u ~ 0; fY =v~O. (1) (UV)l = 1,
Entonces el sistema se escribini de la siguiente manera: Examinemos ahora dos sistemas de ecuaciones:

(u 2 - v2 ) v = ~ ,1 (6) (7)
(2)
(u + v )u = 3v-I
2 2

El sistema (2) tiene la solucion evidente Es evidente que toda solucion del sistema (2) diferente de la
(3), se encontrani entre las soluciones de estos sistemas.
Multiplicando la segunda ecuacion del sistema (6) por U, en
u= 0; v = o. (3)
Viliud de la primera ecuacion, hallaremos que if = 2. de aqui,
°
Considerando a continuacion que u -:;: y, por consiguiente,
(en virtud de las ecuaciones) v -:;: 0, multiplicando los primeros y
tomando en consideracion (1), obtenemos.
4~ ~
u =--J2, v =-
segundo miembros de las ecuaciones (2) entre sf, obtendremos: 2
Analogamente hallamos la soluci6n del sistema (7) que satisface
u -v
4 4 3
(4) lacondici6n(I): Vii ~3
2 U=-- - V= -
2' 2
ALGEBRA/Resoluciones y Soiw.:iol1t:s 131

Es facil de comprobar que las dos soluciones satisfacen tarnbien


al sistema (2). Asi pues, el sistema inicial tiene tres soluciones:

(0, o}, Por medio de la comprobaf!on descu12~imos que el sistema


inicial tiene solo una solucion ~ a 2 , a 2 J.,R
94. Elevando ambos miembros de la primera ecuacion al
95. Hagamos
cuadrado,obtendremos:
I 2 2 a
,jx -y =x--.
2
(1)
Fx = U 2 ° Y fY = v 2 0. (1)
2
Entonces, el sistema adquiere la forma
En virtud de la segunda ecuacion tenemos:

I 2 2 3a 2 (2)
,I x + y =--x. (2)
, 2
Elevando ahora ambos miembros de la segunda ecuacion del
sistema inicial al cuadrado, obtendremos: Este sistema se descompone evidentemente en dos sistemas:
U - v= 0,1 (2 ')
.Jr--z-z ,f
4
I 2
'V x + y 2
x - y = -a - x 2 . y
u4 + u 2v 2 + v4 = b2
2
De aquf, en virtud de (1) Y(2) u2 + uv + v2 = a,l
( (2 U)
u 4 + u 2 v 2 + v4 = b2 .J

Resolviendo el sistema (2') hallamos que 3u" = b2 , de donde,


teniendo en cuenta (I ), obtenemos:
i
Abriendo los parentesis, hallaremos que x = a 2 . A con-
,(b'w fb~m
tinuacion, de la ecuacion (1) hallaremos facilmentedos valores u=--- -- v= - - -- . (3)
dey: 3 ' 3
132 PROBLEMAS DE MATEMAnCAS ELEMENTALES

Pasando al sistema (2"), transformemos ambas eeuaeiones en easo eontrarioel sistema (5), y junto con el tarnbien el sistema
de la forma siguiente: (2"), no tiene solueiones que satisfagan la eondiei6n (1).
Resolviendo el sistema (5), obtenemos:
u 2 + v 2== a _ uv, (u 2 + v 2)2 == b 2 + u 2v 2.
De aquf hallamos uv y u2+v2:

2 - b2
uv== -a - - 1
2a ' Como resultado tenemos:
(4)
2
u +v =
2 a 2 +b 2
2a
r
'J u =_d ) 3a2 _b 2
2l V 2a
+ hb 2 _a 2
- ~ 2a
1j ,
No es dificil demostrar que el sistema de eeuaeiones (4) es
equivalente al sistema (2"). De las eeuaeiones (4) obtenemos: V~~[ ~3a22:bi +~3b22:a2}
(u+v\2 =-
3a2 -b
--
2 l
I
) 2 'I Es faeil de ver que, en virtud de la eondiei6n (6), los dos pares
a 2 ~
2 de valores (u, v) no son negativos; en efeeto, ya que a 2 ~ b2 ,
(u-v)2 = 3b -a
2a
.j (5)
entonees 3a2"b 2 e» 3b 2-a2 .
Asf pues, euando se eumple la eondiei6n eomplementaria (6),
el sistema inieial tiene tres solueiones:
Prestemos atenei6n a que el segundo miembro de la primera
eeuaei6n del sistema (4), en virtud de (I), debeni ser positivo;
tambien debera ser positivo el segundo miembro de la segunda
eeuaei6n del sistema (5). De este modo, deberemos suponer
eumplida la eondiei6n

(6)
ALGEBRAlResoluciones y Soluciones 133

As) pues, todos los valores buscados de r se determinan por


la desigualdad
r ~ 1.
97. Si hacemos x y
-+-=U
y x

2 2 2
Y notamos que'£'- + ~ = U - 2, entonces, la expresion dada
se transforma faliime~te en la forma

3u 2 -8u+4. (1)
Si se perturba la condicion (6), solo la primera de elIas.
Si x e y tienen distintos signos, entonces, U < 0 y el trinomio
3. Desigualdades algebraicas (1) es positivo. Si x e y son de signos iguales, entonces, es faeil
verque U ~ 2.
96. Para que el trinomio cuadnitico Puesto que las rakes del trinomio cuadrado (1) son iguales a
1- y 2, para U ~ 2 el trinomio no es negativo. Asi pues, siendo
u < 0 y u ~ 2, el trinomio no es negativo y, por 10 tanto, la
expresion inicial no es negativa para cualesquiera valores de x e
sea positivo para todos los valores dex, es necesario y suficiente y reales y no iguales a cero.
que a> 0 y que el discriminante D del trinomio sea negativo. En
nuestro caso tenemos 98. Notemos que x 2 _ x + 1 > 0 para todos los valores de x,
puesto que el discriminante del trinomio cuadrado es igual a
= r2 -I > o·
a (1)
- 3 < 0 y el cociente de x 2 es positivo; por esta razon, tenemos
D = 4(r _1)2 _4~2 -1)='-8(r -1)< O. (2) derecho a multiplicar ambas desigualdades por el denominador.
Como resultado obtendremos:
Las desigualdades (1) Y (2) se cumplen simultaneamente
cuando r > 1. Sefialemos ademas, que si r = 1 el trinomio - 3x 2 + 3x - 3 < x 2 + ax - 2,
examinado en el problemaes identicamente igual a I.
x 2 + ax - 2 < 2x2 - 2x + 2,
134 PROBLEMAS DE MATEMAnCAS ELEMENTALES

o bien
(2)
4x 2 + (a - 3)x + 1 > 0,
x2-(a+2)x+4>0.
Puesto que siempre ~ a 2b 2e 2d 2 ::::: abed (el signa > para el
caso cuando sea abad < 0), entonces, confrontando (1) Y (2)
La primera desigualdad es justa para todos los val ores de x
llegamos a la demostraci6n de la desigualdad propuesta.
solamente cuando el discriminante del trinomio cuadrado es menor
que cero, es decir, cuando (a - 3 r-
16 < O. Por razon amiloga
100. El sistema dado es equivalente al siguiente:
la segunda desigualdad se cumple con la condici6n de que sea
x2+(x+a)2+2x~l, y=x+a.
La desigualdad
Resolviendo conj untamente las dos desigualdades
2Y
(a - 3)2 -16 < 0 y (a + -16 < 0 respecto aa, obtenemos:

-4<a-3<4, -1<a<7 tiene la (mica soluci6n respecto ax solamente cuando el discrimi-


y nante del trinomio es igual a cero:
-4<a+2<4, -6 < a<2.
De aqui tenemos definitivamente que - 1 < a < 2. es decir,
a 2 -2a-3=0;
99. En virtud de Ia desigualdad (1) pagina 20 tenemos que deaqui
al = 3, a2 = - 1.
a 4 +b 4 ::::: 2a 2b 2, e 4 +d 4 ::::: 2e 2d 2.
Sumando estas desigualdades obtenemos que 1)Sia=3,entonces, x2+4x+4=0 y x=-2, y=1.
2)Cuando a=-1, x 2 =0 y x=O, y=-1.
a 4 +b 4 +e 4 +d 4 ::::: 2(a 2b 2 +e 2d 2 ). (I)
101. Escribamos el sistemade desigualdades dado en la forma
De acuerdo con la desigualdad (3) pagina 20 haciendo
2 2 ' siguiente:
u = a b y v = e 2d 2 , tenemos que
ALGEBRAIResoluciones y Soluciones 135

Ii X 2 2x I< -3,
, I x - I,I < 1.
v + ~ > IX2
-
- 2x I I I 2
'2 '
y<2- l x-l l · Ala segunda de estas desigualdades la satisface el unico numero
entero x = 1, que satisface tambien a la primera. Por consiguiente,
' 2 I
Puesto que siempre I, x - 2x,.:2: Y I x -11.:2: 0, entonces en este caso tenemos una soluci6n mas del problema: X3 = I'Y3
I
= I. As! pues, el sistema de desigualdades se satisface con tres
pares de numeros enteros.
1
- - < y<2.
2
102. En la parte izquierda de la desigualdad hay solamente n
Los unicos numeros enteros de y que satisfacen a esta
°
desigualdad son y I. Por consiguiente, el sistema de desigllal-
dades dado, examinado para los valores enteros de x e y, puede
sumandos y, ademas, los primeros n «1 sumandos son
estrictamente mayores que el ultimo. Por eso

ser conjunto solamente para los valores y == 0 e y == 1. Exami-


1 1 I 1 1
- - + -- + ... + - >n - ==-.
nemos ambos casos. n+1 n+2 2n 2n 2
Primer caso. si y = 0, el sistema de desigllaldades toma la 103. Designemos la parte izquierda de la desigualdad a demostrar
forma por Sm' Es facil ver que en este caso

1 1 1 1
S . -S ==-- - + -- + - - - - - .
mTI m 3m+4 3m+3 3m+2 m+1
A la segunda de estas desigualdades la satisfacen solamente
los numeros enteros: 0, 1 Y2. Por sustituci6n es facil convencerse
°
de que y 2 satisfacen tam bien ala primera desigualdad, pero 1
°
no la satisface. Asi pues, para el caso y = se han hallado dos
Reduciendo los quebrados a un comun denominador, halla-
remos:
2
soluciones: Sm+ I-Sm = (3m + 2 )(3m + 3)(3m + 4 »0.

XI ==0, YI==O: x2==2, Y7==0. Asi pues, Sm+! > Sm' Puesto que
Segundo caso. Siy= I, el sistema de desigualdades inicial
1 1 I
S == - + - +- > 1
conduce al sistema siguiente: I 2 3 4 '
136 PROBLEMAS DE MATEMAnCAS ELEMENTALES

Por 10 tanto Demostremos que


(n -k + l)k 2 n (1)

para n 2 k 2 1. En efecto,
es decir, Sm > 1, 10 que se exigfa demostrar.
nk - k 2 + k - n = k (n - k) - (n - k) = (n - k )(k -1) 2 O.
104. Escribamos una serie de desigualdades evidentes: .
De este modo, ya hemos demostrado que
1 ] 1 ]
-<-=---
22 ]·2 1 2 ' (n.1)2>_n.
n
] ] 1 ]
-< - =--- Notemos que si el numero k es mayor que la unidad y menor
32 2·3 2 3'
que n, en la f6rmula (1), como se deduce de (2), tiene lugaruna
desigualdad estricta. Esto, por 10 visto, trae consigo una desigualdad
1 1 1 1 estricta en la f6rmula (3). Para n > 2 este numero k puede ser
-<---=----
n 2 (n-I)n n-I n' hallado. Por consiguiente, en este caso, es justa la desigualdad
estricta (n!)2 > n".
Sumando estas desigualdades miembro a miembro, obten-
dremos:
106. Es facil comprobar que para la construcci6n de un triangulo
1 1 1 1 n-l con los lados (1, by c, es necesario y suficiente que estos numeros
2+2+"'+2<1- - = --,
2 3 n n n a, b y c satisfagan a las tres desigualdades siguientes:
10 que se exigfa demostrar. a+b -c > a,}
a+c-b > 0,
105. Escribamos ambas partes de la desigualdad a demostrar en (1)
la forma siguiente: b+c-a > O.
Demostrar que el cumplimiento simultaneo de estas desigual-
(nl)2 = (l.n)[2 .(n ... l)] ... [k(n ... k + I)] ... (n.]} dades es equivalente al cumplimiento de la condici6n puesta en
11 fadtores el problema. Admitamos que sea
n" n·n ... n.
'----v---'
n factores
ALGEBRAIResoluciones y Soluciones 137

Si el triangulo puede ser construido, entonces, las desigual-


dades (1) se han cumplido y, por consiguiente, D < O. Con
Puesto que q = - p, la expresion anterior se puede escribir esto, en el senti do directo la confirmacion queda demostrada.
enlaforma En el sentido inverso, si D < 0, entonces,
K = pa 2 + (I - P)62 _ p(1 _ p};2 = c 2p2 +
(a + b- c )(b + c - a)(c + a - b) > 0. (2)
+ (a 2 _ b2 _ c 2 )p + b2,
Demostremos que de aqui se desprenden las tres desigualdades
donde a, bye son constantes y p puede tomar cualesquiera (1). En efecto, supongamos que solo uno de los parentesis de la
parte izquierda de (2) es positi vo y los dos restantes son negativos,
valores.
As! pues, K representa un trinomio cuadrado con relacion a °
por ejemplo a + b - c < y b + c - a < 0. Sumando estas
desigualdades obtenemos que 2b < 0, 10 cual es imposible. De
p. En el caso general, segun sea la magnitud de p, el trinomio K
puede tomar valores de diferentes signos. La desigualdad indicada este modo, la confrrmacion queda demostrada en sentido inverso.
en eI problema es equivalente a que K > 0 para todos los val ores
de p. Para esto, como es conocido, es necesario y suficiente que 107. Transformemos la parte izquierdade la desigualdad de la
el discriminante del trinomio manera siguiente:

(2 b2 4 (x + y)(x + z)x (x + y + z) + y2 Z2 =
D =\a - -c 2)2 -4b 2c 2
=4(x 2 +xy+xz+ YZ){x2 +xy+xz)+ Y"Z2 =
sea negativo (hemos tornado en consideracion que el coeficiente
=4(x 2 +xy+xzy +4yz(x 2 +xy+xz)+ y2 z 2 =
de p2 es igual a c 2 > 0).
El discriminante puede ser presentado en la siguiente forma: = [2~2 + xy +xz)+ yzy.

D= (a 2 _b 2 _c 2 r
_4b 2 c=
2
La expresion obtenida no es negativa para cualesquierax,yy
z reales, 10 que se exigia demostrar.
= (a 2 _ b2 _ c 2 - 2bc )(a 2 - b2 - c 2 + 2bc)=
= [az _(b+C)2] [az _(b-C)2]= 108. Designando la parte izquierda de la desigualdad 'por z,
transformemos z de la siguiente manera:
= (a+b +c )(a- b- c )(a+b -c)(a -b +c)=
= -(a+b+c )(a +b -c )(b +c -a)(c +a -b). z = x 2 + 2xy + 3i + 2x + 6 y + 4 = {x + y + 1f + 2(y + If + 1.
138 PROBLEMAS DE MATEMATlCAS ELEMENTALES

Si X e y son reales, los dos prirneros sumandos no son negativos


y, por 10 tanto, z ~ 1.

109. Puesto que x = I-;y, entonces, ladesigualdad ademostrar


es equivalente a la desigualdad

112. Notemos que la desigualdad dada se reduce a cero cuando


b = c, c = a y a = b. Por esta raz6n, por el teorema de Bezout,
se divide sin resto por las diferencias a - b, a - c y b - c.
que se transforma facilmente a la siguiente desigualdad evidente: Disponiendo los sumandos segun las potencias decrecientes de
a y dividiendo entre a - b, obtendremos:
100i -40y+4 = (lOy-2)2 ~ O.
a3(b 2 _c 2 )+ a2(c 3 _b 3 )+b 3c2 _c 3b2 =
110. Yaque d > 0 Y R ~ r > 0, entonces,
=(a - b)[a 2(b 2 _c 2 )+ ac 3 (c -b)+ bc 2(c -b)).
d 2 R2 - r2 > 0 y 2dr > o.
Saquemos a continuaci6n de la expresi6n entre corchetes el
Por consiguiente, esta desigualdad es equivalente a la desi- factor (b - c) y dividamos el polinomio que queda entre a-c.
gualdad Como resultado obtendremos:
d 2 + R2 _r2 ~ 2dR.
a (b
3 2 _ c2 )+ b (c a )+ c (a
3 2 _ 2 3 2 _ b2 ) =
Reduciendola a la forma (d - Rr ~ r2, obtendremos
i d - R i ~ r 0 -r ~ d - R ~ r. Porconsiguiente,
=-(b - a)(c -b )(c - a)[ac +bc +ab].

R- r ~ d ~ R + r. Puesto que por la condici6n a<b<c y a, bye son de un mismo


signo, la expresi6n a la derecha es negativa.
111. Multiplicando ambas partes de la desigualdad a demostrar
por a+b+c, obtendremos una desigualdad equivalente cuya parte 113. Tenemos:
izquierda es igual a
ALGEBRAiResoluciones y Soluciones 139

dedonde 115. Examinemos tres casos:


1 + ak ~ 2;;;;:.
1) x ~ 0; entonces, x 8 - x 5 + x 2 + 1 > 0, puesto que los
- X
Escribiendo estas desiguaidades para k = 1, 2, ... y
muitipiicandoias miembro a miembro, obtendremos: cuatro primeros sumandos no son neoativos'
I:> ,

2) 0 <x < 1; transformemos el polinomio a la forma

x 8 + (x 2 - x 5 )+ (l-x)= x 8 + x2(I_ x 3)+ (i -x)


Aquf, por 10 visto, todos los sumandos son positivos, por
114. Es suficiente examinar el caso cuando a y b son de un mismo
consiguiente, tambien el polinomio sera mayor que cera;
signa (es decir, son positivos), puesto que en ei caso contrario
3) x ~ 1; escribamos el polinomio en la forma
uno de estos numeros es mayor que 1 y ia desigualdad es evidente.
Tenemos:
x 5 (x 3 - I)+ x(x -1)+ 1.
a 2 + b 2 = (a + b)2 - 2ab = 1- 2ab, Los dos primeras sumandos no son negativos; por consi-
a 4 +b 4 =(1-2ab)2 _2a 2b 2. guiente, tambien en este caso

Pero si a + b = I, entonces, 0 ~ ab ~t, puesto que


116. Tenemos:
a+b12
ab ~ (- - I
2) 4 (1)

(vease la formula (3) en la pagina20). y el ultimo suman do de la suma entre parentesis es igual ax" si n
es par y a nxn~1 si n es impar. Segun la condicion, - 1 < x < I, de
Por consiguiente, donde se desprende que (~k )x 2k < (~k) para todos los vaiores
a4+b4~(1_2.~)2 -2.~=~.
enteros de k. Por eso,
4 16 8
140 PROBLEMAS DE MATEMATICAS ELEMENTALES

donde An es el valor del polinomio (1) para x = ± 1, es decir, Es facil ver, que el numerador del quebrado derecho, para
An=2n. ° < x ::;; 1- no supera a 2 y el denominador es ~ I. Por eso,

117. La desigualdad a demostrar es equivalente a la desigualdad


x
E2(aj2 + a22 + ... + an2) + 4 (2
xl + x22 + ... + Xn2)+
-
As! pues, la desigualdad propuesta es justa para los valores de
± 4E(Xj a l + X2a2 + ... + Xnan) ~ 0.
x =I- °Y que satisfacen la condicion (1). Siendo
que es justa; puesto que la parte izquierda es igual a x = 0 y x I > 1- la parte izquierda de la desigualdad pierde el
r

sentido.

119. Sea, para mayor certeza, x ~ y. Entonces, haciendo


118. La expresion bajo la ra!z cuadrada debeni ser ~ 0, por
2:' = a ::;; 1, obtendremos la siguiente igualdad equivalente:
eso, x
I 1
--::;;X::;;-. (1)
2 2
Para los valores de x que satisfacen la condiciop (1) y no Elevando ambas partes de (1) a la potencia mn, obtenemos la
son iguales acero, ~ 1 _ 4x2 < 1. Por esta razon, si - 2" : ; x < 0, desigualdad
entonces, la desigualdad indicada en el problema se cumple,
puesto que su parte izquierda es negativa.

°
Si < x ::;;
I
2: '
entonces, liberando el numerador de la parte
izquierda de la lITacionalidad, obtendremos
Es facil ver que esta desigualdad es justa, puesto que
0::;; a::;; 1 y n ~ m.

120. Hagamos

I--Jl=~ 4x2 4x
Xn = ) a + ,fa + ... + -J a ,-
x - (1+ 'h -4x2)x (1)
n radicales
ALGEBRAiResoluciones y Soluciones 141

Es facil ver que Xn = ~+ Xn-I (n = 2, 3, ... ), y por numerador y denominador de la parte izquierda de la desigualdad
consiguiente, x~ = a + Xn-I' Notemos a continuaci6n que inicial son diferentes de cero.
Xn > X n _ 1, puesto que al pasar de n» I an el ultimo radical interior Aprovechando a continuaci6n que
Fa se sustituye por un numero mayor ~ a + Fa. En vista de xn ="Ij'2 + xn_l,
esto, x~ < a + xn y, por consiguiente, las magnitudes que nos
interesan satisfacen la desigualdad transformamos la parte izquierda de la desigualdad inicial, de la
manera siguiente:
(2)
2-~Xn_l+2 ~xn_l+2-2
Las raices del trinomio a la izquierda, son iguales a 2 - xn_1 = (Xn-l + 2)- 4 = {Xn_1 + 2 + 2 - xn + 2'
x
(2) _
- -
1+ -.)1+ 4a
---- . Pues t 0 que x n < 2,entonces, I > 41 ' I0 que se eXlgla
. ,
x +2
n
2 demostrar.
Ya que los numeros x n satisfacen la desigualdad (2), entonces,
todos elIos entran en las rakes X(l) y x(2) (vease la pagina 20). 122. Como es conocido, para cualesquiera numeros reales ay b
Por consiguiente, tiene lugar ladesigualdad
1+-J1+4;
xn< 2 (n=2,3, ... ), (3) 2 +b 2
la. bl:-: ; a (vease la f6rmula (l) en la pag. 20).
10 que se exigfa demostrar. Para n = 1 tenemos que x1 = Fa y la 2
desigualdad (3) es evidente. Aprovechando a continuaci6n que el valor absoluto de la suma
no es mayor que la suma de los valores absolutos de los sumandos
121. Designemos la expresi6n con k signos radicales por xk : correspondientes,obtenemos:

~2+ ~2+ ... +~2+-J2 =xk' la lb1 + a2 b2 + ... + anbnl :-: ; ia1bll + la2b2 + ... + lanb n
1 ! :-::;;

af + b]2 ai + bi a~ + b~
Observemos quexk<2. En efecto, sustituyamos bajo el ultimo ~ +- + .. . + =
2 2 2
signo radical interior 2 por 4. Como resultado, todas las rakes
2 2 2 2 2 2
se extraeran sucesivamente y la parte izquierda resultara igual a aJ + a2 + .. . + an + bJ + b2 + ... + bn 1+ 1
= 2 <
- - 2 - =1 .
2. Por 10 tanto, x k<2. De aquf, en particular, se desprende que el
142 PROBLEMAS DE MATEMATICAS ELEMENTALES

10 que se exigia demostrar. 4. Ecuaciones logaritmicas y exponenciales,


identidades y desigualdades
123. Si n = 1, entonces, Xl = I y, por consiguiente, Xl ~ I, as!
que la confirmaci6n es justa. Supongamos que es justa para tOOos 124. Como se ve de la ecuaci6n, esta tiene sentido solamente
los val ores de m, donde 1 ~ m ~ n = 1; demostremos su validez para a> 0, a =F-I y b > 0, b =F- I. Para la resoluci6n de la
para m = n. Si todos los numeros XI' x 2 ' ••. , xn son iguales a la ecuaci6n empleamos la f6rmula de paso a logaritmos con otra
unidad, entonces la confirmaci6n esjusta. Pero si aunque sea base:
uno de estos numeros supera a la unidad, entonces, en virtud de b loaa _ Cloga
la iaualdad X IX 2 ... x n = 1 existira otro men or que la unidad. t:>-
t:> Clogb
Supongamos que la numeraci6n sea tal, que XII < 1, X n-i < 1. De
(vease la f6rmula(2) en la pag. 24). Aqu! c es una base arbitraria
la suposici6n de la inducci6n y la condici6n (c > 0, C =F- I). La eleeci6n de la base C en este problema es
indiferente, s610 haee falta reducirtodos los logaritmos a una
XI X2·· .Xn-2(Xn-IXn ) = 1
misma base. Se puede, por ejemplo, tomar como base eomun a
se desprende que
xl + x2 + ... + xn-2 + Xn-lXn ~ n -1,
°
a, por cuanto a > y a =F- 1. Entonces la ecuaci6n se transforma
a la forma
es decir,
Xl + x2 + ... + xn-2 + xn-lx n + 1 ~ n.
- - og 22 - 2al ogx al og -1
-alogxal
al og 2 b
Puesto que (xn -1)(1 - Xn-l) > 0, entonces,

y par consiguiente,
Xn + xn-l - xnxn-l -1 > ° o despues de la simplificaci6n

(olog2+2 a logb) alogx =3 a log 2 x.


De aqui, la primera soluci6n es:
As! pues,
°logx = 0, es decir, x = 1.

y la confirmaci6n queda demostrada. La segunda soluci6n es:

° logx = 1(a'Og2 +2 a10gb )=laIOg2b2=alOg~J2b2-.


ALGEBRAIResoluciones y Soluciones 143

es decir, Deaquf
=~2b2.
x
125. Pasemos a los logaritmos de base 2: haciendo uso de la
(1_3 10gx) [1- (1+ 310 g x)] = 0
formula (2), pagina 20 obtenemos: y, por consiguiente,

- I- . 1
= (3 IogX )1 = I, xI =3;
2 10gx 210gx-4 210gx-6·
Esta ecuacion es equivalente ala siguiente: (3 10gx)2 = 0, x2 = I;
1
x3 = - .
9
Deaqui
{2\ogX)1 =2, xI =4,
128. Pasemos en la ecuacion a los logaritmos de base 2. Sobre
la base de la formula (2), pag. 24, tendremos
(2 10gx L 3,
= x2 = 8.
1_210gx ') 2 2 4
- ::---=---+ -log x+ log X = 1.
126. Potenciando con relacion a la base 2, obtenemos 1+210gx

9x - 1 + 7 = 4(3 x - I + 1). Multiplicando ambas partes de la ecuacion por el denominador,


Deaqui pasamos todos los terminos a la parte izquierda y la descompo-
nemos en factores.
Por consiguiente, Como resultado obtenemos:

(3 x - I ) I - "
- J, XI =2; (210gx-l)(210g4x+2 210g3i+ 210g 2 x+2 210gx+l)=0.

127. Pasemos en la ecuacion a los logaritmos de base 3. Sobre Parax> 1, el segundo factor, por 10 visto, es positivo y no se
la base de la formula (2), pag. 24, tendremos: reduce a cera. Igualando el primer factor a cera, establecemos
que para X > 1, la ecuacion inicial tiene la (mica raizx = 2 .
1- 3 10gx 31 2 -1
3 + og X - •
1+ logx
144 PROBLEMAS DE MATEMATICAS ELEMENTALES

129. Pasemos en la ecuaci6n a los logaritmos de base


a(a > 0 y a 1:- I, en el-caso eontrario, 1a expresi6n .; log2x Siendo log a :! ~2 + 1) se debe tomar solamente la rafz
no tendria sentido). En virtud de la f6rmula (2), pag xxxx, xl = b2 . Para a < lOb laecWlci6n no tiene rakes.
obtenemos
131. Pasando ell la ecuaci~')n a los logaritmos de base a, la
°log2x + °log2x = O. reducimos a la fonna
° log a Fx ° log~ ° log ax
2
° I
J
lalog~l _
I '
Ia ' " '
. I . i r
IX [
1
De aqui hallamos: Ilog;Z;ar 1+-- - i+ 1- - - =a.
V \ 0 logx ) ~ I a a logx
t
1) ° log 2x = 0, x = no satisface a la eeuaci6n inieial (el
logaritmo de base 1 del numero a no existe); Despues de 1-; ulteriores transformaciones obtenemos:
2) alogax=alog(a2Fx), x = a 2.
Respuesta: x = «. =a.
130. Empleando la igualdad x log b = ~, transformemos la
ecuaci6n inicial en la ecuaci6n equivalent~~ x
Teniendo en cueota que las rakes cuadradas aqui tienen sentido
b log [x(210ga-x)]= 2. aritmetieo, esta ecuaci6n se puede eseribir asi:

De aquf, despues de la poteneiaei6n, obtenemos: "'Iogx + 11·,- !alogx -11= 2a~ alogx . (1)

x 2 - 210ga· x + b 2 = O. Exam incIlIOS ahora dus casos:


I) Supongamosque
Una vez resuelta estaecuaci6n hallaremos:
°logx > 1. (2)
xl,2 = loga± ~log2 a-b 2 .
Entonces, la igualdad (1) adquiere la forma

Para a;;::: lOb y loga 1:- t(b 1)


2+ ambas ralces son 'flogx = a~ alogx ,
positivas, es decir, diferentes de la unidad y, como es faeil de donde
eomprobar, satisfaeen a la eeuaci6n inicial.
ALGEBRAlResoluciones y Soluciones l45

Es facil de ver que la condici6n (2), en este caso, se satisface 12 - I -42 = 0.


solamente cuando a> 1.
2) Supongamos que Sus rakes son: tl = 7 Y'2 = --6. Puesto que t = rx+1 ~ 0,
omitimos t 2 • A la raiz II Ie corresponde el valor x = 48. Por
O<alogx:S; 1. (3) comprobaci6n nos convencemos de que este valor satisface a la
ecuaci6n inicial. Asi pues, la ecuaci6n tiene la (lllica raiz x = 48.
Entonces, la igualdad (1) toma la forma
133. Pasando en la ecuaci6n a los logaritmos de base a, obte-
2 = 2a~alogx. nemos:
Oeaquf
1+ alog{p-x) = 2alog(p-q)-alog4.
a log(x + q) a log(x + q)
Sefialemos que la condici6n (3) se cumple solamente en el
caso en que a ~ 1. Puesto que, de antemano, a =F- 1 (de 10 De aquf, despues de la simplificaci6n Y potenciaci6n,
contrario laecuaci6n inicial perderiael sentido), entonces, tambien obtenemos la ecuaci6n cuadratica
la segunda raiz existe solamente con la condici6n de que a> 1.
Hemos agostado todas las pasibilidades, puesto que los valores (x + q)(p -x) = !(p _q)2.
de x para los cuales a log x :s; 0, por 10 visto, no pueden 4
satisfacer la ecuaci6n (1). Asi pues, para a > 1 la ecuaci6n Las rakes de esta ecuaci6n son:
examinada tiene dos rafces:

Es facil de comprobar que ambas rakes satisfacen a la


Para 0< a < 1, la ecuaci6n no tiene rakes. desigualdad

132. Tenemos p > xl,2 > -q.

log(-Jx + I + 1) = log(x - 40) . y, par consiguiente, tambien a la ecuaci6n inicial.

Suponiendo que sea .[;+1 = t, despues de la potenciaci6n


obtenemos la ecuaci6n
146 PROBLEMAS DE MATEMAnCAS ELEMENTALES

134. Despues de simples transformaciones que emplean la f6rmula una vez resuelta la cual hallaremos:
de paso de un sistema de logaritmos a otro, reducimos la ecuaci6n
dada a la forma de un sistema de logaritmos a otro, reducimos la (IogX)1 =1, xl =10; (logx)2 = 5, x2 = 105.
ecuaci6n dada a la forma
136. Pasando en la ecuaci6n a los logaritmos de base 10,
-Is 1ogx obtenemos:
I
I '- 10gx +3 = --16.
3
I, -v5 (4-X)
log - -
1+ \ 10 = (log logn _1)_1_.
logx logx
Haciendo 010gx = t, despuesdelasimplificaci6nyelevaci6n
al cuadrado, obtenemos la ecuaci6n De aqui, despues de simples transformaciones, obtenemos la
ecuaci6n
t2 + t - 2 = 0.
( 4 -x '\ logn
log, x -- I=log--.
Sus raices son: II = -2, 12 = 1. Ala primera raiz Ie corres- \ 10) 10
t
ponde el valor x = que, como es feici! comprobar, satisface
tambien a la ecuaci6n inicial. A la segunda raiz Ie corresponde el Despues de la potenciaci6n tendremos:
valor x = 15, que no satisface a la ecuaci6n inicial.
")

X4 - 4x + logn = 0,
135. Aprovechando que 0,4 = i Y 6,25 = (~)2 ,reducimos la de donde, I~--

ecuaci6n inicial a la forma xl,2 = ±--J4 -Iogn.

Ahora, un examen no complicado conduce a los siguientes


resultados.
a) Si 0< n < 10 4 y n"* 10 2 , entonces la ecuaci6n tiene dos
rakes diferentes:
Igualando los exponentes, obtenemos la ecuaci6n

log2 X- 610gx + 5 = 0,
ALGEBRAIResoluciones y Soluciones 147

b) si n = 10 3 , entonces, se tiene una sola raiz x = 3 Es facil de ver que toda esta infinita serie de valores de x
(prescindimos dex = 1); para n -10 4 , obtenemos tambien una satisface a la ecuaci6n inicial.
sola raizx = 2.
c) Si porfin 11> 104 , laecuaci6n notiene ralces. 138. De la segunda ecuaci6n hallamos:
2
137. Pasemos en la ecuaci6n a los logaritmos de base 2. Como x+y=---. (I)
x-y
resultado obtendremos la ecuaci6n
Colocando esta expresi6n de x + yen la primera ecuaci6n,
obtendremos:
?
2 10g a

-Iogsen x 2 2 10gsen x
+ 1= °'
o bien
Deaquf
2 2 2 10g a
log sen x =- - - -
2
Puesto que la magnitud a la izquierda es estrictamente positiva Pasando a los logaritmos de base 3, transformemos la ultima
(sen X::F I, de 10 contrario el simbolo sen x log 2 perderia el ecuaci6n a la forma
°
senti do), entonces, 210g a < y, por consiguiente, para a > 1, la
ecuaci6n no tiene rafces. Admitiendo que sea < a < 1, °
obtenemos:
Puesto que 210g3+1::FO,de aquf 31og(x-y)=O y
2
logsen x
,
= :t,/i- -
I 210ga
- - . x- y = 1. Junto con la ecuaci6n (1) esto da el sistema
~ 2
Puesto que 210g sen x < 0, prescindimos del signa mas delante
x+y = 2,1
del radical. Entonces I 2 10ga
x- y = I.]
sen x = 2-'J -~
Resolviendo estas ecuaciones, obtenemos:
y k _J~2Ioga 3 I
X= - y= -- .
X = ( - 1) 'arc sen 2 v 2 + nk (k = 0, ± 1, ... ) . 2' 2
148 PROBLEMAS DEMATEMAnCAS ELEMENTALES

Por comprobaci6n nos convencemos de que el par de numeros Realizando la potenciaci6n de la primera ecuaci6n, obtenemos
hallado es la soluci6n del sistema inicial = 5 7 , de donde
x . 5Y
(2)
139. Mediante la logaritmaci6n de la primera ecuaci6n respecto
a la base c, tendremos: Colocando el valor dex de la ecuaci6n (2) en la segunda ecuaci6n
del sistema (1), obtenemos laecuaci6n 512+y2 _ 2y = 1, que tiene
a Clogx = b Clogy. (1) las rakes
De la segunda ecuaci6n haHamos: YI ;:::: 4, Y2;:::: 3.
Clogx Como resultado obtenemos dos soluciones:
Clogx-Clogy =--.
Cl ogy

Colocando aqui Clog y de la ecuaci6n (1), obtendremos:


141. Efectuando la logaritmaci6n de la primera ecuaci6n con
I-it b relaci6n a la base y, obtenemos la siguiente ecuaci6n cuadnltica
Cl a C b . cl ogx = _
ogx-t; logx = -;;' 0 bIen b
respecto a Y logx:
a
Por potenciaci6n obtenemos:
h2
o bien x = co(b-a). que tiene las rakes
1
Ahora, de la primera ecuaci6n del sistema hallamos: Yl ogx = 2, Ylogx = - .
2
!! _b_
y=x b =c b - a . Si Y log x = 2 , entonces:
140. Haciendo uso de la identidad logaritmica a 0 10gb = b, (1)
escribamos el sistema en la forma siguiente:
En virtud de la identidad a log b = ~II ,de la segunda
6 10gx + y = 7,l ecuaci6n obtendremos:
oga
(I)
xY = 5 12 .1 Ylog(y -3x )=Ylog4,
ALGEBRAIResoluciones y Soluciones 149

dedonde cuyas rakes son:


y-3x = 4. (2) xI=Glogb Y x2 = 1.
Junto con (l), la ecuaci6n obtenida nos da la siguiente ecuaci6n La respuesta definitiva es:
cuadnitica para la determinaci6n de y:
xI=Glogb, YI=b loga; x2 = 1, Y2 = 1.
3i-y+4=0.
143. Pasemos en la primera ecuaci6n a los logaritmos de base x;
Esta ecuaci6n no tiene rakes reales. Si Y log x = t, entonces entonces la ecuaci6n toma la forma
x = -JY e y =x 2.
En este caso, en virtud de (2), obtenemos la ecuaci6n 3 (XlOgy+~1 ~J = 10.
Xlogy
x 2 -3x-4 = O.
Respuesta: x = 4, YY = 16. Haciendo aqui x log y = t, obtenemos la ecuaci6n

142. Realizando la logaritmaci6n de la primera ecuaci6n respecto 3t 2 - lOt + 3 = 0,


a la base a, hallaremos:
cuyas ralces son: tl = 3 Y t2 = t·
En el primer caso
x log y = 3, y = x 3 y, en virtud de la segunda ecuaci6n del
sistema inicial, x4 = 81. Puesto que x > 0 e y > 0, en este caso
Pasemos en la segunda ecuaci6n a los logaritmos de base a. obtenemos una sola soluci6n:
Entonces
2 GIogx=--- Glogy Glogb 2GI xI =3, YI =27.
=- ogy.
Glogb Glog~ Suponiendo a continuaci6n x log y = t, hallamos una solucion
mas:
De aqui x = ~. Colocando y = ~ en (1), obtenemos la x2=27, Y2=3.
ecuaci6n
144. Pasemos en cada una de las ecuaciones del sistema a los
logaritmos de base 2. Como resultado obtendremos el sistema
150 PROBLEMAS DE MATEMAnCAS ELEMENTALES

1.
x = y2. (2)

(1) Empleando (2), de la primera ecuacion hallaremos que


y = V4. Por consiguiente,
3 2
x=25, y=25.
Puesto que x 1=1 (de 10 contrario el primer miembro de la
primera ecuacion del sistema inicial no tendria senti do) 2 10 g x 1= 0 146. Transformemos el sistema, pasando en la primera ecuac ion
Y el sistema (1) se puede escribir en la forma a los logaritmos de base 2, en la segunda, a los de base 3 y en fa
tercenl, a los de base 4. Como resultado obtendremos:
210gx+ 210gy=210g 12,1
2 12 12 2 "1
logx+- logy+ - logz= log4, I
210g{x + y) = 3.J 221
~ 13 13 3 I
.Jlogy+_ logz +- logx= log9, r'
Despues de la potenciacion, obtenemos: 2 2
4 14 14 4
xy = 12, x +Y = 8, \ogz+- logx+- logy= logl6.J
2 2
dedonde
XI = 6, YI = 2; x2 = 2, Y2 = 6. Despues de la potenciacion, obtenemos el sistema
145. Pasando en cada una de las ecuaciones del sistema a los
logaritmos de base 2, obtendremos: x~ =4,l
y-Jxz =9,
x 210gy = Y-JY(1-2 10gx }1 (1)
,-- J
zjry = 16.
(1)

2 2\ogx =3 210gy.j
Multiplicando las ecuaciones del sistema (I) miembro a
De la segunda ecuacion del sistema (1) hallamos Xl = y, de miembro hallaremos:
donde
ALGEBRAIResoluciones y Soluciones 151

Puesto que x > 0, y > 0, Z > 0, entonces, 148. Dividiendo ambos miembros de la ecuaci6n entre 4\
hallaremos:
xyz = 24. (2)

Elevando la primera ecuaci6n del sistema (1) al cuadrado y


1 -l-(3JX
4
.- 1(3\x
= - 13 - -.
13 \4) 2
I
1

empleando (2), obtendremos:


Deaquf
16 2
X= - = - .
24 3

Amllogamente hallamos que y = 21; Yz = 332 . Por compro- y, por consiguiente,


baci6n nos convencemos de que los tres numeros hallados son 3
X=-.
la soluci6n del sistema. 2
149. Colocando el valor de y de la segunda ecuaci6n en la primera.,
147. Pasando en la primera ecuaci6n a los logaritmos de base 2 obtendremos:
y realizando a continuaci6n la potenciaci6n, obtendremos:

i -xy=4. (1) De aquf, 0 x = 1,0 bien


La ecuaci6n (1), junto con la segunda ecuaci6n del sistema 1 2
x+ -- =-2x+-
inicial, da el sistema x 2 x2
y, por consiguiente,
x2 + i = 25,1
X=
1
VS.
i -xy = 4.] (2)

Respuesta:
Este sistema tiene dos soluciones que satisfacen a las xl =Yl =1 ,
condicionesy> x,y > 0, a saber:
1
Yl - -i2' Y2 = 4. 150. Haciendo aX =u y aY = v, escribimos el sistema en la
forma siguiente:
152 PROBLEMAS DE MATEMATICAS ELEMENTALES

u1 + V 1 = 2b,} Ul= -fi{-Jb+c--Jb-c),


2
UV=C.

vl = -fi (-Jb + c + -Jb - c).


De estas dos ecuaciones se desprende: 2
Hemos hallado dos soluciones del sistema (1), ademas, al
(u + vf = 2(b + c) , (u - vf = 2(b - c) . cumplir la condicion (2) todos los valores de las incognitas, por
Puestoque los valores buscados de U y v deben ser positivos, 10 visto, son positivos. Las dos soluciones correspondientes al
la primera ecuaci6n se reduce la ecuaci6n sistema inicial son:

(1) xl=Glogu1 , Yl=Gl ogv1 ; x2=Glogu1 , Y2=Glogv2'

La segunda ecuaci6n demuestra que la solubilidad del sistema Ahora podemos confmnar que para que el sistema sea soluble
requiere, ademas de que sean positivos los numeros by c, el es necesario y suficiente que b > 0, c > 0 y b? c. Al cumplir
cumplimiento de la desigualdad estas condiciones el sistema tiene dos soluciones.

b? c. (2) 151. Multiplicando ambas ecuaciones entre sf, obtendremos:


Al mismo tiempo,
U - v = ±-J2(b - c). (3)
De aqui, en virtud de que x e Y son positivos, se desprende
Resolviendo conjuntamente las ecuaciones (1) Y(2), en el caso
que 0 bien xy = 1,0 xY -:F1, yentonces
del signa mas, obtendremos:
x+ y=2n. (I)
Ul = -fi (J b+ c + -J b - c ), Examinemos al principio el segundo caso. La primera ecuacion
2
del sistema inicial toma la forma.x2 n)l', de donde
vl= -fi(-Jb+c--Jb-c).
2 (2)
En el caso del signa menos, obtenemos:
ALGEsRAi'Resoluciones y Soluciones 153

Colocando este valor de yen la ecuaci6n (1), obtenemos que y por consiguiente, en virtud de la primera ecuaci6n

x 2 +x-2n = o. 324 = 2 x - y ·81.


Esta ecuaci6n tiene la unica raa positiva Deaqui 22 = 2x - y , esdecir,

x- y = 2. (1)
(3)
Resolviendo la ecuaci6n (1 )juntocon la primera ecuaci6n del
El valor correspondiente dey 10 hallamos haciendo uso de (2): sistema inicial, obtenemos dos sistemas:

Yl =! (J8n + 1 - 1 f (4) X- y =2} X- y =2}


= 3~ = -3~
4 (2) (3)
3x + y 3x + y
En el segundo caso, cuando xy = 1, y = -;-, y la primera
ecuaci6n del sistema inicial adquiere la forma Soluci6n del sistema (2):
5 3
xl =4' y\ =- - .
4
En virtud de que x y n son positivos, esta igualdad puede ser
Soluci6n del sistema (3):
valida solamente en el caso en quex = 1. De este modo hallam os
9
una soluci6n mas: X z = 1'Yz = 1. Y2 = --.
4
152. Transformamos el sistema a la forma Por comprobaci6n nos convencemos de que los dos pares de
numeros satisfacen al sistema inicial.

153. Hagamos ;; = u. Si u = 1, es decir, p = q, entonces,


cualquier par de numeros iguales y positivos satisfacen al sistema.
Por esta raz6n, consideraremos que U:l: 1. De la segunda
De la segunda ecuaci6n hallamos: ecuaci6n obtenemos que x = yX. Realizando la logaritmaci6n
de la primera ecuaci6n y empleando esta igualdad, tendremos:
324 = 2x - Y (3x+ y)2(X- Y ),
154 PROBLEMAS DE MATEMAnCAS ELEMENTALES

y log y (a - yX-I)= 0. Deaqui,


2=Glog(c -b )+Glog(c + b).

Puesto que y > 0, entonces, 0 bien logy =0,0 bien a = yX-I. 1


2= + - :----
En el primer caso obtenemos que x] = 1, y] = 1. En el segundo c-b log a c+b log a
caso,obtendremos:
-.!L -.L y, por consiguiente,
x2 = a a-I, Y2 = a a-I.
Ambos pares de numeros satisfacen tambien al sistema inicial.

154. Efectuando la logaritmacion de am bas ecuaciones, 156. Utilizando Ja fonnula nlogm = -;;--1
1 ·obtendremos f:kil-
ogn
obtendremos el sistema mente que
ylogx = xJogy,l
(1) b2 - k toga = 2kb toga
xlogp = yJogq.J

. I I· , log q
de I que determmamos yx = logp =a;
guiente.
a re actOn por conSl-
In (' .
b·-I toga-a l ' 10gb )20) = I 11 (
2kb
1)
toga-kGtogb 2

x=ay. (2) k=O k=O 2

Si p = q, el sistema tiene un numero infmito de soJuciones del


tipo x = y =a, donde a es cualquier valor mayor que cero. Si
p -j; q, entonces colocando el valor de x de la formula (2) en Ja
primera ecuaci6n del sistema (1 ), hallaremos:
-.!L _ 1_
x=aa-I, y=aa -l
Por consiguiente, para la condici6n p -j; q el sistema tiene
unasoluci6n.
- - _.._ - -
155. Realizando la logaritmaci6n de las dos partes de la igualdad n
a2=c2 "b 2 ,obtenemos: *) EI sfmbolo I
k=O
ak significa la suma ao + at + a2 + ... + an-
ALGEBRA/Rcsoluciones y Soluciones 155

157. De aqui se desprende que

b log 10ga
b
,vI b NI C
og~= ogt;' (1)
a b loga =(a"IOgb) blog h loga =bb log h loga = bloga .

puesto que el factor : :OgC 1= O. Realizando la potenciacion de


158. Tenemos que
la igualdad (1) obtenem~gs~
a aq ... ~aq
n{n-l)
_ _ . n-l ) _ n -2- b C
C - a}a2 ... an - - a q . (2)
a b
Empleando la formula de pasode un sistema de logaritmos a Asi pues b es el valor medio proporcional entre a y c. Rea-
otro, obtenemos: Iizando a continuacion la logaritmacion de la igualdad (2) respecto
a cualquier base Ny efectuando los computos en orden inverso,
ci b _ alog~ _ A
og - b - ( I) . conc1uiremos la demostracion de la afirmacion propuesta.
loge n+ n ~- al ogq
Pero 160. Se debe considerar N 1= I, de 10 contrario, el quebrado en
a logq = b logq = alogb = A la parte derecha de la identidad se hace indetenninado. Dividiendo
b loga qlogb B la identidad a demostrar entre a log N blog N Clog N la
sustituimos por la siguiente identidad equivalente:
Por eso,
Cl og b = 2AB
I 1 I 1
+ + =---
2nB + n(n -I)A' alogN b logN clogN abclogN'

159. Utilizando la igualdad a 10gb = ~I} ,transformemos la Pasando aqui a los logaritmos de base N, obtendremos:
o"a
fonnula dada de la manera siguiente: 0
NI oga+ N logb+ N logc= N logabe.
N __ 1 _ __
1_ NI b ,
loge N loga N logb _ oga. "Iogc
Puesto que es evidente que la ultima identidad tiene lugar, el
N log a = 1 1- = N log fb N Jog a .
problema queda resuelto.
NJogb NJogc
156 PROBLEMAS DE MATEMAnCAS ELEMENTALES

161. Tenemos:
Gl ogx 1
-- 0< X < r210g2.
Gblogx
163. Ya que x > 0, la desigualdad dada es equivalente a la
10 que se exigia demostrar.
desigualdad
'd 'd I .. b Cloga
162. Emp Ieand 0 Ia 1 entI ad ogantmlca loga = Clo b'
transformemos la parte izquierda de la desigualdad dada ~e la Pero a> 1, por eso, realizando la logaritmaci6n de la ultima
manera siguiente: desigualdad respecto a la base a (esta operaci6n conduce tambien
a una desigualdad equivalente), obtendremos:

Glog2 x> 2.

De aqui hallaremos defmitivamente:


o bien G log x > .f2, y, por consiguiente, x > a J2 ;
°
o bien G log x < -.f2 yentonces < x < a --Ii .

Entonces, la desigualdad dad1!- adquiere la forma: 164. Por el senti do del problema x > 0, por eso, la desigualdad
dada es equivalente a la desigualdad
_ :IOgx > 1.
210g2 Glogx(x+l)< Glog (2x+6).

t
Pu~sto que 2 > 1 y > 1 Y por la propiedad de los logarit- Puesto que a> 1, entonces x (x + 1) < 2x + 6, 0 bien
mos ~ log 2 > 0, entonces, la desigualdad anterior es equivalente
a la desigualdad x 2 -x-6 < 0.
Resolviendo esta desigualdad cuadnitica con la condici6n de
De aqui, observando ademas que por el sentido del problema que x > 0, obtenemos que
x> 0, obtendremos definitivamente:
l)<x<3.
ALGEBRAIResoluciones y Soluciones 157

165. La desigualdad dada es equivalente a la siguiente:

o< x 2 - 5x + 6 < 1.
Puesto que x 2 - 5x + 6 = (x - 2 ){x - 3), la desigualdad Puesto que el numerador de la ultima expresi6n es positiva
0< x 2 - 5x + 6 esjusta para

x<2 [ en
2 2 2
efecto, 1+ log x- log x =(
2
-"2 2 +"4'
log x)
1 3
1 la
y
x>3. desigualdad se reduce a 10 siguiente:

Resolviendo a continuaci6n la desigualdad x 2 - 5x + 6 < I, 210gx (2Iogx-I» O.


hallamos que se cumple cuando
Esta ultima desigualdad se cumple siendo x > 2 y siendo
O<x<1.
5--15
-
5+-J5
-<x< - --.
2 2 167. Por el sentido del problema x > 0 y, por 10 tanto, la
-- 5- -J5 5 + -J5 ,., desigualdad dada es equivalente a la desigualdad
Puesto que -J 5 > 2, entonces - -- < 2 Y - -- > -'.
"2 2
Por eso, la desigualdad inicial tiene IJgar cuando 2 x3-~log x - 2 logx > I.

Realicem_os la logaritmaci6n de esta desigualdad respecto a la


5--15
---<x<2 y
5+-J5
3<x<---. base 2 y hagamos y=2 10g x; obtendremos una desigualdad
2 2 equivalente
166. Reduciendo la parte izquierda a un comun denominador
hallamos que
que, despues de descomponerel trinomio cuadnitico en factores,
se puede escribir en la forma
y, por consiguiente,
y (1 - y)(3 + y) > O.
158 PROBLEMAS DE MATEMATICAS ELEMENTALES

Esta desigllaldad puede ser cumplida cuando, y s610 cuando, EI numero Z = Y + ~ tiene el mismo signo que el numero y y
o bien los tres factores son positivos, 0 bien uno de ellos es posi- !z! ~ 2 para todos los valores de y (vease (2), pag. 20). Por
tivo y los otros dos son negativos. En el primer caso, es decir, eso, si z> 0, entonces, la desigualdad Z ~ -2 cos a puede ser
cuando cumplida solamente en el caso en que z = 2 (y = 1) y
y > 0, 1- y > 0, 3 + y > 0, cos a = -1, es decir, si en la desigualdad in icial x = 2 Y

hallamos que ° < y < 1 y, por consiguiente,


a = (2k + l)n (k = 0, ± 1, ±2, ... ). Para estos valores tiene lugar
el signo de igualdad.
Pero si z < 0, es decir,y < 0, entonces z ~ -2 Y la desigualdad
1 < x < 2. (1)
(I) se cumple para todos los val ores de a, de donde se desprende
que la desigualdad inicial se cumple, ademas de los casos de los
EI segundo caso se divide en tres subcasos, con la particulari-
dad de que se obtiene un sistema de desigualdades no contra- °
valores hallados, siendo <x < 1 y para todos los valores reales
de a.
dictorio solamente en uno de los subcasos, cuando

y < 0, 1 - Y > 0, 3 + Y < 0. 169. La desigualdad inicial es equivalente a la siguiente:

De aqu! y < -3 y, por 10 tanto, 0<410g(~2 - 5)< 1.

° 1
< x <-.
8
(2) I
De donde 1 < x-? - 5 < 4 0 6 < x-? < 9 0 \/6 < IiXiI < 3.
As! pues, la desigualdad inicial tiene lugar cuando, y s610 Respuesta: -16 < x < 3 y - 3<x < --16.
cuando, 0 bien

obien
° 8
I
< x < -.
5. Combinatoria y binomio de Newton
1 < x < 2.
170. Escribiendo por separado las relaciones entre el primer
168. Haciendo 2 10gx = y y notando que x log2 = 21~g; =~, miembro de la proporci6n y el segundo y entre el segundo y el
escribamos la desigllaldad dada en la forma tercero, despues de las simplificaciones correspondientes,
1 obtendremos:
y+ -- + 2cosa ~ O.
Y
ALGEBRA/Rcsoluciones y Soluciones 159

(n + 1)! . (n + I)! _ n - m+ 1 Eseribiendo esta misma suma en forma del polinomio


(m + I)! (n - m)!' m! (n - m+ I)! - m + 1
(n + 1) ! . (n + I)! n - m+ 2
y abriendo los parente sis en el segundo miembro de la igualdad
m! (n - m+ I)! . (m -I)! (n - m + 2)! m
(1), obtendremos: si m < k, entonces
En virtud de la eondiei6n del problema obtenemos dos
eeuaelOnes:
~--=- m + 1 = 1 n- m+25 Si m 2 k, entonces
m+I ' m 3
Resolviendolas coqjuntamente obtendremos que m = 3, n = 6. an'
n+
= ( m+1
I) .
173. De la eondiei6n del problema se desprende:
171. Tenemos:

(I +X2 _x 3y= 1+(; )(x 2_x 3)+(~)(X2 _X 3)2 +


+ (;)(X2 _x 3Y+(~)(X2 _X 3)4 +

r
+ (~)(X2 _ X3)5 + ... + (x 2- x3
Una vez resuelta esta eeuaci6n respecto an haJJaremos que
n=I1.
El termino COmlll1 del desarrollo
Examinando los sumandos del segundo miembro, es faeil ver ,- 1 ,II
que.0 figura solamente en el euarto y quinto terminos. Utilizando (
X--JX +;4 j
esto haJJamos faeilmente el coefieiente de X8. El es igual a
3(~)+ (~). Se puede eseribir en la forma
172. Los sumandos de la suma dada forman una progresi6n con Cm(II)x%(i l-m)-4m.
el denominador 1+x. Por eso
(l+x)k +(l+X)k+l +... +(l+xY =
i
Por la eondici6n del problema (11 - m) - 4m = 0, de donde
m = 3. Por consiguiente, el termino buseado es igual a C/ 11).
(l+xy+l-(l+x)k (1)
- - ~-- - ~~

x
160 PROBLEMAS DE MATEMATlCAS ELEMENTALES

174. Hagamos x + ~ = u, entonces Las partes izquierda y derecha de la desigualdad (1) son
numeros no enteros y la diferencia entre ellas es igual a la unidad.
(l+x+~6 J 10
=(l+u)
\0 10 10 2
=1+C )u+C)u +···+Co)u ,
10 10 Por eso, existe solamente un numero entero k que satisface la
desigualdad (1). Captando que 1,72 < .fj < 1, 73, mediante el
calculo directo establecemos que
donde
k k 64,64> k > 63, 135.
k
u =( X+~6 ) =X
k
+C~
k k-2
.6+···+C~
k k-25
·6
5 6
+·'·+7·(1)
Por consiguiente, k= 64.

Para el sumando que no contienex en laexpresi6n (1) debe


satisfacerse la condici6n k - 2s = 0; por consiguiente, este 176. El termino com un del desarrollo es Tk+l =Ck(n)cl.
sumando sera igual a Cs (2s). 6 s . Reuniendotodos los tenninos Si Tk = Tk+l' entonces Ck_l(n)a k 0 bien
semejantes hallaremos que el sumando que no contiene x en la
expresi6n inicial sera igual a n!ak - 1 n!a k
(k-l)!(n-k+i)! - k!(n-k)!'
0). (~). 64+
1+ (I~). (? ).6 + (If)· (i)· 6 2+ (I~). (~). 63+ (18 de don de
+ (lg). (ISO ).65. k= n+l
1+1-·
a
175. Las desigualdades Tk+1 > Tk Y Tk+l > Tk+2' despuesde Obtenemos la condici6n buscada: 1 +.;
debe ser el divisor
las simplificaciones correspondientes, toman las fonnas del numero n + 1.
.fj 1 1 .fj Si ahora Tk = Tk+1 = Tk+2' entonces, esto es equivalente a
- > -- - - - - > - -. las igualdades
k 101-k' 100 -k k + I
a
Una vezresuelta cada unade estas desigualdades con relaci6n
ak,obtendremos:
(n-k+l)(n-k) = k{n-k) ="k(k+l)
obien
101.fj k .1 00.fj-l k k+ 1
- - > >-~-- (1) - ---==a - -==a
.J3 + 1 .fj + 1 n-k+l ' n-k '
ALGEBRA/Resoluciones y Soluciones 161

10 que conduce a la condici6n n + 1 = 0, que es imposible de yes equivalente a la igualdad


cumplir.
n+l(n)
1+ (n+1 )+-- n+l(n) n+l(n)
I +-- 2 + ... + - - k + ... +
177. En el desarrolloentranin: n terminos tipo 2 3 k+l
x~{i=I,2, ... ,n),n(n-l) terminos del tipo xjxii,j= + n + 1(n-I
n) + 1 = 2n+ I .
n
1,2, ... , n, i"* j) yporfm C3 (n) terminosdel tipo XiXjXb donde
i,j y k son numeros diferentes. Asf pues, el numero de terminos Puesto que
diferentes no semejantes entre sf es igua\ a
n+l(n)_n+l n! _ (n+l)! _(n+l)
k+l k - k+lk!{n-k)!-(k+l)!(n-k)!- h I '
n+n(n-I)+ n(n-I)(n-2) = n(n+ 1)(n+2).
6 6 el primer miembro de la ultima igualdad es igual a
178. Los divisores del numero q son, por 10 visto, los numeros
PI, P2'·· ., P k Ytodos los productos posibles por 2, por 3, etc. 1+ (ntl)+ (n;I)+ ... + (k~i)+ ... + (n~I)+ 1 = {I + l)n+1 = 2n+l,
La cantidad de tales divisores es igual a
con 10 cual el problema queda resuelto.

180. El termino comun del primer miembro de la igualdad puede


EI hecho de que todos los divisores obtenidos no son iguales ser transformado de la manera siguiente:
entre sf y que no existen otros divisores se desprende la unicidad
de la representaci6n de un numero en forma del producto de
numeros simples.

179. La igualdad a demostrartiene la forma

Por eso, el primer miembro de la igualdad puede ser presentado


en la forma siguiente:
162 PROBLEMAS DE MATEMATICAS ELEMENTALES

nx l(n01 Xl-xt- + (nj-l )x(l_x)n-Z + ... + (~=l)xn-l J=


1

=nx[x+l-X]n-l =nx.
=(2n -1)(2n -3) ... 3·1.
181. Toda division de la baraja, sefialada en la condicion, es
En fonna reducida esta multiplicacion a veces se de nota con
equivalente a sacar 16 no ases de entre 32 no ases y dos ases de
el sfmbolo (2n -1)!!
entre cuatro ases. La primera extraccion puede realizarse por
Se puede obtener el mismo resultado razonando de distinta
C I6 (32) metodos y la segunda por C/4). Puesto que la sacada
manera. Designemos por kn el numero de divisiones para el caso
de 16 no ases puede ser combil1ada con cualquier sacada de
cuando la cantidad de elementos es igual a 2n. Examinemos 2n
dos ases, entonces, el numero total de metodos de la division
elementos. Por cuanto el orden de disposicion de los pares no
indicada de la baraja es igual a C1i32) Cz( 4).
tiene importancia, se puede considerar como primer par a aquel
en el que figura el primer elemento. Los pares que contienen el
182. La cantidad buscada de nltmeros es igual a la cantidad de
primer elemento pueden ser formados por 2n -1 metodos. Una
variaciones que se pueden formar de 10 cifras en series de 5, es
vez elegido el primer par, los 2 (n -1) elementos restantes pueden
decires igual a 10·9·8·7·6 = 30240.
ser divididos en pares por k n _ , metodos. Por eso,
kn = (2n -1 )kn _ l • Con ayuda de esta relacion es faeil hallar que
183. Enumeremos ciertos n sitios y formemos la division lIenando
sucesivamente cada uno de los sitios indicados con un par de kn = (2n - 1}(2n - 3) ... 5.3.1.
elementos.
AI primer sitio el par puede ser elegido por C2(2n) metodos; 184. De la cantidad total de 11 ! pennutaciones debemos substraer
una vez elegido el primer par, el segundo puede ser elegido por aquellas en las que los elementos a y b son vecinos. Para formar
C2 (2n-2) metodos, el tercero por C2 (2n-4) etc. Como tal permutacion es neeesario tomar eierta permutaeion con los
resultado obtendremos C/2n) C2 (2n - 2) Cz (2n - 4) ... C z(2) n-2 elementos restantes (en total son (n-2)!) y unirla a la
divisiones entre las cuales, sin embargo, figuranin todas las pennutaeion elegida con los elementos ay b de modo que resulten
divisiones que difieren por el orden de disposicion de los pares. uno al lado del otro. Esto, por 10 visto, se puede haeer por 2 (n - 1)
Por consiguiente, la cantidad de divisiones que nos interesan sera metodos (el factor 2 esta relaeionado aquf con que a y b pueden
iguala ser eambiados de sitio). Asf pues el numero de permutaciones en
ALGEBRAIResoluciones y Soluciones 163

las que ay b son vecinos es igual a 2(n - 2)! (n -1) y el numero 186. Primera resoluci6n. Supongamos que en la recta superior
de permutaciones que nos interesa es igual a se encuentran n puntos y en la inferior m puntos (fig.1). Dividamos
todos los segmentos de uni6n en haces de segmentos, con la
n!-2(n - I)!= (n -1)!(n - 2). particularidad de que en uno de los haces reunimos todos los
segmentos que unen el punto
185. Si entre los 5 billetes sacados resultaronjusto dos billetes
fijado de la recta inferior (por n puntas
premiados, entonces los otros tres seran no premiados. De 8
billetes premiados dos se pueden elegir por C/8) metodos, de
ejemplo, el A) con todos los ~~::::;;;:;=~A~=:::=~ __
puntos de la recta superior.
50 - 8 = 42 billetes no premiados tres se pueden elegir porC(42)
.' Esta claro que la cantidad de
metodos. Cada metoda de elecci6n de dos billetes premiados
tales haces es igual amy que
puede combinarse can cualquierade los metodos de elecci6n de
la cantidad de puntos de
tres no premiados. Por eso, el numero total de metodos es igual a
intersecci6n de los segmentos
pertenecientes a cualesquiera v
(~) . (j2)= 8 · 7. 42 · 41 · 40 =326240. dos haces es la misma para
m puntas
Fig 1
1·2 1· 2·3
cualquier par de haces. Si
La cantidad de metodos de elecci6n de 5 billetes entre los cuales designamos esta cantidad por kn' entonces la cantidad total de
por 10 menos dos seran premiados es igual a la sum a de las puntos de intersecci6n de
cantidades de metodos por los cuales se sacanjusto dos premiados,
todos los segll1entos sera igual al producto de kn por la cantidad
justo tres premiados:justo cuatro premiados y justo cinco billetes
de cOll1binaciones de los haces en series de dos, es decir,
premiados. Por consiguiente, esta cantidad es igual a
m(m-l)
k n C 2 (m) = k n .
2
Para calcular el numero kn dividamos todos los segmentos que
8·7 42 ·41· 40 8·7·6 42·41 unen los n puntos de la recta superior con los dos puntosA y B de
= - - . - - - - + -- . - - +
1·2 1·2·3 1·2·3 1·2 la recta inferior en haces de segmentos, reuniendo en un haz dos
8·7·6·5 42 8·7·6·5·4 segll1entos que unen el punto fijado de la recta superior (por
+ - - - -.-+ =
1·2·3·4 1 1·2·3·4 ·5 ejell1plo, el C) con los puntas A y B. EI numero de tales haces es
=326240 +48216 + 2940+ 56 =377 452. igual any el nUll1ero de puntos de intersecci6n de los segmentos
164 PROBLEMAS DE MATEMATICAS ELEMENTALES

pertenecientes ados haces es igual a la unidad (punto de 188. Puesto que en el alfabeto dado a cualquier signa por separa-
intersecci6n de las diagonales del trapecio ABCD). Por eso do (punto 0 raya) y a cualquier par de signos Ie corresponde
cierta letra, el numero de procedimientos por los que se puede
kn = C2 (n) = n(n -1) . leer la cadena continua de x signos no depende de la construcci6n
2
con creta de esta cadena yes igual al numero de todas las divisiones
Por consiguiente, el numero total de puntos de intersecci6n de posibles que forman la cadena de signos en grupos de uno 0 dos
todos los segmentos que unen los n puntos de la recta superior con signos vecinos. Designemos este numero por Pn .
los m puntos de la recta inferior es igual a Distribuyamos todos los metodos posibles en que se puede
leer la cadena dada compuesta de n signos en dos categorias.
n{n -1) 111{m -1) A la primera categoria referimos los metodos en los que el
2 2 primer signa de la cadena se lee como una letra independiente.
Segunda resoluci6n. Cada punto de intersecci6n de los seg- EI numero de metodos de la primera categorfa es igual al numero
mentos puede ser obtenido eligiendo dos puntos en la primera de metodos en que se puede leer la cadena compuesta de n - 1
recta, cosa que se puede hacer por Cim) procedimientos, y signos (que quedan despues de eliminar el primero), es decir,
dos puntos en la segunda recta, 10 que se puede hacer por C2(n) igual a Pn-I.
procedimientos. Combinando tados los pares posibles de puntos, A la segunda categoria referiremos los metodos en que los
obtendremos dos primeros signos se leen como una sola letra. Elnumero de
metodos de la segunda categoria es igual al numero de metod os
m(m-l)n{n-I)
C 2 ()
m . C 2 ()
n = 4 puntos de intersecci6n. en que se lee la cadena compuesta de n - 2 signos (que quedaron
despues de eliminar los dos primeros), es decir, igual a Pn-2.
187. Cada paralelogramo se determina con la elecci6n de dos Puesto que cada metodo de lectura de la cadena dada
rectas de la primera serie, 10 que se puede hacer por C/n) pertenece a una, y s610 a una, de las dos categorfas indicadas, el
procedimientos y con dos rectas de la segunda serie, cosa que numero total de metodos es igual a la suma de los numeros de
se puede realizar por C/m) procedimientos. De este modo, el metodos de la primera y segunda categorias, es decir,
numero total de paralelogramos es igual a (I)
Pn = Pn-I + Pn - 2·
Esta igualdad representa una f6rmula de recurrencia por la
que se puede calcular sucesivamente P" para cualquier n, si se
ALGEBRAIResolucioncs y Soluciones 165

conoce PlY P2 • Pero en el problema dado P I = I (para la cadena


1 1
de un signa existe s610 un metodo de la primera categoria) Y 2s+-s y 2s--s km.
P2 = 2 (para la cadena compuesta de dos signos existen dos k k
metodos: uno de la primera categoria Yotro de la segunda). Pero si dos cuerpos se mueven a velocidades constantes,
Empleando la f6rmula (I), hallaremos sucesivarnente: entonces la relaci6n de las velocidades de los cuerpos, siendo
iguales los tiempos consumidos, es igual a la relaci6n de los
P3 = P2 + PI = 2 + 1 = 3,
carninos recorridos por ellos. Por eso, para la detemlinaci6n de
P4 = P3 + P2 = 3 + 2 = 5, s tenemos la ecuaci6n
Ps = P4 + P3 = 5 + 3 = 8.
Yasi sucesivarnente. Definitivarnente obtendremos:
De aqu! s = 2ak km.
P I2 = 233.
191. Si dos cuerpos se mueven con velocidades constantes,
entonces, en un mismo trayecto del camino, la relaci6n de sus
6. Planteamiento de ecuaciones velocidades es inversamente proporcional al tiempo consumido
por los cuerpos. Supongamos que sea v la velocidad del tercer
189. Supongamos que sea x el menor de los factores. Entonces, autom6vil; t, el tiempo de movimiento del segundo autom6vil
de las condiciones del problema se desprende directamente que hasta el momento en que 10 alcanza el tercero. Entonces,

x(x + 10)-40 = 39x + 22 40 t - 0,5 50 t +1


o bien v v t + 1,5
Dividiendo miembro a miembro la primera ecuaci6n entre
de donde XI = 31, x 2 = -2. Prescindimos de la ra!z negativa, por i
la segunda hallaremos que t = de hora; luego hallamos que
consiguiente, los factores son 31 y 41. v = 60 km/h.

190. Hasta el primer encuentro el primer ciclista recorri6 s + a km 192. Supongamos que hasta el momenta de encuentro pasaron
y el segundo s - a km, donde s es la distancia deA a B. Hasta el x horas. El camino desde el punto de encuentro hasta el punto B
segundo encuentro los ciclistas recorrieron respectivamente el ciclista 10 pas6 enx horas y el transeunte enx+t horas. Puesto
166 PROBLEMAS DE MATEMATICAS ELEMENTALES

que para un mismo camino el tiempo es inversamente proporcional 195. Designemos por lla distancia entre los puntosA y B Ypor
ala velocidad, entonces VI y v2 las velocidades de las motocicletas. En el tiempo t la
X+f=k , primera motocicleta paso el camino igual a p + t - q y la segunda
x el camino q + /- p. Por eso,
deaquf f
X= - -. l+p-ql
k -I v\ = ,I
t ~
193. Designemos la distancia de A a B por x y la distancia entre l+q-p i (1)
Bye por y. Entonces, teniendo en cuenta que en todos los v2 = .1
t j
casos de que se habla en el problema el tiempo de movimiento
es el mismo, obtendremos el sistema de ecuaciones Por otro lado, la reiacion de las velocidades es igual a la relacion
de los caminos recorridos hasta el primer encuentro, es decir,
~+2::=x+Yl ~_l-p
3,5 . 4 3,75 ~
v2 P
x + y 14 Y x I
- - = - +--+. - '11 Colocando aquf v\ y v 2 del sistema (1), obtendremos la
3,75 60 3,75 4) ecuacion para la detenninacion de l. Resolviendola hallaremos
Resolviendo este sistemahallamosquex= 14 km ey= 16 km. que 1= 3 p - q. Colocando este valor de I en la formula (1),
obtendremos:
194. Seax la longitud del camino por ellugar llano,y la longitud 4p-2q 2p
del camino cuesta arriba. Tenemos el sistema de ecuaciones v\ = , v2= - '
t t

1" + ~ + 11,5 - (x + y) = 2 ~ 'I·


196. La diferencia entre los tiempos de retraso del avion en el
3 4 5 10' primer y segundo vue los igual a tl~~2 horas esta enlazada con
11,5 - (x + y) + .:: + 2:: = 3 ~.J que el camino de dkm fue recorrido a distintas velocidades: en
3 4 5 10
el primer vuelo la velocidad era V km/h y en el segundo w kmlh
(en los demas trayectos del camino las velocidades eran
Sumando las ecuaciones del sistema hallaremos que x = 4.
respectivamente iguales). De aq UI obtenemos la ecuacion
ALGEBRAlResoluciones y Soluciones \67

d d
11 - 12 De aqui obtenemos:
,
60 v w
a:~ = (r-q):(p-r).
de la que hallamos que la velocidad inicial del avion es igual a
La resolucion es posible sip > r> q 0 bienp < r < q.
W = 60vd km/h. Para hallar el peso maximo de la nueva aleacion examinemos las
60d + V(/2 - 11) relaciones
p
y Q
197. Designemos el peso del pedazo cortado por x. Supongamos ir
!
- ai11 Ip-{
que la primera aleacion contenia 100 a% de cobre y la segunda
100 b%. Entonces la cantidad en peso de cobre en la primera
P Q
Si Ir - ql = IP - rl ' entonces, el peso maximo es igual a
aleacion despues de fundir su resto con el pedazo cortado de la
segunda aleacion sera igual a a{m - x)+ bx y el peso de cobre
en la segunda aleacion despues de fundir su resto con el pedazo P + Q _ P - q P = L- q Q.
r-q p-r
cortado de la primera sera igual a b{n - x)+ ax. De la condicion
del problema P 0
Si [r _ qi < !p-=- ri' entonces, e\ peso maximo es igual a
a(m -x)+ bx b(n -x)+ ax p- r p- q
m n P+ - - P= - - P.
r-q r-q
Una vez resuelta esta ecuaci6n, obtenemos (teniendo en cuenta
que a -:f::- b): P 0
mn
x =- -. Si por fm ir _-ql > p -=- r' entonces, el peso maximo es igual a
, 1

m+l1
198. Supongamos que la relacion de los pesos de los pedazos a
alear es igual a a : ~.
Entonces up pq
TQO+ TQO _ r
a +~ 100
168 PROBLEMAS DE MATEMATICAS ELEMENTALES

199. Supongamos que cada obrero trabaja t dias y que A gana 6 t- 7 64,8 t- 7 3
- .--=
x rub los, B gana y rublos. De las condiciones del problema 5 t-l 72' t-l 4
obtenemos el sistema de ecuaciones
De aqui t= 25 y, porconsiguiente,

(t -I)~ = 72,
t x = 75 rub los, y = 90 rublos.

(t -7)Y = 64,8, 200. Designemos por II el tiempo transcurrido hasta el primer


t (1) encuentro, por t2 el intervalo de tiempo hasta el segundo encuentro
(t -I)~ - (t - 7)~ = 32,4. y por Rei radio de la circunferencia. En el intervalo de tiempo II
t t el primer cuerpo recorria el camino vII yel segundo, eI camino

De las dos primeras ecuaciones hallamos: ai- . La sum a de estos caminos es igual a la longitud de la
?

circunferencia, por 10 tanto,


t -I 72 1-7 64,8
2
t x t y all
vII + - = 2rcR. (1)
Entonces, de la ultima ecuacian obtenemos: 2
En el intervalo de tiempo (2 ambos cuerpos recorrieron un
y x mismo camino igual a la longitud de la circunferencia, asf que
72- - 64,8- = 32,4,
x y
at2
2
o bien vt2 = 2rcR, - = 2rcR.
20(~r -9(~)-18=0.
2
2
Eliminando de aqui 12, hallaremos que R = L. IW
Colocando
este valor de R en la formula (I) obtendremos la siguiente ecuacion

De aqui y = tx (prescindimos de la raiz negativa). Dividiendo cuadnitica respecto a 11:

ahora la segunda ecuacian del sistema (1) entre la primera y atf 2v2
-+vtl - - =0.
2 a
sustituyendo l. por su valor, hallamos:
x
ALGEBRAIResoluciones y Soluciones 169

Resolviendo esta ecuacion y prescindiendo de la rafz negativa


(puesto que por el sentido del problema debe ser > 0),
obtendremos defmitivamente:
'I II =
216(ql +q2)
ql = 540 min (9 horas),

216(ql +q2)
tl = (-J5 -1)~.
a
t2 =
q2
= 360 min (6 horas).
201. Designemos par q, y q2las capacidades de los grifos (en Existe una segunda soluci6n:
11m in) y porvel volumen de la piscina. EI tiempo requeridopara
llenar la piscina haciendo uso de cada grifo por separado sent II = 360 min, '2 = 540 min.
iguala
202. Anotemos con y el peso especifico del agua y con s el area
(1) de la seccion transversal del tubo. La presion atmosferica Pa se
hall a con ayuda de la formula
La primera condicion del problema conduce ala ecuacion
Pa = ye.
1 I 13
ql . - t2 + q2 . - II = - v.
3 3 18 Si PI es la presion bajo el piston en su posicion superior,
Empleando la igualdad (1) obtenemos la ecuacion cuadnitica entonces, segun la ley de Boyle-Mariotte, para el aire contenido
entre el piston y el nivel del agua, tenemos que PI (b - x)s = Pahs

(!ll)2 _~!ll+ 1= 0,
q2 6 q2

· ,ql 2 ql 3 Did d· . ,
cuyas soIUClones seran q2 = 3' -;;; = "2. e a segun a con IClOn
del problema se desprende que
Fig. 2 Fig. 3

(fig. 2). La ecuacion de equilibro de la columna de Ifquido tiene


De (l ) hallamos las magnitudes buscadas:
la forma P a -- PI = yx. Esto conduce a la ecuacion
170 PROBLEMAS DE MATEMATICAS ELEMENTALES

o bien
he
e - --:::: x x 2 -112x + 832:::: 0.
b-x
(y se simplifica), es decir, conduce a la ecuaci6n cuadnitica , I ~-

DeaqUi x = 56 ±,,3136 -832 = 56 ±-)2304 :::: 56± 48, es


decir, x = 8 cm.

De aquf hallamos que 204. Supongamos que el reloj se adelanta en x minutos al dfa.
Entonces 61 marcara el tiempo exacto a los; dfas. Si el reloj

marcase 3 min menos, pero se adelantani en x + t minutos al


203. SeanP I Ypzlas presiones del aire que se encuentra bajo el dia, entonces, marcarfa la hora exacta a los ~ dias. Por
piston en las posiciones I y II respectivamente (fig. 3) Y y, el x+"2
peso especifico del mercurio. Las ecuaciones de equilibrio de consiguiente,
las columnas de mercurio de 12 cm Y x cm de altura senin 3 2
respectivamente -·- +1= -
x+t x'
76y - PI :::: 12y,l
~ (1)
76y - P2 :::: xy.j de donde
2 3
La ley de Boyle-Mariotte para el aire que se encuentra bajo el x +-x -1 = 0.
piston de la ecuacion 2
Resolviendo esta ecuaci6n hallaremos que x = 0,5.
PI' 29 l : : p2(36-x).
4 205. Si x es la cantidad inicial de los depositantes eyes el
Colo cando aquf las expresiones para PI Y P2 de (1), porcentaje que paga la caja de ahorros, entonces
obtendremos la siguiente ecuaci6n cuadrada respecto ax:
y 111 y n
x+x-'- - =p x+x - --=q.
29 L 64:::: (76-x)(36-x), 100 12 ' 100) 2
4
ALGEsRAiResoluciones y Soluciones 171

Multiplicando la primera ecuaci6n par n, la segunda par m y Colocando esta expresi6n para v2 en la primera ecuaci6n,
substrayendo de la primera la segunda, hallaremos: obtendremos la siguiente ecuaci6n cuadratica respecto a VI:
pn-qm
X= .
n-m Vf - 2rcR VI _ 2 rcR . 2rcR = o.
Volviendo de nuevo al sistema inicial y restando de la primera T T t
ecuaci6n la segunda, obtendremos: Resolviendo esta ecuaci6n, hallamos:
rcR
xy (m-n)= p_q.
1200 VI =T - UI + ~7 + 1
1 ,----- )

Deaqui y, a continuaci6n,
y = 1200(p - q) %.
qm-pn
206. Anotemos con VI y v2 las velocidades de los puntos y 207. Supongamos que sea v el volumen de soIuci6n en el frasco
supongamos que sea VI > v2 • La primera condici6n del problema y x, el porcentaje de sal en la soluci6n.
nos da la ecuaci6n
2rcR 2rcR A la pro beta se vierte ~ de solucion y se concentra por
- - ---=t.
v2 VI
evaporacion hasta que el porcentaje de sal en esta aumente el
La segunda condici6n significa que durante el intervalo de doble. Puesto que la cantidad de sal en este caso no varia,
tiempo T, el punto que se desplaza a mayor velocidad recorreni entonces, el volumen de solucion en la probeta disminuini dos
por la circunferencia un camino en 2 7r R mayor que el otro punto.
Esto nos da la segunda ecuaci6n veces y el peso del agua evaporada sera igual a {n .
Despues de echar de nuevo la solucion concentrada al frasco, en
este habra la misma cantidad de sal que al principio, es decir,
De la segunda ecuaci6n hallamos: v I~O pero la cantidad de soluci6n disminuye en 2vn· De aquf
2rcR obtenemos la ecuaci6n
v2 =Vl - - - .
. t
172 PROBLEMAS DE MATEMAnCAS ELEMENTALES

x Seg(ln la condicion del problema


V 100 X +P
v- l2n 100'
de la que hallamos que
[ x
18 1-- + 1-
x ( )2lJ' +2 = 12 [1--+
x ()2]
-
x
+X- - 2.
x
30 \30 30 30 30
x = (2n -1) p.
de donde obtenemos la ecuacion
208. Supongamos que en el primer recipiente habiax litros de
alcohol; entonces en el segundo habra 30 - x litros. Oespues de x 2 -30x+ 200 =O.
lIenar el primer recipiente con agua, un litro de la mezcla obtenida Esta ecuacion tiene las rakes
contenia 3xO de alcohol y 1- 3xO de agua. Oespues de llenar el
segundo recipiente con la mezcIa obtenida en el primero, en el XI =20, x2=10.
to
segunqo recipiente resultaron 30 - x + x litros de alcohol y Asi pues, en el primer recipiente habia 0 bien 20 I (entonces
(1 - 36)x litros de agua. Un litro de esta nueva mezcla contiene en el segundo habia 10 I), 0 bien 10 I (entonces en el segundo

~ + (~) 2 litros de alcohol


habia20 I).
1-
30 30 .
209. Supongamos que seax la distancia desde la orilla de partida
hasta el punto en que C dejo el bote. Sefialemos al principio que
Oespues de echar 12 litros de la nueva mezcla al primer
A subio al bote a la misma distancia hasta la segunda ori lIa (Ia
recipiente, en este resultaron
orilla opuesta). En efecto, la manera en que A y Cvencieron el
paso difiere solamente en que Cal principio 10 paso en el bote y
X
12 [ 1- - + -
(x)2lJ+ -(30
x - x) litros de alcohol despues a nado y A al contrario. Puesto que los dos nadan a una
30 30 30 misma velocidad v, ademas, v t:. VI Y el tiempo que pierden en
realizarel paso es el mismo, entonces, por 10 visto, las distancias
yen el segundo indicadas deberan ser iguales.

l () 2]
Oespues de esta observacion es facil componer la ecuacion
x x
18 1--- + -
30 30
.
htros de alcohol. x+s-2(s-x) =~~
v
ALGEBRAlResoluciones y Soluciones 173

Aqui el primer miembro expresa el tiempo que pierde el bote


En total el tren estuvo en camino 8 + 5 (s6-v6V ) horas (teniendo
en vencer el camino hasta el encuentro con A y el segundo
miembro, el tiempo consumido por A hasta el encuentro con el en cuenta las dos horas de parada forzosa). Este numero de
bote.
De la ecuacion obtenida hallamos: horas constituye una hora mas de las ~ horas previstas por el
horario. De este modo obtenemos la ecuacion
s{v + VI)
8 + 5(s - 6v) = 1 + ~.
x= .
3v+ vI
6v v
De aqu!, el tiempo consumido en el paso es igual a
Desarrollando un razonamiento analogo respecto al segundo
T = s - x + ~ = ~ v + 3vI . tren, compongamos una ecuacion mas:
v vI vI 3v+ vI
Observacion. Se hubiera podido resolver el problema sin la ~+2=8+ 150 + 5(s-6v-150).
observacion preliminar sobre la igualdad de las distancias v 2 v 6v
indicadas mas arriba. Sin embargo, en este caso, hubiera sido De este sistema de ecuaciones hallamos que s = 600 km.
necesario introducir varias incognitas y la resolucion hubiera
resultado mas dificultosa. 211. Designando la velocidad del bote en agua muerta por v y la
velocidad de la corriente por w, obtendremos un sistema de dos
210. Designemos la distancia buscada por s km y la velocidad ecuaclones
del tren por v km/h. En las 6 horas hasta su parada, provocada -a- +--=T,
a }
por la acumulacion de nieve, el tren recorrio 6v km y el trayecto v+w v-w

restantedel camino de (5 - 6v) kmde longitud 10 paso en 5(s-6v) _a_ = To + a - b + ~ = To + a + b .


6v v-w v+w v+w v+w)
horas, ya que la velocidad del tren en este trayecto del camino

era sV.
6 Resolviendo este sistema respecto a las incognitas

v~w y v~w y tomando sus reciprocas, hallamos:


174 PROBLEMAS DE MATEMATICAS ELEMENTALES

de donde x = 15 h (la segunda rafz por ser negativa no vale).


v+w= 2a+b y V-W= a(2a+b) .
T-To T(a+b)+Toa
213. Designemos la velocidad buscada del tren por v krnIh y la
De aquf se desprende que velocidad segun el grafieo por VI lan/h. EI tren pas61a primera

~oI horas y en veneer la segunda mitad y en


r
1 2a + b a(2a + b)
v="2lT-To + T(a+b)+Toa '
1 mitad del camino en

la parada perdi6 la primera vez II ~10 + 210 horas y la segunda


r
1 2a + b a(2a + b)
w="2lT-To - T(a+b)+Toa .
1 I

I~ + 1~ horas. Pero como ambas veces el tren lIeg6 aB a tiempo,


entonees,
10 10 1 10 10 1
212. Seax el tiempo que permaneci6 abierto el segundo grifo, v, -=---+ - - =-+ - .
la velocidad de salida del agua del primer grifo y w, la velocidad vI vI + 10 20' vI v 12
de salida del agua del segundo grifo (v y W se miden en m 3/h). De la primeraecuaci6n hallaremos VI:
Tenemos:

v(x + 5)+ wx = 425'}


1 1
10 ~ - vI + 10
( J= 20'
1 100

2vx = w(x + 5),


(v + w)17 = 425. Esta ecuaei6n tiene s610 una rafz positiva VI = 40.
De la segunda ecuaci6n hallaremos que v = 60 kmlh.
De la segunda y tercera ecuaciones obtenemos:
214. Sea la distaneiaAB igual as km y las velocidades del primero
v = 25 x + 5 ; SOx y segundo aviones iguales respeetivamente a vJ y v 2 • Entonces,
W=--- .
3x+5 3x+ 5 en virtud de las condiciones del problema tenemos un sistema de
tres ecuaciones:
Coloeando estas expresiones en la primera ecuaci6n, haHamos:

3x 2 - 41x - 60 = 0,
ALGEBRAlResoluciones y Soluciones 175

,
2:, + ~ = 2: 2 - ~ ,I ~+~=14f
u+v u-v '
24 96 72 .
_s__ ~=b
2v2 2v, '
--; = u + v + u - v .J

~_b=_s_ . Para su resoluci6n hagamos ~ = z. Multiplicando ambos


4v, 4v2 )
miembros de la segunda ecuaci6n por y, hallaremos:
Hagamos 24=~+~.
z+1 z-l
s s
- =X -=y. Libenindonos de los denominadores, obtenemos:
2v, ' 2v2

De la segunda y tercera ecuaciones hallamos: 24z2 -168z = 0.


3 5 Puesto que z"* 0, entonces z = 7. Por consiguiente, U = 7v.
x= - b y= - b
2 ' 2 Colocando U = 7ven la primera ecuaci6n del sistema, hallamos:
y de la primera:
2~ + 96 = 14
8v 6v '
de donde
Pero,
v = 2 km/h, U = 14 km/h.
V2 _~ =2
v, y 5 216. EI camino recorrido por cada cuerpo en t sse determina
por la f6mlula
Ahora es facil hallar que at 2
s=vot+-.
8a 8a 2
v, = 3b ' v2 = 5b Y s = 8a.
Para hallar v0 y a para cada uno de los cuerpos, colocamos
215. Sea u la yelocidad del bote en agua muerta y v la yelocidad en esta f6rmula los datos numericos citados en las condiciones
de la corriente. Entonces tenemos el sistema del problema:
176 PROBLEMAS DE MATEMATICAS ELEMENTALES

1) para el primer cuerpo: a Por lacondicion del problema tenemos:


para 1 = 1 tenemos que 25 = vo + -, 10 6
2 3s--+ - -::o;4. (1)
1 v+l v-I
para 1 = 2 tenemos que 50 - = 2vo + 2a;
3 Es necesario que sea v > 1, puesto que de 10 contrario la
,1 1 5 12 lancha no podra moverse rio arriba. Pasemos del sistema de
de aqUl a = -, Vo = 25 - - y 5] = 24 - t + - .
3 6 6 6' desigualdades (1) al siguiente sistema equivalente:
2) parael segundo cuerpo: a
para 1 = 1 tenemos que 30 = Vo + -, 3{v 2 - 1)::0; 16v -4::0; 4{v 2 -1).
2
1 As! pues, es necesario que se cumplan a la vez dos desigual-
para 1 = 2 tenemos que 59 - = 2vo + 2a; dades:
2
,1 1 1 (2
deaqUla=-2' vo=30+"4 y 52=30"41-4. 3v 2 -16v + 1 ::0; 0
y
En el momenta cn que el primer cuerpo alcanza al segundo
tendremos que 51 = 52 + 20; de aqu! obtenemos la ecuacion
cuadnitica para la determinacion de t: La primera desigualdad queda satisfecha si

12 -131 - 48 = o. 8-J61 8+J61


---::o;v::o; .
Resolviendo esta ecuacion hallamos que 1= 16. Prescindimos 3 3
de la segunda ra!z, ya que es negativa. La segunda desigualdad se satisface si v < 0 0 bien v> 4. Pero,
puesto que v> 1, tenemos defmitivamente:
217. Supongamos que sea v la propia velocidad de la lancha.
Entonces, el tiempo en que se encuentra en movimiento la lancha
4::o;v::o;8+J61.
sera igual a
3
IO 6 218. Designemos por x el volumen de agua en el recipiente A
1=-- +--.
v+l v-I antes del transvase. Entonces el volumen inicial de agua en los
ALGEBRAIResoluciones y Soluciones 177

recipientes Bye sera igual a 2x y 3x respectivamente y el volumen Resolviendo esta ecuaci6n, hallamos:
total de agua sera x + 2x + 3x = 6x.
Despues del primer transvase de A a B y de B a C la x = 500.
profundidad en los tres recipientes se hizo igual y, por 10 tanto,
Asf pues, la cantidad inicial de agua en cada uno de los
los volumenes de agua son entre sf como las areas de la base, es
recipientes era la siguiente:
decir, como 1:4:9. Por eso, despues del primer transvase los
volumenes de agua en los recipientes A, Bye tendran enA, 500 I,
respectivamente los siguientes valores: en B, 1000 I,
en C. 1500 I.
1. 6x =lx
1+4+9 7'
219. Supongamos que el numero buscado tiene la formaxyzt
9. 6x = 27 x. (aquf las letras x, y, z Y 1 significan cifras de los 6rdenes
1+4+9 7 correspondientes). Seglin las condiciones del problema obtenemos
Despues del segundo transvase de CaB estos volumenes se el sistema de ecuaciones siguiente:
hacen respectivamente iguales a
= ,
x 2 + 12 13)
y2 + z2 = 85,
3 12 4 27 4 (1)
-x -x+ 128- -x-128-.
7' 7 7' 7 7 xyzl - 1089 = Izyx.
Despues del tercer transvase de B aA el volumen de agua en
A result6 igual a x-I 00 yen B igual a En virtud de la substracci6n por orden, en la tercera ecuaci6n
del sistema (1) 0 1 = 9,0 bien
l (x - 100)·4 = 2(x -100) .
2
(10+/)-9=x,
Sumando los volumenes de agua en todos los recipientes es decir,
obtenemos una ecuaci6n de primer grado respecto ax: (2)
x=t+l.
27 4 Pero, de la primera ecuaci6n del sistema (I) se desprende que
(x -100)+ 2(x -100)+ - x -128- = 6x.
7 7 t < 4 y, por 10 tanto, tiene lugar la igualdad (2). Entonces, de la
178 PROBLEMAS DE MATEMATICAS ELEMENTALES

primera ecuacion del sistema (1) obtenemos [a siguiente ecuacion de donde (a - vt}w
para [a determinacion de t: x= .
v-w
Desde el inicio del movimiento hasta el primer encuentro paso
dedonde un tiempo igua[ a
1=2.
a+x
II = - - - .
De la formu [a (2), ahora se desprende que x = 3 Y la tercera v
ecuacion del sistema ([) toma la forma Colocando aqui el valor hallado de x, obtendremos:

3yz2-1089 = 2zy3. (3) I-wI


tl = - - ·
v-w
Observemos a continuacion que z < 9 (si z = 9, entonces, de
la formula (3) se desprende que y = 0, pero en este caso no se Sea'! la integral del tiempo entre dos encuentros consecutivos.
satisface [a segunda ecuacion del sistema (1 ». De la formula (3) Entonces
haJ[amos: v'!-w'!=l,
de donde I
(z - 1+ 10) - 8 = y, '!=--.
o sea, v-w
z=y-l. (4) Los encuentros sucesivos tendnlnlugar en [os momentos de
tiempo t l , II + t , II + 2t, ... E[ momento del enesimo encuentro
Por fm, de [a segunda ecuacion del sistema ([) y de (4) hallamos sera
que z = 6 e y = 7. Asi pues, e[ numero buscado es 3762.
v - wt-i--l(n-l)
tn = -.
220. Al principio hal [amos [a distanciax desde e[ punto de partida v-w
hasta e[ lugar del primer encuentro. La ecuacion del tiempo de 221. Designemos e[ peso especifico del primero de los meta[es
movimiento de ambos puntos tiene [a fom1a que componen la aleacion por YI, e[ del segundo por Y2 y del
agua por y. Supongamos que seax el peso del primer metal en la
a+x x a[eacion. Seglin el principio de Arqufmedes [a perdidade peso de
----- -( =-,
v w la aleacion en el agua sera igual a
ALGEBRAIResoluciones y Soluciones 179

222. Designemos la distancia desde el punto A hasta la


desembocadura del rfo por s, la distancia desde la desembocadura
del rio por ellago hasta el punto B por s I' la velocidad del barco
(sin remolque) porvy la velocidad de la corriente del rio por VI.
Analogamente para los metales puros esta perdida sera igual a
p p Hacefaltadetenninar ~
v
= x.
--yy - yo De las condiciones del problema componemos tres
YI Y2
ecuaClOnes:
Estas perdidas son conocidas e iguales respectivamente a B y
C. de aqu! hallamos que s x
- -+ - =61
v + VI 2 '
Y B y C
y]
=p = s
Y2 P - --=79,
x
Por consiguiente, la perdida en peso de la aleaci6n es igual a v-v] +-
2
A = B x + C (p - x). s
p p - +=411.1
Deaquf v] J
A-C De la primera ecuaci6n tenemos que
x=--P.
B-C V + VI _ 2
Para la solubilidad del problema es necesario que B ::f- C. A (1)
s -122-x'
continuaci6n del hecho de que ~ es un numero incluido entre 0 y
de la segunda ecuaci6n hallaremos:
1, se deriva la desigualdad
v-v] _ 2
A-C (2)
O<--<l. s -IS8-x'
B-C
de la tercera ecuaci6n obtenemos:
Deaqui sedesprendequeo bienB> A>C,o bienC> A> B.As! v]
pues, para que el problema sea soluble, es necesario y suficiente = -- -- (3)
s 411-x
que el numero A este incluido entre los numeros B y C.
180 PROBLEMAS DE MATEMATICAS ELEMENTALES

Substrayendo de la igualdad (l) la igualdad (2) Yvaliendonos de Con la condici6n de que en el trozo AB la corriente es la misma
la igualdad (3), obtendremos la siguiente ecuacion respecto ax: que en el trozo BC, el camino AC requiere

S + s\
122 - x 158 - x 411 - x ' - - = 5,5 horas. (3)
o bien v+vl

x 2 - 244y + 4480 = 0. . detenmnar


ES necesano, . Iare I ',
aClOn S+S,
v-v, .

Resolviendo esta ecuaci6n hallaremos:


Reduciendo las ecuaciones (1), (2) y (3) a un denominador
comun y multiplicando ambos miembros de la ecuacion (3) por
= 20, x2 = 224.
Xl
*
v 0, obtendremos el sistema
Por 10 visto, el valor x 2 = 224 no sirve, puesto que el primer
miembro de la ecuacion (I) no puede ser negativo. (s + SI)V = 6v (v + VI) - SVI>}
(s + s\)v = 7v(v - v\)+ SVI, (4)
223. Designemos la distanciaAB por s y la distancia BC por s I; (s + SI )v = 5,5 (v + VI )v.
supongamos a continuaci6n que sea v la velocidad de la lancha y
v, la velocidad de la corriente (se supone que s y S I se expresan Sumando las dos primeras ecuaciones y haciendo uso de la
en unas mismas unidades de longitud; v y v, son velocidades tercera,obtendremos:
calculadas por hora).
Para el movimiento de la lancha rio abajo desde A hasta C
tenemos:
S sl
De aqu!, v = 6v ,. Pero de la tercera ecuacion del sistema (4)
-+--=6. (1) tenemos que S+ S
v v+vl __ I = 7.5,5.
Para el movimiento de la lancha rio arriba desde C hastaA VI
Por consiguiente,
tenemos:
_S_I_ +~=7.
(2)
v-vI v
ALGEBRAIResolucionc~ ) ~)llluciones 181

224. Designemos por v la capacidad del vasa y sea al el y, por consiguiente,


contenido de acido en porcientos en el despues del primer
(
mezclado, a2 el contenido de acido en porcientos despues del ll--;-a)k (p-q)=r-q.
segundo mezclado etc. Tenemos: Respuesta:

(v-a)p+aq _ a
v
al'
-'----'--=------=- -
v= J~'
1-~--
{v-a)aJ +aq p-q
=a2'
v
225. Al final del primer ano el deposito aumento en JAto rub los y
el depositario retiro B rublos. Por eso, a principios del segundo
{v - a)ak_2 + aq ano el deposito componia una cantidad en rublos igual a
=ak- J,
v
p \
(v-a)ak _J +aq
-'------"----"'---'----~ = r.
II = A ( 1+ -
100)
1- B.
v
Al final del segundo ano el deposito componia una cantidad
Multiplicando la s-exima ecuacion por (v~a y-s yefectuando iguala
la suma, obtenemos: 12 =~(l+L l_B=A(l+L 12 -Brl+(L+l)]rublos
100) 100) I 100 '
(
v-a k a I v-a v-a
- - ) P+ - qll+ - -+ - -
(12 + ...
v v v v) y al final del tercer ano P2 = Ak1"B( I+k+k2 ) rublos, donde

k=I+L.
100
Es evidente que al final del enesimo afio la cantidad del depOsito
sera igual a
Deaqui
-- p+ -a q (~Y-I_
lrv_a)k
v v-a --r
V -I
v
182 PROBLEMAS DE MATEMATICAS ELEMENTALES

es decir, a

y asf sucesivamente. Por fin, al final del enesimo ano la cantidad


Por la condicion del problema es necesario hallar tal numero de madera era
n que Pn :;:: 3A. Entonces

(1)
Es necesario determinar x.

E! problema tiene sentido si aumenta la cantidad del deposito, Haciendo, parasimplificar laescritura, 1+ I~O = k, de la ultima
es decir, si
Ap > 100B. igualdad obtendremos que x = kan_l - aq. Expresemos 0,,_1 por
su valor de la ecuacion anterior. Obtendremos:
Puesto que, ademas,p>O, A>O, B>O, entonces la expresion
que figura en la parte derecha de la desigualdad (1) tiene senti do.

226. Al final del primer ano en la parcela forestal habia una Pero,
cantidad de madera igual a

a ( 1+ L
\ 100)
'I - x = a] , Por consiguiente,

x = k 3a n-3 - 2
k x - kx - aq.
al final del segundo ano,
Continuando del mismo modo, expresaremos por fin ao mediante
a l y obtendremos la siguiente ecuacion respecto ax: -
a1(I+L l-x=a2'
100 )
x = k n a - x (kn-l + k n - 2 + ... + k) - aq.
al final del tercer ano,
ALGEBRAIResoluciones y Soluciones 183

Deaqui
Deaqui
x=a kn - q (k_I)=a(l+tfot -qL.
k -1 n (1 + 1 100L)n _ P = qn + Pn-l
~ 100 11 2
227. Antes del transvase la concentracion de alcohol q; era:
= ql1 + qn-l +~ qn-2 + Pn-3 = ~ + ql1-1 + qn-2 + PI1-3 =
2 22 22 2 2 22 23 23
en el primer recipiente ql = I,
en el segundo recipiente q2 - t' ...
= qn
2
+ qn-l +
22
+3L;- y~ =_l_+ __l _ ~
... 2n- 1 2n- i 2e- 1 22 e- 2

en el enesimo recipiente qn = kL 1 • ... + - n -+


1
- 11 -.
1
12 - 1 k 2 -

Supongamos que despues de todos los transvases las


concentraciones se hicieron respectiva mente iguales a:
Para k -::1= 2, la ultima suma es igual a
PI> P2 , ···, Pw Entonces, PI = 1 y, siendo i> 1, Pi se
detemlina por la ecuaci6n _ 1_ _ 1_ 1 2 n - 1 - k n- I 1
1 kl1 - 1 ')11-1

qi + Pi=l (i=2, ... ,17).


Pn = 2k t --~ + 2 n - 1 = (2k t- (2 - k) + 2
1 n- 1

2
Esta ecuaci6n se obtiene dividiendo la cantidad de alcohol
v v 228. La fracci6n tiene la fonna +1' donde pes un nlunero
p -
qi 2 +Pi-l 2·
entero mayor que cero. Las condiciones del problema se escriben
que result6 en el i-esimo recipiente despues de llenarlo con el en forma de desigualdades
contenido del (i -l)esimo recipiente, por el volumen total v del P+2 I P -3 1
- -> - 0 < - -- < -
recipiente. p2 + 1 3' p2 _ 4 10
De este modo,
Transfonnemos la primera desigualdad a la fonna
... , P =Hn+Pn-1
n 2
184 PROBLEMAS DE MATEMAnCAS ELEMENTALES

Resolviendo la correspondiente ecuacion cuadrada, obten- 7. Problemas diferentes


dremos:
3±m 229. Tenemos:
Pl,2 =
2

De la desigualdad 0 > p2 - 3 P - 5 obtenemos P2 < P < Pl.


Pero P2 < 0 y P > 0, por eso

O<P<Pl=
3 +m
2

Es facil ver que P I se encuentra entre 4 y 4,5. Por consiguiente,


,
de la segunda desigualdad se desprende que p, como numero
entero, puede tomar solamente uno de los cuatro valores
230. Supongamos al principio que x*- a. Multiplicando y
siguientes;p = 1,2,3,4. Colocando estos valores en la segunda
dividiendo el producto a examinar por x»a y empleando
desigualdad
sucesivamente la formula para la diferencia de los cuadrados de
p-3 1
0<-2-<-· dos numeros, obtendremos:
P -4 10
+~ =
)( 7 2 )( 4 4) ( 2,,-1 2,,-1 \
(x-a)(x+ax- +ax +a ... x
hallamos que P *- 1, P *- 2 Y P *- 3. As! pues,
x-a
4 P - 4 ( 2 _a 2 )(x 2 +a 2 )(x 4 +a 4) ... ( x 4 +a 4) ... (x 2,,-1 +a 2,,-1 ) =
=x
p=,~
P -1
15·
x-a

x-a x-a
ALGEBRAIResoluciones y Soluciones 185

Supongamos ahora que x = a. Entonces, el producto que se x = -a el producto es igual a 3n a 2 (2 n -I) Ypara x = ael resulta
examina sera igual a igual a a 2 (2 n -I). '

232. Es evidente que

231. Multipliquemos y dividamos la expresion dada por el


Sk -Sk-I = bk (k = 2, 3, 4, ... , n) (1)
y
producto
(2)
(x+a )(x 2+a2X' x 4+a4)... (2
x
n- 1
+a 2
n l
- )
,
Colocando en la suma

que para todos los val ores reales de x oj:. -a difiere de cero. Se
ve facilmente que el resultado se puede escribir de la siguiente
en vez de b I' b l , ... bn sus valores de (1) y (2) obtendremos:
manera:
ali1 + a2 b2 + ... + anbn = alSI + a2 (S2 -SI)+ a3(S3 - S2)+···
~3 + a 3 Xx 6 + a 6 Xxl2 + a l2 ).. .( x 3.2n- 1
+ a 3.2n - l )
... + aAsn -Sn_l) = SI (al - a2)+ S2(a2 -a3)+···
(x + a Xx 2 + a 2 Xx4 + a 4). .. ( X2n- 1 + a 2n -1 )
... +Sn_l(an_I-an)+anSn-

233. Multiplicando ambos miembros de la igualdad por 2,


En el numerador y denominador de esta fraccion figuran pasamos su parte derecha a la parte izquierda. Despues de
productos analogos al examinado en el problema anterior. Por simples simplificaciones obtenemos:
eso, multiplicando el numerador y el denominador par el producto
(x- a)(:>? - a3 ) reduciremos laexpresion dada ala fonna 2(a 2 +b l +c 2 -ab - ac-bc)= a 2 -2ab +b 2 +a l _

3.2 n 3.2 n 2n + 1 2n 2n 2n +1 -2ac+c 2 +b 2 -2bc+c l =


x -a x-a x +a x +a
------
3 3 2n 2n 2 2
x -a x -a x +ax+a
Nuestro metodo pierde su vigor cuando x = ±a. En estos Puesto que a, by c son numeros reales, esto es posible cuando,
casos, sin embargo, un simple computo demuestra que para y s610 cuando, a = b = c.
186 PROBLEMAS DE MATEMATICAS ELEMENTALES
- -- - - - - - - - -- - - -- - - - - -- - - -- - - - - --
234. Multi PIiquemosa 2 + b 2 + e 2 -be - ea-ab por a + b + e. de donde directamente se desprende la confirmaci6n del
Despues de simples c6mputos haHamos que el producto es igual problema.
a a 3 + b3 + e 3 _ 3abe , es decir, de acuerdo con la condici6n
del problema, es igual a cero. De aqu! se desprende la validez de 236. Hagamos Pn = an - an-I . Entonces, de la condici6n del
la confmnaci6n del problema. problema se desprende la f6rmula Pn = Pn-l + 1 que muestra
que los numeros Pn forman una progresi6n aritmetica con la
235. Puesto que p =t ° y q =t 0, se puede escribir: diferencia de I. Por eso Pn = P2 + n - 2. ahora hallamos:

an = {an -an_J+(an_1-an_2)+ ... +(a 2 -al)+al =


= Pn + Pn-I + ... + P2 +al =(n-l)P2 +(n-2)+
+ (n - 3) + ... + I + al =
. .( ) (n - 2) (n -I)
=(n-I) a2 -al +al + 2

o defmitivamente

an = (n -1)a2 - (n _ 2)al + (n - 2)(n -1).


2
Sumando las dos primeras de estas igualdades y restando la
237. Primera resolucion. La relaci6n dada se puede escribir
tercera duplicada, hallamos: en dos formas

an -aan-l = ~(an-l -aan_2),


an - ~an- l = a(an_l - ~an-2)·

Tomando en consideraci6n que todas las magnitudes son reales, Suponiendoque an -aan_1 = un Y an -~an-l = vn halla-
deducimos que remosque

!i_EL=O,
P q De las ultimas relaciones se desprende que
ALGEBRA/Resoluciones y Soluciones 187

11-1 An-l n-2 An-2


a -I-' a-I-'
an = - - - - a 2 -al3 al'
obien a-13 a-13
an -aan_1 = I-'An- 2(a2 -aa] ') , 238. Tenemos que xI + x2 = 3a, xlx2 = a 2. Por eso
al1 -l3al1 -1 = a n - 2(a2 -l3a]). 2+X22 = ( ) 2 -2XI X2 = 7a 2 = 4'
7
xl XI +X2

Eliminando de aquf an-I' obtendremos definitivamente que


de donde a 2 = i. Por eonsiguiente, son posibles dos valores
de a, a saber:
1
a2 = - .
2
Segunda resolucion. Suponiendo en la relaei6n inicial que n 239. Hallamos:
es sueesivamente igual a 3, 4, ... , hallaremos:
Yl = (x] +X2)2 - 2xlx2 = p2 -2q,
a" - fJ2 a - fJ Y2 = (XI +X2)3 -3(Xl +X2)XIX2 = _p3 +3pq.
a = (a+fJ)a,-afJa, =- - - a -afJ - - a
3 " ' a-fJ 2 a-fJ l'
2 fJ2 Los eoefieientes de la eeuaei6n euadnitiea y2 + ry + s =0
a4= (a + fJ )a3- afJa2=(a + fJ) a - a2-
a-fJ euyas rakes Y 1 e Y, son iguales a

r = - ( Yl + Y2 ) = P3 - 2
P - 3 pq + 2q,
s = YlY2 = ~2 -2q)(- p3 -'-3pq).

240. Tenemos que xl + x2 = - 12,


a
Xl X2 = ~a . Con ayuda de
Ahora, es faeil demostrar por el metoda de indueei6n eompleta estas f6nnulas hallamos:
que la f6rmula general es
188 PROBLEMAS DE MATEMATlCAS ELEMENTALES

(a l +blxr +(a z +b2x)2 +(a3 +b3 x)2 =


=bI2(A+X)2 +bi(A+X)2 +b;(A+X)2 =
1
= (. A,
fb2I + b 2 + bi? + -VIb 2 + bi? + b 2)2
2 l 3 X ,

y, por consiguiente, la suma expuesta en el problema representa


el cuadrado de un polinomio de primer grado. Asi pues, las
condiciones (2) y (3) son necesarias y suficientes.

don de B 7:- 0. Suponiendo que sea x = -1, obtenemos:


242. Designemos las raices de la ecuaci6n por x I y x 2• Entonces
+ x 2 = -p, X I X 2 = q.
XI

Si XI Yx2 son negativos, entonces es evidente que p > y q > 0. °


°
Si XI = ex + if3, ex < y P7:- 0, entonces, x2 = ex - if3 Y obte-
nemosque

P=-XI-X2 =-2ex>0
Puesto que todas las magnitudes son reales, de aqui se deducen
tres igualdades: y
q=XI X 2 =ex 2 +p2 >0 .
GI = Abl> G2 = Ab2 , G3 = Ab3 . (2)
Supongamos que, al contrario, se conoce que p > y q > 0. °
donde A = 1· Ademas. en este caso, debe cumplirse la siguiente Entonces, en el caso en que XI y x 2 son reales, de la igualdad
condici6n: XI . x2 = q se desprende quex i y x 2 son de un mismo signa y de
bI2 + b22 + b32 7:- 0, (3) la igualdad XI + x 2 = - p se deriva que las raices son negativas. Si
XI = ex + if3, x2 = ex - iP, f3 7:- 0, entonces, XI + Xl = - P =
de 10 contrario, los tres numeros sedan ceros y el primer miembro
de (1) no dependerfa de x.
=2a, XI y por consiguiente, ex es negativa.
Supongamos ahora que, al contrario, se han cumplido las
condiciones (2) y (3); entonces
ALGEBRAI'Resoluciones y Soluciones 189

243. Puesto que las rakes de la ecuaci6n x2 + px + q


positivas, el discriminante de la ecuaci6n es
= °son Es evidente que la afirmaci6n es justa si se exige que sean
simultineamente

D = p2 -4q ~ 0, (1) 1 - pr > °y p - 2rq < 0,


es decir que
y los coeficientes satisfacen las desigualdades 1
r>-, (7)
p = -xl - x2 < 0. (2) p
q=xlx2 >0. (3)
r>L. (8)
Supongamos ahora que Y1 e Y z son las rakes de la ecuaci6n 2q

q/ +(p-2rq)Y+l- pr = 0. (4) As! pues, para los valores negativos de r que satisfacen las
condiciones (7) y (8)'Y 1 e Yz son positivos. Cuando no se cumplen
EI discriminante de esta ecuaci6n es igual a estas condiciones, una 0 ambas rakes de la ecuaci6n (4) no son
positivas.
Dl = 4r2q2 + p2 -4q

yen virtud de (1) no es negativo cualquiera que sea el valor de r. 244. Supongamos al principio que p ::j:. 3. Para que las rakes de
Por consiguiente, Y1 e Yz son reales para todos los valores de r. una ecuaci6n cuadnitica con coeficientes reales sean tambien
Teniendo en cuenta (2) y (3), por las f6rmulas de Viete, para reales es necesario y suficiente que el discriminante D de esta
r ~ 0, obtenemos que
ecuaci6n no sea negativo. Tenemos que
1- pr
YlY2 = - - >
q
° (5) D = 4p2 -24p(P-3)= 4p(18-5p).

y, por 10 tanto'Y l ey z son de un mismo signo.Acontinuaci6n, Por eso D 2 ° cuando

p-2rq 0-:; p -:; 3,6. (I)


~+h=-
q
>0 W
Las rakes r~alesxl y X z seran positivas cuando, y s610 cuando,
y, por consiguiente, para r 20, Y1 e Yz son positivos, con la cual su suma y su producto sean positivos, es decir, cuando
el problema queda res!lelto.
190 PROBLEMAS DE MATEMATlCAS ELEMENTALES

2p 6p - b + ~ b 2 - 4a(c + A) > 0
xl +X2 =- - >0, Xl . X2 =- - > O. (2)
p-3 p-3
y, por 10 tanto, la rafz
El sistema de desigualdades (l) y (2) se satisface en el caso
en que
-b+ ~b2-4a(C+A) <0 .
3<p ~ 3,6. 2a
Asf pues, ambas admisiones conducen a una contradicci6n.
Observemos ademas que siendo p = 3 la ecuaci6n examinada
La afirmaci6n queda demostrada.
en el problema tiene una sola rafz X = 3 > O. Por esta raz6n,
todos los valores buscados quedan detenninados por la condici6n
246. Las raices Xl' x 2 de la ecuaci6n X2 + X + 1 = 0 satisfacen
3~ p ~ 3,6. ·' a Ia ecuaClOn
tam b len "'-'l-OP
A. « -
3m
. or eso, xI,2 = Xl,3n2 = Xl,3p2 = 1,
245. Demostremos la afirmaci6n pro oposici6n. Supongamos de donde se deriva la afirmaci6n.
"*
que sea a O. Entonces, para las raices XI y X 2 tenemos:
247. Colocando el valor de y de la segunda ecuaci6n en la primera,
- b± ~b2 -4a(c+A) obtendremos la ecuaci6n
XI2 = - .
. 2a
(1)
Examinemos ahora dos casos:
1) Supongamos que sea a > O. Elijamos 'A de tal modo que que, por la condici6n del problema, tiene raices reales para todos
se cumpla la desigualdad [os va[ores de ')". Demostremos que en este caso a = O.
Supongamos 10 contrario. Entonces, para eI discriminante D de
b2
A>--C. Ia ecuaci6n cuadratica (1) es justa la desigualdad
4a
En este caso, es evidente que b 2 - 4a(c + A) < 0 y, por (2)
consiguiente, la ecuaci6n dada tiene raices irreales. cua[quiera que sea e[ valor de A. La parte izquierda de la
2) Supongamos que a< O. Entonces, con la condici6n de que desigua[dad (2) tiene la fom1a
A> -C, tenemos:
ALGEBRAIResoluciones y Soluciones 191

250. Por las f6nnulas de Viete (vease la pagina II) tenemos:

y para valores 10 suficientemente grados en valor absoluto de Ie


es negativa. Por ejemplo, para Ie = I~ el miembro izquierdo de Val iendonos de estas igualdades, obtenemos:

laecuaci6n (1) es igual a«321. Esto qQ~duce a unacontradicci6n. YI + Y2 + Y3= xlx2 + x2 x3 + x3 x I = I,


248. En la forma reducida, la ecuaci6n dada tiene el aspecto YIY3 Y2Y3 + Y3YI = Xl X2 X3(Xl + X2 + X3) = -2,
..L

YIY2Y3 = (XIX2 X3)2 = 1.


x2 - ~ + q + 20 2 )X + pq + (p + q )0 2 = O.
Por consiguiente, la nueva ecuaci6n tiene la fonna
Calculando el discriminante D de esta ecuaci6n cuadratica
obtenemos: Y 3 -Y 2 -2y-I=0.

D y
= (p + q + 2a 2 -4[pq + (p + q )a 2 ]= (p _ q)2 + 4a 4 . 251. Sobre la base de la f6nnula de Viele tenemos:

Puesto que D ~ 0 para todos los valores reales de a,p y q, la Xl + x2 + x3 = 1,


ecuaci6n cuadratica, y al rnismo tiempo la ecuaci6n inicial, tiene xlx1 + x2x3 + x3xl = 0,
ralces reales.
xlx2x3 = 1.
249. Examinemos el discriminante de la ecuaci6n cuadratica dada En virtud de estas igualdades

y
D = (b 2 + a 2 _c 2 _4a 2b 2 = YI + Y2 + Y3 = 2(Xl +x2 +x3)= 2.
= (b 2 + a 2 _c 2 -2ab )(b 2 + a 2 _c 2 + 2ab)= Puesto que Yl = 1- xI, Y2 = 1- x2 e Y3 = 1- x3' entonces,
= [(a-b)2 _c 2] [(a+b)2 _c 2 ].
YIY2 + Y2Y3 + Y3Yl = (1- XI )(1 - X2) + (1- x2 )(1 - X3) +
Puesto que a + b > c y ia»bj< c, entonces, (a+b)2> c 2 y + (1- x3)(I- Xl)= 3 - 2(xl + x2 + X3)+ xlx2 + x2x3 + xlx2 =1
(a-b )2< c2 . Por consiguiente, D < O.
192 PROBLEMAS DE MATEMATICAS ELEMENTALES

y, portin, Para que con los segmentos de longitudesx 1,x2 y X3 sepuede


construir un triangu\o es necesario y suticiente que

La nueva ecuaci6n tiene la forma (2)

y3 _ 2i + y + 1 = o. Este hecho fue demostrado en la resoluci6n del problema 106.


Para obtener la condici6n planteada en el problema,
252. Supongamos que sea expresemos la parte izquierda de la desigualdad (2) por medio
de p, q y r. Con este tin, valgamonos de la dependencia entre las
XI = p-d, x2 = p, x3 = p+d. rakes y los coeticientes de una ecuaci6n
Entonces, xI + x2 + x3 = 3 p; por otra parte, por la f6rmula
de Viele xI+x2+x3=-a. De aqui 3p=-a y, por Xj + X2 + X3 = -p, XI X2 + XI X3 + X2 X3 = q,
consiguiente, XIX2X3 = -r.
a
x2 = P = --. Escribamos ahora la condici6n (2) en la forma siguiente:
3
Colocando esta raiz en la ecuaci6n, obtenemos:
deaqui

- p3 _2p2(Xj +x2 +x3)-4p(xjX2 +x\x3 +X2X3)-8xjX2X3 > 0

y, por consiguiente,
de donde
23 1 p3 _ 4 pq + 8r > O.
c=--a +-ab.
27 3
°
253. Supongamos que xl > 0, x2 > 0, x3 > son las raices
de la ecuaci6n dada. Ateniendonos a la indicaci6n, estudiemos la
254. Sea Xo la raiz comun de las ecuaciones. Colocando Xo en
ambas ecuaciones y substrayendo una ecuaci6n de la otra,
hallamos:
expresi6n
Xo = 32 - q\ 7= 0.
(XI +x2 -X3)(X2 +x3 -XI)(X3 +X\ -X2)· (I)
Pl- P2
ALGEBRAIResoluciones y Soluciones 193

Supongamos que x2 + ax + b es el cociente de la division del


Iv+ 1
trinomio x3 + PIx + qi entre X - xo. Entonces, Xo = - - . (2)
Iv
x 3 + Plx+ qi = (x-xo)~2 + ax+ b). Asi pues, si existe la raiz comun, ella esta enlazada con Iv por
la fOrmula (2). Se puede comprobar facilmente que la fraccion
Igualando en esta identidad los coeficientes de x 2 a los
terminos independientes, hallaremos: a = Xo y b = uq/xo' De At efectivamente satisface a ambas ecuaciones (por 10 visto,
aqui se deriva que las otras dos rakes de la prim era ecuacion se
es suficiente establecer este hecho solamente para la segunda
determinan por la formula
ecuacion). As! pues, las dos ecuaciones (I) tienen una raiz comun
para cualesquiera valores de Iv :f: 0. Esta raiz se determina por
la formula (2).

256. Primera resoluci6n. Supongamos quex" x 2 y X3 son las


y las de la segunda ecuacion, por la fonnula rakes del polinomio P(x). Por las formulas de Viete tenemos:

de donde se desprende facilmente que


222
Xl + x2 + x3 + 2P = O.
255. Es facil comprobar que para A= 0, las ecuaciones no tienen
raiz comun. Seaxc la raiz comun de las ecuaciones para cierto Puesto que XI' Xl Y X3 son reales y difieren de cero
valor de Iv :f: 0. Entonces,
(q :f: 0), entonces, xf + xi + xi > °
y, por consiguiente,p < 0.
4g - x~ - Xo - (Iv + 1) = a,} (1) Segunda resoluci6n. Se ve facilmente que entre las tres ralces
40 - Xo - (Iv + I) = o.
del polinomio P(x) habra dos desiguales. En el caso contrario

Multiplicando la segunda igualdad por Xo y restandola de la P(x) representaria un cuba exacto: p(x) == {x - xO)3, 10 que,
primera, hallaremos: evidentemente no tiene lugar.
194 PROBLEMAS DE MATEMAT'CAS ELEMENTALES

Supongamos ahora quex, Yx 2' son dos rakes desiguales del


polinomio Y seax, <x2 . Supongamos, al contrario, que p 2': 0;
~ 3
entonces, xl < X2 Y pXl:-::; PX2. De aquf se desprende: Con esto la afinnaci6n queda completamente demostrada.
3 3
P (xl) = xl + pXI + q < x2 + pX2 + q = 0, 258. Sean a, 13, YI las rakes de Ia primera ecuaci6n Y a, 13, Y2
las rakes de la segllnda. En virtud de las f6rmulas de Viete tenemos:
puesto que P(x 2 )=0. L1egamos a la conclusion de que P(x)<O.
Esto contradice a que x, es la raiz de P(x). Por consiguiente,
a+I3+YI=-a, (1)
p<O.
a(3y 1 = -18, (2)
257. Supongamos que xl' x 2 YX3 son las rakes de la ecuacion (3)
a+I3+Y2=0,
dada. En virtud de las formulas de Viete tenemos:
al3Y2 = -12. (4)
XIX2 + xl x 3 + x2x3 = 0, (1)
De las ecuaciones (1) Y(3) obtenemos:
xlx2x3 = b > 0. (2)

Supongamos al principio que las tres rakes son reales. Yl - Y2 =-a (5)
Entonces, de la condicion (2) se deriva, que, por 10 menos, una
de elias es positiva. Si al mismo tiempo resultaran positivas dos Yde las eClIaciones (2) Y(4)
rakes, entonces, en virtud de Ia misma formula (2) deducirfamos
que la tercera raiz tambien es positiva, 10 que contradice ala 1i 3
(6)
condici6n (1). De este modo, para el caso en que las tres raices Y2 2
sean positivas el problema queda resuelto.
Resolviendo conjuntamente las ecuaciones (5) Y(6), hallamos:
Admitamos ahora que x, es una raiz irreal de la ecuaci6n,
entonces, como es sabido, la eCllaci6n tiene una rafz compleja
YI = -3a, Y: = -2a. (7)
conjugada x2 + Xl. Puesto que en este caso XIX2 = XI Xl > 0,
de la igualdad (2) obtenemos; De este modo, si para ciertos val ores de a Y bIas eClIaciones
tienen raices comunes; las terceras raices de estas ecuaciones se
ALGEBRAIResoluciones y Soluciones 195

determinan por las formulas (7). Colocando y\ = -3a en la Asi pues, la parte izquierda de la igualdad a demostrar satisface
primera ecuacion y Y2 = -2a en la segunda, obtenemos: ala ecuacion cubica
x 3 -6x-40=0. (1)
-18a 3 + 18 = 0
y Es facil comprobar que parax=4laecuacion (1) se satisface.
-18a3 - 2ab + 12 = O. Dividiendo la parte izquierda de la igualdad (1) por x «4
obtendremos la siguiente ecuacion para la determinacion de las
De aquf se determina el unico par de valores reales otras dos ralces:

a = 1, b = 2. (8) x 2 + 4 x + 10 = O.
Esta ecuacion tiene rakes irreales, puesto que su discriminante
Colocando estos valores en la ecuacion hallaremos facilmente
D =--24 < O. Por 10 tanto, la ecuacion (I) tiene la unica raiz real
que
x = 4 Y puesto que A es notoriamente un numero real, entonces
x 3 + x 2 + 18 = (x + 3)~2 - 2x + 6) A = 4, con 10 cual el problema queda resuelto.
y
x 3 + 2x + 12 = (x + 2)~2
260. Es facil ver que la expresion que se exam ina se reduce a
- 2x + 6).
cero si dos cualesquiera de los numeros a, bye son iguales entre
Por consiguiente, para los val ores indicados de a y b, las sf. De aqui, segun el teorema de Bezout, se desprende que debe
ecuaciones tienen efectivamente dos rakes comunes. Estas rakes dividirse sin resto por cada una de las diferencias
se determinan por la formula
(b - c), (c - a), (a - b).
Xl, 2 =-I±-J-S.
Esto sugiere que la expresion dada es el producto de los
259. Designemos la parte izquierda de la igualdad por A. Tenemos: factores indicados. Efectivamente, tenemos:

A3 = 20 + 14-12 + 3~~0 + 14-J2)i~20 -14J2 + a 2 (e - b)+ b2 (a -e)+ e 2 (b - a)== a2e _ a2b +


+ b2 a -b 2e + e2b - e2a == a 2 (e - b)- a(e 2 _ b2 )+
+ 3~)20 + 14--J2 ~(20 -14J2 )2 + 20 -14-12 =
+be(e-b)=(c-b)[a 2 -ae-ab+be]= (I)
= 40 + 3 ~ 400 - 2 . 14 2 A = 40 + 6A. = (e -b ) [a{a -e )-h{a-e )]= (e - b)(b -a)(e - a).
196 PROBLEMAS DE MATEMATICAS ELEMENTALES

Puesto que a, bye son por pares diferentes, la afirmaci6n El primer miembro deesta igualdad se descompone faciimente
queda demostrada. en factores

261. Observemos que para x = - y la expresi6n dada se reduce a 2(b + c)+ ab(e +b)+ ac(b + c)+ bc(b + c)=
a cero. Por consiguiente, por el teorema de Bezout, se divide sin
= (b + c)(a 2 +ab + ae + be)= (b + c)(a+ b)(a+c).
resto entre x Iy. Para efectuar la divisi6n representemos x + y +
z en forma de la sum a de dos sumandos: (x + y) y z. Elevando Puesto que el ultimo producto es igual acero, entonces, por
esta suma al cuba obtenemos: 10 menos uno de los facto res es igual a cero, de donde se deduce
la resoluci6n del problema.
[(x + y)+z]3 _x 3 - i _z3 =
= (x + y)3 + 3(x + y f z + 3(x + y)z2 _ x3 _ y3 = 263. Sean a y 13 las ralces del trinomio cuadratico x 2 + px + q.
Si el binomio x 4 _ 1 se divide entre e\ trinomio indicado sin resto,
=3(x+ y)[z2 +z(x+ y)+xy].
entonces, a y 13 son ralces del binomio. Se ve facilmente que es
El trinomio cuadrado respecto a z, que figura a la derecha
entre corchetes, puede ser facilmente descompuesto en factores, justo tambien 10 opuesto: si a y 13 son rakes del binomio x 4 -1,
puesto que es evidente que sus rakes son «x y «y. Como resultado
entonces, este se divide entre x 2 + px + q sin resto *).
obtenemos:
Las raices del binomio x4 -1 son los numeros, 1, -1, i, -i.
Por eso, tiene lugar la descomposici6n

262. Multiplicando ambos miembros de la igualdad dada por


abc(a + b + c) la reduciremos a la forma
En virtud de 10 dicho mas arriba los trinomios que nos interesan
(ab + be + ac )(a + b + c) - abc = O. pueden ser solamente aquel\os que representan un producto de
los dos factores que figuran enla parte derecha de (1).
Abriendo los parentesis, obtenemos:

*) En cste caso, si a = 13, el numero a debera ser raiz multiple tambien


del dividendo.
ALGEBRAIResoluciones ySoluciones 197

Componiendo todas las combinaciones posibles hallaremos Para que este polinomio se divida por x-I sin resto, en virtud
Ci ( 4)=6 trinomios. del teorema de Bezout debe cumplirse la igualdad

(x -I )(x + I) = x 2 - 1, n - a(n - 2) = o.
(x -1)(x - i) = x 2 - (1 + i}x + i, Por eso la divisibilidad tienen lugar para cualquier numero
entero positivo
(x -1)(x + i) = x 2 - (1- i}x - i,
n
(x + 1)(x - i) = x 2 + (1 - i) x - i, n>2 y a=--.
n-2
(x + 1)(x + i) = x 2 + (1 + i}x + i, 265. De las condiciones del problema se desprende que
(x - i )(x + i) = x 2 + 1.
Con estos por 10 visto se agotan tOOos los trinomios buscados.
pea) = A,}
pCb) = B, (1)
264. Representando el polinomio dado en la forma pee) = c.

xn - 1- ax(xn- 2 -1), Dividiendo el polinomiop(x) entre (x-a)(x-b)(x-e),


representemoslo en la forma
dividamoslo por la diferenciax« 1 valiendonos de la formula
p (x) = (x - a )(x - b )(x - c )q (x) + r (x) (2)
x - 1
k+l
k
- - - =I+x+ ... +x (1) *) Es evidente que r(x) es un polinomio no superior al segundo
x-I orden. Escribiendolo en la forma
Como resultado en el cociente obtendremos el polinomio
(3)
x n-l +x n-2 + ... +x+ 1- axx 1)
(n-3 +x n- 4 + ... +x+.
Coloquemos en la identidad (2) sucesivamente
*) La f6rmula (I) se comprueba con facilidad directamente, es mas, ella x = a, x = b, x = c. En virtud de la igualdad (1) obtendremos
coincide con la f6rmula de la suma de k lI::rminos de una progresi6n el siguiente sistema de ecuaciones para la determinacion de los
geometrica. coeficientes I, my n del polinomio (3):
198 PROBLEMAS DE MATEMAnCAS ELEMENTALES

266. Por 10 visto, la formula es justa para n = I. Supongamos


fa 2 + ma + n = A. "1I
" que la formula esjusta paracierto numero n; demostremos que
Ib 2 + mb + n = B, ~ entonces ella sera tambienjusta paran +1. Designando por Sn la
I (4) suma que figura en la parte izquierda de la formula a demostrar,
le 2 + me + n = C.j tendremos:

Resolviendo este sistema hallaremos: Sn+l = Sn + (n + I);n + 2) _ (11 + I)(n: 2)(11 + 3) =


1= (A-B)(b-e)-(B-C)(a-b)
(a-b)(b-e)(a-e) , = (n+l)[(n+l)+I][(n+I)+2]
6
m = (A - B)(b 2 _e 2 )_ (B -C)(a 2 _b 2 )
De aqui, de acuerdo con el metoda de induccion matematica,
(a-b)(b-e)(e-a) , se deriva la validez de la formula para cualquier valor entero
n = a2 (Be -Cb)+ a(Cb 2-Be 2)+ A(Be 2-Cb 2) positivo de n.
(a-b )(b -e )(e - a) .
267. SeaSn lasumaquefiguraen la parte izquierdade la formula.
Observacion: Para x = a, x = b y x = e, el polinomio bus- Para n = 1, ambas partes de la formula coineiden. Demostremos
eado rex) toma respectivamente los valoresA, B y C. Es faeil que si la formula esjusta para cierto numero n, entonees sera
eomprobar que tal polinomio, no superior al segundo orden, es tambien para n + I. Tenemos:
el siguiente:
Sn+l -S ('1)2_-
- n + 11 T
n(n+l)(2n+l) + (n + 1)2_-
A (x-b)(x -c) B (x- a)(x -c) C (x-a)(x -b) 6
(a - b)(a - e) + (b - a)(b - c) + (e - a)(e - br (5) = (n+l)(2n 2+7n+6)= (11-+-1)[2n(n+ 2)+3(n+ 2)] =
Puesto que el sistema (4) tiene una sola solucion, entonees,
6 6
existe solamente un polinomio con la propiedad indicada y, por = (n + I)[(n + 1)+ 1][2{n + 1)+ I]
eonsiguiente, r (x) coincide con el polinomio (5). 6
Por consiguiente, la formula es justa para eualquier n entero
positivo.
ALGEsRAIResoluciont:s y Soluciones 199

268. Paran= 1es facil convencersede lajustezade laafinnaci6n.


(coscp + isenCPt+! -(cosncp + i sen ncp)(coscp+ i sencp) =
Supongamos que la f6nnula seajusta para cierto valor de n ~ 1.
Designemos por S" la suma que figura en la parte izquierda de la = (cosncp coscp-senncp sen cp)+ i(cosncp sencp+ senncp coscp) =
f6nnula. Tenemos: = cos{n + 1) cp + i sen{n + 1) cp.
I Por consiguiente, la f6nnula es justa para cualquier valor entero
Sn+!=Sn+(n+ln+2
)( )(n+3 )= positivo de n.
_ n(n+3) , 1
270. Es evidente que a + b = 1 y ab = -1. Aprovechando este
- 4(n+l}(n+2)T (n+l)(n+2)(n+3)"
hecho se puede escribir que
Deaqui

S +1 = n3 + 6n 2 + 9n + 4 = (n + 1)(n 2 + 5n + 41_
n 4(n+l)(11+2)(n+3) 4(n+1)(n+2)(n+3) - - - --- -
_ (n+l)[(n+l)+3] J5 ~5
- 4[(n+l)+I][(1I+l)+2]"
obien
Por consiguiente, la f6nnula es justa para cualquier valorentero
positivo de n. de don de

269. Por 10 visto, la f6rmula es justa para n = 1. Supongamos


que ella sea justa para cierto valor de n ~ I, es decir, De aqui se deduce que si para cierto valor de n, los numeros
an- I yan son enteros v positivos. entonces, tambien an+l y' por
. I " '

(cos cp + i sen cp) n = cos ncp + i sen ncp. (1) consiguiente an+2' all+3,· .. seran numeros enteros y positivos. Pero
como a] = 1 y a 2= I, por 10 tanto, para 11 > 2 todos los valores de
Para convencerse de la justeza de la f6rmula para 11 = 1, an seran enteros y positivos.
multipliquemos am bas partes de (1) por cos cp + i sen cp. De
acuerdo con la regIa de multiplicaci6n de numeros complejos,
obtendremos:
200 PROBLEMAS DE MATEMAnCAS ELEMENTALES

271. Para n = 1 la desigualdad es justa. Supongamos que sea Abriendo aqui los corchetes, transformemos cada uno de los
justa para cierto valor de n. Multiplicando ambas partes deesta n + 1 sumandos por la f6rmula
desigualdad por 1 + an+l > 0, hallaremos que

(k )(a)k(b )n-k(a + b- n) = (k )(a)k(b )n-k[(a - k)+ (b - n+ k)] =


(1 + a1)(1 + a2 )··· (I + an )(1 + an+1)~
~ (1 +a1+a2 + ... +aJ(I +an+ 1 )=
=(k )(a)k(a - kXb )n-k + (k )(a)k (b )n-k(b - n+ k) =
= (k}(a)k+l(b)n-k + (k)(a)k(b)n-k+l (k=O, 1, ... ,n).

Como resultado obtendremos:


Puesto que la sumaa1an+l+a2an+l+'" +apn+I>O, entonces de
aqui se deduce que la desigualdad tambien es justa para n + 1.
(a+ b)n+l = (o)(aA (b)n + (o)(a)o (b)n+l + (r )(a)2 (b )n-l +
272. Ante todo nos convencemos de que la f6rmula es valida + (r)(a)l (b)n +"'+(I:Xa)k+l (b)n-k + (k)(a)k (b)n-k+l + ... +
+(~)(a)n+l (b)o +(~)(a)n (b)l'
para n = 1. En efecto, para n = 1 la f6rmula tiene la forma

Despues de reducir los terminos semejantes tendremos:


Valiendose ahora de la definici6n de patencia generalizada, se
hace evidente que ambas partes de la f6rmula (1) son iguales a
(a +b)n+! = (o)(a)o (b )n+l + l(o)+ (r )J (aA (b)n +
a+b y, par consiguiente, efectivamente tiene lugar la igualdad.
Supongamos ahora que la f6rmula es valida para cierto valor + [(1 )+ (~)](a)2 (b )n-l + ... +[(1:)+ (k:J (a )k+l (b )n-k + ... +
+ [C~l)+ (~)](a)n (b A+ (~)(a )n+l (b )0'
de n y demostremos que sera tambien valida para n + 1. De la
defmici6n de potencia generalizada tenemos:
Aprovechando, a continuaci6n, el hecho de que
(a+btl =(a+bt (a+b-n)=
=[(~)(a)o(bt +(~ )(a~(b )n-l + ... +
+(~)(a)k(btk + ... + (~)(at(b )o](a +b - n) y la identidad facil de comprobar
AWEBRAIResoluciones y Soluciones 201

De acuerdo con la f6rmula( 4) en lapagina 42 eI valor minimo


de ret) y, porconsiguiente, de ret), se obtiene en el momento de
obtenemos: tiempo
avl + bV2
(a+bLl =(n;I)(aMb)n+l +(ntl)(a~(bt + to = 2 2'
VI + V2
+{n;l )(a)2(b )n-l +... +(~:: )(a )k+Jb )n-k +... +
Valiendonos acontinuacion de la formula (3), hallamos la
+(n:l )(a t(b h+ distancia minima entre los trenes:
+ (~:: )(a t+l (b )0'
r{to) =
Porconsiguiente, hemos demostrado que si la f6rmula, expuesta ~vl +vi .
en la condici6n del problema, es valida para cierto valor de n,
entonces es tambien valida para n + 1. Pero ella es justa para n = 1, 274. En el momenta de tiempo I el coche se encuentra ala
por consiguiente, de acuerdo con el metodo de inducci6n distancia de 40 t Ian del punto A y la motocicleta a la distancia
matematicacompleta, es valida para todos los val ores enteros de 3; (2 + 9 Ian del mismo punto. Por consiguiente, la distancia
positivos de n.
entre ellos es igual al valor absoluto de la diferencia 16P + 9
273. Sea r(/) la d istancia entre los trenes en el momento de tiempo «40t. Designando esta diferencia por y(t), tracemos el gratico
I. Entonces del trinomio cuadrado y(t) (fig. 4). Este grafico representa una
parabola que, para los valores II = -i- Y t2 = 2-i-, intersectael
r2(t)=(a-v1t)2 +(b-v2t)2 = (V12 +vnt 2-
eje t. Del grafico esta claro que la ordenada de mayor valor
- 2(av1 +bv2)t + a2+ b2. absoluto y, para la condicion 0::;; t ::;; 2, corresponde al vertice
de la parabola. El vertice de la parabola se encuentra en el eje de
Observemos que si r(t) tiene el valor minimo para 1 = to'
simetria que cruza el eje ten el punto
entonces, tambien r(t) adquiere el valor minimo cuando t = to'
Esjusto y viceversa. El problema se reduce a la determinacion II + 12 5
to = - - = - .
del valor minimo del trinomio cuadrado r (t). 2 4
202 PROB LEMAS DE MATEMATICAS ELEMENTALES

As! pues, la distancia maxima entre el tren y la motocicleta se 276. Primera resolucion. Por 10 visto es suficiente examinar
alcanza al cabo de 1 hora 15 minutos despues de iniciar el solamente los valores positivos dex. De acuerdo con laconocida
movimiento y es igua\ a 16 kIn. desigualdad (3), pag. 20 tenemos:
ax 2 +b 1~2- ~
275. Designemos la expresi6n que se - - - ~ 'l ax b = x -v ab. (1)
y analiza por y y transforme-mosla de 2
la sigu iente manera: Por consiguiente, para todos los valores de x > 0
y=21 og 4 x+ x x
y = ~ I = ------,====. (2)
~ + 1221og2 x(1Iog8- 2 1ogx )= ax 2 +b 2x\l ab 2-J ab
t
=llogZ x(zlogZ x-12 2 1ogx+36)=
Puesto que (1) pasa a ser una igualdad cuando ax1 = b,
=zlog2 x(6- Z1ogx)2 . I.
entonces, para Xo =~! tenemos:
Hagamos a continuaci6n
2 10gx =z, de modo que O~z~6. 1
Entonces, el problema se reduce a la Yo = 2-J ab . (3)
determinaci6n del valor maximo de
lavariable En virtud de (2) este es precisamente el valor maximo de la
funci6n.
Segunda resoluci6n. Resolviendo la relaci6n
Es suficiente hallar el valor maximo de z(6-z) con la condici6n x
de que 0 ~ z ~ 6, puesto que cuanto mayor es un numero positivo y= 2 (4)
ax +b
tanto mayor es el cuadrado de este numero. El trinomio cuadrado
z(6-z)=- {z- 3)2+9 a1canza su valor maximo paraz=3.As! pues, respecto a x, obtendremos:
el valor maximo se consigue cuando z = 3 yes igual a 81.
1 ± ~I- 4aby2
x=-- - - - . (5)
2ay
ALGEBRAIResoluciones y Soluciones 203

De la formula (5) se desprende, que para todos los val ores x2 + 1 r;:;
--:2:-2+2,, 2 (2)
reales de x debe cumplirse la desigualdad 1 - 4aby2 :2: O. De x+1
aqU) Yel signo de igualdad en esta formula tiene lugar cuando
(6)
x=J2-l.
Puesto que para cierto valor real de Xo la funcion (4) adquiere
278. Tomemos el eje numerico y marquemos en ellos puntosA,
elvalor Yo = 2"J~a b (de la formula(5)hallamos que Xo = ~ (f),
I a B, C y D correspondientes a los numeros a, b, C Y d. EI punto
entonces, en virtud de (6) este valor es el maximo. con la abscisa variable x 10 designaremos por M (fig. 5)
Examinemos los cinco casos siguientes.
277. Por medio de las transformaciones evidentes, obtenemos:

~2 + 1 = x-1 :. _2_ = -2 + [x + 1 + _ 2_ ] .
M

x
A
a
B

b
C
.
c
.
D
d
..
x+l x+l x+l

En virtud de la desigualdad (3), pag. 20 Fig. 5

1) x::;; a; entonces
x+ 1+ _ 2_ :2: 2 /(x+ 1)-{2 ) = 2J2, (1)
x+l ~ x+l
<p (x) = MA + MB + MC + MD = AB + 2MB + 2BC + CD.
con la particularidad de que el signa de igualdad en (1) tiene
lugar solamente en el caso cuando De aqu! esta claro que <p (x) adquirira su valor minimo en el
2 - caso en que eI punto M coincida con el puri.toA y que este valor
1+ x = - -1-' es decir, para Xo = -02 - 1. sera igual a
x+
3AB + 2BC + CD.
As) pues, para todos los valores de Xo =:2: 0
204 PROBLEMAS DE MATEMATICAS ELEMENTALES

2) Q < X s, b; en este caso 279. Sea r el m6dulo y cp el argumento del numero complejo
z(r~O,Oscp<2n). Entonces, z=r(coscp+isencp) y la
cp (x) = AM + MB + MC + MD = AB + 2MB + 2BC + CD. ecuaci6n dada toma la forma

La funci6n cp (x) adquiere su valor minimo cuando el punto M r2(cos 2cp + i sen 2cp) + r = O.
coincide con el punto B; este valor es igual a
De aquf, 0 bien r =0 y Z = Zj = 0, 0 bien r cos 2qJ+ 1 +
AB+2BC+CD + ir sen 2<p = 0 y, por consiguiente,
3) b s, x s, c; para estos valores de x la funci6n cp (x) es
constante e igual a sen 2cp = 0,1
AB+2BC+CD. rcos2cp + 1 = 0.)
4) c s, x < d; la funci6n cp(x) adquiere su valor minimo para A la primera ecuaci6n la satisfacen los valores
x = c; este valor es tambien igual a
cp = 0, n / 2, n, 3n / 2 y puesto que en virtud de la segunda
AB+2BC+CD. ecuaci6n cos 2cp < 0, quedan solamente los valores
5) x ~ d; el valor minimo de la funci6n cp (x) es igual a cp =~ y cp = 3n / 2. Ademas, de la segunda ecuaci6n hallamos
AB + 2BC +3CD. en ambos casos que r = 1, asf que obtenemos dos soluciones
mas:
Comparando los resultados obtenidos vemos que el valor
minimo de la funci6n cp(x) es igual aAB+ 2BC + CD, 0 bien ( 3n . 3n'\ .
z~
J
= llcos-
2
+ I sen -2 ) = -I.
b- Q + 2(c - b) + d - c = d + c - b - Q.

La funci6n cp (x) adquiere este valor en el caso cuando 280. Representemos a Z en la forma z = x + iy. Entonces, la

b s xsc. ecuaci6n I~=~I = I toma la forma


ALGEBRAi'Resoluciones y Soluciones 205

Queda colocar en la f6rmula dada en vez de z su valor.


Tendremos:

2
De aqui x = 6 y, par 10 tanto, Z = 6 + iy. Colocamos este valor
l-z
= I_l+i = I-i
2
en la ecuaci6n 1~=1~1 =}. Entonces, despues de las
correspondientes simplificaciones la ecuaci6n toma la forma y, a continuaci6n,
y 2 -25y+136=0.
Deaqui

25±-J625-4·136 25 ± .)625=5« 25±9


Yl,2 = Observemos que para n z 2 tenemos i 2n = ~4 )2 n-2 = 1. Por
2 2 2
consiguiente, en virtud de (1 ), para n z 2 , tenemos que
esdecir'Y I = 17;y2 =8.
Respuesta: Zl = 6 + 17i; Z2 = 6+8i.
l-z 2
n +! 1 Y P=(I+l). ( 1- 2 1
=1- 2 2n 2n
J.

281. Para simplificar la escritura hagamos l;i = z. EI producto


que se examma Para n = 1 tenemos:

1-z
2n+!
=1-
II(2.J2 =45
tiene la misma forma que el producto en el problema 230. Respuesta:
Designemos este producto, para simplificar la escritura, por P.
Procediendo del mismo modo que en el problema 230,
hallaremos:
2n+l
1- Z
P=---
l-z
206 PROBLEMAS DE MATEMATICAS ELEMENTALES

282. Puesto que la subs- la + hil = 1 Y a + bi * -1.


traccion de numeros
complejos geometrica- Demostremos la necesidad. Supongamos que se satisface la
mente se efectua por la condie ion (1). Entonces
regIa del paraielogramo, 11- ix l
el modulo de Ia diferencia la + bil = _I_ I = l.
de dos numeros comple- 11 + ixl
jos !z'-z'1es igual a la
distancia entre los corres- puesto que 11- ixl = 11 + i~ = ~. A continuacion,
pondientes puntos del
1- ix
-----';:f-----'-----~~ plano complejo. Por - - *-1
consi-guiente, la condi- 1+ ix '
. x, cion Iz- 2Sil s ISla sa- ya que de 10 contrario tendrlamos que 1 - ix = -I - ix, es decir,
Fig. 6 tisfacen los puntos del 2= o.
plano complejo que se encuentran dentro y en el margen del Demostremos la suficiencia. Supongamos que sea
circulo con el centro en el punto Zo = 2Si y de radio igual a 15 la +hil = 1 Y a + bi * -l. Hagamos arg{a + bi) = a, donde
(fig. 6). del dibujo se ve que al numero de argumento minimo Ie - 11: < a < 11:. Observemos que a *
11: en virtud de la condici6n
corresponde el punto Z I en el que la recta trazada desde el punto *
a + bi -1. Ahora tenemos:
oes tangente a la circunferencia. Del triangulo rectangulo OZI Zo
hallamos que xI = 12 e YI = 16. El numero buscado sera a + bi = la + bil(cos a + isena + isena). (2)
zi =12+16i. Pero
l-tg 2 ~ 2tg~
283. Demostremos que para representar el numero complejo cosa = 2, sena = 2.
a + bi en la forma 1+ t o02 ~2 1+ tg 2 ~
1- ix Colocando estas expresiones en la parte derecha de la formula
a+bi=--- (1)
1+ ix (2), tendremos:

es necesario y suficiente que . (I + i tg ~ Yt8%)


1 + i tg ~
a + bl = ~I+ito~ )(1-ito~ = I-ito~
1 - ix
= -l+ix'
02 02 02
ALGEBRAIResoluciones y Soluciones 207

donde x == -tg 'i. 285. El angulo entre dos rayos vecinos es igual a 2n1t • Designemos
por dI' d 2• ... las distancias desde A hasta las bases de las
284. Sea z == r(cos<p+ isen<p); entonces perpendiculares bajadas sucesivamente a los rayos que parten
del punto A (fig. 7). Es evidente que tenemos:
~ ~J(;2 cos2<p + 1)2 + ~2sen2<p)2 == ~ r4 + 2r2 cos2<p + 1;
IZ2 + ==

I z+~Jz2+11
Zl r
==1;
La longitud de la k-esima perpendicular es igual a

r 4 + r2 (2cos2<p-l)+ 1 == O. '\ k-l


2TC 2TC 2TC
Lk =:: d k - 1 sen- == dsen- (
n n
COS-j
n
1

Hagamos r2 == t; izl adquiere su valor maximo cuando


adqu:ere ru vabnn fucin 0 t. Tenemos:

1- 2cos2<p ± ~(I- 2cos2<p)2 - 4


t== .
2
Puesto que nos interesa el valor maximo de t, delante de la
raiz debe tomarse el signa mas. Es facil ver que el valor maximo
de t se consigue cuando cos2<p == -I, es decir, cuando

<p == ~ + 1m. Este valor es igual a 3+f .Por consiguiente, el valor Fig. 7
maximo Izi es igual a La longitud total de la quebrada de m eslabones sera

2TC I 2TC
dsen- ~ Il+cos-+lcos -
( 2TC) 2 +"'+,(2TC)
cos -
m- 1 1 .
nl n n ~ n
~ ~
208 PROBLEMAS DE MATEMATICAS ELEMENTALES

La longitud L de la quebrada enredada infinitamente se 287. Puesto que pes divisible entre 37, se puede escribir
obtendni al aumentar ilimitadamente my se expresani con la
suma de los terminos de una progresi6n geometrica infmitamente
p = 100a+ lOb +c = 37k,
decreciente cuyo denominador es q = cos 211n y el primer termino donde kes un numero entero. Es evidente, luego, que

d sen 211 :
q = 100b + 1Oc + a = lOp - 999a = 370k - 37·27 a.
n
sen 211 TC Porconsiguiente, q tambien es divisible entre 37.
L =d n = d cotg - . Razonamientos amilogos sirven tambien para el numero r.
l-cos 211
n
n
Al aumentar n la longitud I, aumenta y al aumentar ilimitada- 288. Tenemos:
mente n aumenta ilimitadamente L.
A =n 3 +(n+I)3 +(n+2)3 =3n 3 +9n 2 + 15n+9.
286. Primera resoluci6n. Sea labcde el numero (aqui las letras Por 10 visto, es suficiente demostrar que
a, b, c, dye significan cifras de los 6rdenes respectivos).
Evidentemente, e = 7, ya que 1abcde x 3 = abcde I. Despues de B = 3n 3 + 15n = 3n(n 2 + 5)
multiplicar 7 por 3 el dos pasa al siguiente orden, por eso, el es divisible entre 9. Si n = 3k, donde k es un numero entero,
producto d x 3 debeni terminar con la cifra 5. Por consiguiente, entonces, B es divisible entre 9. Para n = 3k+ 1, n 2+5 =
d = 5. Tenemos que labc 57· 3=abc 571. Razonando =9k2+6k+6; para n = 3k+2, n 2+5=9k2+ 12k+9. En ambos casos
amiIogamente hallamos que c=8, b=2 y, por fin, a = 4. EI numero
n2+5 es divisible entre 3. Por consiguiente, en todos los casos B
buscado es 142857.
esdivisible entre 9.
Segunda resoluci6n. Supongamos de nuevo que 1abcde es
el numero buscado. Hagamos abcde = x, entonces, el numero
289. Primera resoluci6n. La suma Sn se puede representar en la
buscado es igual a 105 + x. Segun la condici6n del problema
siguiente forma
tenemosque
Sn = n 3 + 3(n 2 + 2n + I)- n = (n -I)·n· (n + 1)+3(n + 1)2.
(105 + x )3 = lOx - l.
El primer sumando es divisible entre 3, puesto que es el
de dondex = 42857. Por consiguiente, el numero buscado es
producto de tres numeros enteros sucesivos (uno de ellos es
142857.
ALGEBRAI'Resoluciones y Soluciones 209

obligatoriamente mUltiple de tres). Por consiguiente, tambien la


120 _n{n+l) (n-l)n (n-2)(n-l) ~ 2·3 ~
9.ll11 as es divisible entre 3. - + + + ... + + + .
n
222 222
Segunda resolucion. Realizamos la demostracion por
induccion. Para n = 1, S[=12 es divisible entre 3. Supongamos La parte derecha de la igualdad es igual a
que para cualquier valor de n la suma Sn es divisible entre 3.
Tenemos: n{n + 1){n + 2)
6
(vease el problema 266, pagina 198), por consiguiente, para la
Por consiguiente, Sn + I tambien es divisible entre 3. determinacion de 11 obtenemos Ia ecuacion
n(n+1)(n+2)=720. (1)
290. Las bolas se han colocado en la base de la pinimide en
forma de un triimgulo equihitero. Supongamos que ellado de Esta ecuacion tiene una solucion evidente n = 8. Para hallar
este triangulo contiene n bolas. Entonces, en la base de la pirfunide las demas soluciones de esta ecuacion pasamos 720 a la parte
izquierda y dividimosel polinomio obtenido entre n-8. EI cociente
habnin n + (n -1) +(n -2)+ .. . +3+2+1= n(~+l) bolas. de la division sera igual a n 2 + I In + 90. Puesto que las raices
La segunda capa de la pinimide tienc de este ultimo polinomio son irreales, la ecuacion (1) no tiene
mas soluciones enteras que n = 8. As! pues, en la base de la
(n -1)+ (n-2)+ ... + 3 + 2 + I = (n -1}n figura piramidal hay
2
bolas. La tercera tiene n(n+l)=36
2
(n-2){n-1) bolas.
2
291. Puesto que Ia cantidad de cajones llenos es igual am, la
bolas y as! sucesivamente. La ultima, la capa superior, se com pone cantidad de cajones metidos sera igual a mk. De aqu) se deduce
de una sola bola. En total en la pinimide hay 120 bolas. Por que la cantidad de todos los cajones (junto con el primero) es
consiguiente. igual a mk+ I. Por consiguiente, la cantidad de cajones vados es

mk + 1- m = m{k -1)+ l.
210 PROBLEMAS DE MATEMATICAS ELEMENTALES

GEOMETRIA 293. Sea AD la bisectriz del angulo recto A en el /). ABC Y


DE 1. AC (fig. 9). Puesto que
n x
L DAE = "4' entoces, AE = DE = .fi'
A PLANIMETRlA donde x = AD es la longitud buscada. Es evidente que
x
1. Problemas de calculo b- 72
ED CE
292. Tracemos la bisectriz del anguloA (vease la fig. 8). Esta AB CA b
Deaquf
bisectriz intersectani ellado BC en el punto D y la dividira en
partes proporcionales a b y c. Observemos a continuaci6n que bc.fi
el /). A CD es semejante al /). ABC puesto que tiene el angulo C x=-- .
b+c
comun y el angulo CAD es igual al B. De aquf
294. En el trianguloABC (fig. 10) 0 es el punto de intersecci6n
AC BC b a de las medianas AD y BE; AC = b, BC = a. Hallemos AB = c.
0
CD AC ab b Supongamos que OD = x Y OE = y. Valiendonos de la
b+c propiedad de las medianas, de los triangulosAOB, BODy AOE
Por consiguiente, hallaremos:
2 2 b2 2 2 2 2 2 ')
I ') 4x +y = - , 4x +4y =c, 4x +16y =a-.
a = '1/ b- + b. 4
Eliminando x e y, obtendremos:

B
C

A~B :~c
5
Las condiciones de existencia del triangulo con los lados a, b
c
y c, adquieren la fonna
Fig. 8 Fig. 9
GEOMETRiA/Planimetria (Resoiuciones y Soluciones) 211

1 1
La primera desigualdad, evidentemente, se cumple cualesquiera SACD + S DCB = 2. by sen 2a + 2. ay sen 2a
que sean a y b, mientras que la segunda se transforma en la 1 1
siguiente: S ACE + S ECB = - bx sen 2a + -ax sen a,
2 2
5
a --ab+b 2 <0.
2
1 1 1
2 S ACD + SDCE + SECB = 2 by sen a + 2 xy sen a + 2ax sen a.

~
Ba

AbE
c

y
ox
D

Fig. 10
C
LiTI
A
b
~v

DEB

Fig. II
c
x
a Igualando las partes izquierdas de estas igualdades y teniendo
en cuenta la condici6n del problemaobtendremos un sistema de
tres ecuaciones:

2a cos a = x+a~,l

Resolviendo esta desigualdad respecto a ~ , definitivamente


2b cos <X ~ ~, l
Y +b
x I
obtendremos:
1 a ~=m.1
- < - <2. y 11 J
2 b
Resolviendo este sistema obtendremos:
295. Admitamos que L.ACD = LDCE = LECB = a y
CE = x, CD = y (fig. 11). Para el area del triangulo ABC se
pueden escribir las tres expresiones siguientes:
x= (n 2 - ml lab
n(bm - an) ,

296. Designemos por S el area del triangulo dado ABC (fig. 12)
y hagamos ~~ = x. Entonces el area del !l ADE sera igual a
212 PROBLEMAS DE MATEMATICAS ELEMENTALES

x 2S y el area del ~ ABE, igual a xS. La condici6n del problema


conduce a la ecuaci6n

xS-x 2S= k2 , Sumando estas igualdades miembro a miembro, hallaremos:


resolviendo la cual obtendremos:
S = (JS; +JS; +jS;Y-
I ,
x=
1 ± ,11- 4f 298. EI tercer lado del triangulo, igual a la altura bajada a este
1
lado, 10 designamos por x. Haciendo uso de dos expresiones
2
para eI area del triangulo dado, obtendremos la ecuaci6n
Elproblemaessolublesi S > 4k2 ytieneunaodossoluciones
- 2 2
en dependencia de que sea S > 4k 0 S = 4k 1 2 /b+c+x c+x-b x+b-c b+c-x
respectivamente. 2x =~ 2 ' 2 ' 2 ' 2 .
B Resolviendo esta ecuaci6n, hallaremos:

~c E
Fig. 12 Fig. 13 La condici6n indispensable para la solubilidad del problema
es la condici6n
297. Designemos por S el area del triangulo dado ABC. Los (2)
trianaulos con areas iauales as1, S2 y S., obtenidos segun Ia
I::> I::> 0
Si se cumple esta condici6n, ambos val ores de x 2 en (1) son
construcci6n indicada en la condici6n del problema, son
positivos. Es facil comprobar que si se cum pie (2) tam bien se
semej antes al ~ ABC (fig. 13). Por eso, sus areas son entre s i
cumpliran las desigualdades
como los cuadrados de sus lados semejantes, de don de
GEOMETRiAIPlanimetria (Resoluciones y Solucilllles) 213

ademas, el signa de igualdad tendni lugar solamente en el caso Tenemos:


en que seax = 0. Esto ultimo tiene lugarcuando, siendo b = c, en
la igualdad (1) se toma delante de la raiz cuadrada eI signo menos.
1 1
SB[AC: =2 AB,.AC, senA=2ABcoSA.ACcosAsenA=
Por consiguiente, en el caso cuando b = C el problema tiene una
sola solucion 1 2 ?
= -AB· AC sen A cos A = SABCcos- A
2

Si b :t:- c, el triangulo existe solamente en el caso en que se yanalogamente


curnpla la desigualdad (2). Resolviendo esta desigualdad respecto
a ~, hallaremos que es equivalente a las siguientes desigualdades:
Colocando estas expresiones en (1), despues de las transfor-
(3) maciones evidentes, obtendremos:

SABC
[ [ [ = 1 - cos 2 A - cos 2 B - cos 2 C.
Por consiguiente para b :t:- c existen dos triangulos, si ambas (2)
desigualdades (3) se cumplen con el signo <, y uno, cuando por SABC
10 menos una de las desigualdades se cump!e el signo =.
Si el ~ ABC es obtusangulo (fig. 15), en vez de (1) tendremos:
8

y correspondientemente, en vez de (2)

SABC
I I [ = cos 2 A + cos 2 B + cos 2 C - I. (3)
A IC.- _ _---L_ _ _---''''' C A "------''----------'''''' C SABC
8, 8,
Fig. 14 Fig. 15 Por fin, si el ~ ABC es rectangulo, S A[ B[C[ = 0, 10 que, como
es facil comprobar, resulta tambien de las formulas (2) y (3).
299. Supongamos al principio que el ~ ABC es acutangulo
(fig. 14). Entonces 300. 1) Sean BO y CO las bisectrices de los angulos internos del
~ ABC (fig. 16). Es facil ver que los triangulos BOMy CON
son isosceles. Por consiguiente.lvfN = BM + CN.
214 PROBLEMAS DE MATEMAnCAS ELEMENTALES

2) La dependenciaMN = BM + CN es valida tambien para el regulares y la suma de sus lados es igual al lado AB = a del
caso de bisectrices exteriores. triangulo ABC. Por consiguiente, la suma de sus alturas es igual a
3) Si una de las bisectrices es interiory la otra exterior (fig. 17), la altura del L\ ABC, por 10 tanto,
entonces, de los triangulos interiores BMO y CNO halla- ~

mos que MN = CN - BM , cuando CN > BM, y que PD+ PE +PF = a."j 3


MN = BM - CN , siendo CN < BM As! pues, en este caso 2
La SUl11a BD + CE + AF es igual ala suma de los lados de los
MN = !CN - BM j. triangulos rayados mas la suma de las mitades de estos lados 0
B B sea,
3
BD+CE +AF = - a.
2
Por consiguiente,

A ""------'--------'-----'---"'" C
o PD+PE+PF
Fig. 16 Fig. 17 BD + CE + AF = -fj'
Los puntos My N coinciden solamente en el caso (3), si el L\ 302. Sea 0 el punto de interseccion de las medianas en el L\
ABC es isosceles (AB = AC) . ABC (fig. 19). En la prolongacion de la mediana BE trazamos
ED = OE. Segun la propiedad de las medianas los lados del L\
301. Tracemos a traves del punto P tres rectas paralelas a los -t
CDO son iguales a de los lados del triangulo cOl11puesto por
lados del triangulo (fig. 18). Los tres triangulos formados (rayados las medianas. Designando el area de este (dtimo por SI'
en el dibujo) tal11bien son tendremos: 9
B SI = - SCDO'
4

£
B
~ Por otro lado, el L\ CDO esta formado por dos, y el L\ ABC
~. por seis triangulos equidimensionales al L\ CEO, Por eso,
.4~C 1
A E C D SCDO = -:; S.,.jBC .
,)
Fig. 18
GEOMETRiAIPlanimetria (Resoluciones y Soluciones) 215

Por consiguiente,
AB = r eotg a + coto~.
~ 3 2 °2
Oeaqui
SABe 4 a !3 2R
eotg - + eotg - - = - = 5.
2 2 r
303. Supongamos que sea ABC el triimgulo dado (fig. 20). EI
Ademas,
area del L1 COB es igual a ! ar, y el area del L1 COA es igual a a . 13 =1t- ( a 13\
--t- - y cotgl - + - I= I,
~ br. Sumando estas es decir,
224 \. 2 2)
C
a
eotg-eot o -
13 ~ 1
A
2 °2
a 13 = 1,
coto ~ - + eotg -
°2 2
de donde
A LL----~----~B
cotg a cotg~ = 6.
Fig. 20 Fig. 21 2 2

magnitudes y expresando el area del L1 ABC por la fonnula de Por consiguiente, eotg a y eotg ~ son iguales a las rakes
2. 2
Heron, obtendremos: de la ecuaeion cuadratiea x 2 ~ 5x + 6 = O.
,., Defmitivamente obtenemos:
r = --'"'- ~b ~ p(]J ~ a)(]J ~b )(]J ~c),
a+ 1 1
a = 2aretg-, 13 = 2arctg -,.;-.
donde 2 -'

p= ~ (a+b+c) . 305. Oesignemos par a y bios lados del reetangulo dado y par
2 <p el angulo entre los lados de los reetangulos dado y eireunserito
304. SeaR el radio de la eircunferencia circunscrita y r el radio (fig. 22). Entonees, los lados del reetangulo cireunserito seran
de la cireunfereneia inserita. Entonees (fig. 21) AB = 2R Y igua\esa
216 PROBLEMAS DE MATEMAnCAS ELEMENTALES

a cos <p + b sen <p y a sen <p + cos <p > BC Y una sola soluci6n cuando AB = Be. (EI punto El en la
fig. 23 corresponde a la segunda soluci6n).

~r\21\E'
307. Ellado lateral se ve desdeel vertice de la base inferior bajo
"i
el angulo (fig. 24) y la linea media es igual al segmento desde
este vertice hasta el pie de la altura bajada desde el vertice
E B opuesto, es decir, h cotg ~. Por consiguiente, el area del trapecio
esiguala
Fig. 22 Fig. 23 2 a
S:=: h cotg-.
2
Segun lacondici6n del problema
308. Los puntos medios de las diago-
(a cos <p+b sen <p)(a sen <p+b cos <p):=: m 2 , nales E y F del trapecio se encuentra
L~- -
sobresu linea media MN(fig. 25). Pem,
de donde hallamos que
ME:=: FN:=: t. Por consiguiente,
- - - - - --

2{m
2 -ab)
sen 2<p : = : .
a 2 +b 2 EF :=: b + a _ a == b - a .
La condici6n de solubilidad del problema sera 2 2
Fig. 24
o: ; sen 2<p ::;; 1, 10 que es equivalente a las siguientes desigual-
dades: 309. EI paralelogramo esta compuesto por 8 triangulos
~ a+b equidimensionales al trianguloAOE. La figura (el octaedro)
"ab ::;; m::;; r;;' obtenida por medio de la construcci6n indicada tam bien esta
,,2
compuesta por 8 triangulos equidimensionales al MOQ (fig.
306. Si el L.AED == L.DEC (fig. 23), tam bien el
L.CDE :=: L.DEC, de donde CE = CD. Por consiguiente, E es 26). Puesto que OP :=: 10A (por la propiedad de las medianas
el punto de intersecci6n del lado AB con la circunferencia
circunscrita desde el centro C con el radio CD. EI problema es en el tJJAE) y OQ:=: tOE, entonces
soluble si AB ~ BC, ademas, tiene dos soluciones cuandoAB>
GEoMETRiAIPlanimetria (Resoluciones y Soluciones) 217

I 312. El area buscada es igual a la suma de las areas de dos


SpoQ =6 SAOE ' sectores euyos angulos son 2a y 2/3 (fig. 28) menos el doble
Por eonsiguiente, la relaei6n buseada es igual a i. del area del triangulo con los lados R, r y d:

S = R 2a + r2/3- Rd sena.
a
Para hallar los angulos a y /3 tenemos dos ecuaeiones:

~ R sen a = r sen /3,

" I'
R cos a + r cos /3 = d.

-1'b;,~
b D
Fig. 25 Fig. 26 B 0 c
1-

310. Es evidente que KLMN es un paralelogramo (fig. 27),


~ /N-·----·. D
ademas KL = %A Q. Por eonsiguiente, A s
Fig. 27 Fig. 28
2 212 12
SKLMN =SSAQCS =S2 a =Sa. resolviendo las cuales, hallamos:
311. A las dos euerdas dadas de longitudes 2a y 2b les d 2 +R2 _r2
eorresponden los angulos eentrales 2a y 2/3, donde eosa=---- -
2Rd
a b d 2 +r2 _R2
sen a =- , sen /3 = - . cos /3 = - - - --
R R 2rd
Por consiguiente,
EI areo igual a 2 (a ± /3) esta formado por la euerda 2e, donde
218 PROBLEMAS DE MATEMATICAS ELEMENTALES

313. Sea K el punto de contacto de las dos circunferencias de 314. Sean 01 y 02 respectivamente los centros de las circunfe-
radios r y r I y P el pie de la perpendicular bajada desde el centro rencias de radios R y ry 03 el centro de la tercera circunferencia.
02 de la terceracircunferencia a 001 (fig. 29). HaciendoKP=x, Supongamos que seax el radio de la tercera circunferencia y P
tenemos: el punto de tangencia de esta con el diametro 0P2 (fig. 30).
(1) Aplicando el teorema de Pitagoras a los triangulos 0,0] y - j

0P3P, obtendremos la igualdad

02032 = 03 P 2 + ( 02 0 l +~OP32 -03 P 2)2


J

Colocando aq u i los valores 0 2 0 3 = r + x, 03 P = X,

0 2 0 1 = R-r, OP3 = R-x, obtendremos una ecuacion res-

r
pecto a la incognita x:

Fig. 29 Fig. 30 (r+x)2 = x 2 +( R-r+~(R-x)2 _x 2


EI valor de x se determina de la ecuacion Resolviendo esta ecuacion, hallaremos que
R-r
x=4Rr )2'
(R+r
yes igual a
315. Sean 01' 02 Y 03 los centros de las tres circunferencias
x= -
r-rl
- R. °
iguales y el centro del circulo de radio r (fig. 31). Designemos
r + rl por SOl 0 20 3 , el area del i'l0P203, por SAOlB el area del
Colocando este valor de x en (1), obtendremos: sector A02 B; entonces, el area buscada sera

4~rrl S = ~ (SOP?03
j -
- 3S40 ,B
-
- n:r2 ). (I)
AB=--R.
r +r[
GEOMETRiAIPlanimetrfa (Resoluciones y Soluciones) 219

SiR es el radio com un de las 002 = (a + b) - b = a.


tres circunferencias, entonces
Haciendo 0 1D = x, escribamos la segunda igualdad de (1 )
j3 enlaforma
R= - (R+r).
2
(a + r)2 + b 2 - 2bx = (b + r)2 + a 2 - 2a(a + b - x),
De don de
de donde hallaremos que

r-)
0.,

Fig. 31 j3 (3+2--J3 r. a-b


R=-----r:::r= x=a+ - - r.
2-"'J3 a+b
A continuaci6n, hallamos:
Ahora, la primera igualdad de (1) tom ani la forma de una
ecuaci6n con una inc6gnita r

( a+b-r ) 2 = ( a+r )2 +b 2 -2b ( a+ -a - -r).


b "\
\. a+b

y por la f6rmula (1) obtenemos definitivamente:


a

~
r - (23 "\ l
yr Jr2.
~
A
S = l12+ 7-J3 - \.6 + 2-13
D 0 O2 I]
316. Sea 03D ..1 OP2 (vease la fig. 32). Tenemos que

2 2 2 Fig. 32 Fig. 33
00 3 =OP3 +Op -20p·op=
= Op; +00; -2002 . D02 , Resolviendo esta ecuaci6n, obtendremos definitivamente que
ab(a + b)
r = --=----'-----'-_::_
a 2 + ab + b 2 ·
220 PROBLEMAS DE MATEMAnCAS ELEMENTALES

317. Designemos poray b lasdistancias desde el punto dado A Supongamos que sea r el radio comun de las circunferencias
hasta las rectas dadas I J y l2' Y por x e y las longitudes de los que se examinan. Entonces, ellado del poligono construido por
catetos del triangulo buscando (fig. 33). Observando que nosotros seni igual a 2r y su area sera

~ = sen <p, ~ = cos <P, tendremos dos ecuaciones: 2


a = nr cotg - .
1t
n
1 2
-xy=k
2 .
Ad mltamos, . .,
a contmuaclOn, que
A
jJ = 1t{n - 2) sea e I angu
' I
0
n
Transformando estas ecuaciones, obtenemos el sistema interior del poligono. Para el area buscada S de la "estrella"
obtenemos la expresi6n
xy = 2k 2 ,}
r2 2 1t r2
b 2 x 2 +a 2 y2 = 4k4. S=a-n - j3=nr cotg--n - j3.
2 n 2
Resolviendolo, obtendremos que
Luego, es facil ver (vease la fig. 34) que

x =~I~k2 +ab ± ~e -abl, a 1t


--r =r tg -.
bl 2 n
k' ~- ,
y = -1>1 k 2 + ab +- -J k 2 - ab l · c
a I

, (} .
El problema es soluble si k 2 ~ ab Y tiene dos soluciones

~
siendo k 2 > ab Yuna sola soluci6n cuando k 2 = abo

318. Uni.::ndo los centros de las circunferencias obtendremos un


poligono semejante al dado. El centro del poifgono obtenido coin-
Fig. 34 Fig. 35
cide con el centro del poligono dado y sus lados son respecti-
vamente paralelos a los lados del mismo (fig. 34).
GEoMETRiNPlanimetrfa (Resoluciones y Soluciones) 22 I

de donde es decir, los radios de las circunferencias forman una progresi6n


aritmetica cuyo denominador es
a
l-sen -
2
y, por consiguiente, a
1 +sen-
2
2 n cotg ~ - (n - 2)~
S=~ n 2 321. Supongamos que el angulo minimo entre los rayos reflejados
y el plano P sea igual a a (fig. 37). Semejante lingulo 10 forma el
4 (l+tg;f .
rayo que pasa por el borde del espejo C despues de ser una vez
re£lejado en el punto B. Segun la condici6n del problema
319. En las denotaciones de la fig. 35 tenemos que CF II DA; por consiguiente, LOCB = LOBC = a. De la
condici6n de reflexi6n en el punta B se desprende que
LCGF=~(FA+AC), LCDB = ~(FA + BC) . LOBF = a. Por esta razon, en el triangulo OBFtenemos:
2 2 o
EI cuadrilatero DEGF sera inscrito si, y s610 sl,
LCGF = LCDB, es decir, si AC = BC.

320. Sea 0 el vertice del angulo agudo a y Ok el centro de la


k-esima circunferencia (fig. 36). Entonces,

y
Fig. 36 Fig. 37

LBOF = 2a, LOFB = 180 0 - 2a -a = 180 0 -3a.


Por consiguiente, a
I-sen- Designemos la distancia desde el espejo hasta el plano por h,
2
a y el radio del circulo iluminado AD por r. Puesto que el radio del
I +sen- espejo es igual aI, entonces
2
222 PROBLEMAS DE MATEMATICAS ELEMENTALES

Luego
h
-~ =tga. (1)
r -J
tga= sen 2a = ~ =~=2--fj,
Del triangulo OBF, segun el teorema de los senos, hallamos: I + cos 2a '1'3
1+ _ 2 + ~v/3
2
por 10 tanto, de (1) obtendremos:
OF = sen a_.
sen 3a 1 (1+-03
') (2- -voJ
<;:;-) = --fj-l
h= ~ -.
En virtud de la semejanza de los triangulos CBF y DBA, sus 2 2
alturas son proporcionales a sus lados, asf que 322. Es necesario examinar diferentes casos en dependencia del
AD h+sen 2a valor de la relaci6n ;.
= ,
FC sen 2a
1) ; ~ J2. Las circunferencias no cortan al cuadrado, S = a 2 .
o bien
r h+ sen 2a
1 + --~
sen a sen 2a (2)
sen 3a

Resolviendo conjuntamente las ecuaciones (1) Y(2), hallamos:


2 cos 2a
r=----~
a
2 cos 2a-1
Fig. 38 Fig. 39
Colocando aquf la magnitud dada en el problema a = 15°,
obtendremos: 2) '~-" ::; L < '\1~
2. Es evidente que en este caso S = a 2 - 8cr
"- a '
donde cr es el area del triangulo curvilfneo rayado (Fig. 38).
r =- -v3
,==--- = -3 +-,J3
--. Tenemos:
--J 3 -J 2
GEOMETRiNPlanimetria (Resoluciones y Soluciones) 223

donde <p = arcsen ~. Para hallar x observemos que Por consiguiente,


,- i2 2 ( / -)-- -2 )
x-v 2 + Ij r - a = a, I
I
2 2

al \j 4r- - a - a
dedonde 1 2 -y 4r - a - a \
0= - r arcsen ~ -
/ 2 2 2 2'\j 2r 8
a -',,;r -a
X=
4) ~ :S h .EI area buscada es igual a cero.
Por consiguiente,

B C

3)~<L
"'; 2 a
<Ji
2
. Aqu! S=80
'
donde 0es el area del A
~ D

triangulo curvilfneo rayado (fig. 39). Tenemos: Fig. 40 Fig. 41

1 2 1 a 323. Tenemos (fig. 40):


0=-r <p-- - x
2 2)2'
donde (1)
x
<p = arcsen - . A continuacion,
r
Observando que S3 = SI = A~
S2 S[ OC
a ~
= - +xo.}2, de donde S3S4 = S]S2' Pero, evidentemente, tenemos que
2
haHamos:
\j1
4r 2 2 -a
-a
X=
224 PROBLEMAS DE MATEMAnCAS ELEMENTALES

Por consiguiente, de (1) obtenemos que 2. Problemas de construccion

325. Sean 01 y 02 los centros de las circunferencias dadas.


324. Designando por a, b, c y d las longitudes de los lados y por Trazamos la recta 0IA y a traves del centro 02 de la segunda
m, n las longitudes de las diagonales del cuadrillitero (fig. 41); circunferencia una recta paralela a 0IA que corta la segunda
circunferencia en los puntos My N
por el teorema de los cosenos tenemos:

n 2 = a 2 + d 2 - 2ad cos <p,


n2 = b 2 + c 2 + 2bc cos <po
Deaquf "

(be + ad)n 2 = (a 2 + d 2 )be + (b 2 + c 2 )ad = (ab + cd) (ae + bd) .


Fig. 42 Fig. 43
Por consiguiente,
(fig.42). La recta MA intersectara la segunda circunferencia en el
n 2 = ab+cd (ae+bd) . punto PI' La recta 02PI intersectanlla 0IA en el punto C I . De la
bc+ad semejanza de los triangulos M02P I y ACll se deduce:
Amllogamente hallaremos:

m 2 = ad + be (ac+.
bd)
ab+cd Por consiguiente, la circunferencia de centro C I y de radio
CIA es la circunferencia buscada. La segunda resoluci6n se
Multiplicando estas igualdades entre sf, obtendremos el teo- obtiene con ayuda del punto N 10 mismo que la primera con
rema de Ptolomeo: ayuda del punto M Si una de las rectas MA 0 NA resulta tangente
ala segunda circunferencia, entonces queda una soluci6n y la
mn= ac+bd.
segunda dara esta tangente (el centro de la circunferencia se
encontrara en el infmito). Esto tendra lugar cuando, y s610 cuando,
eI punto A coincida con el punto de tangencia de una de las cuatro
tan gentes comunes a las circunferencias dadas.
GEOMETRiAIPlanimetria (Resoluciones y Soluciones) 225

326. Sea 0 el centro de la circunferencia dada y AB la recta (la segunda pasa a la recta /); si I pasa par N, la soluci6n sera la
dada (fig. 43). EI problema se resuelve de forma amiloga al circunferencia dada y si I pasa por M, el problema tiene una
anterior. En el caso general tiene dos resoluciones. Habra los cantidad infinita desoluciones.
casas particulares siguientes: 1) la recta dada intersecta la
circunferencia y el punta dado A coincide can uno de los puntas 328. Valiendonos de la hipotenusa dadaAB = c como diametro
de intersecci6n (no hay ninguna soluci6n); 2) la recta dada hace tracemos una circunferencia con su centro en el punto 0 (fig.
contacto con la circunferencia y el punta A no coincide con el 45). Tracemos OE -1 AB Y marquemos sabre OE el segmento
punto de intersecci6n (tiene una soluci6n); 3) la recta dada hace OF = h. La recta paralela a AB que pas a por F intersectara la
contacto can la circunferencia y el punta A coincide con el punta circunferencia en el punto buscado C. EI problema es soluble si
de intersecci6n (tiene una cantidad infinita de soluciones). E D
c
327. A traves del centro 0 de la circunferencia dada trazamos
una recta perpendicular a la recta dada I, que carta la A ~----!:c--L---~B
circunferencia en los puntas My N (fig. 44). La recta MA
intersectara la 1 en el punto PI' El punta C I es el punto de
intersecci6n de la perpendicu lar a la recta Ilevantada en el punta A E
PI can la recta ~A. De la semejanza de los triangulosAOMy
ACll se desprende que CIA = CIP\. Por consiguiente, la Fig. 45 Fig. 46
circunferencia de centro C\ y radio CIA es la buscada. La segunda
resoluci6n se obtiene con ayuda del punto N 10 mismo que la h ::;; ~. Las longitudes de los catetos a y b se hallaran con ayuda
primera con ayuda del punto M S i la recta 1no pasa por ninguno del sistema de ecuaciones
de los puntos A, My Ny el punto A no coincide con M 0 con N, a 2 + b2 = c2 l
entonces el problema siempre tiene dos soluciones.
ab = hcJ
Supongamos que A no coincide con M 0 con N; si 1 pasa a
traves de Mode N, entonces el problema tiene una sola soluci6n Resolviendo este sistema obtendremos:
(Ia segunda circunferencia coincide con la circunferencia dada); 1('2 /2 'I
a = -! -V c + 2hc + v c - 2hc)
si 1pas a por A, el problema no tiene ninguna soluci6n. 2\ '
Supongamos que A coincide con M; si I no pas a por ninguno ] f l'2 '2 ')
de los puntos My N, entonces el problema tiene una sola soluci6n b=-!,\c +2hc-"'I/c -2hcj'
2\
226 PROBLEMAS DE MATEMAnCAS ELEMENTALES

329. Tomamos el segmentoAB y sobre la rectaAB trazamos el ABC bajemos las perpendiculares OM, ON y OP a los lados del
segmento AE = AD (fig. 46). Tomando como base a BE triangulo (fig. 48). Entonces AP = AN, BP = BM y CM = CN.
construimos el triangulo BCE con los lados BC y EC = CD. Por consiguiente, las circunferencias de radiosAP, Bl'viy CN
Sobre el segmentoAC como base, construimos el MCD con cuyos centros son A, B Y C tend ran contacto una con la otra en
los lados AD y CD. El cuadrilatero ABCD es el buscado, puesto los puntos P, My N.
que tiene los lados dados y el LDAC = LCAE (los triangulos
ACDy ACE son iguales por construccion). B. Caso de tangencia interior. Desde el punto a de
interseccion de la bisectriz del angulo C con las bisectrices de los
330. Admitamos que sean H, S y Mlos puntos de interseccion angulos externos A y B bajemos las perpendiculares OM, ON y
de la altura, la bisectriz y la mediana respectivamente con la B
circunferenciacircunscritaK que tiene porcentro el punto a (fig.
47). Tracemos la recta SO y a traves del punto H una recta
paralela a SO que cortara por segunda vez a Ken el punta A.
Tracemos la recta A.M que intersectani a
SO en el punta P. A traves de P tracemos
una recta perpendicular a SO que cortara a
K la circunferencia en los puntos B y C. EI
Fig. 48 Fig. 49
triangulo ABC es el buscado, puesto que
c AH 1- BC, BS = SC y BP = Pc. E I OP a los lados (0 las prolongaciones de los lados) del triangulo
problema es soluble si, y solo Sl, H, Sy M ABC (fig. 49). Entonces,
no se encuentran en una misma recta. la
tangente ala circunferencia Ken el punta H AP=AN, BP=BM, CM=CN.
no es paralela a SO y si los puntos H y M se
Fig. 47 Por consiguiente, las circunferencias de radiosAP, BMy CN
encuentran a diferentes lados de la recta SO,
cuyos centros son los puntosA, By Ctendran contacto una con
pero no en un ITIISma diametro de la
la otra en los puntos P, AI YN.
circunferenciaK.
Obtendremos dos soluciones mas tomando las bisectrices del
angulo interno A y los extemos Bye 0 del angulo intemo B y los
331. A. Caso de tangencia exterior. Desde el punto a de
angulos externosA y C.
interseccion de las bisectrices de los angulos intemos del triangulo
GEOMETRiA/Planimetria (Reso1uciones y Soluciones) 227
- -- - - - - - - - - - -

332. La resoluei6n se basaen la siguiente propiedad: si las alturas hA y


h8 del triangulo inserito ABC eortan a la eireunfereneia en los puntos PQ = AE = 1
QM EB '
Al y B I, entonees el vertiee C divide el areoAIB l por la mitad (fig.
50). Esto se desprende de la igualdad de los angulos de donde PQ=QM; luego,
LA1AC y LBjBC, cadaunodeloseualesesiguala ~ - LACB.
MN PQ
Construccion. A traves del punto A trazamos una recta en la CD CD'
direcci6n dada hasta su intersecci6n con la circunferencia en el
de donde MN=PQ. La segunda soluci6n se obtiene con ayuda
punto A I; admitamos que sea BI el punto de interseeei6n de la
del punto medio E' de la base CD de la misma manera que la
altura hH con la cireunferencia; hallamos el punto medio C del
primera con ayuda del E.
area AiBI y trazamos AC; trazamos BIB.l. AC; el triangulo
ABC es el buscado.
334. Supongamos que se ha construido el cuadrado ABCD,
La segunda soluei6n AB' C' se obtiene si se toma el punto
ademas, B es el vertice dado, E y F son los puntos dados (fig.
medio C' del segundo areoAIB I.
52). El vertice D debera encontrarse en la circunferencia
B construida tomando a EF como diametro. Admitamos que BD
corta ala circunferencia en el punto K. Entonces. EK=EF, puesto
que LADB = LBDC.

1f:5.-.---I-----'!c
Construccion. Tomando EF
como diametro, construyamos una
B, circunferencia y desde su centro
levantemos una perpend icular a EF
Fig. 50 Fig. 51 hasta su intersecci6n con la circunfe-
rencia en los puntos K y K'; unamos
333. Unamos el punto medio E de la baseAB con el vert ice Cy B con K y prolonguemos BK hasta
hallemos el punto Qde intersecci6n de las reetas ECy AD (fig. su intersecci6n con al circunferencia
51). La recta PQMN paralela aAB es la buscada. En efecto, en el punto D; tracemos las rectas Fig. 52
DE y DFy a traves del punto B, las
228 PROBLEMAS DE MATEMATICAS ELEMENTALES

perpendiculares BA Y BC a estas ultimas rectas. ABCD es el 336. Para la construcci6n es suficiente conocer la altura h=KL
cuadrado buscado. La segunda soluci6n se obtiene haciendo uso del rectangulo.
del punto K. El problema tiene siempre dos soluciones, excepto Supongamos que sea KLMN el rectangulo buscado y que
en elcaso cuando el punto B se encuentra en la circunferencia de KN se encuentra sobre el lado AC (fig. 54). Si se traslada el
diametro EF. En este ultimo caso el problema no tiene soluci6n vertice B paralelamente a la base AC y se conserva al mismo
si el punto B no coincide con uno de los puntos K 0 K. tiempo la magnitud hconstante, entonces se conservaran tambien
constalltes las magnitudes de [a base y de la diagonal del rectangulo
335. Primera soluci6n. Tracemos AD 11MB hasta su (puesto que LM constituye la misma parte deAC que BH - h de
intersecci6n con la prolongaci6n de BC en el punto D (fig. 53). BlI). Por consiguiente, para hallar h el triangulo dado ABC puede
En el segmento CD haHamos el punto N tal, que ser sustituido por cualquier otro con [a misma baseACy [a misma
altura BH. Es mas c6modo tomar un triangulo con un angulo
recto en la base. De aqu! obtenemos la siguiente construcci6n.
CD =k.
CN Trazamos a traves de B una recta para[e[a aAC y a traves de C
una recta perpendicular aAC; desde el vertice del angulo recto
La recta MN es la buscada, ya que el area SABM = S DBM' C, con una abertura del compas igual ala [ongitud d de la diagonal
por consiguiente, S ABC = SDMC Y de acuerdo con la construc- dada, hacemos una intersecci6nL, en la hipotenusaAB,; a traves
ci6n S DMC = kSNMC' de L, trazamos una recta paralela a AC; los puntos L y M de
La segunda soluci6n la obtendremos valiendonos del punto intersecci6n de esta recta con los lados AB y BC son los vertices
N, tal, que del rectangulo buscado. Segun que la altura del trianguloAB,C
bajada desde C sea menor, igual a mayor que la magnitud dada
d, el problema tendra dos soluciones, una 0 no tendra soluci6n.
D
Entonces

SASC = k.
SABN,M
N
Teniendo en cuenta la posibilidad de una construcci6n analoga
partiendo del vertice C (en vez del A), es facil convencerse de L---'":'_ _ _--' C A
K M N
A
que si k =I:- 2 el problema tiene siempre dos soluciones y si k = 2,
s610 una. Fig. 53 Fig. 54
GEOMETRiAIPlanimetria (Resoluciones y Soluciones) 229

337. Inscribimos en el angulo dado la circunferencia dada. Desde Para que el problema sea soluble es necesario que
los puntos de tangenciaA y B en los lados del angulo, trazamos
a
los segmentosACy BD iguales allado dado del triangulo (fig. R+r? - - .
a
55). cos -
Inscribamos en el angulo dado una segunda circunferencia de 2
Pero,
modo que haga contacto con los lados del angulo en los puntos
Cy D. Tracemos una tangente comun EF a las circunferencias
a
R =r+atg -
construidas. Demostremos que el ~ SEF, obtenido de esta 2
manera, es el buscado. Para ello, es suficiente demostrar que y, por consiguiente, <;lebera ser
AC = FE. No es dificil convencerse de que el perimetro ~ SEF
a a
es igual a 2SC; por otro lado, este perimetro, evidentemente, es 2r+a tg-? - - ,
igual a 2 (SA + EL + LF). As! pues.
2 a
cos -
o bien 2
SC =SA+EL+LF, SA+AC =SA+EF, a
2r l-sen -
_ ? _ _~2"'__
es decir, a a
cos -
AC=EF. 2

como 10 cual el problema queda resuelto. 338. Tomando como centro eJ punto B,
Esta claro que el problema tiene dos Fig. 55 trazamos una circunferencia que hace
soluciones si las circunferencias no se inter- contacto con la recta CD (fig. 56).
c D Desde el puntoA (siA y B se encuentran
sectan y una soluci6n cuando estas tienen contacto. En el caso
cuando las circunferencias se intersectan el problema en insolu- A a distintos lados de la recta CD, 0 desde
ble.Supongamos que sea a el angulo dado, r y R los radios de
Fig . 56 e 1 punto A ' · ' · a A respecto a
, Slmetnco
las circunferencias y a ellado dado del tri:ingulo. La distancia CD, si A y B se encuentran a un mismo lado de CD) trazamos la
entre los centros de las circunferencias es igual a tangente AK a la circunferencia construida. EI punto M de
a intersecci6n de AK (0 A'K) con la recta CD es precisamente el
a buscado. En efecto LAMC = LKMD = 2LBMD.
cos-
2
230 PROBLEMAS DE MATEMAnCAS ELEMENTALES

3. Problemas de demostraci6n
B
339. Sea BO una mediana del triangulo ABC; construyamos a
base del triangulo ABC el paralelogramo ABCD (fig. 57). De
triangulo BCD tenemos que 2BO < BC + CD y, puesto que, CD
= AB, entonees
A
AB+BC
BO< ----- -.
2 Fig. 57 Fig. 58

Del ~AOB Y ~BOC tenemos: 341. Vease la resoluci6n del problema 301.
AC
BO-i-->AB, 342. Sean a, b y c las longitudes de los lados del triangulo,
2 opuestos respeetivamente a los angulos A, By C. Demostremos
AC que la longitud LA de la biseetriz del anguloA se expresa par la
BO+--- > BC.
2 f6nnula

Sumando estas desigualdades, obtendremos: A A


2bc eos- - 2cos-
L 4 = --~ = ~--~'
BO > AB+BC _ AC. . b+c I I (I)
- +-
2 2 b c
340. Supongamos que sea D el punto de intersecei6n de las
alturas, 0 el centro de la eircunferencia circunscrita y E y F los En efeeto, el area del triangulo ABC es igual a
puntos medios de los lados Be y AC (fig. 58). Los triangulos
ADB y EOF son semejantes puesto que I 1 A 1 A
LABD = LOFE Y LBAD = LOEF (como angulos con S4BC = -- bcsen A=-c!A sen - +- bIA sen -
. 2 2 2 2 2
lados paralelos). Por consiguiente,
De aqui se deduce !a f6rmula (1). Analogamente, para la
OE EF I
bisectriz lR del angulo B obtendremos la formula
AD AB 2
GEoMETRiAlPlanimetria (Resoluciones y Soluciones) 231
~- ~ - ~ ---- ----- - - -- - - - - -- .- ~ - -~- -- -~- -----

B Multiplicando miembro a miembro estas igualdades,


2cos - obtendremos:
t - 2
B-1 (3)
- +- RB · PC ' QA = PB·QC·RA.
a c
344. Sea el L4KB = a, el L AFB = 13 y el LACB = Y (fig.
Admitamos que sea a > b; entonees LA > LB, y puesto que 60). Tenemos que a = ~ y, puesto que

ademas 0 < 1<} y 0 < 1<i por 10 tanto, cos 1 < cos1· to ~
1
=-
1
tob ,v =-
to fJ 2' ,, '
.)
As! pues, el numerador de la fracci6n (1) es menor que el
entonces
numerador de la fraeci6n (2). Ademas, el denominador i + i: 1 1
-+ -
de la fracci6n (1) es mayor que el denominador J~a + 1c de la tg(f3+y)= ~T = 1.
\- _ .-
fracci6n (2), ya que i > ~. Por eonsiglliente fA < fw 2 3

345. Valgamonos del teorema inverso a1 teorema de Pitagoras:

B~ .
si la suma de los euadrados de dos lados de un triangllio es iguaI
al cuadrado del tercer lado, este triangulo es rect{mgllio. En el
A K F ( easo dado, la relaei6n

Fig. 59 Fig. 60

343. Sea, el L CPQ = a y L PQC = 13 (fig. 59). Segun el se eumple, plIesto que es equivalente a 1a igualdad evidente ab =ch.
teorema de los senos tenemos:
346. Primera resoluei6n. Traeemos AE de manera tal, que
RB BP PC CQ AQ AR L EAC = 20 0 Y BD...L AE (fig. 61). Puesto que ell1CAE es
sen a sen (a-:--~)' sen 13 sen a sen(a -"- 13) sen 13 semejante al ABC, entonees
232 PROBLEMAS DE MATEMATlCAS ELEMENTALES

Elevando ambos miembros al cuadrado y realizando las


CE a
simplificaciones correspondientes, hallaremos que esta relacion
a b' es equivalente a la relacion a demostrar.
dedonde
2 2
Scgunda resoluci6n. Puesto que a=2b sen 10°, la relacion a
a " a demostrar es equivalente a la siguiente:
CE=- y BE=b--.
b b

Por otro lado, el LBAD = 60°, en virtud de 10 cual obien

BD= J3 b,
2 .
AD=~ La ultima igualdad se cumple en virtud de la formula general
2
y, puesto que AE = a, ED = ! - a. Por eso, sen 3a = 3sena - 4sen 3a.

347. En todo triangulo, frente a mayor lado se encuentra mayor


angulo. Por esta razon, si en el f1 ABC (fig. 62)
AC<ABM, B
Por consiguiente, B

10 que es equivalente a las dos


a2 desigualdades:
b- - =
b
AM<BM, MC<BM, AL--------'-,--------"C

entonces, Fig. 62

LABM<LBAM, LMBC < LBCM.

Sumando estas desigualdades, obtendremos:


Fig. 61
LABC < BAM + LBCM -1t - LABC
GEoMETRiAiPlanimetria (Resoluciones y Soluciones) 233

dedonde 1t
B B
2LABC < 1t 0 bien LABC < -.
2

Analogamente se examinan los casos A C ~ 2BM.

348. Primera resoluci6n. Sea QQ'IIAC y N el punto de


intersecci6n de AQ' con QC (fig. 63). En la figura, con arcos
Henos se designan los angulos cuyos valores son evidentes.
Demostremos que
QP l.AQ'. (1)

En efecto, NC == AC; pero, AC == PC puesto que el


I'1ACP es is6sceles. Poreso, NC == PC y, porconsiguiente, el
·"--='-----..=L>c
1'1 NCP es tambien is6sceles, de 10 que se desprende que
Fig. 63 Fig. 64
LCNP == LNPC == 80°.
Demostremos que estos triangulos son en efecto semejantes. Para
ello, en virtud de la igualdad de los angulos ABP y PCQ, es
De aqui, obtenemos facilmente que LQ'NP = 180°-
suficiente verificar que en virtud de la igualdad de los angulos
- 60° - 80° = 40° y, puesto que LNQ' P == 40°, el triangulo ABP y PCQ, es suficiente verificar que
QQ' P es igual al QNP. De aqui se deduce (I). Ahora esta claro
que el L Q' PQ == 50° y, por consiguiente, el LQPA = 180° - AB PB
== (I)
- 50° - 50° = 80°. CQ CP
Hagamos AB == 1; entonces, del triangulo is6sceles CQB
Segunda resoluci6n (vease la fig. 64). Es facil ver que el
tenemos:
angulo P==80° cuando, y s610 cuando, I'1ABP (J)I'1PCQ (los
angulos cuyos valores se desprenden directamente de las co== - - -
~ 2cos20°
condiciones del problema, se dan en la figura con arcos lIenos).
234 PROBLEMAS OE MATEMATICAS ELEMENTALES

Por otro lado, puesto que PC = AC, Esto significa que los triangu los CAB y CBD son semejantes y
LA = LCBD. Ademas, LB = LBDA = LDBA. Por consi-
PC = 21 sen 10°, BP = 1- 21 sen 10°. guiente, LA = L.B + LDBA = 2L.B.

Colocando estas expresiones en (1), obtendremos la igualdad 350. Supongamos que sea OC la mediana del triangulo OAB I.
equivalente: Admitamos que eI punta D se encuentre en la prolongaci6n de
(2) OC y OC=CD (vease la fig. 66). Demostremos que
.1.AOD = llOAIB. En efecto, AO=OA , segun laconstrucci6n .
Es facil revelar la validez de esta ultima igualdad observando Luego AOBP es un paralelogramo, en virtud de 10 cual
que AD=OB,=OB. Par fin LOAD = LApB, puesto que los lados
de estos angulos son perpendiculares entre Sl:
senl O"cos20" = sen{l 0" + 20")-sen{l 0" - 20") =..!. -..!.sen 10". AO J.. ~Al Y OBI J.. OB segun la construcci6n, y
2 4 2 AD li OBI.Par consiguiente, /j. AOD = II OAIB Y dos de los
lados de uno de el los son respectivamente perpendiculares ados
B lados del otro. Por eso, los terceros lados son tambien

~c
perpendiculares, es decir, OD .l AlB.

DCA b

Fig. 65

349. Sea dado el II ABC (fig. 65). Sobre la prolongaci6n del


lado AC trazamos AD = c. De la igualdad a 2 = b 2 + bc se
deduce:
a b +c
b a
GEOMElIUA/Pianimetria (Resoluciones y Soluciones) 235
- ------ ~---- - - - - - --

351. Sea ABC un triangulo acutangulo y AD, BE YCF sus alturas


que se cruzan en el punto o (fig. 67). Cada uno de los cuadrilateros S = abe.
BDOF, CEOD y AFOE estan inscritos en cierta circunferencia. 4R
Por el teorema del producto de la secante por su parte externa, Por otro lado, S = rp, donde pes el semiperimetro. Igualando
tenemos: ambas expresiones, tendremos:
abe
AD· AO= AB· AF = AC. AE, BE- BO= BC· BD= BA· BF, rR = ~--. (I)
4p
CFCO=CA·CE=CB·CD.
En las condiciones del problema
Sumando estas igualdades, obtendremos:

2(AD. AO+ BE· BO+CF ·CO)= a+b+e 3


p= ' = - b.
= AB·AF + BC- BD+CA-CE + AC · AE + 2 2
+ BA· BF +CB·CD = AB(AF + BF)+
Coloquemos este valor de pen (I), obtendremos:
+ BC(BD+CD)+CA(CE + AE)=
= (AB)2 + (BC)" + (CAr, 6rR = ae.
con la cLlal el problemaqueda resuelto. En el caso de un triangulo 353. Sea z la longitud de la bisectriz y m y n las longitudes de los
obtllsangulo, el producto corresponde al anglilo obtuso debe segmentos en que esta divide a la base del triangulo (fig. 68): Por
tomarse con el signa menos. el teorema de los cosenos

352. Seglll1la condici6n del problema b - a = c - b, 0 a 4- C := 2b. /I


Para calcular el producto Rr valgamonos de las expresiones para
a I
/\ \_
el area del triangulo en funci6n del radio de la circunferencia
circunscrita 0 inscrita y ellado. /
/ \\
/ \
/ ~. \

Es conocido que S = ~ be sen A, y por el teorema de senos


sen A = 2oR' de donde Fig.6R Fig. 69
236 PROBLEMAS DE MATEMAnCAS ELEMENTALES

a 2 == Z2 + m 2 - 2mzcosa, DM senB
b2 == Z2 + n 2 + 2nz cosa. ME senC

Pero en el f... ABC


Multiplicando la primera de estas igualdades par n y la segunda
senB AC
por m y suman do los resultados, obtendremos: -- = AB
senC
Por consiguiente,
na 2 + mb 2 = (m + n)(z2 + mn). (1)
DM AC
En virtud de la relaci6n i!. = Qn tenemos:
In
ME AB

na 2 2m
+ mb == na b
- + mb -na = ab (m + n ) .
n m 355. Sean BD, BE YBF respectivamente una altura, una bisectriz
y una mediana en el triangulo ABC. Supongamos que AB < BC.
Colocando esta expresi6n en (1), obtendremos la igualdad Entonces
requerida LA> LC, LCBD> LABD,
dedonde
ab = z2 + mn.
1 1
LCBD > - (LABD + LCBD) == - LB,
En el caso en que sea a = b y m = n, la igualdad demostrada 2 2
expresani el teorema de Pitagoras: a 2 = z2 + m 2 . es decir, LCBD> LCBE. Por 10 tanto, la bisectriz BE pasa
por dentro del LCBD y el punto E se encuentra entre D y C.
354. Segun la condici6n del problema BD = EC (fig. 69). Si M Luego,
es el punto de intersecci6n de BC con DE, entonces, del
f...BDM ydel f...ECM tenemos: AE == AB < 1 AE<EC,
EC BC '
de donde,
BD DM EC ME
sen <p
= sen f3 ' sen<p senC' 1 1
AE < -(AE +EC)= -AC,
2 2
de donde
GEOMETRiAiPlanimetria (Resoluciones y Soluciones) 237

es decir, AE < AF. Por consiguiente, el punto F se encuentra Dividiendo miembro a miembro (1) entre (2), obtendremos:
entre E y C. Asf pues, el punto Ese encuentra entre D y F, 10
que era necesario demostrar. x
y
356. Supongamos que en el triilllgulo ABC, BD es una de las
10 que habia que demostrar.
bisectrices, BM, una de las medianas y BN, una recta simetrica
con BM respecto a BD (fig. 70). Si SABN Y 5MBCson las areas de B

~ A
los respectivos triangulos, entonces
B
,0
2SABN = xhB = nesenLABN,
y 'I'
B ex C
p
D

x y
Fig. 71 Fig. 72
donde hB es la altura bajada a AC Fig. 70 357. Las rectasAP; BQy CR dividen al trianguloABC en seis
desde el vertice B. Puesto que triangulos: ~AOR, ~ROB, ~BOP, ~COQ, ~QOA (fig. 71).
LABN = LMBC, entonces Aplicando el teorema de los senos a cada uno de ellos,
x+ y ne obtenemos:
X=- · -. (I)
2 rna
AR AO AO AQ
Analogamente sen<p seny sen 13 sen \If
BO BR BP BO
2SNBC = yhB = nasen LNBC,
seny sen (ql + \If)' sen \If sena
x+y
2S ABM = - - hB = mesenLABM. CQ CO CO CP
2
sen (cp + \If) - sen 13 ' sena sen<p
Puestoque LNBC = LABM, entonces
x+y na Multiplic~ndo miembro a miembro todas estas igualdades entre
y= - ---. - . sf, haHamos:
2 me
238 PROBLEMAS DE MATEMATICAS ELEMENTALES

AR- Bp·CQ =BR·AQ·CP. y pasen por los vertices de este (fig. 73). Tracemos tangentes a
lacircunferencia K 2, paralelas a los lados del/}. AI BI C I , confonne
358. Supongamos que sea K el centro de la circunferencia
ala siglliente regia: la tangenteA 2B 2, paralela alladoA,B" hace
circunscrita al triangulo ABC, 0 el centro de la circunferencia
contacto con Kl en un punto que se encuentra en el mismo arco
inscritaen el mismo triangulo y Del punto medio del arcoAC
AB que el vertice C, etc. Los segmentos de las tangentes trazadas
(vease la fig. 72). Cada uno de los angulos LOAD y LAOD es
forman el trianguloA 2BP2"
igual a la semisuma de los angulos en los vertices A y B del
trianguloABC. De aqui se desprende que OD=AD.

Por el teorema de las cuerdas que se cruzan dentro de una


circunferencia, tenemos que

MO· ON = BO · OD.
lJ.
Luego, si OE 1- AB Y FD es el diametro, entonces los Fig. 73
triangulos BOE y FDA son semejantes, de donde
BO:OE=FD:AD, as! que BO· AD = OE . FD 0, puesto que Entonces el !\ A1B1C 1 se encllentra dentro del/}. A2 B2 C 2 Y
AD = OD, BO· OD = OE· FD. Porconsiguiente,
estos dos triangllios son semejantes. Por esta razon, el radio R'
MO·ON=OE·FD. de la circunferencia inscrita en el /}. AI B1C 1 no es mayor que el
radio R de la circllnferenciaK2 inscrita en el !\ A2 B2C 2 es decir
Colocando en esta igualdad los valores MO = R + l, ON =
=R-l,OE=r,FD=2R, obtendremos R2 _/2 = 2Rr, loque R'$. R; por otro lado, la relacion entre los radios de las
era necesario demostrar. circllnferencias inscritas en los triangllios semejantesA 1B,C, y
ABC es igual a la relaci6n entre los lados semejantes de estos
359. Primera resolucion. Supongamos que seaABC el triangulo trianglllos, es decir, A~~l = 2. Asi pues, R'=2r. Confrontando esta
dado, K I la circllnferencia inscrita de radio r y Ko_ la circunferencia
circunscrita de radio R. Construyamos un triangulo auxiliar igualdad con Ja desigualdad R' $. R, obtendremos defin itivamente:
AIB,C, de modo que sus lados sean paralelos a los del /}.ABC
GEOMETIUA/Planimetria (Resoluciones y Soluciones) 239

2r ~R. Tercera resolucion. De la f6nnula {2 = R2 - 2Rr demostra-


da en el problema anterior, se desprende que R2 - 2Rr ~ 0, de
Segunda resolucion. Sean ry R los radios de las circunfe- donde R ~ 2r.
rencias inscrita y circunscrita, S el area del triangulo dado,p el
semiperimetro yay b dos de sus lados. Entonces, 360. Sean a y b las longitudes de los catetos y cia longitud de la
h ipotenusa. Comparando las dos expresiones para el area del
r S 1 absenC 2Rsen A sen Bsen C triangulo,obtenemos:
R pR 2 pR R (sen A + sen B + sen C) .
S = -1 (a + b + c}r = -1 hc,
Pero,
2 2
de donde
r c
A+B A-B (1)
sen A +senB+senC = 2sen - - -cos - - + h a+b+c
2 2
A+B A+B A+B A B Puesto que a+b>c, entonces,
+2sen - - cos-- =4sen - - cos-cos -- =
2 2 2 2 2
ABC r c
= 4cos - cos-cos-. - <--=0,5.
222 h c+c

Por consiguiente,
Luego, en virtud de la relaci6n c 2 = a2 + b2 , la desigualdad

-r = 4 sen -A BC
sen -- sen -
a 2 + b 2 ~ 2ab es equivalente a la desigualdad 2c 2 ~ (a + b r,
R 2 2 2· o bien a2 + b 2 ~ c-fi. De aquf que
E\ problema se reduce a la demostraci6n de la desigualdad r c I -
- ~ - --.~ = -,-- = ,/2 - 1 > 0,4.
ABC I h C"\j 2 +c 'J 2 + 1
sen-sen - sen - ~-
2 2 2 8
361. Supongamos que sean A, By C los angu\os del triangulo, a,
(vease el problema 644). by c los lados opuestos a estos angulos y P=a+b+c. La relaci6n
necesaria se desprende de las igualdades
240 PROBLEMAS DE MATEMATICAS ELEMENTALES

aka = bkb +ckC = Pr (1) segmentosAA" BB, Y CC, (fig. 74). Los puntosA 3 , B3 y C3 se
(b +c)ka +(c+ a)kb + (a +b)kc = PR, (2) encuentran en las !ineas medias del L1 ABC y, ademas, no en los
extremos de estas lineas, puesto que de 10 contrario, por 10 menos
sumando las cuales obtendremos que uno de los puntosA"B" C, coincidiriacon un verticedel triangulo
ABC. Puesto que toda recta que no pasa por uno de los vertices
del triangulo A 2B 2C 2 no corta al mismo tiempo sus tres lados,
La igualdad (1) es justa en virtud de que sus partes izquierda entonces, los puntosA 3 , B 3, C 3 no se encuentran en una misma
y derecha son iguales al area duplicada del triangulo. Para recta.
B
demostrar la igualdad (2) observemos que

ka = RcosA, kb = RcosB, kc = RcosC (3)

yque
bcosC + ccosB = a,
ccosA + acosC = b,
acosB + bcosA = c, Fig. 74 Fig. 75

de don de, sumando miembro a miembro estas igualdades, 363. Si h, es la altura del L1 DON, h8 la altura del L1 ABC Y
obtendremos la igualdad
S / 1()C Y SABC las areas de los respectivos triangulos, entonces
(b + c)cosA + (c + a)cosB + (a + b )cosC = P, (fig. 75),

que despues de multiplicarla por R y hacer uso de (3), coincide SAOC


- -
hi aD
=---= -
AF
con (2). SABC hB AB AB

yanalogamente
362. Admitamos que sean A2 B 2 , B2 C 2 , C 2 A 2 las lfneas
SA08 _ BE SWR
- -- =-
CN
medias en el L1 ABC Y A 3, B 3, C 3 los puntos medios de los S4HC - BC' SAse CA
GEOMETRiAIPlanimetria (Resoluciones y Soluciones) 241

Sumando estas igualdades, obtendremos: A"B'1!AB, B"C liBC Y A"C1IAC (fig. 77). Esevidenteque
el t:. ABC se encuentra dentro del t:. A"B"C" y, por 10 tanto
AF + BE + CN = SAoe + SBoe + SAOB = SABe = l. A"C"> AC. Puesto que
AB BC CA SABe SABe
~A"B"C"(j)~ ABC,

364. 1) Examinemos la circunferenciaK de radio r' inscrita en entonces


el cuadrado y tracemos las tan gentes a esta circunferencia rOO A"C'
-=~-->l
A'B'IIAB y B'cIIBC (fig. 76): r AC '
dedonde
x = -Ii rOO> -Ii r.
365. Supongamos que sea Mel punto de intersecci6n de las
alturas AA I, BBI Y CC I del triangulo ABC; P el centro de la
circunferencia circunscrita de radio R; C 2 , A2 Y B2 los puntos
medios de los ladosAB, BCy AC; OM=OP; ON 1- AC; A 3 , B3
Fig. 76 Fig. 77
Y C 3 los puntos medios de AM, BM y CM
B
Esta claro que el ~ A' B' C se encuentra dentro del ~ ABC,
por 10 cual A'C < AC . Puesto que ~ A' B'C (j) ~ ABC,
entonces
r' A'C
- = - -<1
r AC '

de donde x = 2r' < 2r. Fig. 78 Fig. 79

(fig. 78). Demostremos que el punto 0 equidista de A" B, YCi ,


2) Examinemos la circunferencia K" de radio r" circunscrita
donde i = 1,2,3. Puesto que ON es la linea media en el trapecio
al cuadrado y tracemos las tan gentes a esta circunferencia
MB I B2P, entonces OB I =OB2 • De la semejanza de los triangulos
242 PROBLEMAS DE MATEMATICAS ELEMENTALES

AMB YPA/32 hallamos que BM=2PB2 y, por 10 tanto, B 3M=P B 2· y SM IIAC. De analoga forma se demuestra que PN IIAB y
Del paralelogramo MB 3P B2 tenemos que OB 3=OB2 • Pero
que RQIIBC.
1 R 2) Para demostrar que los vertices del hexagono MNPQRS
OB3 -=-BP-=-
2 2 se encuentran en una misma circunferencia, es suficiente demostrar
que cuatro cualesquiera de sus vertices consecutivos se encuentran
(como linea media en el triangulo P MB). Por consiguiente, en una misma circunferencia. Esto se desprende del hecho de
que por tres puntos no pertenecientes a una misma recta puede
ser trazada solamente una circunferencia. Se tienen dos tipos de
cuatro vertices sucesivos del hexagono que se exam ina: tales, en
De manera analoga se demuestra que GAl = GA 2 = GA3 = los que los puntos medios se encuentran en distintos lados del
~ ABC (RSMN, MNPQ, PQRS) y tales, en los que los puntos
medios se encuentran en·un mismo lade del ~ABC (NPQR,
QRSM, SMNP).
Examinemos los cuatro vertices RSMN y NPQR (de distintos
366. Supongamos que AAl' BBI Y CC I son las alturas del tipos). De la proporcionalidad obvia
I
triangulo ABC, que se cruzan en el punto 0, CIM BIN .l BC. Bel = BO = B~
AIPllcIQ.lAC, BIRIIAIS .lAB (fig. 79). BR BBl BN
1) Demostremos que SMIIAC. Tenemos que se deduce que NR IIAICI. Por eso
~ BAI A IJ) ~ BCIC
como triangulos rectangulos con el angulo
agudoABC comun. De aquf LMNR -= LBAIC1 = LBAC = LBSM.
BAI BA
BCI BC Por consiguiente, LMNR + LMSR = rc Ylos puntos R, S, M
y N pertenecen a una misma circunferencia. Luego,
Porconsiguiente, ~AIBCIIJ)~ABC Y LBAICI -= LBAC.
En el ~ AI BC I los segmentos A IS Y C 1M son alturas. Por esta LPNR + LPQR = rc-(LPNC + LBNR) + rc-LAQR =
razon, repitiendo los razonamientos anteriores, mostremos que = 2rc - (LABC + LBAC + LACB) = rc,
LBSM = LBAICI.Por consiguiente, LBSM=LBAC
GEOMETRiAlPlanimetria (Resoluciones y Soluciones) 243

de donde se deriva que los puntos N, P, Q y R estin dispuestos donde res el radio de la circunferencia inscrita, y R es el radio de
tambien en una misma circunferencia. De forma amlloga se lleva la circunferencia circunscrita.
a cabo la demostracion para los demas conjuntos de cuatro
vertices. 368. Supongamos que en el triangulo ABC, ABC es el angulo
recto, BD es la altura, BE es la bisectriz y BF la mediana (fig.
367. Sean A I' B I YC I los puntos de tangencia de la circunferencia 81). Puesto que BF = FC, entonces LCBF = LACB. Pero
inscrita con los lados del t-.ABC, y D el centro de esta
circunferencia (fig. 80). Puesto que los segmentos de las tangentes LABD = ~ - LBAD = LACB.
a una circunferencia trazadas desde un mismo punto son iguales 2
entre sf, entonces Porconsiguiente, LABD = LCBF Y

LDBE = LABE - LABD = LCBE - LCBF = LFBE,

10 que era necesario demostrar.

Fig. 80 Fig. 81 369. La disposicion simetrica de los triangulosABC y AIBIC I


respecto al centro 0 de la circunferencia inscrita significa que los
respectivos puntos de dichos triangulos se encuentran en una
misma recta con 0 y equidistan de este (fig. 82). En particular,
Al mismo tiempo, OC=OCl' OB=OB I YBCBIC I es un paralelogramo, por 10 tanto,
BC=BIC I. Analogamente AC=AICI' AB=AIBI Y
t-. ABC = t-. A, B,C,. Examinando los paralelogramos ABA IB I'
BDBPI,ACA1C 1y ECE1C j hallamos queAD=ApJ'AE=AIE j
Por consiguiente,
y, puesto que LA = LA" t-. ADE = t-. A,D, E,. Analogamente
AC + BC = CAl + AIB+CBJ+BJA = BJD+ t-. B, EK, = t-. BE, K Y t-. DC1K = t-. DpK1•
+A[D+BCJ+AC[ =2r+2R, Introduzcamos las siguientes denotaciones:
244 PROBLEMAS DE MATEMAnCAS ELEMENTALES

S es el area del L1 ABC, Par consiguiente,


S1' el area del L1 ADE, 2
( 2pr )
S2' el area del L1DC[K, S _ a(hA _2r)2
a --2r
a r
2(P)2
-a
S3' el area del L1 KBEI , 1- 2h
A 2h,4 S
AB=c, BC=a, AC=b,
A Analogamente,

Empleando la formula de Heron, obtenemos:


B

S2S2S?;S2 = rI2(p-at(p-b)4(p-c)4 S 2 =r12 S4 =r16


Fig. 82 1 - 3 s6 p4
M
hA' hB Y he son las alturas bajadas de los vertices A, By C.
Entonces,
S = pr = Eh A = bhR = chc.
~-+--\---~ C
222

Sea AM una altura en el L1ADE y AN una altura en el


L1 ,4BC; entonces
Fig. 83
S _ DE · AM
1- 2
370. En las denotaciones de la fig. 83 tenemos:
De la semejanza de los triangulosABC y ADE, hallamos:
2 ? ?
MA =MO-+AO--2MO·AOcosa,
DE = a(hA -2r).
MC l =M0 2 + C0 2 +2MO.COcosa.
hA
GEOMETRiAiPlanimetrfa (Resoluciones y Soluciones) 245

Puesto que AO=CO, entonces, sumando estas iguaJdades, Observemos que I1C1BC = I1ABA1, ya que
obtendremos: CjB=AB, BC=BA\ y LCjBC=600+LABC=LABA\.
Por 10 tanto, LOC1B = LOAB y el cuadningulo OAC1B esta
(1) inscrito en cierta circunferencia. Por consiguiente,
LAOB = 120°. De forma analoga demostremos que
Amilogamente
LBOC = 120°. Pero entonces tambien LAOC = 120°, de
donde se desprende que el cuadrangulo A OCB, esta inscrito en
esta circunferencia. Pero, de aqu! se deriva, al mismo tiempo,
Por consiguiente, la diferencia que LAOB1 = LACB1 = 60°. Por esta razon, la formula (1) es
valida.

372. En las anotaciones de la fig. 85 tenemos:


no depende de la posicion del punto M.
LPBR = LABC
371. Sea 0 el punto de intersecci6n de las rectas AA I Y CC I y
(vease la fig. 84). El problema quedara resuelto si se demuestra
PB BR
que
AB
= BC

Esto significa que I1PBRrfJ I1ABC y anaiogamente


11 QRC C/J 11 ABC. Valiendonos de este hecho obtendremos:

LAPR = LAPB - LBPR = LAPB - LBAC,

de don de

LAPR+ LPAQ= LAPB+ 2LPAB = 7t,


Q as! que PR IIAQ. De forma analoga demostraremos que
Fig. 84 Fig. 85 QR !I AP.
246 PROBLEMAS DE MATEMATICAS ELEMENTALES

373. Designemos por hE' he y hD las distancias desde los vertices o


B, Cy Ddel paralelogramo hasta la rectaAO(fig. 86). Eneste
caso tiene lugar la siguiente propiedad: la mayor de estas distancias
es igual a la suma de las otras dos. Por ejemplo, si AO cruza al
lado BC, (como en la fig. 86), entonces, trazando
BE IIAO y CE .1 AO, de la igualdad de los triangulosBECy A!r---~
AD 'D hallaremos;
Fig. 86 Fig. 87

con la misma regIa de eleccion de los signos, pero respecto a la


Analogamente, si AO cruza el lado CD, entonces
recta CO.
hs = he + hD ; si AO no cruza los lados BC y CD, entonces,
he = hs + hD · De esta propiedad, para el caso expuesto en la
fig. 86, se deduce directamente la igualdad de las areas de los
374. Construyamos a base del trapecioABCD el trianguloAMD
y unamos el punto M con el punto medio F de la base AD (fig.
triangulos:
87). Entonces
SAGe =SAOD -SAOS'
ME=BC MF= AD.
En general, evidentemente, se puede escribir la formula 2 ' 2

S Aoe = I S AGD ± S AOS I '


donde se toma el signo mas, si los puntos By D se encuentran a
un mismo lado de A 0, y el signa men os, si los puntos B y D se
encuentran a distintos lados de A O.
Fig. 88 Fig. 89
La repeticion de este razonamiento para la recta CO en vez
de laAOconduce a la formula analoga
Por consiguiente,

S Aoe = ISeo/) ± SeaB I EF= AD-BC.


2
GEoMETRiAIPlanimetria (Resoluciones y Soluciones) 247

375. Supongamos que seaABCD el trapecio dado con las bases 8

AD y BC, Yque BE ~ AD y CF ~ AD (fig. 88). Tenemos: c

AC 2 - AF2 = CD 2 - FD2,
BD2 - ED2 = AB2 - AE2.

Sumando estas igualdades, obtendremos:


Fig. 90 Fig. 91
AC 2 + BD2 = AB2 + CD 2 + AF2 - FD2 + ED2 - AE2 =
= AB2 +CD 2 + AD(AF -FD+ ED-AE)= Por consiguiente MBA'M' es un paralelogramo y
A 'B=M'M=2MN. Puesto que por la condicion del problema
= AB2 + CD 2 + AD· 2EF = AB2 + CD 2 + 2AD . BC. BC+AD=2MN, entonces, BC+CA '=A 'B. Par consiguiente, el
punto C se encuentra en el segmento A 'B: en el caso contrario
376. Sea dado el trapecio ABCD con sus bases paralelas AD y tendriamos que en el f..BCA', BC+CA'>A'B. De aqui se
BC, E es el punto medio de BC, F el punto medio de AD yO el desprende que BC II MN IIAD es decir, queABCD es un trapecio.
punto de interseccion de las diagonales (fig. 89). Los triangulos
AOFy COE son semejantes (esto se deriva de la semejanza de 378. Hallemos la expresion para el area de un cuadrilatero en
los triangulosAOD y COB). Por est a razon, LAOF = LCOE, funcion de sus diagonales y el angulo formado por estas. Sea 0
es decir, EOF es una recta. el punto de interseccion de las diagonales del cuadrilateroABCD
y LBOA = a (fig. 91). Entonces, el area del cuadrilatero dado
377. SeaABCD el cuadrilatero dado y los puntos My Nlos sera igual a
puntos medios de los lados AB y CD respectivamente (vease
la fig. 90). Giremos el cuadrilatero AMND 180 0 en el plano l
S4ECD ~ S AWl + SClJD + S4QD + S E()e =} AO· OB· sen a +
del dibujo alrededor del vertice N. Entonces, el vertice D
coincidira con el C, y los vertices My A ocuparan las
posiciones M' y A'. Ademas, los puntos M, N y M' se +-~OC ·OD·sena +~BO.OC ·sena +~ AO·OD·sena =
2 2 2
dispondran en una misma recta y, al mismo tiempo, tendremos
que M'A'!!MB y M'A'= MB. =~ BD · AC ·sena.
2
248 PROBLEMAS DE MATEMATICAS ELEMENTALES

De esta formula se deduce, precisamente, lajusteza de la 10 que significa que PQRS es un recmngulo. Los triangulos BAB,
afrrmacion a demostrar. y CDC I son isosceles, ya que sus bisectrices son perpendiculares
a sus bases. Por esta razon, BP = PB" D,R = RD y, por 10 tanto,
379. Sea Mel punto interior del poligono convexo y AB su lado PR IIAD. As! pues, PRDB, es un paralelogramo y
mas cercano al punto M . Demostremos que el pie de la
perpendicular P bajada desde el punto M aAB se encuentra en PR=B]D=AD-AB] =AD-AB.
AB y no en su prolongacion (fig. 92). En efecto, si P se encontrara
381. Sean 01' 02' 03 Y 04 los centros de los cuadrados
fuera deAB, MP cortaria a cierto lado I del poifgono en el punto
construidos a base de los lados del paralelogramo ABCD (fig.
Q, ademas, en virtud de la convexidad del poIigono, MQ < MP.
94). Tenemos:
Pero la distancia DM de M a I es menor que MQ y, por
consiguiente, tambien menor que MP, 10 que contradice a la
eleccion delladoAB.

380. Sean AA I, BB I, CC I YDDI las bisectrices de los angulos


7t 7t
internos del paralelogramoABCD, que forman en su intersec- LO]BO, = LMBN + - = LDCB + - = L03CO,.
- 2 2 -
cion el paralelo~ramo PQRS (fig. 93): Es evidente que
BB] I DD] y AAIli CC, . Ademas,

1
LAPB = 7r -(LBAP +LABP)= 7r --(LBAD+ LABC)=
2
1 1 N,----,
=7r--7r= - 7r
22'

~c
C

B~
A~D
A C B D
J J o
Fig. 94 Fig. 95
Fig. 92 Fig. 93
GEoMETRiAIPlanimetria (Resoluciones y Soluciones) 249

De lamismamanerasedemuestraque ° 2 °3 = 0304 = 04Q 1t 1t


yque L.MEB + L.BNlvl = - + - = 1t.
2 2
De aqu] que L.MND = L.MAD Y L.MEN = L.MBN. Pero
L.MAD = L.MBN, puesto que cada uno de ellos abarca la mitad
Por eonsiguiente, 0P20304 es un cuadrado. del areo AM. Asf pues, L.MND = L.MEN. De analoga forma
se establece la igualdadL.NDM = L.ENM.
382. Admitamos que sean AP, BQ, CR y DS las bisectriees de De la semejanza de los triangulos DMN y NME obtenemos
los angulos internos del euadrilatero ABCD (fig. 95). Sean, que
ademas, A, B, C YD los valon!s de estos angulos. Entonces, DM MN
MN
= ME'
1 1 1 1
LASD = 1t - - A- - D L.BQC = 1t- - B- - C.
2 2' 2 2 10 que habfa que demostrar.
Sumando estas igualdades, obtendremos:

LASD.+ L.BQC = 21t - -1 (A + B + C + D ) = 21t - -1 21t = 1t.


2 2
Por consiguiente, los puntos P, Q, R YSse eneuentran en una
misma eireunfereneia.

383. Sean A y BIos puntos de tangeneia, M un punto arbitrario


de la eireunfereneia y MN .1. AB, MD.l AC, MEl. BC
(vease la fig. 96). Demostremos que los triangulos DMN y NME D
son semejantes. Con este fin, observemos que a los euadrilateros Fig. 96 Fig. 97
ADMN y NMEB se les puede eireunseribir eireunfereneias, puesto
que 384. SeaABC el triangulo inserito en la cireunfereneia, D el punto
1t 1t de lacireunferencia y L, My Nlos pies de las perpendiculares
L.MNA + LADM =-+ - = 1t (fig. 97). Unamos el punto M con el Ny el punto Neon el L, y
2 2
y demostremos que los angulosANMy LNC son iguales.
250 PROBLEMAS DE MATEMATICAS ELEMENTALES

Observemos, para ello, que

LANM = LADM, (1)


A JL--L-l------=:.;B
puesto que al cuadrilatero MAND se Ie puede circunscribir una
circunferencia. Por la misma causa

LLNC = LLDC; (2)


Fig. 99
Fig. 98
por otra parte,
que los tres segmentos indicados se cruzan en un mismo punto.
LADC = LMDL. (3) Por ejempJo, demostremos que los segmentos 0IAI y 0~2 se
dividen por la mitad en el punto de su interseccion B (vease la fig.
En efccto, LADC + LB = 180°, puesto que en suma estos
98). En virtud de la igualdad de las circunferencias deducimos
dos angu\os abarcan la circunferencia completa; al mismo tiempo,
que 0 1"'1
A 0 3 0 Y 0 IA 2 0 3 0 son rombos. De aqui se deriva que
LMDL + LB = 180°, ya que al cuadrilatero MELD se Ie puede 11 003 Y 0 IA 2 son i~.!:Uales
los segmentos 0 :fC"1' y paralelos. Por
~
circunscribir una circunferencia. Por consiguiente, la igualdad (3)
es justa. Del dibujo esta claro que, en este caso,
esta razon, 0IA~ P2 es un para\elogramo y el punto B de
interseccion de sus diagonales 0IAI y 0;12 divide aestas por la
LLDC = LADM, mitad.

y, entonces, de (1) y (2) se desprende la igualdad 386. Sea 0 el centro de la circunferencia menor (fig. 99). Entonces,
AK IIOC, puesto que AK ~ BK y OC ~ BK. Ademas,
LANM=LLNC, OA=OC. Por consiguiente,
que era necesario demostrar. LKAC = LACO = LCAO.

385. Demostremos que cada dos de los tres segmentos 0IAI' 387. Del examen de Ja fig. 100 esta claro que
o-;t.l..2
A y OA, se dividen por la mitad en el punto de su interseccion.
" oJ
De aqu! se desprende R-r R
r u
GEoMETRiAiPlanimetria (Resoluciones y Soluciones) 251

2a - - _ Por consiguiente,

--~~~------=D

Fig. 100

pero, esta igualdad es equivalente a la igualdad a) b)


D
1 1 1
- =-+ - .
r R G
388. Son posibles tres casos. Estos tres casos estan representados
en la fig. 101, G, b, c. En el primer caso las tangentes fijas son
paralelas, el angulo COD=a+I3=~, por eso

CE· ED = OE 2, es decir, AC· BD = r2, donde r es el radio


de la circunferencia. En los casos segundo y tercero, valiendonos c)
de las anotaciones, faciles de comprender en la fig., hallamos Fig. 101

que a + 13 ± Y = -i, es decir, a ± Y = ~ -13, de donde se 389. Supongamos que sea Mel punto de intersecci6n de las
desprende que ~ AOC es semejante a ~BDO y, por 10 tanto, cuerdas perpendiculares entre sf AB y CD (fig. 102). Tracemos
AK I CD, entonces, BK es el diametro, AK < CD y
AC OB
AO BD BK2 =AB2 +AK2 <AB2 +CD 2 .
252 PROBLEMAS DE MATEMATICAS ELEMENTALES

luego, KD = AC y, por 10 tanto,

~------7lB

390. Sea AC=CD=DB (fig. 103): Tracemos OE l.AB.


Entonces, OE es una de las alturas, y OC una de las medianas
B
del !1. ADD. Dado que la bisectriz del !1. ADD se encuentra entre
la mediana y la altura (vease el problema 355), entonces, Fig. 104 Fig. 105

LAOC < LCOD. Por eso,

AE . AD = AC 2 + EC 2 + BE· EC = AC 2 + EC . BC =
= ACJ. + (BC - BE)BC = AC 2 + BC 2 - BE· BC,

o defmitivamente

D AE·AD+BE·BC = AB2.
Fig. 102 Fig. 103
392. SeanAy BIos puntos dados, Del centro de lacircunferencia
391. Admitamos que seaAB el diametro de la circunferencia y E dada, R el radio de esta circunferencia y r el radio de las
el punto de intersecci6n de las cuerdas AD y BC (fig. 104). circunferencias iguales inscritas, cuyos centros son 01 y 02 (fig.
Tenemos: 105). Entonces,

AE· AD = AE2 + AE . ED = AC 2 + EC 2 + AE . ED.


Por la propiedad de las cuerdas que se cruzan Tomando la derivada de la proporci6n, obtendremos:

AE . ED = BE· EC. OA OB
GEoMETRiA/Planimetria (Resoluciones y Soluciones) 253

Porconsiguiente,OP2 1IAB. y, dado que

393. Sean r 1 y r 2 10s radios de las semicircunferencias inscritas AC>AM, BD>BM, MC=MD.
en la semicircunferencia dada de radio R (fig. 106). Puesto que
entonces,
R=r 1+r2' entonces, el area sombreada es igual a
AC +BD >AB >1AC -BD l ·
Si la rectaAB cruzaa lacircunferencia, son posibles dos casos:
a) la cuerda cortada par la circunferencia en la recta AB, se

.,
-
Pero, encuentra en el segmento AB; b) esta cuerda se encuentra fuera
del segmt'''~~ A D

cJ1
A

Por consiguiente, N

S = !rr.h 2 . C" ," Ii


4 --- £
394. Si la recta que une los puntos A y B no corta a la I

circunferencia dada, entonces las tangentesAC y BD pueden ser It:)


B
trazadas de manera tal, que el punto de su intersecci6n M se
Fig. 108 Fig. 109
encuentre en los segmentosAC y BD (fig. 107). En el triangulo
AMB tenemos:
En el caso a) (fig. 108) tenemos:

AB > AE + BF > AC + BD.

puesto que las hipotenusasAEy BF en los triangulos recmngulos


AECy BFD son mayores que los catetosACy BD.
Fig. 106
En el caso b) el segmentoAB se encuentra dentro del angulo
Fig. 107
CAC' (fig. 109). Tracemos por el punto B una circunferencia
concentrica a la dada. Supongamos que esta circunferencia corta
AM +BM >AB >1 AM -BM i, a AC y a AC' en los puntos EyE'. Entonces, EC=BD y
AE>AB. Por consiguiente,
254 PROBLEMAS DE MATEMAnCAS ELEMENTALES

AB < AE == AC - EC == AC - BD. de donde,


2 a 2 senysen(a+f3+y)
x = .
395. Introduzcamos las siguientes anotaciones (fig. 110): sen a sen f3 + sen ysen (a +f3+ y)

LPCM== LQCN= a, LNML=LNKL==y, LLCP=LQCK= jJ, Amilogamente se determina que


QC=x, PC=y, AC=CB=a. 2 a 2sen y sen (a + f3 + y)
y ==
sen a sen f3 + sen y sen (a + f3 + y) .

As! pues, x = y.

396. Sean B 1, B 2 , B3 Y B410s puntos medios de los arcosA1A z'


A~3,A~4Y A4A I (Fig. Ill). Sea, ademas, a 1 el angulo central
L correspondiente al arcoA,B;Ci=l, 2, 3, 4). Designemos por <p al
Fig. I IO
angulo formado por los segmentos BIB3 y B 2B 4 • Entonces,
Fig. III

De acuerdo con el teorema de los segmentos de las cuerdas


que se cruzan de una circunferencia, tenemos que y, puesto que
NQ.QK =AQ.QB=a 2 _x 2 .

Aplicando el teorema de los senos a los triangulos NQC y entonces,


1t
QCK,obtenemos: <p = -.
2
NQ = xsena QK = xsen f3 , 397. Elijamos los puntosA y Ben la linea quebrada de manera
sen(a+f3+Y)' seny
que dividan su perimetro en dos partes iguales. Sea 0 el punto
Por consiguiente, medio del segmento AB. Tracemos, tomando el punto 0 como
centro, una circunferencia de radio ~, donde pes el perf metro
NQ . QK = x 2sen a sen f3 = a2 _ x 2, de la quebrada. Demostremos que esta circunferencia es la
sen y sen (a + f3 + y)
buscada. Supongamos 10 contrario, 0 sea, que existe un punto
GEoMETRiAIP1animetria (Resoluciones y Soluciones) 255

M de la quebrada exterior a la circunferencia descrita. La longitud BP CQ


de la parte de la quebrada que contiene a M no es menor que tgLBAP= - y tgLCAQ=~
AP AQ
AM + BM, es decir, AM + BM s ~. Pero,
seran numeros racionales y, por 10 tanto, sera tambien racional el
AM+BM:::::2MO.
numero
En efecto, del paralelogramoAMBD(fig. 112)tenemos: BP CQ
--~

tgLBAP - tgLCAQ AP AQ
tgL BAC = -----=-- - -=----=-
DM = 2MO < BM + BD = AM + BM. 1+ tgLBAP tgLCAQ 1+ BP CQ·
Puesto que MO >~, entonces, de la desigualdad APAQ
AM + BM::::: 2MO se desprende que AM + BM >~. Por esta raz6n, es imposible que el LBAC = 60°, puesto
Obtenemos una contradicci6n. que tg 60° = F3 es un numero irracional. Por consiguiente, el
~ ABC no puede ser regular.
398. Tracemos por el vertice A del triangulo dado ABC la recta
AD paralela a una de las rectas dadas x e y y que no corta al 399. Supongamos que las rectasA 1By AB1 se crucen en el punto
triangulo. A continuaci6n,
N
Oyque sea OD 1.. AB (fig. 114). Puestoque t-,. ABA1 VJ t-,. DBO

A~B
Y t-,.BAB 1VJt-,.DAO, entonces,
...... \ ,,"
OD BD OD AD
........\,," " a AB' b AB
b
Fig. 112 Fig. 113
Deaqui,

bajemos desde los puntos B y C las perpendicu lares BP y CQ a OD - (~. ~ 1_ AD + BD - 1


- a + b) - AB -.
AD (fig. 113). Supongamos que las distancias desde los vertices
del trianguloABChasta las rectasxe y se expresan por numeros
Por consiguiente, la distancia
enteros. Entonces las longitudes de los segmentosAP, AQ, BP Y
CQtambien se expresaran por numeros enteros. En virtud de
eso.
OD=~~
a+b
256 PROBLEMAS DE MATEMATICAS ELEMENTALES

no depende de la disposici6n de los puntosA y B (si se conservan 401. SeaABCel triangulodado,AB=BC, BOliAC, Oeleentro
las magnitudes de a y b). de la eireunfereneia que haee eontaeto eonAC; DyE los puntos
A de interseeei6n de esta eireunfereneia con AB y BC (fig. 116).
Prolonguemos elladoAB hasta su segunda interseeei6n con la
B c eireunferencia en el punto F. Demostremos que FE.l Bo.
a Observemos que LOBF = LOBE, puesto que estos angulos
son iguales a los angulos en la baseAC del trianguloABC. Luego,
A
BF = BE; en efeeto, si fuera BF> BE, entonces, trazando en BF
el segmento BE '= BE, tendriamos que los triangulos OBEy OBE'
Fig. 114 Fig. 115 son iguales y que OE '= OE, 10 eual es imposible, puesto que el
punto E' se eneuentra dentro del cireulo de radio OE; de analoga
400. Si K es el punto de tangencia del segmento MN con la forma se demostrara que es imposible la desigualdad BF < BE.
circunferencia (fig. 115), entonees KM=MK y KN=NC, de donde Pero la biseetriz BO del triangulo FBE debera ser tambien su
altura, 10 que era neeesario demostrar. Por esta raz6n,
MN=BM+CN. (I) 1
LDFE = LABC no depende de la posici6n del punto Oen la
recta BO. Por eonsiguiente, la magnitud del areo DE, euya mitad
Pero, MN < AM + AN. Por eso
se mide por el LDFE, durante la rodadura de la eireunfereneia
2MN <BM +AM +CN +AN=AB+AC, permaneee constante.

de donde B

MN < AB+AC.
2

Por otra parte, MN> ANy MN> AM, puesto que MN es la


hipotenusa del triangulo AMN. Por eso 2MN> AN + AMy, en
virtud de (1) tenemos que 3MN> AN + NC + AM+ MB = AB +
+AC. Por consiguiente, Fig. 116
Fig. 117
MN> AB+AC.
2
GEoMETRiAIPlanimetria (Resoluciones y Soluciones) 257

402. Val iendonos de las denotaciones introducidas al resolver el 404. Supongamos que ellado AB del cuadrilatero ABCD cruza
problema 324, hallamos: ala circunferencia y que los lados BC, CD y DA hacen contacto
con ella en los puntos E, Fy G(fig. 118). Puesto que CE=CFy
n2 = ab + cd(ac+,
bd) m2 = bc + ad (ac+.
bd) DF=DG, entonces, la desigualdad AB+CD>BC+DA es
bc+ad ab+cd equivalente a la desigualdad AF>BE+AG, que fue demostrada
Dividiendo miembro a miembro estas igualdades, obtenemos: en la resolucion del problema 394.

n ab+cd 405. Supongamos que ellado AD del cuadrilatero ABeD no


m bc+ad corta a la circunferencia y que los lados BC, CD y BA hacen
contacto con esta en los puntos F, E Y G (Fig. 119). La
403. Sea ABC un triangulo regular con los lados ay rJ' r2 y r3las
desigualdad
distancias desde el punto M de la circunferencia circunscrita al
triangulo hasta los vertices de este (fig. 117). Observemos, al AD+CB < DC + BA
principio, que para la posicion del punto M dada en la fig. 117
tendremos que es equivalente a la desigualdad

AD<DE+AG,

En efecto, si trazamos DM= rz' obtendremos el triangulo que fue demostrada en el problema 394.
equil<itero BMD. De aqui se desprende que LABD = LCBM,
en virtud de 10 cual ~ ABD = ~ CBM y, por 10 tanto, AD = r 3 • 406. SeaR el radio de semicircunferencias dadas. Si r l , r z'·· .,rn
Aplicando al triangulo BMC el teorema de los cosenos, son los radios de las circunferencias inscritas y d l , d z' ... , dn sus
obtenemos: diametros (Fig. 120), esta claro que al aumentar inconmen-
surablemente n la suma d l + d z + ... +dn tiende a R, es decir,

(1)
Por consiguiente,
258 PROBLEMAS DE MATEMATICAS ELEMENTALES

G A Tenemos:

Pero,

E
d 1 + d 2 + ... + d = R 1_1- + _1 . + ... +
n ,,1.2 2·3
1
n(n + 1)
=J
B C,---..::o,..~=---....JD = R(I-~ +~_~+ ... +~ __
2 2 3 n
I_j\ = R_n_.
n+l n+l
Fig. 118 Fig. 119

Ademas, tenemos: Colocando esta expresion en (2); hallaremos:

R
R d n+ 1 = 2rn+l = (n+l )(n+2 ).
21j =d1 - - ;
1· 2
R Haciendo en la igualdad (l)R=I, obtendremos:
(R + rz ) 2 = R Z + ( R - d1 - r2 )2 , 2rz =d7 = - .
- 2·3
1 1 1
- + - + ... + + ... =1.
1. 2 2 .3 n(n + 1)

407. Sea 0 el centro de la mesa de biHar, B el primer punto de


R R rebotacion y C el segundo punto de rebotacion. Demostremos
quesi el LABC *- 0, entoncesel ~ABC es isosceles (fig. 121).
Fig. 120
Enefecto,el ~BOC esisosceles,poriotanto, LOBC = LOCB.
Por la de reflexion (el angulo de incidencia es igual al angulo de
Supongamos que sea d n = ~~. Demostremos que rebotacion) LOBC = LOBA Y LOCB = LOCA. As! pues,
LABC = LACB. Por consiguien-te, el centro 0 se encuentra
R en la altura AD trazada al lade Be. La posicion del punto B,
d n + 1 = (n+l)(n+2)" hacia el cual hay que dirigir la bola para que despues de rebotar
GEoMETRiAIPlanimetrfa (Resoluciones y Soluciones) 259

de B YC pase par el punto A, se puede fijar dandonos el angulo Si suponemos ahora que LABC = 0, obtendremos la segunda
LBOD = a. Tenemos: soluci6n del problema: los puntos B y C se encuentran en los
extremos del diametro que pas a porel puntoA.
OD = Rcosa BD = Rsena , BA= BD =_ BD
cos2(~ - a) cos2a·
408. Sea S el vert ice del angulo dado a, A lei punto del primer
Puesto que BO es la bisectriz del encuentro del rayo con el espejo, SB, elladodel angulo, en el
angulo B en el triangulo ABD, que se encuentra el punto A " y SBo el otro lado del angulo.
entonces, Designemos los siguientes puntos de encuentro del rayo con los
BD OD lados del angulo par A 2, A 3 , ••• , de manera que el trayecto del
BA
= OA rayo dentro del angulo tendra la forma de una linea quebrada
AAIA~3 ... (fig. 122).
o bien

Rcosa
- cos 2 a = ,
a Fig. 121

de donde obtenemos la ecuaci6n para el cos a

"
cos~
R 1
a + - cosa - - = 0.
2a 2

Resolviendo esta ecuaci6n, hallaremos: Fig. 122

Tracemos sucesivamente, en sentido de rotaci6n de SBo hacia


SBI' los angulos B I SB 2 , B 2SB 3 , ••• , iguales al angulo
a = LBaSBI • Tracemos en el lado SBm (m = 2, 3, 4, ... ) el
segmento SA' m= SAm (los puntos A' I Y Al coinciden) y
Prescindimos de la segunda raiz puesto que, en virtud de que demostremos que los puntos A' I' A' 2' ... se encuentran en una
R> a . da el valor de cosa < - 1. misma recta. Para ello es suficiente demostrar que cad a tres
260 PROBLEMAS DE MATEMAT]CAS ELEMENTALES

puntos sucesivosA'm,A' m+l,A'm+2 se encuentran en una misma


recta (suponemos aqu! m = 0, 1, 2, ... ). Observamos que n<.Q.::;n+1.
a
tl.A'mSA'm+! = tl.AmS~+!, en virtudde 10 cual
Para aclarar las condiciones con las cuales el rayo, despues
de cierta cantidad de reflexiones, pasani de nuevo por el punto
A, construyamos una serie de puntos C], C2 , ••• de manera que
Analogamente tl.A'm+!SA'm+2= tl.Am+!SAm+2 y, por el punto C 1 sea simetrico al puntoA respecto dellado SB 1, el
consiguiente, punto Cz sea simetrico al C] respecto allado SB2 , etc., en general,
de modo que el punto Cm sea simetrico al punto em -1 respecto
dellado SBm . Es evidente que el hecho de que el rayo pase de
nuevo porel puntoA esequivalente aque pase la recta lpor uno
Pero, por la ley de reflexi6n (el angulo de incidencia es igual al de los puntos C m (m = 1,2, ... ).
angulo de reflexi6n).
Para fonnular analiticamente esta condici6n introduzcamas el
angulo y = LASBo y distinguiremas dos casos:
a) el punto Ck por el que pasa la recta I es tal, que kes un numero
Por consiguiente, par;
b) el punta Ck es tal, que kes un numero impar.
En el caso a) (este caso esta representado en la fig. 122, donde
k= 6) LASCk = ka . Puesto que tl. ASCk es is6sceles, entonces
De este modo, el trayecto del rayo, la quebrada AA lA2' ... ,
ha resultado desarrollado en la recta I(AA' ]A' J Puesto que esta LSAC =~_ ka
k 2 2
recta puede cruzar solamente un numero fmito de lados SBm' por
consiguiente, el numero de reflexiones del rayo es finito. Por otro lado, el mismo angulo es igual a y + 1t -13, par
Esta claro que si SBn es el ultimo lado que corta la recta I, consiguiente,
entonces na < 13, y {n + l)a;:::: 13. Asi pues, el numero de
rcflexiones es igual a un tal numero entero n que satisface a las
dcsigualdades
GEOMETRiAiPlanimetrfa (Resoluciones y Soluciones) 261

dedonde, con la particularidad de que EAF es tangente en el punto A ala


circunferencia dada (fig. 123).
k = 2[3 - 2y - 1t . Demostraci6n. Sea N un punto dellugar geometrico buscado,
(1)
a obtenido con ayuda del punto Mtomado en el arco inferior AB.
En el caso b) tendremos que Segun la construccion el triangulo NMB es isosceles y, por 10
tanto,
LASCk = (k + I)a - 2y
L BNA = ~ L BMA = ~ L BCA.
y, como anteriormente, obtendremos la relacion 2 2

2:_ (k+l)a-2y =y+1t-[3, Por consiguiente, el punto N se encuen-


2 2 tra en la circunferencia de centro C que
pasa por los puntosAy B. Luego, el punto
de donde N debera encontrarse dentro del lingulo
k + 1 = 2[3 - 1t . BAE, es decir, se encuentra en el arco BE
(2) de la circunferencia de centro C.AI
a
contrario, si N se encuentra en este arco,
Si razonamos ala inversa, nos convenceremos facilmente de entonces
que el cumplimiento de una de las relaciones (1) Y(2), para un Fig. 123
valor entero de k, conduce a que la recta I pase por el punto Ck • L BNA = ~ LBCA = ~ LBMA.
Por consiguiente, el rayo pasara de nuevo por el punto A cuando, 2 2
y solo cuando, (1) 0 (2) sea un numero entero par.
de donde se desprende que LBNA = LNBM Y que el
f:..NMB es isosceles. As) pues, el punto N se obtiene de la
construccion indicada. De analoga forma se efectua la
4. Lugar geometrico de los puntos demostracion en el caso cuando el punto M se encuentre en el
arco superior AB.
409. Ellugar geometrico buscado esta compuesto por dos arcos
de circunferencias: el arco BE con su centro en el punto medio C 410. Ellugar geometrico buscado se compone de dos rectas ly
del arco AB de la circunferencia dada y el arco BF con centro en k dispuestas simetricamente con respecto de la perpendicular
el punto medio del segundo arcoAB de la circunferencia dada, com un BB' a las rectas paralelas dadas trazada a traves del punto
262 PROBLEMAS DE MATEMATICAS ELEMENTALES

O. La recta 1 para por el punto C perpendicularmente a OC, 411. En el caso de rectas que se
ademas, B"C= OB(fig. 124). cruzan, ellugar geometrico buscado

~M~
se com pone de cuatro segmentos
A B
que forman el rectangulo ABCD;
cuyos vertices se encuentran en las
C B m
K rectas dadas I y m y a una distancia
Fig. 125 de estas igual a la distancia dada a
D
" (fig. 125).

Demostraci6n. Sea el punto Mtal, que MK .1/, ML.1 m


N
y MK + ML= a, donde a es la longitud del segmento dado.
Fig. 124
Tracemos por el punto MIa rectaAB de tal manera que OA =
Demostraci6n. Sean My Nlos puntos obtenidos durante la =OB, y .MN!IOB. Sea AP .1 OB y Qel punto de intersecci6n
construcci6n con ayuda de la secante AA". La demostraci6n se de AP con .MN. De la igualdad AN = MN se desprende que
lIeva a cabo solamente para el punto M (para el punto N se MK = A Q y, por consiguiente,
realiza analogamente). Sea MP.1 B"C; entonces,
AP=AQ+QP=MK +ML=a.
L OAB = L A"MP (como angulos con lados perpendiculares).
Por esta razOn, los triangulos rectangulos OAB y A" MP con iguales Por consiguiente, el punto A es un vertice del rectangulo
hipotenusas OA y A" M, son iguales. Por consiguiente,A" P = OB mencionado. Lomismo esjusto parael puntoB, as! que el punto
=B" C. De aqu! se desprende que si E es el punto medio de OM, M se encuentra en uno de los lados de este rectangulo. Al contrario,
entonces, los puntos M, A", C Y 0 se encuentran en una si A1 se encuentra en uno de los lados de este rectangulo, entonces,
circunferencia con centro en el punto E y, por consiguiente, razonando ala inversa, obtendremos que MK + ML = AP = a.
MC .1 ~C, es decir, el punto M se encuentra en la recta I. AI Si las rectas I y m son paralelas y la distancia entre elias es
contrario, si 11/[ es un punto de la recta 1y el angulo MA" 0 es igual a h, ellugar geometrico buscado existe solamente cuando
recto, entonces A" P = B" C = OB, de don de se deriva la igualdad a ~ h, y representa un par de rectas paralelas a las dadas para
de los triangulos OAB y A" MP y, por fin, la igualdad OA=A" M. a> h, y toda la zona entre I y m cuando a = h.
Por consiguiente, el punto M se obtiene de la construcci6n
examinada. 412. En el caso de rectas que se cruzan, ellugar geometrico
buscado se com pone de ocho semirrectas que son las prolonga-
GEOMETRLA.!Planimetria (Resoluciones y Soluciones) 263

ciones de los lados del rectanguloABCD indicado en laresolucion la disposicion de estos segmentos en las rectas I y m. En efecto,
del problema 411 (fig. 126). La demostracion es amiloga a la al cambiar esta disposicion, las areas de los triimgulosA.MB y
demostracion dada en el problema anterior. CMD no varian. Por esta razon, es suficiente examinar el caso
Si las rectas dadas I y m son paralelas y la distancia entre elias particular cuando los segmentos AB y CD tienen un extremo
es igual a h, ellugar geometrico buscado existe solamente cuando comun en el punto de interseccion de las rectas I y m. En este
a:::;; h, y representa un par de rectas paralelas a las dad as en el caso los segmentosAB y CD seran los lados de un triangulo; el
caso en que a < h, 0 la parte de un plano que se encuentra fuera tercer lado del cual se encuentra en uno de los cuatro angulos
de la zona entre I y m, cuando a = II. formados al cruzarse las rectas I y m. Por ejemplo, en la fig. 127
coinciden los extremos A y C y el tercer lade en BD.
Sea A1un punto dellugar geometrico buscado, que se encuentra
dentro del angulo BAD. Entonces el area del triangulo BMD sera
iguala
I ! 2 !
fig. 126 Fig. 127
S HMD = I SAMB + SCMD - S4BDi = I a - S ABO i .
De aqui se desprende que la distancia del punto Mala recta
413. Si el segmento AB se encuentra en la recta I y el segmento BD no depende de su posicion en la recta PQI!BD. Para los
CD en la recta m, entonces, el lugar geometrico buscado se puntos P y Q se cumplen las relaciones (I ).
compone de cuatro segmentos que forman el paralelogramo Al contrario, supongamos que sea M un punto cualquiera en
PQRS, en el cuall y 111 son diagonales y la posicion de los vertices la recta PQ, donde los puntos P y Q han sido construidos de
P y Q se determina de la relacion acuerdo con (I). De las relaciones
?
AP SAFD
a- CQ SCQB a2
(1) ,
AB SARD SARD CD SCOR SABlJ

donde hp y hQ son las distancias desde los puntos P y Q hasta las se deduce
rectas m y I (fig. 127).
AP CQ
Demostraci6n. Observemos que para las rectas I y m fijadas,
AB CD'
ellugar geometrico buscado queda determinado por las longitudes
de los segmentos AB y CD y la con stante a, pero no depende de
264 PROBLEMAS DE MATEMATICAS ELEMENTALES

es decir, PQjIBD. Por eso, AMCD es un paralelogramo y, por 10 tanto, AMIIDC. Pero
DC .1 BC, puesto que ABCD esta inscrito en K y el angulo
S AMB + SCMD = SAHD + SBMD = SABD + SBPD = SAPD = a 2 . BAD es recto. Por eso AM .1 Be y M es el punto de intersecci6n
de las alturas del ~ ABC. Por consiguiente, M pertenece allugar
Por consiguiente, el punto M pertenece allugar geometrico geometrico buscado.
buscado. Los demas lados <leI paralelogramo PQRS se obtienen
de forma analoga al hacer coincidir otros extremos de los 415. Sea Oel centro de la circunferencia dada y R su radio (fig.
segmentos, a saber: QR si B == C, RS cuando B == D y SP en el 129). Ellugar geometrico buscado es la recta I perpendicular a
caso en que A == D. la recta OA y que corta a esta recta en el punto B de manera que

OB= R2 (1)
414. Ellugar geometrico buscado es una circunferencia sirnetrica
OA
a la circunferencia dada K con respecto de la cuerda dada AB
(fig. 128). Demostraci6n. Tracemos por el punto M una recta I .1 OA
Demostraci6n. Tracemos en la circun- que cortara a la recta OA en el punto B. Supongamos que sea C
ferenciaKlacuerda AD.l AB. Suponga- el punto de intersecci6n del segmento OM con la cuerdaKL. De
mos que el ~ ABC esta inscrito en K y la semejanza de los triangulos OACy OMB se deduce:
que sea Mel punto de intersecci6n de las OB OM
alturas de este triangulo. Es facil verque
OC OA'
AMCD es un paralelogramo: DA//CM
como perpendiculares aAB y DC AM de donde
como perpendiculares a BC(DC .1 BC,
OB= OM.OC.
puesto que BD es el diametro de K). Por (2)
OA
esta raz6n, el punto M se encuentra en la
circunferencia K' obtenida desplazando la SegUn la construcci6n, KC es una de las alturas del triangulo
Fig. 128
circunferencia Kala distancia AD en rectangulo OKM, por consiguiente,
sentido de la cuerda DA. Es evidente que esta circunferencia
K' es simetrica a la K respecto a AB. Al contrario, sea M un
punto en K' y MC .1 AB. Puesto que MC = AD, entonces
GEoMETRiAIPlanimetria (Resoluciones y Soluciones) 265

Sustituyendo esta expresion en (2) obtendremos la igualdad esta circunferencia, entonces L P MQ = -I. Tracemos a traves
(1). del punto B RS JI AM, entonces
AI contrario, sea M un punto cualquiera de la recta I
perpendicular a OA y tal, que OB se determina por la igualdad
(1). Tracemos la tangenteMKy KC 1- OM. Supongamos que
KC corta a la recta OA en el punto A'. Entonces, repitiendo la
primera parte de la demostracion hallaremos que OB se determina
par la formula (1) sustituyendo OA par OA'. De aqui obtendremos
que OA'= OA, es decir, el puntoA' coincidinicon el puntoA, 10
Fig. 129 Fig. 130
cual significa que el punto M pertenece allugar geometrico
buscado. AM AQ P AM AP P
BR BQ q BS BP
=q (2)
416. Sea
de donde BR = BS y BM es una mediana en el triangulo RMS.
Puesto que el ~ RMS es rectangulo, BM= BR y, en virtud de (2).
AM P
Tracemos las bisectrices MP y MQ de los dos angulos
BM q
adyacentes con el vertice My los lados MA y ME (fig. \30).
Entonces, por la propiedad de las bisectrices tendremos: Por esta razon el punto M pertenece allugar geometrico que
AP P AQ P seexamma. "
- =- y- =- . (1) Para expresar el diametro PQ por medio de la longitud a del
BP q BQ q
segmentoAB, de las relaciones
De aqui se desprende quela disposicion de los puntos P y Q p
no depende de la del punto M. Puesto que, ademas, PB = AB - AP = a - - P B,
q
L P MQ = -I, entonces, el punto M se encuentra en la circunfe-
rencia K de diametro PQ. Al contrario, supongamos que los BQ= AQ-AB = -P BQ-a,
puntos P y Q se han construido de acuerdo con (1) y que K es la q
circunferencia de diametro PQ. Si el punto M se encuentra en
266 PROBLEMAS DE MATEMATICAS ELEMENTALES

hallamos: 418. Ellugar geometrico buscado es una recta trazada por dos
posiciones cualesquiera del ultimo vertice.
PB=a-q~ , BQ=a-q~ , Demostracion. Sea, por ejemplo AIB]C] DI El una de las
p+q p-q posiciones del poHgono deformable y A2 B2C2 D2 E2 otra de elias.
dedonde Los vertices A, B, C y D de este poligono se deslizan respec-
tivamente por las rectas lA' I B' Ie y I D (fig. 132). Tracemos la
recta I por las posiciones E[ y E2 del ultimo vertice.

Si P = q, entonces, el lugar geometrico buscado sera,


evidentemente, la perpendicular ala rectaAB trazada desde el
punto medio del segmentoAB.

417. Ellugar geometrico buscado es la perpendicular al segmento e c,


Ie
AB trazada por su punto medio E.
Demostraci6n. El triangulo ADB es isosceles, ya que
Fig. 131 Fig. 132
LCAD = LCBD, como angulos queabarcan igualesarcos CD
en iguales circunferencias (fig. 131). Por esta razon, el punto D
Supongamos que el vertice en la recta IA ocupola posicion A,
se encuentra en la perpendicular al segmentoAB trazada par su
yen la recta If)' la posicion D. Ellado paralelo aA2E2 cortara a
punto medio E. AI contrario, si tomamos cualquier punto D en
esta perpendicular, que no coincida con el punto E, entonces las I en el puntoE', y ellado paraleloaD2E2' enel puntoE". Segun
circunferencias que pasan por ACD y BCD son iguales. Esto se la construccion
desprende, por ejemplo, de las igualdades E'E2 _ AA2 _ BB2 _ CC2 _ DD2 _ E"E2
CD CD E2E] - A2AI B2B] C2C1 - D].D] - E2EI '
R] = = - - - = R2 ,
2 sen a 2 sen 13
de donde
donde a = LBAD Y 13 = L CBD.
GEOMETRiAIPlanimetria (Resoluciones y Soluciones) 267

es decir, los puntos E' y E" coinciden. Esto significa que el ultimo 1
vertice se hall ani sobre la recta 1en el punto E == E' == E". LAMB = - (13+ y). (1)
2
Lo inverso es evidente, puesto que la posicion del poligono
deformable puede ser construida comenzando desde cualquier Si por 10 menos uno de los puntos C y D resulta en el area 13,
punto E en la recta I. entonces el punto M sera exterior a K y

419. Ellugar geometrico buscado es una circunferencia que pasa LAMB = -1 (a -y). (2)
2
por los extremos de la cuerda AB y uno de los puntos MI
obtenidos de la construccion indicada en las condiciones del
problema.
Demostraci6n. Introduzeamos previamente algunas denota-
eiones. Existini una, y solo una, posicion CPI de la cuerda CD
en la que C,DdI AB y cuando en la cireunferencia dada K se
puede elegir tal direccion de giro y, al moverse en la euallos
extremos de las cuerdas se eneontranin en la sueesion A, B, C I Y
DI (esta eleeeion pude ser indeterminada solamente en el caso
y
de la igualdad AB = CD, euando las reetas AC y BD son
paralelas). Designemos por a la euerdaAB de la eircunferencia Fig. 133
dada K, sobre la que se hallan los puntos C I y D I' por 13, otra En el primer caso M se encuentra sobre el arco AMIB de la
cuerdaAB y por y, aquella de las euerdas CPI sobre la que no cireunferenciaKI, puesto que de aeuerdo con (1) el LAMB no
se hallan los puntosAy B.Aeontinuaeion, anotemos con MI el depende de la posicion de CD y, por consiguiente, es igual al
punto de interseccion de las rectasAC I y BDl' EI punto MI se L AMJB. En el segundo caso, debido a que la suma de los
encuentra dentro de K. Sea KI la circunfereneia eireunserita al miembros dereehos de (1) y (2) es igual a
~ ABM, (fig. 133). Demostremos que, cualquiera que sea la
posicion de la cuerda CD, el punto de interseccion de las rectas 1 1
2(a + (3)2' 2n = n,
AC y BD se encontrani sobre K I •
Mientras ambos puntos Cy D se encuentren sobre el area a,
el punto M se encuentra en el arco AB de la eireunfereneia K I ,
el punto M se encontrani dentro de K y, entonces,
exterior a K.
268 PROBLEMAS DE MATEMATICAS ELEMENTALES

Es evidente que es justo tambien 10 inverso, es decir, que vez ala circunferencia 0 1 en el punto Q, y a AR en el punto S,
cualquier punto M de la circunferencia KI puede ser obtenido entonces
eligiendo adecuadamente la posici6n de la cuerdaCD. AQ·AP=AS·AR. (2)

420. Designemos lacircunferencia .. ~ De (I) Y(2) se desprende la igualdad


dada por 0 y la recta dada por L .IV' '. A

(fig. 134). Sea Mel segundopunto"" Q


AM AQ
de intersecci6n de la recta PQ con .~.. 0 p 0, L (3)
0: Tomemos una circunferencia AN AS
cualquiera 0 1 que pasa por los
Fig. 134
puntos P y Q y que corta por se- De la igualdad (3), en virtud del teorema inverso al teorema
gunda vez a la circunferencia 0 en el punto R y a la recta Len el sobre la proporcionalidad de los segmentos cortados por rectas
punto S. Sea N el segundo punto de intersecci6n de la recta RS paralelas en los lados de un angulo, se deriva que MNIIQS, 10
con la circunferencia 0. que era necesario demostrar.
Demostremos que MNjIL Con este fin, apliquemos el De este modo, para cualquier circunferencia tipo 0 1, el punto
siguiente conocido teorema de la planimetria: si se conocen una N puede determinarse como el segundo punto de intersecci6n
circunferencia y un puntoA, entonces, para cualquier recta que de la recta que pasa por My que es paralela a L, con la
pas a por A y que corta a esta circunferencia en los puntos A I Y circunferencia O. Esta construcci6n detennina un mismo valor
A 2 , el producto de los segmentos AAJ • AA2 es una magnitud del punto N independientemente de la elecci6n de la circunferencia
constante que no depende de la elecci6n de la recta. 0 1• Por consiguiente, todas las rectas posibles RS obtenidas para
Designemos por A el punto de intersecci6n de las rectasPQy diferentes circunferencias 0 1 cortan a la circunferencia 0 en el
RS. Al principio apliquemos el teorema mencionado a la puntoN.
circunferencia 0, al punto A y a las rectas AP y AR. Puesto que Los casos excepcionales cuando de (1) Y(2) no se deriva (3),
AP corta por segunda vez a 0 en el punto M, y aAR en el punto por ejemplo, cuando coinciden los puntos R y P 0 los Q y s, 0
N, entonces cuando PQII RS, pueden ser exam inados como I [mites para el
AM· AP = AN· AR. (1) caso genera y se pueden emplear los razonamientos de
continuidad.
Apliquemos, ahora, el mismo teorema a la circunferencia 0 1,
al punto A y a las mismas rectas. Puesto queAP corta par segunda
GEOMETRiAIPlanimetria (Resoluciones y Soluciones) 269

5. Determinacion de los valores maximos y minimos Observando que

421. Si A es el vert ice del angulo recto del ~ ABC Y C YB se


encuentran sobre las rectas paralelas dadas II y 12 (fig. 135),
cotg a
2 + cotg (TC4 -2 a) =
entonces
2
cos a sen (TC 4 - 2a) + cos (TC4 - 2aJ sen 2a
AB=~a_ , BC=~b_.
sen <p cos<p a (TC ai
sen 2 sen l4 - 2 )
Par consiguiente, el area del triinguloABC sera igual a
TC
1 ab 2sen - 2
SABe =-AB·AC= -~ 4 _ __ ---;-_~-
2 sen 2<p
cos( a - 2:J - cos 2: - .J2 cos(a - 2:) - 1 '
De aqul se desprende que SARC tendra su valor minimo igual a \ 4 4 4
ab, cuando <p = *. obtenemos:
R 1

~ ~
r - J2 cos(a - 2:
4)
1-1 .
~ B E /, f-O. 2R La magnitud 1f tiene valor minimo cuando cos ( a - ~ ) = 1,
Fig. 135 Fig. 136
es decir, (en virtud de la limitaci6n 0 < a < ~ ) cuando a = ~;
422. Si R es el radio de la circunferencia circunscrita y r el de la en estecaso
inscrita (fig. 136), entonces
R 1 ,.
- = ----r:.=-- = -V 2 + l.
2R = rcotg -a + rcotg (TC
- - -aJ . r --v2-1
2 4 2
270 PROBLEMAS DE MATEMATICAS ELEMENTALES

M 423. Supongamos que del Transformando esta expresion a la forma


rectangulo ABCD cortamos un
triangulo con el vertice C, de tal
a manera que se obtenga el (1)
pentagono ABEFD (fig. 137).
b NEsta claro, que el rectangulo
deducimos que S tendra su valor maximo cuando ~ - x == 0, es
Fig. 137
buscadoABICPI debera tener
decir, cuando x == ~. Designemos par Co la posicion del vertice
el vertice C I sobre el segmento
C I, correspondiente a x ==~.
EF. El problema consiste en hallar la posicion de este vertice.
Para hallar el punto C I, prolongamos los lados AB y AD del Observando que la expresion (1) para S decrece al aumentar
rectcingulo hasta su interseccion con la prolongacion del segmento I~- xl, es decir, al moverse el punto C I desde el punto Co hacia
EF, formando el trianguloAMN. Sea el vertice M 0 hacia el vertice F, haHamos que son posibles los
tres casos siguientes:
AM == m, AN == n 1) El punto Co se encuentra sobre el segmento EF; en este caso,
y el vertice C I del rectangulo buscado coincide con Co'
2) El punto Co se encuentra sobre el segmento ME; entonces,
C 1 debe tomarse coincidente con E.
De lasemejanzade los triangulosAMNy DICINtenemos: 3) El punto Co se encuentra sobre el segmento FN; en este caso,
el punto C 1 debe tomarse coincidente con F.
C]D] n-x Queda hallar el criterio para distinguir estos casos con ayuda
m n de las magnitudes a, ai ' b y b l dadas en las condiciones del
de donde problema.
Primeramente hallemos la magnitud n. De la semejanza de los
triangulos ECFy NDFtenemos:
Porconsiguiente, para el areaS del recmnguloABICPI' igual n-b h)
a AD) . C)D), obtenemos laexpresion a-a) a]
dedonde
GEOMETRiAIPlanimetria (Resoluciones y Soluciones) 271

si no se cumple la desigualdad izquierda de (3), el vertice C I


(2) coincide con el punto E, y si no se cumple la derecha, con el
puntoF.
Observemos, ahora, que el punto Co resultara dentro del 424. Describamos una circunferencia
segmento EF si se cumplen las desigualdades que pase por los puntos A y B Y que
haga eontaeto con el segundo lado del
b-bl <x<b. angulo (fig. 138). El punto de tangen-
cia sera el punto buscado, puesto que
Sustituyendo aqul x = 1, por el valor conocido de n,
para cualquier punto C' perteneeiente
obtendremos: Fig. 138
a esta recta el anguloAC'B se mide
b-b <~+Jl(a-a
2 2a
)<b. por la semidiferencia de los arcos AB y AIB I, mientras que el
I l I L ACB se mide por la mitad del arcoAB.
Observemos a eontinuaci6n, que (OC)2 = OB· ~A. Por
Estas desigualdades se pueden transformar facilmente a la consiguiente, el problema se reduce a la construeci6n eonocida
forma de la media aritmetica de las longitudes de los segmentos dados
Q b OAyOB.
-1<-- - <l. (3)
QI bl
425. Analicemos tres casos posibles de disposici6n del segmento
Si no se observa la desigualdad izquierda, el punto Co resultani AB respecto a f.
en el segmento ME, y si no se cumple la desigualdad derecha, a) ABIII . Para cualquier punto M de la recta I tenemos que
sobre el segmento FN. lAM - BMI ~ 0, ademas, existe un punto Mo para el eual
Definitivamente se obtiene el siguiente resultado: si para los IAMo - BMol = 0.
datos a, b, a l y b l se cumplen las dos desigualdades (3), entonces Este punto es el pie de la perpendicular bajada desde el punto
el vertice C I del rectangulo de area maxima se encuentra dentro medio del segmento AB a la recta I. EI punto M para el cualla
de los \imites del segmento EF y el lado x de este rectangulo se magnitud lAM - BMI tend ria su valor maximo, no existe. Esto
calcula por la f6rmula se desprende de que lAM - BMI ::::; AB y la igualdad es posible
solamente en el caso cuando A, By M se encuentran sobre una
misma recta.
272 PROBLEMAS DE MATEMATICAS ELEMENTALES

b) AB 1- I. Puesto que IAM _ BMI ::; AB, entonces, para el Tenemos:


punto de interseccion de la recta 1con la rectaAB, la magnitud S = 20"+x+ y.
lAM - EMI tiene su valor maximo igual a la longitud deAB. El
punto M para el cualla magnitud lAM - BMI seria minima, no Es evidente que
existe. X a y b
c) La recta AB no es paralela y no es perpendicular a I. Es = ,
0" b' 0" a
evidente que lAM - BMI adquirini su valor minimo si M es el
punto de interseccion de la recta leon la perpendicular al punto Por consiguiente,
medio del segmento AB. La magnitud lAM - BMI tendra su
valor maximo cuando el punto M sea el punto de interseccion de S +
= 0"(2 ~ +~) = 40"
b a
+0"-,-(a_-_abb--,-)_2
ABconl.
EI valor minimo S = 40" se obtiene para a = b, 10 que era
426. Sea lY1N una posicion cualquiera de la secante, APII ON Y necesario demostrar.
AQ 1 0M(fig. 139).
427. Sea a + b = q (fig. 140). De acuerdo con el teorema de
c
p .. los cosenos
0" \ a ..··
......
O«-----:Q~---h-T
b c 2 = a 2 +b 2 -2ab cosrp = a 2 +(q _a)2 -2a{q -a)cosrp =
Fig. 139 Fig. 140 = q2 +2a2(1+cosrp)-2aq(1+cosrp)=
Introduzcamos las siguientes denotaciones: = q2 1-C;S9' +2(1+cosrp) (a-{Y-
X =area A APM,
y = area AAQN, Puesto que q y <p son invariables, el valor minimo de c sera
0" = area !:t.APQ, cuando a = %= a;b ,
es decir, cuando sea a = b.
S = area !:t. OlY1N,
428. Primera resoluci6n. Examinemos el !:t. ABC con base
a=AM,
A C Ydesignemos por a, bye las longitudes de los lados opuestos
b=AN. respectivamentea los angulosA, By C; hagamos a + b + c = p.
GEOMETRiAlPlanimetrfa (Resoluciones y Soluciones) 273

De las relaciones Desde el punto D describamos una circunferencia de radio


a c b AD = DB, prolonguemos AC hasta su interseccion con la
circunferencia en el punto My unamos el punto M con los puntos
sen A = sen{A+B) = senB Dy B. Obtendremos:
hallamos:
AD + DB = AD + DM > AM = AC + CM.
p = b +b sen A + b sen {A + B) = b-b- sen(A + BI. Pero en el triangulo BCM
sen B sen B B 2)
sen -
2 LCBM = LACB-LCMB = LCMB,
Puesto que b >
cuando sea
° y sen 1> 0, p tendnl su valor maximo puestoque LACB = LADB ysemideporel arcoAB, mientras
que el LAMB se mide por la mitad del arco AB. Por
B
A+ - =-.
1t consiguiente, CM = CB Y AD + DB> AC + CB.
2 2
429. Designemos por R] y R2 los radios de las circunferencias
En este caso A = C y el L! ABC es isosceles. circunscritas respectivamente a los triangulosACD y BCD, Y
Segunda resolucion. Tracemos con la base dadaAB como hagamos L ADC = <p, AC = b y BC = a (fig. 142). Tene-
cuerda un segmento que abarque el lingulo dado <p (fig. 141) Y mos:
examinemos los dos triangulos inscritos en este segmento: el
b a a
triangulo isoscelesADBy el no isosceles ACB. 2R,= - - , 2R2 = = --.
sen<p sen (1t - <p) sen <p

De aqui :~ =!. Los radios R] y R2 seran los minimos cuando


sea <p = ~; en este caso D sera el pie de la altura CD.

430. Cada una de las circunferencias cortadas debera hacer


contacto con dos de los lados del L!ABC (vease la fig. 143);
Fig. 141 Fig. 142 ademas, las circunferencias deberan tener contacto una con la
274 PROBLEMAS DE MATE MATI CAS ELEMENTALES

otra. En el caso contrario el radio puede ser aumentado. Por Del L\ Al BCI se desprende que ~ a = d sen -'}. Por eso
esta razon, los centros de las circunferencias se encuentran en
dos bisectrices de los angulos internos, por ejemplo, en lasAOy
i
CO, donde 0 es el centro de la circunferencia inscrita en el I = \j12 2
d -a =
a 2a
J1- 4 sen -
L\ ABC. Si res el radio de lacircunferencia inscritaen el L\ ABC a ' 2
2sen -
y P es el radio de las circunferencias cortadas, entonces, del 2
L\ A OC tenemos que B y, por consiguiente,
r-p r
a 3 J3 I 2 a
2p b' v= 011 - 4 sen -
a ~ 2'
8sen-
de don de hallamos que 2
p _ b -1 2r dedonde
--; - b + 2r - - b + 2r . c
De esta formula se desprende
Fig. 143
que p sera maximo cuando como
b se toma ellado mayor.

432. Sea H la altura de la piramide y ala longitud dellado de la


B. ESTEREOMETRIA
base. Examinando los triangulos semejantes OMSy ABS (fig.
145) hallaremos:
1. Problemas de calculo

431. Sea a ellado de la base, d la diagonal de la cara lateral del


h
prismay Ila arista lateral (fig. 144). Tenemos:
a H (1)
Ii
GEOMETRiAiEstereometria (Reso\uciones y Soluciones) 275

Colocando esta expresi6n en (1), hallaremos facilmente que

En resumen, para el volumen V obtenemos la siguiente


expresi6n:
16 b3 h3
B V =3 (h2 _ b2 )\I2b2 _ h 2 .

Fig. 144 Fig. 145


433. Sea Hla altura de la piramide, x la altura de la cara lateral,
trazada desde el vertice de la piramide, R el radio de la
Amilogamente, de los triangulos OKS y CBS obtendremos: circunferencia inscrita en la base, rei radio de la circunferencia
circunscrita a la base ya ellado de la base. De la semejanza de

~
los triangulos CA IBI YCAB (fig. 146) obtenemos:
b H-h R s
a H (2)
H r
2
de donde
Dividiendo miembro a miembro la igualdad (1) entre la (2)
tendremos: H= -~.
r-R
I'H2 - 4h2 h
~ H2 -4b 2 = b-fi' Pero, del L1 ADB tenemos que
B
dedonde R Fig. 146
r= - -
1[
cos -
n
276 PROBLEMAS DE MATEMATICAS ELEMENTALES

y, por 10 tanto, 0, defmitivamente,


h
H = - --
7t
1- eos-
n
Puesto que para el area de la base y el volumen tenemos las
_ sen .~ Hl-COS:)[3V(I-COS~;)
Slat - n n,lI 2 7t + 7t
h2
2
lj.
siguientes f6rmulas \1 nh sen - nh tg - (l-cos2:)
j n n n
1 2 27t 1
Sbase =n - r sen- Y V=-Sbase H ,
2 n 3
entonees, 434. Sean My N los puntos medios de las aristas ES y DS (fig.
6V 147); es faeil ver que AMNC es un trapecio, ya que
27t· MNIIED y EDI!AC. Es evidentetambien que
Hnsen -
n 1
MN=-q.
Colocando aqui el valor haHado de H, haHamos: 2

Haciendo uso de la f6rmula (1) para el cuadrado de la mediana


6V(I- eos~) de un triangulo en la resoluci6n del problema 370, hallaremos:
r=
'\ nhsen -
27t

a 7t Luego,
-!
Puesto que x = R2 + H2 Y 2 = r sen -;;' la superfieie
lateral es igual a
I
n - xa = nr sen -7t \jIR 2 + H -~ , puesto que L ABK = ~~. Si KL es el segmento que une los
2 n
puntos medios de la base del trapecioACNM, entonees
GEOMETRiAiEstereometria (Resoluciones y Soluciones) 277

c
SB = MB = a".)3 .
2
La altura de la cara lateral es

(aqui aprovechamos que sen f~ = ~+l). As) pues, el area


buscada sera igual a
Por eso

S _ 3a 2 .fi
lat - 4
435. Sean E y F los puntos med ios de las aristas de la piramide
y, puesto que el area de la base es igual a
regular triangular SABC y D el punto medio del segmento EF
(fig. 148). Dado que la secci6n es perpendicular ala cara CSA , a 2 .fj
6ntonces, el LSDB es recto. Prolongando SD hasta su Sbase = - 4-
intersecci6n con la recta AC en el punto M, examinemos el
triangulo MES. EI punto D , obviamente, divide al segmento SM entonces
por la mitad. Puesto que, ademas, BD -.l MS, entonces, el
triangu 10 MBS es is6sceles; SB = ME. Supongamos que ellado
de base de la piramide es igual a a. Entonces,
:_R_:C 436. Sea ala longitud dellado del cuadrado que se encuentra en
la base del prisma, lla longitud de la arista lateral del prisma y d
la diagonal de la cara lateral (fig. 149). Designemos por Ssec el
area de la secci6n; se ve facilmente que la superficie total del
prismasera igual a 4(S - S sec); poreso, es suficiente determinar
Ssec . Tenemos:
1( c'-'---------'-''''' 8
II

Fig. 147 Fig. 148


278 PROBLEMAS DE MATEMATICAS ELEMENTALES

a = d f2sen a. S Ssen a- - - --
\j';' 2' = - -- - - --
sec oa ~ a --
I sen a + 2sen~ - + 2-J2 sen - ~ /cos a
1=,Jd 2 -a 2 =a~1-2sen2~ =d,Jcosa . 2 2 '
En conclusion, despues de las correspondientes simplifica-
Luego
ciones, hallamos que la superficie total del prisma es igual a
a2 la
S=S +-+2-=
sec 2 2 sen a + ,J2cos a
a r=:::-:::) Stotpris = 4(S - Ssec ) = 4S _ _ _ -=2'--_ __ _ _
= d-o(sen
- - a + sen-? -a +'\}2
r;:; sen -\j cos a a a c --
cos - +sen-+~2cos a
2 2 2 2 2
437. Ellado de la base de la pinimide es igual a a = 2r sen a
.' (por ellema conocido al teorema de los senos): La arista lateral
:~,
'. ' (Fig. 150) es S

a 1 a
1= - - - = 2r cos-. c
2 a 2
sen -
2

Fig. 149 Fig. 150


Deaquf
Por esta razon, la altura de la pinimide sera
d2 =_ ___ _ _ 2S_ _ _ _ _ _
sen a + 2sen 2 a + 2-fi sen a ,Jcos a
2 2
y, par consiguiente,
GEOMETRiAiEstereometria (Resoluciones y Soluciones) 279

y, por consiguiente, el volumen de la pinimide sera igual a Puesto que D'C11 DC, entonces

a 2 .fj 2 3 2 I., 2 a 2 -
D'C=DC OM =a sen a .
V = 3h - 4- = 3r sen a ~ jcos 2 -sen a.
ON sen 3a
438. Sea ABC D' la secci6n indicada de la piramide OABCD. Aplicando el teorema de los senos al triangulo P MN, hallamos
Tracemos el plano auxiliar OPN a traves del vertice 0 de la que
piramidey los puntos medios de sus aristasAB y CD (fig. lSI).
PM sen 2a
Es facil ver que el plano OPN es perpendicular aAB y CD, y
que los segmentos OP y ON son iguales. PN sen (rc - 3a)'

o dedonde

PM=asen 2a.
sen 3a

Ahora, obtenemos el area buscada de la secci6n ABC D':

s = ~(AB + D'C)PM = ~(a+a sen 2a)


2 2 sen 3a)
asen 2a
sen 3a
=

A~--a'---~D
o sen 2 2a cos a
=a" - - - --
Fig. lSI Fig. 152 sen 2 3a

Aplicando el teorema de los senos al triangulo OPM, hallamos: 439. Empleando las denotaciones de la fig. 152, examinemos t
parte del desvan OSBMN. Esta parte consta de dos pinimides.
OM sen a La primera piramide tiene como base SBM y su vert ice es 0; su
volumenes
OP sen 3a
280 PROBLEMAS DE MATEMATICAS ELEMENTALES

La segunda pinimide tiene corrio base BMN y su vertice es Sea a ellado de la ba...e de la pinimide. Entonces
0; su volumen sera

SK = a.fj .
6 cos a

II( a.fj )1 + (a) a ~3'( + )


Asi pues, el volumen del desvan es igual a 12 2
SB =, - = 1 3 cos 2 a .
a 2h ~ 6 cos a 2 6 cos a
V = 8 (V; + V2 ) = - .
2
Del triangulo isosceles ADB hallamos facilmente su altura
440. Sean BM y CM las perpendiculares trazadas desde BM:
los vertices B y C de la base (fig. 153) a la arista lateral SA. BM= a
EI angulo BMC formado por estas perpendiculares es el ~1+3 cos 2 a'
buscado. Designemoslo por 13. Es obvio, que
De este modo, en virtud de (1)
s s

13 ~1 + 3 cos 2 a
sen-=---'--------
A
2 2
c
y, por consiguiente,

B B flo
f-' = 2 arc sen
~I +3 cos 2 a .
Fig. 153 Fig. 154 2
441. Tracemos un plano por la arista SA y el punto N del pie
de la perpendicular AN al segmento BC (fig. 154). Sea NM
13
sen- = - .-.
BK
(1) la altura del triangulo ASN. EI segmento NM, por ser
2 BM perpendicular a AS y BC, evidentemente, es igual a d.
Designemos por a ellado de la base de la piramide . Entonces
GEOMETRiAIEstereometria (Resoluciones y Soluciones) 281

SA= a Pero,
a
2sen-
2
b <p a <p
y la altura de la pinimide es igual a OF=- tg- OE=- tg-
2 2' 2 2'

SO=..JSA2-A02 = a 19-12sen2a. y obtenemos la eeuaeion


6 sen a ~ 2 a· tg a = b· tg 2a, resolviendo
2 la eual, hallaremos:
Puesto que AN· SO = AS· d, entonees
~a-2b *)
tga= - -
6d a
a= .
oJ3J9 -12 sen 2 ~ Luego, obtenemos:

Como resultado, tenemos a <p ~a -2b


SO=OE.t g a="2 tg "2 - a-;

S
base
=a+b(OE+OF)=(a+~)\
2 2
~
g2'

y, por fin, el volumen de la piramide sera igual a


442. Sea AD = a, Be = b (fig. 155). Traeemos el segmento
EF que une los puntos medios de las bases del trapeeio. Es
evidente, que el angulo diedro adyaeente aAD es menor que
el cingulo adyaeente a Be. Sea LSEO = a; entonees
LSFO=2a.
Tenemos:
*) Este resultado demuestra que siendo a ~ 2b el problema no tiene
SO = OF . tg 2a = OE . tg a. sentido.
282 PROBLEMAS DE MATEMATICAS ELEMENTALES

443. Sea SL 1. AB, SK 1. AC Y SM perpendicular al plano


P (fig. 156). Segun la condici6n del problema SA = 25 cm, EF =a'A,
SL = 7 cm y SK = 20 cm. Por el teorema de Pitagoras
hallamos facilmente que AK = 15 cm y AL = 24 cm. y del tlMKO obtenemos que
Prolonguemos el segmento KM hasta su intersecci6n con
ellado AB en el punto Q. Es facil ver que el L AQK = 30° OM= KM =~ ,fi
por consiguiente, AQ = 30 cm. De aqui que sea LQ = 6 cm y cos p 2 cos P
EI area de la secci6n es igual a

Del triangulo rectangulo SML hallamos que


I 1 jj'A jj
- (AD+EF)OM=-(2a+'Aa)- - - a= - - 'A('A+2)a2 .
2 2 2 cosp 4cosp

444. Supongamos que sea S el vertice de la piramide, SO su EI area de la base, como el area de un hexagono regular
altura, BN = NC (fig. 157). Designemos por a el lado de la con ellado a, es igual a 6· u 24-fi , y la relaci6n buscada de las
base de la piramide. Hagamos provisionalmente ~';J = 'A. areas es igual a
Entonces, de la semejanza de los triangulos hallamos facill 1
mente que - -'A('A+2). (2)
s
6cos P
Por consiguiente, el problema se reduce a la determinaci6n
s de 'A . Para este fin hagamos L SNO = <po Entonces, por el
B teorema de los sen os, del tl SOM obtenemos:

sen (2: - pi
SM = SO 2 ) = SO cos p .
60'
A~'-""------'K':---- c sen (p + <p) sen (p + <p )
A

Fig. 156 Fig. 157


Puesto que SO = SN . sen <p, entonces
GEOMETRiAIEstereometr[a (Resoluciones y Soluciones) 283

SA a la respectiva cara (fig. 158). Designemos los angulos


A = SM = cos [3 sen <p = 1
(3) buscados por [31 y Y1
SN sen ([3 + <p) 1+ tg [3 cotg <p .

Queda determinar cotg <p . Para ello, observemos que


Supongamos, a continuacion, que LACB = a' y
a a L A CD = a". Haciendo CA = a, de los tricingulos rectangulos
SN =-cotg - oN=a4 CBA, SBA y SBC haHamos:
2 2' 2 '
SB a sen a ,
-J a~ cotg 2 --3
SO= SN 2 -ON 2 =- a tg Y1 = - = , = sec y tg a .
2 2 CB a cos y cos a

y, por cons-iguiente, Analagamente obtenemos:

R-
Colocando este valor en la formula (3), obtendremos:

a
cotg 2 - -3
A= 2 .
~ cotg 2 ~ - 3 + -fj tg [3 EI problema se ha reducido, por consiguiente, a la determi-
nacion de tg a' y tg aU. Tenemos que a'+a"= a. Calcu-
445. Desde cierto punto S que no coincida con el vertice C y lando por diferentes metodos el segmento SA, haHamos:
que se encuentre en la arista de angulo triedro, que no es
lade del angulo plano a, bajemos las perpendiculares SB y SA = a sen a' tg y
SD a los lados del cingulo plano indicado y la perpendicular SA = a sen aU tg [3.
284 PROBLEMAS DE MATEMATICAS ELEMENTALES

De aqui sen a'=sen a" tg ~ cotg y y, por consiguiente,

sen a'= sen (a -a') tg ~ =


tg Y
= (sen a cos a'-cos a sen a')tg ~ cotg y.

Como resultado, dividiendo ambos miembros de la ultima E


igualdad entre cos a', obtenemos:
F A
, sen a tg 13 cotg y Fig. 159 Fig. 160
tg a = - --..:::....:.---=""""'----
1+ cos a tg 13 cotg Y
S!J.PAB 1
Cambiando de lugar a 13 y y, hallamos: StlQAB k

" sen a tg y cotg 13 Dado que las areas de los triangulos son entre Sl como sus
tg a = - ---=-----------"-'---
1 + cos a tg y cotg 13 alturas, bajadas a la base comun AB, para el coseno del angulo
diedro de la base tenemos:
De este modo, en resumen obtenemos:
PR 1
cosq>=-= - .
sen a tg 13 cotg Y QR k
tg y, =
1 + cos a tg 13 cotg Y
De aqui se desprende que la apotema de la base de la
sen a tg y cotg 13
tg
A'
t--' ,= 1+ cos a tg y cotg 13
. piramide es igual a

446. Puesto que la sum a de los angulos internos del poligono


regular es igual ann, la cantidad de lados del polfgono sera
n + 2. Sea PQ la altura de la piramide (fig. 159). Examinemos A continuaci6n, hallamos el lado de la base
una cara lateral cualquiera de la piramide, por ejemplo el 2h n
AQAB Y su proyecci6n sobre la base, es decir, el L1 PAB . a= tg--.
De la condici6n del problema se deduce:
-Jk2 -1 n+2
GEOMETRIAiEstereometria (Resoluciones y Soluciones) 285

Puesto que el area de la base es


S = ~(2a+~J.!2
2 2 2
= 8 = 2(~ahJ
5ah
4 2
1 trap
S=-(n+2)ad,
2 s
entonces, el volumen de la piramide sera

V = ~Sh = ~ (n + 2)h 3 tg_1t _.


3 3 e-l n+2
447. El cuerpo obtenido es un octaedro cuyos vertices se
encuentran en los centros de simetria de las caras del cubo
(fig. 160). EI volumen del octaedro es igual al doble del
volumen de la piramide cuadrangular regular EABCD de
t
altura ~ y el area de la base ABCD de la c~al es igual a a 2 .
Por consiguiente, el volumen buscado es 19ual a
Fig. 161
1 a 1 ~ a3
2· - ·-·-a- = -
322 6 y, por consiguiente, la relaci6n buscada es igual a %

448. Es facil ver que en la secci6n se obtendra un trapecio 449. Sea A 1BCP el tetraedro dado y ABCDA 1B 1CP1 el
is6sceles ABCD (vease la fig. 161). Sea P el punto medio del paralelepipedo obtenido de la construcci6n indicada. Es facil
lado EF de la base de la piramide. Examinemos el ~SPR en comprender que las aristas del tetraedro son las diagonales
el que entra la altura SO de la piramide. El segmento KO, de las caras laterales del paralelepipedo (fig. 162). EI tetraedro
evidentemente, es la altura del trapecio ABCD. Dado que puede ser obtenido eliminando del paralelepipedo cuatro
KOIISR, entonces KO = ~ h, donde h es la apotema de la piramides equidimensionales: ABDA 1, BDCC 1, A 1B,C 1B Y
piramide. Es obvio tambien, que AB = 2a, donde a es la Ap,Cp. Puesto que el volumen de cada piramide es igual a
longitud del lade de la base de la piramide y que 16 del volumen del paralelepipedo, la relaci6n entre el
DC = *EF = tao De aqui que sea volumen Vpar del paralelepipedo y el volumen Vtetr del
tetraedro sera
286 PROBLEMAS DE MATEMATlCAS ELEMENTALES

y
Vpar Vpar
4 =3. Jj
~etr -6 vpar
vpar
QS 3 a Jj
cos f3 = - = - - = -.
450. Es fkil ver que los vertices extemos de los tetraedros SA a 3
se encuentran en los vertices de cierto cuadrado. Para
deterrninar la longitud de su lado, tracemos por el vertice S Por eso SA y OB son paralelos y, por consiguiente,
de la pinimide y por el vertice extemo A de uno de los
tetraedros un plano perpendicular a la base de la pinimide OB =SA =a.
cuadrangular (fig. 163). Este plano pasara por el pie 0 de la As! pues, la distancia buscada es igual a a12.
altura de la piramide, por el pie Q de la altura del tetraedro y
por el punto medio M de la arista KL. Bajando la 451. Supongamos que el plano secante ha sido trazado por
perpendicular AB al plano de ia base de la pir?mide, cierto punto de la diagonal HP del cuba dado (fig. 164).
examinemos el cuadrilatero SOBA. Su lado OB es la mitad Examinemos al principio las secciones que cortan a la diagonal
de la diagonal del cuadrado mencionado y debe ser en los puntos del segmento OP. Separemos la secci6n QRS
deterrninado. Es facil revelar que SOBA es un rectangulo. En que pasa por tres vertices del cubo, ella, evidentemente,
efecto, haciendo L. OMS = a, y L. ASM = {3, hallamos: pertenece al conjunto que se examina. Es un triangulo equila-
teto cuyo lado es a12. Es facil
calcular que la distancia desde
esta secci6n hasta el centro del
.
cu b 0 es 19ua 6- ' E s
I a -a.J3
evidente, que si x ~ a.J3 en la
A K secci6n se obtienen tr~angulos
Fig. 162 Fig. 163 equilciteros. Puesto que la rela-
1 ci6n entre los lados de los trian- L
OM 2a Jj gulos en cuesti6n es igual a la
cos a = -OS = Jj = 3 relacion entre sus distancias
- a Fig. 164
2 hast~ el punto P, entonces
GEOMETRlAiEstereometria (Resoluciones y Soluciones) 287

MN OP-x
~ .

QR OP- a,}3
6 De donde
De aquf, tomando en consideracion que
AB = M)N) - 2BN! =!"!..Ji + x-J6. (2)
~ a-J3 , 2
QR=a'\j2 y OP= - El lado Be se podrfa hallar analogamente. No es diffcil,
2
sin embargo, comprender que BC = BN, y, por consiguiente
hallamos:
a r- /,
BC = ~-.J2 - x'\j6. (3)
MN = 'iJi a- x-J6. (1) 2
2 Sefialemos que en la seccion con
S I. ~6~
a.[j > X ~ 0 , entonces en I · , se 0 b hene
a secclOn . el el plano 11: que pasa por el punto 0 se
;:G_~E;,,<'_~S
hexagono ABCDEF. obtiene un hexagono regular (vease
Los lados AB, FE YCD del hexagono son respectivamente las formulas (2) y (3) para x = 0). Los
paralelos a los lados QR, QS Y RS del triangulo equilatero vertices de este hexagono se
QRS. Por esta razon, en su prolongacion, intersectandose, encuentran en los puntos medios de
forman angulos de 60°. Teniendo en cuenta que AFIICD, las aristas del cuba (fig. 165). Es facil
etc., lIegamos a la conclusion de que todos los angulos del ver que si a una de las dos partes en
hexagono son iguales a 120°. Es facil ver tambien, que las que el plano 11: divide al c!.lbo se L

AB = CD = EFy BC= DE=AF(se debe tener en cuenta que la hace girar 60° en tomo a la diagonal Fig. 165
los lados del hexagono cortan en las caras triangulos isosceles). OP, entonces el hexagono coincide
Con el fin de hallar las longitudes de los lados del hexa- consigo mismo y obtendremos dos poligonos dispuestos
gOIlO, prolonguemos el lado AB del hexagono hasta su simetricamente con respecto al plano 11:. Por consiguiente,
interseccion con las prolongaciones de las aristas PQy PR la secci6n que corta a la diagonal en los puntos del segmento
en los puntos M, y N,. La longitud del segmento M,N, puede HO a la distancia x del punto 0; se obtiene de la correspon-
ser calculada por la fonnula (1). Conociendo MN hallamos diente secci6n del conjunto de pianos secantes ya examinado
J l
el segmento haciendolo girar a 60°.
288 PROBLEMAS DE MATEMATlCAS ELEMENTALES

452. En la proyeccion se obtendni un hexagono regular cuyo


lado seni igual a a'f'. Para convencerse de esto es comodo
representarse el resultado de la proyeccion de todas las s
G*,-+-+---"'*---+---!~ R
secciones posibles del cubo, examinadas en el problema 451
(vease la fig. 164). Todas las secciones indicadas se proyectan
sin modificar sus dimensiones y obtendremos la figura A =---_ _ _----'>1
mostrada en la fig. 166.
Valiendonos de que el lado del triangulo RQS es igual a Fig. 166 Fig. 167
a-fi, del triangulo GOS haHamos:
Determinemos, a continuacion, las longitudes de los seg-
GS.J3 = a-fi mentos QO y QS. Dado que
2 2'
QO GO
de donde GS = af . Puesto que, a continuacion, el lado del HK GK'
hexagono regular AIBICPIEIFI (vease la fig. 164) es igual a entonces, teniendo en cuenta (1), obtenemos que
af, entonces, la relacion buscada resultara igual a h a h
QO = - . - . -4 = -.
(a~)' (af)' 2 2 3a 3
4
3 de don de
QS=~h
453. Sea AEFD el trapecio isosceles que se obtiene en la 3
seccion y sean G y H los puntos medios de sus bases (vease
y
la fig. 167). Bajemos desde el punto H la perpendicular HK
a la base de la piramide. Puesto que H es el punto medio de GQ=, (2aJ2 + (hJ2
3
SN, entonces
Bajemos desde el punto S la perpendicular SM a GH.
HK='2 KN=~ GK = 3a. (I) Entonces, de la semejanza de los triangulos SMQ y GOQ
2' 4' 4
tenemos que
GEOMETRlAiEstereometria (Resoluciones y Soluciones) 289

SM GO
v = a 3 -fi.
QS GQ 80
y, por consiguiente, la distancia buscada es igual a 455. Supongamos que sea AMKN el cuadrilatero obtenido
en la secci6n y Q el punto de intersecci6n de sus diagonales
SM = QS . GO = 2ah . (vease la fig. 169). Al examinar el I1SAC es facil ver que Q
GQ -J9a 2 + 4h2 se encuentra en la intersecci6n de las medianas de este
454. EI cuerpo que se exam ina esta compuesto por dos triangulo. Por eso,
piramides con base comun KMN (fig. 168). La altura OR de s s
la piramide interior es facil de hallar, bajando desde el punto A
P (punto medio dellado KN) la perpendicular PD a la base
de la piramide. El punto D dividira al segmento QL por la
mitad. Valiendonos de este hecho, del I1APD obtenemos:
PD DA 5
RQ QA 4 B"'--------!----'" B "'--------!'----'"

Fig. 168 Fig. 169


De aquf

RQ=i5 pD MN
ED
SQ
=
SO 3
2

y, por consiguiente, y, por consiguiente,

OR=lpD=l.laJt = a;;.
Aquf, hemos aprovechado que la altura de un tetraedro Luego, del triangulo rectangulo SAC hallamos que
regular es igual a aJr
El volumen buscado es igual a
1 1 I 2 2
AK= - SC=-- I/ q +a .
2 2
290 PROBLEMAS DE MATEMATICAS ELEMENTALES

Puesto que AK 1- MN, entonces Observacion. Este problema puede


ser facilmente resuelto por otro proce-
dimiento, Sl se toma en consideracion
la formula
456. Sean NQNlQ l y LMLlMl las secciones paralelas del Sproyec = S cos <p, (1 )
prisma (fig. 170), a la longitud de la diagonal AC de la base
y H la longitud del segmento KK l • Entonces, el area de la donde S es el area de cierto poligono
primera seccion sera dispuesto en el plano P, Sproyec es el area
de la proyeccion de este poligono sobre
S= ~( a+~) =lHa. Fig. 170
el plano Q, y <p es el angulo entre los
pianos Py Q.
EI area de la segunda seccion sera De acuerdo con la formula (I), las areas de las secciones
paralelas examinadas en el problema son entre Sl como las
areas de sus proyecciones. ASI pues, nuestro problema se
reduce a hallar las areas de dos figuras: ~M;CMLA y
Pero, N;Q;CQNA (fig. 171) (las letras con rasgos significan las
proyecciones de los puntos correspondientes sobre la base
LM=~ ~Ml
3
= - a, PT=~H I
I1 T =-H, del prisma).
4' 4 4 ' 4
457. Examinemos la piramide KAEF, que es uno de los
10 que se ve facilmente de la semejanza de los triangulos poliedros (V ease la fig. 172). Consideramos que
correspondientes. En virtud de esto, obtenemos:
AE AF
S'=~aH EB FC
=2
16
y, por consiguiente, Por eso,

AE AF 1
- = - =-
EB AC 3
GEOMETRiAlEstereometria (Resoluciones y Soluciones) 291

y, por consiguiente, Segun el teorema de las tres perpendiculares


OCI -.lAB, OAI -.l BC Y OBI -.l AC, en virtud de 10 cual
(1) los angulos LDCP, LDAp y LDBp son angulos
lineales de los respectivos angulos diedros y segun la
Supongamos, a continuacion, que KM y SN son las alturas
condicion del problema son iguales. De aqui se deduce la
de las pinimides KAEF y SABC. Es facil ver que
igualdad de los triangulos DOC l , DOA l Y DOB l • Para
KM AK 2 comodidad del ca1culo introduzcamos las siguientes
SN
= AS 3 denotaciones:
Por eso DO=H, DCI =DAI =DBI =h,

KM=~SN
3
Es evidente, que r es el radio D
y, por consiguiente, teniendo en cuenta (1), obtenemos que de la circunferencia inscrita en el
2 ~ ABC. EI volumen de la
VKAEF = 27 VSAHC"
piramide ABCD es
La relacion buscada es igual a i5. 1
V =)SoH. (1) c
s
K Del triangulo rectangulo DOC l A
obtendremos:
Fig. 173
c

B
As! pues, el problema se reduce a la determinacion de la
Fig. 171 Fig. 172 -t
apotema h y del radio r. De la formula S3 = ABh Y otras
458. Tomemos la cara de area So como base ABC de la analogas obtendremos las expresiones para los lados del
piramide dadaABCD. SeaDO la altura de la piramide y DA I , triangulo ABC:
DBI Y DC I las alturas de las caras laterales (fig. 173).
292 PROBLEMAS DE MATEMAnCAS ELEMENTALES

AB= 2S~ BC= 2St AC = 2S2 .


S
h ' h ' h So = pr = '/'
de don de
Por consiguiente, el semiperimetro sera
S
1 (AB+BC+AC ) =-+-+-=-.
S3 St S2 S
r = h---.Sl.
p=- S
2 h h h h
Colocando este valor de r en la formula (2), hallaremos;
Luego,
p-AB = S _ 2S3 = S-2S3
h h h
-BC = S-2St p-AC = S-2S2
Ph' h Colocando aqui el valor de h de la formula (3) e
introduciendo el resultado obtenido en la formula (I),
y por la formula de Heron obtendremos definitivamente:

S6 = p(p-AB)(P-BC)(p-AC) =
S S -2St S -2S2 S -S3 S(S -2St )(S -2S2 )(S -2S3)
=Yz. h . h ·- h - = h4 459. Cortemos al cuba por la mitad con ayuda de un plano
diagonal, perpendicular al eje de rotacion, y giremos 90° el
de donde poliedro obtenido. Como resultado obtendremos la
configuracion representada en la fig. 174.
VS(S -2St )(S -2S2 ) (S -2SJ
h= Fa La parte comun la componen el paraJelepipedo rectangular
ABCDD,A,B,C, y la piramide regular SABCD. La altura del
EI radio r de la circunferencia inscrita 10 hallaremos de la paralelepipedo la hallamos del triangulo BBJ:
formula que expresa el area So del triangulo ABC por medio
de este radio y el semiperimetro:
h = B] T = a-fi - ~
2 2
GEOM ETRi.A.iEstereometria (Resoluciones y Soluciones) 293

La altura de la piramide es s

H = a.fi - h = ~.
2 2
El area de la base comun del paralelepfpedo y la piramide
s igual ad.
De este modo, el volumen buscado de la parte comun sera
Fig. 174 Fig. 175

Por esta razon, el volumen del cono es

1 ~ 3 tg a cos~
o bien 1W '")
V= - nr-h= - · ~.
3 [3
V=a 3(-.J~2- 32) . 3 24
sen -
2

461. Sea a el angulo buscado, Ila generatriz del cilindro, II


la generatriz del cono y r el radio de la base del cono y del
460. Sea S el vertice del cono, SO = h la altura del cono, ASB
cilindro (fig. 176). Segun la condicion del problema
el triangulo que se obtiene en la seccion, C el punto medio
de la cuerda AB, y AO = r (fig. 175). Observando que
2nr(r + t) _ 7 r +I 7
LAOC=%, hallamos:
nr (r + II) -"4' r + 11 8

a /f~\
h = COtg a = ~tg a cotg~,
[3 a
CO=-cotg - r= - - - !/ I~\\'
2 2' 2 2 2sen~ . I r \
2
/ i ~\
,I: l R! \\
I: :
L_'_ j_~' _ \
~
R

Fig. 176 Fig. 177


294 PROBLEMAS DE MATEMAnCAS ELEMENTALES

Por consiguiente, Sin embargo, se ve facilmente que tg a = R~r < 1, por


I -t
eso, tg a = y, por consiguiente,
1+ -
__ r -2 o bien
1 + cotg a 7 1
a = arctg - .
1+~ 8' 1+ cosec a 8 2
r
y, por 10 tanto, 463. Sea Ila longitud de la generatriz, R el radio de la base
del cono, x la longitud de la arista del prisma y rei radio de
sen a + 8 cos a -7 = O. la circunferencia circunscrita a la base del prisma (fig. 178).
Examinemos el triangulo formado por la altura del cono, por
Resolviendo esta ecuaci6n hallaremos: la generatriz del cono, que pasa por uno de los vertices del
prisma, y la proyecci6n de esta generatriz sobre la base del
3 3 cono. Tenemos:
sen a = - , a = arcsen - .
5 5
I sen a R
462. Supongamos que sea a el angulo buscado, R el radio
de la base del cono y r el radio de la base del cilindro (fig. I sen a -x r
177). Tenemos:
Puesto que ,.
21tr2 + 21trR = 2(1 +~) ~ =~. (u

1tR2 R R 2 R
x
Pero RI/ = tg a y, por 10 tanto, j = 1 - tg a. Como
r = -- - Y R = I cos a, Fig. 178
1t
resultado obtenemos la siguiente ecuaci6n respecto de tg a: 2sen -
n
4tg 2 a-12tga+5=O. obtendremos que 1t
21 sen a sen -
Resolviendo esta ecuaci6n, haHamos: x= _ _ _ _ _:..o.
n
1t
2sen - +tg a
taa= -5 0
b len
' tga= -1 . 12
o 2' 2
Por consiguiente, la superficie total del prisma sera
GEOMETRiAiEstereometria (Resoluciones y Soluciones) 295

De aqu!
2
1t
M- M
S==-nx2cotg~+nx2 ==n (
1 21 sen a sen - 1 sen a == - y a - arcsen--.
2 n 1t
n J (1+-cotg~).
2 n
h h
2sen- + tg a 465. Sea R el radio de la esfera y a, bye los catetos y la
n hipotenusa respectivamente del triangulo ABC que se
464. Examinemos el trapecio isosceles ABICP obtenido encuentra en la base del prisma (fig. 180). Tenemos:
como resultado de la proyeccion del trapecio dado ABCD h a h
a== - - , b==_h_ , c == - - == - - - - -
sobre el plano perpendicular al eje del cilindro (fig. 179).
cos a sen a sen a cos a sen a
Puesto que el trapecio que se examina esta circunscrito a
una circunferencia, entonces Es evidente que el radio R es igual al radio de la
a+b circunferencia inscrita en el Li ABC. Por eso
AB] == AK + KB] == AM + BjN] ==--
2 2S;"ABC ab h
R==---
Del triangulo rectangulo APB I obtenemos: a+b+c a+b+c 1 + sen a + cos a

y, por consiguiente, el volumen del prisma sera


(-a+2-b)\I == (a- -2-b) + hsen a.
2 2 2 2

v == SI'.ABC2R == (
2h 3
).
. sen 2a 1 + sen a + cos a

466. EI volumen de la piramide es igual a la suma de los


volumenes de las piramides que se obtienen al unir el centro
de la esfera inscrita 0 con todos los vertices de la piramide.
C La altura de cada una de estas piramides es igual al radio r de

~
la esfera inscrita en cada piramide. Si S es el area de la base
de la piramide y SI es la superficie lateral, entonces, el
volumen de la piramide sera
A D B c A
1
Fig. 179 Fig. 180 V==-(S]+S)r. (1)
3
296 PROBLEMAS DE MATEMATICAS ELEMENTALES

Puesto que, por otro lado, 467. Designemos por r el radio de la esfera inscrita y por a la
longitud del segmento OE (fig. 181). Entonces,
V=!hS
3 '
r = a tg a,
entonces, obtenemos para r la formula
hS donde a es la mitad del angulo buscado (vease la fig. 181).
r= - - . (2) Por consiguiente el volumen de la esfera sera
S1 +S
De las condiciones del problema se desprende: D Vesf = ~4 + 1ta 3 tg 3 a.
.)

Puesto que DO = a tg 2 a, y
AB = 2~ a, entonces, el volu-
men de la piramide sera

1 fj ., c 3
B Vpir =-D~AJ5 = ---J 3 a tg2a.
3 4
Puesto que por la condicion
A
del problema
Sustituyendo estas expresiones en (2), hallamos: Fig. 181 Vpir 27~
Vesf 41t

2 1t.,

n,
~
4Ib2~£
a
na cotg- b- - - --
21t
sen -;;
I ., 1t
a 4b- - a 2 cosec-., --
De aquf

~ n
2
r= 4 [ -na cotg -1t + -na ~b--
., a 2J
-
1t~.,
2 ( a+tg- 4b- -a
2)"
J Teniendo en cuenta que a es lin angulo agudo, hallamos:
4 n 2 '\ 4 \ n
1t
a 1 = --
6
GEOMETRiAf'Estereometria (Resoluciones y Soluciones) 297

y entonces,

= arctg~
V 47t 3 a 1t s
a2 - esf = -tg - cotg a cotg -
Vpir n 2 n

468. Sea a ellado y b la apotema del polfgono de n lados que


se encuentra en la base de la pinimide, H la altura de la
pinimide. Entonces (fig. 182, a y b)
s
a
b = r cotg - ,
2
1t a 1t
a = 2b tg- = 2r cotg - tg - ;
n 2 n
el area de la base es D

2 a
S
base
= nab
2
2 1t
- = nr tg-cotg -
n 2
Luego, a) b)
a Fig. 182
H = b tg a = r tg a cotg - .
2
De aquf, el volumen de la piramide sera
.
Vplr = -1 nr 3cotg 3a 7t
-tg a tg - .
3 2 n

Puesto que el volumen de la esfera es


4 3 g) b)
Vesf = - 1tr ,
3 Fig. 183
298 PROBLEMAS DE MATEMATICAS ELEMENTALES

469. Sea a ellado de la base la pinimide, b la apotema de la


base, R el radio de la circunferencia circunscrita a la base, h
la altura de la pinimide, r el radio de la esfera inscrita en la
pinimide, y la altura de la cara lateral bajada desde el vert ice
1~h . nab
4
-1tr
3
n sen ~ ( 1+ 2 cos ~
2 1t
41tcos -
r
de la pinimide (fig. 183, a y b). Entonces, 3 n
s
1t
a = 2R sen - , b = R cos -1t ,
n n
ademas,

Fig. 184 Fig. 185

470. Supongamos que sea a ellado de la base de la piramide


De la ecuaci6n SABeD, h la altura de la piramide y r el radio de la esfera
r b circunscrita a la pinimide (fig. 184). Entonces,
h-r y =i
V 1tr 3
3
(vease la fig. 183, b) haHamos:

hb 1t ~ 1 + 2cos-
R cos- I
1t
I

r = __ = _ _ ___'n-'------'-~_ _ _ ___'n~
Si SE es el diametro de la esfera circunscrita, entonces,
y +b 1 + 2cos2: del triangulo rectangulo SBE se desprende:
n

Por consiguiente, la relaci6n buscada es igual a (a~r ~h(2r-h)


GEOMETRiAIEstereometrfa (Resoluciones y Soluciones) 299

Sin embargo, puesto que del triangulo FOIS tenemos que Como resultado obtenemos el volumen del prisma
-I = h cotg cx, , entonces, eliminando a a, hallamos:
cos 4 \jJ
1 V = ~R3 2
h- 2r _ 1 (6V)3 3 sen cx, cos \jJ sen 2 \jJ
- 2 cotg 2 cx, + 1 - 1 + 2cotg 2cx, 1t 2
s, 472. Tracemos un plano por los vertices
471. Empleando la igualdad de los angulos diedros, asi como SI y S2 de las piramides y el punto medio
en el problema 458, no es dificil demostrar que la A de uno de los lados de la base (fig. 186).
perpendicular bajada desde el vertice a la base se proyecta al EI radio de la semicircunferencia, inscrita
centro de simetrfa del rombo. Es facil tambien ver que el en el triangulo ASI S 2 de tal modo que su
centro de la esfera inscrita se encuentra en la perpendicular diametro se encuentra sobre SIS2'
mencionada. evidentemente, es igual al radio de ,a
Supongamos que sea a el lade del rombo, 2h la altura del esfera inscrita. Sea 0 el centro de la
rombo y H la altura de la piramide (fig. 185). Entonces, el semicircunferencia. Designemos por b la
area de la base es S = a 2sen cx" 0 puesto que a = h s;n a' s,
altura del triangulo AS1S 2 bajada allado
SIS2' Dado que h es la apotema del
Fig. 186 poligono regular de n lados, entonces,
4h2
S= - - . a 1t
sen cx, b = - cotg-.
2 n
Pero, h = R cotg ~ (vease la fig. 185, donde esta represen- EI radio de la esfera R 10 hallaremos calculando por dos
tad a la secci6n que pasa por la altura de la piramide y la metodos el area S del triangulo ASI S 2 • Por una parte,
altura del rombo).
b
Esta tambien claro, que S= - (H+h),
2
2 cos 2 \jJ por otra parte,
H=R+ ~ =R 2 R R R{ ! . , 2 ~2)
cos \jJ cos \jJ S=-S]A+-S]A= - t Jh-+b +'\fH-+b ,
222
300 PROBLEMAS DE MATEMAnCAS ELEMENTALES

Como resultado obtenemos la formula definitiva

!a(H + h)cotg2:
R=-r====~2==~--~~n~======
h 2 + -a cotg 2 -1t + ,JH-~ +-cotg
2 a2 2 1t
- a
4 n 4 n La solucion es posible si R:2 ~en.1!. .
n
473. Sean hI Y h2las alturas de las pinimides, rei radio de la 474. Es fkil demostrar que el punto medio del segmento
circunferencia circunscrita a la base (fig. 187). Entonces, que une los centros de las bases del prisma es el centro de las
a 1t esferas inscrita y circunscrita. EI radio de la circunferencia
- = r sen - . inscrita en la base es igual al radio de la esfera inscrita. Sea r
2 n
el radio de la esfera inscrita, R el radio de la esfera circunscrita.
Del triangulo rectangulo SIAS2 , cuyos vertices son al mismo Examinemos el triangulo rectangulo cuyos vertices son uno
tiempo los vertices de las piramides dadas y uno de los de los vertices de la base, el centro de la base y el centro de
vertices de la base, hallaremos que las esferas. Tenemos que R2 = r2 + rT, donde

rl = -- 1t
- '

n cos -
n
Pero,
De aquf,

I 1
R=r /1+ - -.
s, 21t
De aquf, \1 cos -
1- - - a- 2- - Fig. 187 ! n
7

hI = R+ IR- -- --,
1t 2
La relacion entre el volumen de la esfera circunscrita y el
'\ 4 sen - volumen de la esfera inscrita es
1/ n
GEOMETRlAIEstereometria (Resoluciones y Soluciones) 301

3
resoluci6n del problema 475) es igual a 3: 1. Por consiguiente,

R3
r
3
=(1+~1 2 7t
COS _
J2
la relaci6n buscada de los volumenes es igual a 3 3 = 27.

477. Supongamos que el problema es soluble. Tracemos el


n
plano A,B,C, (vease la fig. 189, a) de tal modo que haga
contacto con la esfera menor y que sea paralelo a la base
475. Los radios de las esferas circunscrita e inscrita son
ABC del tetraedro dado. El tetraedro SA,B,C, esta circunscrito
iguales a los segmentos en que el centro com un de las esferas
a la esfera de radio r. Es facil hallar que la altura de este
divide a la altura del tetraedro. Es facil revelar que la relaci6n
tetraedro SQ,= 4r (vease el problema 475).
de estos segrnentos es 3: I. En efecto, de la semejanza de los
triangulos BQO y BPK (fig. 188) tenemos: Admitamos que la longitud de la arista del tetraedro SABC
B
sea igual ax. Entonces, el segmento AQ = xf, y la altura
R BK
SQ = xf. Luego, (vease la fig. 189, b) tenemos que
r PK'
pero, QO = xf - 3r y del triangulo rectangulo AQO se desprende
BK = BK =3.
PK QK que (xfY +(xf -3rY =R2.
Fig. 188
s
Puesto que las superficies de las esferas son entre sl como
los cuadros de sus radios, la relaci6n buscada es igual a 9. x
3r

0
476. Los volumenes de los tetraedros regulares son entre sl R r
AI
Q,
como los cubos de los radios de las esferas inscritas en estos
tetraedros. Puesto que la esfera inscrita en el tetraedro mayor A Q
esta circunscrita al tetraedro menor, entonces, la relaci6n de a) b)

los radios mencionados de las esferas inscritas (vease la Fig. 189


302 PROBLEMAS DE MATEMAnCAS ELEMENTALES

Resolviendo esta ecuacion cuadnitica hallaremos que °


479. Sea el centro de la esfera, y AS, BS YCS las cuerdas
dadas. Es evidente, que el triangulo ABC es equilMero (fig.
191). Es facil tam bien ver que la perpendicular SO 1 al plano
ABC, al ser prolongada, pasa por el centro de la esfera 0,
En esta formula debe tomarse solamente la raiz con signa
puesto que el punto 01 es el centro de la circunferencia
mas, puesto que SA en todo caso es mayor que 3r, y 3r > r.J6. circunscrita al triangulo ABC.
Es evidente que el problema es posible con la condicion de
que sea R25.

478. Sea --1B,C,D1EJ'; el hexagono regular obtenido en la


seccion del cubo. EI problema se reduce a la determinacion
del radio de la esfera inscrita en la piramide hexagonal regular
S/\B1C,D,~I:; (vease la fig. 190). Ellado de la base de la

piramide es igual a af y su altura es igual a af. Fig. 190 Fig. 191


Valiendonos de que el radio de la esfera inscrita en la piramide
es igual al triple del volumen de la piramide dividido entre Designemos, despues de estas observaciones, la longitud
su superficie total (vease la formula (1) en la resolucion del buscada de las cuerdas por d. Del triangulo SAB hallamos que
problema 466), haHamos:
a
AB =2d sen-
r = a(3 - -J3). 2
4
y, por consiguiente,
Por consiguiente, la relacion buscada sera igual a
-J3 2 ~
qA = AB·- = -oJ3 d sen-.
a
332
Calculando por dos procedimientos distintos el area del
triangulo isosceles SOA, obtenemos:
GEOMETRiAIEstereometria (Resoluciones y Soluciones) 303

entonces,

de donde
Es tambien evidente, que

AD = ~ a 2 _ b 2 F2
2 '
480. EI radio de la esfera inscrita 10 hallaremos por la formula
(vease la formula (1) en la resolucion del problema 466) BD = ~ AB2 _ AD2 = J2 ~ a 2 + b2 .
2
3V
r=- y, por consiguiente,
S'
donde S es la superficie total de la pinimide y V su volumen. 1 I 4 I
Hallemos al principio el volumen de la pinimide. Observemos S!1.ABC = - '1 a - b
2
para ello, que los triangulos rectangulos BSC y BSA (fig. 192)
son iguales, puesto que son iguales sus hipotenusas y tienen Como resultado, despues de las simplificaciones corres-
un cateto comon. En virtud de esto, el triangulo rectangulo pondientes, obtenemos:
ASC es isosceles. Dado que
boJa 2 - b 2
I 2 ~
r= .
AS=CS= '1 a -b-. -Ja 2 +b 2 +2b+ oJa 2 _b 2

s 481. Designemos por r el radio de la esfera inscrita, y por R

~
:h el radio de la esfera circunscrita.
: B Examinemos al principio el triangulo SFE, uno de los lados
: ' ;~"
a' _--:- del cual, el lado SF, es la altura de la piramide, y el otro, el
_~ ___ ~::_-_----- a
SE, es la altura de la cara lateral (fig. 193, a). Sea a el centro
_----- _- - -0
A 0 C de la esfera inscrita. De los triangulos SFE y OFE (fig. 193,
Fig. 192 b) tenemos:
304 PROBLEMAS DE MATEMATlCAS ELEMENTALES

s s s De aqui,

o~
SF 2 +DF2
R=--- - (1)
2SF
Puesto que R = 3r, entonces, colocando aqui las
expresiones halladas anteriormente para SFy DF, obtenemos
F E una ecuaci6n respecto de <p :
b)
a)

c)
r 2cotg 2 ~ tg 2 <p + r 2cotg 2 ~ . 2
3r = 2 2
Fig. 193
2r cotg <p tg <p
2
despues de las simplificaciones correspondientes,
FE = r cotg~,
0,

SF = r cotg~tg <po
2
Hagamos, a continuaci6n, tg~ = z. Observando que
A continuaci6n, es evidente, que tg <p = 1~:2' obtenemos la ecuaci6n

DF=EF·J2 =rcotg~J2.
2
De aqui
Recurriendo a la fig. 193, c, donde esta representada la
secci6n trazada por el centro de la pinlmide y su arista lateral,
hallaremos facilmente que

Doi = O\F 2 + DF2 Puesto que z> 0, son posibles solamente dos respuestas:

o bien tg!h = ~3 +J2


2 7
GEOMETRiA/Estereometria (Resoluciones y Soluciones) 305

y 483. Sea R el radio de la base del cono, ex el angulo entre el

t <1'2=
g 2
!3--fi
"\ 7 .
eje del cono y la generatriz, y r el radio de la esfera inscrita.
En la seccion axial del cono tenemos un triangulo isosceles
ABC (fig. 195). EI radio de la circunferencia inscrita en este
482. En total se obtienen 6 biangulos (por el numero de triangulo es igual al radio r de la esfera inscrita en el cono.
aristas) Y.4 triangulos (fig. 194). Designemos por SI el area Sea 0 el centro de la circunferencia, L OCA = 13. Entonces,
de cada tnangulo y por S2 el area de cada biangulo. Tenemos: es evidente, que tg 13 = j. Pero, segun la condicion del
problema,
(1) 4rrr,2 = 4(~) 2 4
rrR~ R 3
Sea So la suma de las areas de uno de los triangulos y de
B
tres biangulos adyacentes a este triangulo. So es el area del
segmento esferico cortado por el plano de la cara del tetraedro.
Esta area es igual a 2rrRh, donde h es la altura del segmento.
Puesto que la altura del tetraedro se divide por el centro de la
esfera en la relacion de 3: 1, (v ease el problema 475), entonces
1 4
H = R+-R=-R
33'
A c
de donde hallamos que h = 2R - t R = 1R. Luego, Fig. 194 Fig. 195
4 2
SJ +3S2 = 2rrR·-rrR (2)
3
De aqu! que sea 11 = JJ y, por consiguiente, 13 = ~. Puesto
Resolviendo el sistema, compuesto de las ecuaciones (1)
que, ademas, ex + 213 = ~, entonces, ex = ~. Por consiguiente,
y (2), respecto a las incognitas SI y S2' obtenemos:
el angulo buscado sera
2 ~ 2 2
SJ = -rrR~, S2 = -rrR .
3 9
306 PROBLEMAS DE MATEMATICAS ELEMENTALES

n 1
20. =-. sen 0. 2 =-.
3 6
484. Sea r el radio de la semicircunferencia, R el radio de la s
base del cono, lla generatriz del cono y a. el angulo formado
por el eje del eo no y la generatriz.

Por la condicion del problema tenemos que


nR(I+R) 18
(1)
2nr2 5
Introduzcamos en esta igualdad el angulo a.. Con este fin Fig. 196 Fig. 197
examinemos el triangulo isosceles ABC (fig. 196), obtenido Por consiguiente, el angulo buscado del vertice del cono
en la seccion axial del cono. Del triangulo ABC haHamos que esiguala
R = I sen a., r = R cos a. = I sen a. . cos a. . 2 arcsen -5 0
b·len 1
2 arcsen -.
6 6
Sustituyendo estas expresiones en la parte izquierda de
(1), obtenemos: 485. Supongamos que sea h la altura del cono, rei radio de
la base, lla generatriz del cono y a. el angulo formado por la
1 + sen a.
--------,- = 18 generatriz y la altura (fig. 197). Segun la condie ion del
2 sen a. cos 2 a. 5
problema tenemos que nrl = lrnr2; de aqui, I = kr y, por con-
Puesto que cos 2 a. = 1- sen 20., entonces, simplificando el t.
siguiente, sen a. = Del triangulo rectangulo ABC obtene-
quebrado por I + sen a., tendremos que mos: ~
-V k'2 -1
r = 2R cos a. sen a. = 2R 2
36 sen 2 a.-36 sen 0.+5 = 0, k

de donde (k 1
h = 2R cos a. cos a. = 2R ~ ~ J
2
2
GEOMETRiAiEstereometria (Reso1uciones y Soluciones) 307

El volumen buseado del eono sera y, por eonsiguiente,

v = .!1tr2h = ~1tR3
3 3
[e _J)2
k3
q2
x=-(2-q).
4

486. Sea R el radio de la esfera, h la altura del eono y r el El problema es soluble, evidentemente, euando 0 < q < 2.
radio de la base del cono. La relaei6n del volumen del eono
al volumen de la esfera es 487. Sea Rei radio de la esfera, Sesf y Vesf la superfieie y el vo-
lumen de la esfera, Scono y Vcono la superfieie total y el volumen
r 2h q(r)2 del eono, h la altura del eono y r el radio de la base del eono
X= 4R3 ="4 R
(fig. 199). Entonees
Del triangulo SEA (fig. 198) tenemos que r2 = h(2R - h). nR 3
Vesf _ _3_ _
.i 4R 3
De aqui
Vcono -1 nr 2h r 2h
3
Sesf _ 4nR2 _ 4R2
Sco no - nr (/ + r) - r (/ + r r
Observemos, sin embargo que,
/ h-R h
- = - -=--1
r R R
y, por 10 tanto,
/ +r h
=
r R
obtenemos:
Fig. 198 Fig. 199
Vesf _ Sesf

Vcono Scono n
308 PROBLEMAS DE MATEMATICAS ELEMENTALES

Observacion: Se puede obtener el mismo resultado por Es facil ver, que AM==MF y que BD==FD, puesto que 0 es
una via mas corta, valiendose de la siguiente formula: el centro de la circunferencia inscrita en el trapecio; por 10
tanto,
(1) /==r+lj. (2)
donde Scona es la superficie total del cono, y R es el radio de Valiendonos de esta igualdad y de la igualdad (1)
la esfera inscrita en este cono. La formula (1) se obtiene obtendremos:
facilmente de la fonnula correspondiente para la piramide
(vease la resolucion del problema 466) por el metoda del (3)
paso al Ifmite. En efecto, puesto que es evidente que

1
Vesf =="""jSesfR, (2)

ento[,.;es, dividiendo (2) entre (1), obtendremos que

Vesf == S esf == ~
Vcono Sco no n
488. Sea S la superficie total del cono, SI la superficie de la Fig. 200
esfera, r 1 y r los radios de las bases superior e inferior del
cono y / la longitud de su generatriz. Sea, luego, CMDL el Del triangulo MED se desprende:
trapecio obtenido en la seccion axial del cono; 0 el centro de
/2 == (r - r1)2 + 4R2. (4)
la esfera inscrita y AB..l LD Y OF .1 MD (fig. 200).
Tenemos, Eliminando / de las igualdades (2) y (4), hallaremos:

nl(r + lj) + nr~ + nr2 (5)


== m. (I)
4nR2 Con ayuda de esta igualdad, eliminando I de (2) y (3),
obtendremos:
GEOMETRiAIEstereometrfa (Resoluciones y Soluciones) 309
------- ---------------------------- -
(6)
R r
R=~.
Resolviendo el sistema (5), (6), hallaremos: h d+r' d+r
As! pues, en el primer caso, el volumen del co no es
r = R (-J2m + 1 + .,J2m - 3);

'i
2

= R (.,J2m + 1-.,J2m -3).


2
'ITr2 (r +~r2 + (d +r)2
3(d +rr
r
As! pues, si m < ~ el problema no tiene soluci6n: cuando
m = ~ eI cono truncado se transforma en un cilindro. En el segundo caso el problema se resuelve analogamente.
EI volumen del cono resulta igual a
489. Son posibles dos casos: I) el vertice del cono y la esfera
se encuentran a distintos lados del plano tangente; 2) el vertice
del cono y la esfera se encuentran a un mismo lado del plano
tangente.
Examinemos el primer caso. Tracemos un plano por el eje B
del cono y la generatriz del cono Be, de la que se habla en
las condiciones del problema, (fig. 201). Este plano dara en
la secci6n con el como el triangulo ABC y, en la secci6n con
la esfera, una circunferencia con el centro 0; el plano
perpendicular a BC sera intersectado por la recta ME (M es
el punto de tangencia). Tracemos BD 1- AC y OF 1- Be.
Sea BD=h, OD=OF=r, CD=R. Es evidente, que OMEF es
un cuadrado y, por 10 tanto, A ~-------'''''---!F:::"''''-------'C
Fig. 201 Fig. 202

490. Examinemos la seCCIOn axial ABC del cono.


Luego, Supongamos que sea BF la altura del triangulo ABC, Ny M
310 PROBLEMAS DE MATEMAnCAS ELEMENTALES

los puntos de tangencia de la circunferencia, inscrita en el 2xalj + (2a - x )2ar = S. (1)


triangulo ABC, con los lados AB y BC, 0 el centro de la
circunferencia, E el pun to de intersecci6n del arco menor Del triangulo OKS tenemos que
MN con el segmento BF, y D el punto de intersecci6n de los
segmentos MNy BF(fig. 202). Hagamos DM= r, DE = Hy
BD = h. EI volumen buscado seni o bien
I ?
V=-nr-h- -1nH-
? (3R-H ) . (2)
3 3
Pero, Anaiogamente, del triangulo 0IKS tenemos que
cos 2 a
a a a 2
h = r cotg- = R cos-cotg - = R-----''""- r]2 = R2 + {1j -x )2
2 2 2
sen -
a
2 K
y
a
H =R-Rsen-·
2'
por consiguiente,
Fig. 203

o bien

(3)

De (2) y (3) hallamos:


491. Designemos por r y r 1 los radios de las esferas y examinemos
la secci6n de las esferas por un plano que pasa por sus centros 0 R2 +(2a-x)2 R2 + x 2
r= - ------;--'---------;-'--- r] = - --~.
(4)
y 01; sea AA 1=2a, KS=R y AS=x (fig. 203); entonces 2{2a- x) 2x
A]S = 2a - x. La superficie total de la lente es igual a
GEOMETRiAiEstereometria (Resoluciones y Soluciones) 311

Colocando estas expresiones para r y r I en la igualdad (I), 492. Sean VI y V2 respectivamente los volumenes de los
obtendremos la ecuaci6n segmentos esfericos menor y mayor, en los que el plano, que
pasa por la linea de tangencia de la esfera con el cono, divide
a la esfera. Sea, a continuaci6n, R el radio de la esfera, h la
altura del segmento menor, H la altura del cono, y r el radio
o bien
de su base (fig. 204). Entonces
X
2
- 2ax + R 2 + 2a 2 - -S = 0
2n ' VI = ~nh2 (3R - h), 4
V'J = -nR 3- -n h 2(3R - h ) .
de donde .J - 3 3

Is 'J EI problema se reduce a la determi-


x=a+ I--R-
V2n
'J
-a~. naci6n de la relaci6n 1. Designando
por a el lingulo fonnado por el eje
Colocando este valor de x en la f6nnula (4), despues de del conn y la generatriz, del ~ P KO
las simplificaciones correspondientes, obtenemos: haHamos:

R-h
- - =sena,
R

de donde Fig. 204

h
- = I-sen a.
R
r1 = Is 2 'J
A continuaci6n, segun la condici6n del problema
a + ~ 2n - R - a-
I 2
La elecci6n otro signo delante de la raiz cuadrada en (5) -nr H
se reduce al cambio de las designaciones r y r l • k ==3-:--_
.±nR3
3
312 PROBLEMAS DE MATEMAnCAS ELEMENTALES

Expresemos ahora r y Hen funci6n de R y a. Tenemos: El problema tiene dos soluciones, puesto que, siendo
k > 2, ambas raices de la ecuaci6n cuadnitica tienen senti do,
H=~+R =R.l+sena ,
sen a sen a 493. El radio r de cada una de las ocho esferas inscritas 10
1 +sen a hallaremos examinando el triangulo AOe en el plano que
r = H tg a = R . - - - -
cos a pasa por los centros de estas esferas y el centro 0 de la esfera
S (fig. 205, a)). Tenemos:
Por consiguiente,
AB r 1t
-=--=sen-.
AO R-r 8
k=!. (1+sena)3 =!. (1+sena)2
4 sen a (I - sen 2 a) 4 sen a (1 - sen a r De aqui
sen-
1t

Sustituyendo aqui sen a = 1 - i obtenemos una ecuaci6n r = R _ -----'8


"'-----
sen 2: +!
respecto a ~ = Z : 8
k-! (2-zY Trazando una secci6n que pase por el centro 0 de la esfera
- 4 (I-z)z S, el centro O[ de la esfera S[ y los centros de las dos esferas
0, despues de las simplificaciones correspondientes, opuestas de radio r (fig. 205, b),

z2(4k + 1)- 4(k + l)z + 4 = O.


Resolviendo esta ecuaci6n, obtenemos:

2(k + 1)±2~k(k -2)


= --"-----'-----'-------'-------'--
Z
1,2 4k+! ' (1)

Definitivamente hallamos: a) b)

V; zi, (3 -
2 zl, J Fig. 205

V2 = 4- Z 7,2 ~-z~~l
del triangulo rectangulo A001' obtendremos:
GEOMETRiAiEstereometria (Resoluciones y Soluciones) 313

obtenemos la ecuaci6n

o bien R-x = x0}3,


de donde

De aqui
R-r 495. Supongamos que sea r el radio de la base de cada uno
p=R·--,
R+r de los dos con os inscritos. Su parte com un se compone de
dos conos truncados iguales. Designemos por r l Yr2 los radios
o bien
R de las bases superior e inferior respectivamente del conn
p=R·---- truncado y por H su altura. La relaci6n buscada de los
1[
2 sen- + 1 volumenes es
8

494. Puesto que las esferas inscritas son iguales entre sl, sus
centros equidistan del centro 0 de la esfera S. Por A
consiguiente, el centro de simetria del cuba indicado en las
condiciones del problema coincide con el centro 0 de la esfera
S (fig. 206). Sea x el radio buscado de las esferas. Es facil
ver, que entonces la arista del cuba sera AB=2x, y la mitad de
la diagonal del cuba

AO =CO-CA =R-x.
Fig. 206 Fig. 207
Puesto que, por otro lado,
De la semejanza de los triangulos AQZ, AOS y APC (fig.
AO =!. 2x0}3 207) tenemos:
2 '
!l= R-H r2 R
R y r h
314 PROBLEMAS DE MATEMAT]CAS ELEMENTALES

De estas dos ecuaciones haHamos.


Puesto que, ademas, H=h-R y r=-JR 2 -H 2 =

= -J2Rh - h 2 , las dos igualdades anteriores permiten expresar


r] y r 2 en funci6n de R y h:
y, por consiguiente,
2R-h
fj = r2
R
Ya que de la condici6n del problema i = k, entonces
De las ecuaciones (1) y (2) obtenemos:

Por esta raz6n, el area buscada es

496. Supongamos que los radios de las secciones circulares


con las areas S] y S2 sean iguales a RI y R 2 , Y que las distancias
desde el centro de la esfera hasta dichas secciones sean
respectivamente iguales a II y 12 (/\ < 12)' Designemos por R 497. Designemos por r el radio buscado de la base del cono.
el radio de la esfera, por r el radio de la secci6n buscada y Examinemos la figura obtenida en la secci6n trazada por el
por I la distancia desde esta secci6n hasta el centro de la centro de una de las esferas y el eje del cono (fig. 209).
esfera. Entonces, (fig. 208), Observemos que la distancia entre los centros de dos
circunferencias en contacto es igual a 2R. Valiendonos del
(1) hecho, facil de demostrar, de que el centro de la base del
y cono A equidista de los tres puntos de tangencia de las esferas
con el plano P, hallamos:
GEOMETRiAiEstereometria (Resoluciones y Soluciones) 315

AD= 2J3 R.
De aqui , siendo q = 2, obtenemos que x = 1, por
3
consiguiente, r = 1 R. Si q:;t:. 2, entonces
x _2J3 (q-I)=t ,J9q 2- 18q+12
1.2 - 3(q-2)

Dado que 0 < x < 2'[3, entonces, en la formula indicada

Fig. 208 Fig. 209 debe tomarse el signo menos. Siendo q > 2, la segunda raiz,

Es facil ver que el L. SEA = LC01D = 213 y, por consi-


como es facil demostrar, es mayor que 2'[3, Y responde al

guiente, cono que tiene contacto exterior con las esferas; cuando
q < 2, la segunda raiz es negativa.
1t
213= - -a.
2 498. Los centros de las cuatro primeras esferas se encuentran
Tomando las tangentes de los angulos que figuran en ambas en los vertices de un tetraedro regular, puesto que la distancia
partes de esta igualdad, .obtenemos: entre los centros de dos esferas cualesquiera en contacto es
igual a 2R. No es dificil demostrar que los centros de la quinta
2 tg 13
- -
y sexta esferas coinciden con el centro de gravedad del
(1) tetraedro (fig. 210). Sea r el radio de la quinta esfera (la
1 - tg 2 13 tg a
mayor), y p el radio de la sexta esfera. Es evidente, que
De la fig . 209 esta claro que tg~=hf~ R-r}R y que
tg a =r :qR.
Si, ahora, hacemos *
= x, la igualdad (1) nos dara la
siguiente ecuacion respecto a x:
r =p+2R.

Valiendonos de que la distancia desde el centro de gravedad


(1)

hasta el vertice del tetraedro que se examina es igual a Jf R,


3(q - 2 )x 2 - 4-J3(q -I)x + q = O. obtenemos: -
316 PROBLEMAS DE MATEMATICAS ELEMENTALES

el plano, 0 el centro de la cuarta esfera, el radio r de la eual


p+R=-R.
-J6 (2) haee falta hallar (fig. 211). Uniendo los centros de todas las
2 esferas obtendremos, evidentemente, la pinimide triangular
De aqu! regular OABC, en la cual AB-BC=AC=2R,
AO=BO=CO=R+r, y OQ=R-r. EI segmentoAQesel
radio de la circunfereneia eircunserita at L1 ABC, por 10 tanto,

y de la formula (1) AQ _ AB _ 2R
- fj- fj'
Del triangulo AQO, por el teorema de Pitagoras, hallamos:

B
2R\2 (
( fjj )2 ( )2
+ R-r = R+r .
R
Resolviendo esta ecuaeion, obtenemos que r = -::;- .
.J

500. Sean A, B, C y D los centros de las esferas grandes.


""-- - ---' c, Examinemos la proyeccion de todas las esferas al plano A,
B, C y D (fig. 212). Dado que los eentros de las esferas
Fig. 210 Fig.211 pequefias equidistan de los centros de las respectivas esferas
As! pues, la relacioll buscada de los volumenes es grandes, se proyectaran a los centros de gravedad 0 1 y O2 de
los triangulos equilMeros ABC y BCD. Puesto que, ademas,
los radios de las esferas pequefias, segun la condicion del
problema, son iguates, el segmento que une sus centros es
paralelo al plano que se examina y se divide por el punto de
tangencia de las esferas en dos mitades. En virtud de esto, la
499. Supongamos que sean A, By C los centros de las esferas proyeecion del punto de tangencia resultara sobre el segmento
de radio R,A 1, Bl YC 1 las proyeceiones de estos centros sobre Be. De aqui se deduce que las esferas pequefias se proyecta-
GEOMETRiAIEstereometria (Resoluciones y Soluciones) 317

[lin en circunferencias inscritas en los triangulosABCy BCD.


Por esta raz6n, el radio de las esferas pequenas es x = -R+r
- r,os a y h = I cos a,
2

ABJi 2RJi entonces, x = 1, 10 que era necesario demostrar 0).


r= --- - = - -- s
6 6' r E C

de donde

R =Ji.
r Fig.212
B

Fig.213 Fig. 214

2. Problemas de demostracion 502. Examinemos el trapecio ABCD que se obtiene en la


secci6n axial del cono (vease la fig. 213). Sean Ey F los puntos
501. Sean E y F los puntos medios de las bases del trapecio medios de sus bases yO el punto medio del segmento EF.
ABCD obtenido en la secci6n axial del cono (fig. 213).
Tracemos por el punto medio 0 del segmento EF las rectas
OM -.lCD, ON -.lEF, CP -.lAD, y LMON=LPCD=a.
OM ~ CD, ON -.l EF Y CP -.l AD. Hagamos, para
Para la resoluci6n del problema es suficiente demostrar
simplificar la escritura, CD=I, EF=h, OM =x,
que OM = OE. Introduzcamos las denotaciones:
EC=r, DF=R, y LMON=LPCD=a.
EC = r, DF=R; OM=x, OE=~.
2
Para la demostraci6n, es suficiente establecer que x = 1. *) De la igualdad obtenida mas arriba R + r = I se desprende que
Segun la condici6n del problema rei (R + r) = rel 2 y, por 2R + 2r = I + I. Esto signitica que las sumas de los lados opuestos
del cuadrilatero examinado son iguales. Esto ultimo, ya es suficiente
consiguiente, R+r =1. Sin embargo, puesto que de los para que en el cuadrilatero se pueda inscribir una circunferencia. Sin
triangulos OMNy CPD tenemos que embargo, nosotros no nos basamos en este hecho.
318 PROBLEMAS DE MATEMATICAS ELEMENTALES

Entonces,
R+r
X = ON cos a = - - cos a.
2
Por consiguiente, OE = BE = EC y, por 10 tanto,
Del triangulo CPD tenemos: LBOC =90°.

h = CD cos a = ~(R - r)2 + Cp2 cos a. 504. Sea a ellado de la base de la pinimide dada SABCD, a
el angulo diedro formado por la cara lateral y la base, y H la
Pero, puesto que segun la condici6n del problema altura SO de la piramide (fig. 215). Entonces,
CP2 = 4Rr, entonces, s a a
r =-tg - .
2 2
Ademas (vease la f6rmula (1) en
As) pues, x = 1, 10 que era necesario demostrar. la resoluci6n del problema 481).

503. Sea SD la altura de la pinimide SABC, 0 el punto medio


de la altura, y E el punto medio del segmento BC cuya
longitud designaremos por a (fig. 214). R~ H2+(~r
Tenemos: Fig.215 2H
~

DE = a-n. Por consiguiente,


6 '

y, por 10 tanto,
OD = a-!6. R
6 r a
De agu), 2tg a tg--
2
GEOMETRiAiEstereometria (Resoluciones y Soluciones) 319

2a = x,
S
Haciendo tg obtenemos:
R 1+X4
----; = 2x2 (1 - x 2 ). o

Suponiendo, ademas, que sea x 2 = t, reduciremos el


problema a la demostraci6n de la desigualdad
~_ _1 -_ _ _ _ _ _ _ _=~8
1+t 2 - A A
(
2t 1- t ) ~ 1 + -J2 siendo 0 < t < 1.
Fig. 216 Fig.217

Multiplicando ambas partes de la desigualdad por el VOSHC = OA'


denominador y abriendo los parentesis, obtenemos la
VA c<'jBC SA
desigualdad
Analogamente,
(2-12 + 3) (2 - 2(-12 + 1) t + 1~ 0 VOSCA Oc VOSAH _ OB'
para un trinomio cuadrado. Calculando el discriminante del VASBC SC ' VASBC SB
trinomio, descubrimos que es igual a cero. Por consiguiente,
el trinomio no varia de signa cualesquiera que sean los valores Sumando estas igualdades, obtenemos:
de t. Puesto que siendo t = 0 el trinomio es positivo, la OA' OB' OC
desigualdad queda demostrada. - + - - + - - =1.
SA SB SC
505. Las piramides ASBC y OSBC tienen la base SBC comun 506. Sea P el plano del triangulo ABC, PI el plano del
(fig. 216), por 10 cual SllS volumenes son entre sf como las triangulo AIBIC I, y lla lfnea de intersecci6n de P con PI
alturas bajadas a su base comun. Puesto que OA1I AS, la (fig. 217). Designemos por QAB el plano que pasa por A, By
relaci6n de las alturas de las piramidesASBCy OSBC, bajadas O. La recta AIBI se encuentra en el plano QAR y, siendo no
a la base SBC, es igual a la relaci6n de SA a OA'. Por paralela a la recta AB, se cruza con esta en el punto TAB" Este
consiguiente, la relaci6n de sus volumenes es punto se encuentra en los pIanos P y PI y, por 10 tanto, sobre
la recta l. Analogamente, demostraremos que las rectas BC y
320 PROBLEMAS DE MATEMAnCAS ELEMENTALES

BIC I se cruzan en el punto TBe que se encuentra sobre I, y las qo = q02


rectas AC y AIC I se intersectan en el punto TAe que se OB AB
encuentra tambien sobre I.
Asi pues,
qo 1
°
507. Sea 1 el centro de graved ad de la caraASC, y BO I uno
de los segmentos examinados en el problema. Tomemos otra
OB 3

cara cualquiera, por ejemplo, la BSC; designemos su centro 10 que se afirmaba. Si tomamos, ahora, el centro de graved ad
de graved ad por 02 y demostremos que el segmento AO en una cara mas y trazamos el segmento correspondiente,
intersectani al segmento BO I y que el punto de interseccio~ entonces, en virtud de 10 demostrado, este tambien
°
de estos segmentos divide al segmento BO I en la relacion
°
de 1:3, contando desde el punto 1, En efecto, si MI y M2 son
intersectara al segmento BO I y, ademas, en el punto que
dividira a BO I en la relacion de 1:3, es decir, en el punto 0.
los puntos medios de los segmentos AC y BC (fig. 218), Por consiguiente, todos los segmentos examinados se
entonces, es evidente que ABiIM1M2; es facil tambien ver intersectan en el punto 0. Es tam bien evidente, que el punto
°
que 0P2iiM1M2' ya que los puntos 1 y 02 dividen ° divide a cada uno de estos segmentos en la relacion de
respectivamente a los segmentos MIS y M 2S en una misma 1:3, 10 que habia que demostrar.
relacion.
.
°
Por esta razon, ABiIOP2' la figura ABO2 I es un s
508. Desarrollemos primeramente
trapeclo y, por 10 tanto, sus diagonales BO I y AO?, se
un razonamiento auxiliar. Supon-
intersectaran. Designemos el punto de interseccion de las
gamos que sean PP I y QQ I dos
diagonales por 0. Tenemos:
rectas que se cruzan, A, B YC tres
_M J M 2 puntos sobre la recta QQI' con la
AB 2' particularidad de que el punto B
se encuentre entre los puntos A y
Multiplicando estas igualdades entre sf obtendremos que B
CAl' BI Y C I los pies de las per-
Fig. 218 pendiculares bajadas desde los
0P2 -.l puntos A, By C a la recta PP I . Designemos por hA' h R y hC las
AB 3
distancias desde los puntos A, B Y C hasta la recta PP I •
Pero, de la semejanza de los triangulos AOB y ° 00 se
I 2
Demostremos que hR es, por 10 menos, menor que una de las
desprende que distancias hA 0 he
GEOMETRiAIEstereometria (Resoluciones y Soluciones) 321

Con este fin, proyectemos la figura representada en la fig. seccion de la piramide no coincide con su cara, entonces, su
219 sobre el plano n perpendicular a la recta PP [. La recta area es estrictamente menor que el area de una de las caras.
PP [ tendni como proyeccion el punto 0, mientras que los B

segmentosAAI' BB[ y ee[ se proyectan sin variarsu longitud,


puesto que todos ellos son paralelos al plano n. En este caso,
el punto B' resulta entre los puntos A' y C. Examinando It

ahora el triangulo A' OC podemos confirmar que la linea


inclinada OB' es menor que una de las lineas inclinadas OA'
u OC. En efecto, bajando desde el punto 0 una perpendicular
AC (enla fig. 219 no se muestra), estableceremos que el punto
D
B' esta dispuesto mas cerca del pie de la perpendicular que p
Fig. 219 Fig. 220
uno de los dos puntos A' 0 C. De aqui se desprende que hB
es menor que hA 0 que he 509. En vez de comparar las sumas de los angulos pIanos de los
Sea, ahora, ABeD una piramide triangular arbitraria, y EFG vertices S y 8', comparemos entre Sl las sumas de los angulos
una seccion triangular tal, que, por 10 menos, uno de los pIanos de las caras laterales de ambas piramides en cada uno de
vertices F no es vertice de la piramide. Demostremos que, los tres vertices de la base comun.
entonces, el area del triangulo EFG es menor que el area de
s
Demostremos que cada una de estas
uno de los triangulos AEG 0 DEG (fig. 220). sumas de los angulos de la piramide
En efecto, los tres triangulos tienen el lado EG comun y, exterior es mayor que la suma
segun 10 demostrado anteriormente, la distancia desde F hasta correspondiente de los angulos de la
la recta EG es menor que la distancia desde A 0 D hasta esta ,,'5' piramide interior. Demostremos, por
misma recta. Si S /",.EFCi < St1 AEG, entonces todo queda ,/:>'~ -- --> B ejemplo, que (fig. 221)
demostrado. Pero, si S /",.1-:FG < S /",.DEG y, por ejemplo, el punto A ...:,~,:..:: - - - - - _.1\ - - - -

E no es vertice de la piramide, entonces, hacemos uso del LACS+LSCB>LACS'+LS'CB. (1)


razonamiento anterior para el I1DEG, comparando su area c
con loas areas de los triangulos DGA y BDG. Valiendonos, Fig. 221 De la desigualdad (1) y otras ana-
en caso de necesidad, una vez mas del mismo razonamiento logas (para los vertices A y B) obtendremos la solucion del
para el I1BDG, demostremos la afirmacion hecha en el problema. En efecto, sumando las tres desigualdades indica-
problema. De la resolucion expuesta, esta claro, que si la
322 PROBLEMAS DE MATEMATICAS ELEMENTALES

das, estableceremos que la sum a L, de todos los seis angulos es decir, la desigualdad (1).
pianos de las caras laterales en la base la piramide exterior
es mayor que la suma correspondiente L para la piramide °,° ,°
510. Designemos por 1 2 3 Y 04 los centros de las esferas
dadas, y por Pik el plano tangente comun para las esferas de
interior:
centros 0i y OK (i < k).
(2) Tales pIanos son en total seis: P 12 , P 13 , P 23 , P 14 , P24 Y P 34 •
Demostremos primeramente que
Pero, las magnitudes que nos interesan en el problema los pIanos P 12 , P I3 Y P23 se cruzan
por una recta. En efecto, cada uno
suplementan a LYL' hasta 540°(=180°.3) y, por de estos pIanos es perpendicular al
consiguiente, para elias tiene lugar la desigualdad de sentido °
plano 10.,° 3, puesto que este es
contrario. As! pues, nos queda demostrar la validez de (1). perpendicular a la linea de los
Prolonguemos el plano ACS' hasta su intersecci6n con la centros de las esferas divididas por
piramide exterior. Examinando el angulo triedrico CS'S" B, el mismo, que se encuentra sobre
Fig. 222
deducimos que este plano.
Ademas, es facil ver que los pIanos que se examinan (fig.
222) pasan por el punto 04 de intersecci6n de las bisectrices
LS'CS"+LS"CB > LS'CB.

Adicionando a ambas partes de la desigualdad el LACS',


(3)
del triangulo °\° °
2 3 , As! pues, los pIanos P 12 , P I3 Y P 23 se
cruzan, en efecto, por cierta recta que, como hemos estable-
obtendremos: cido de paso, es perpendicUlar al plano de los centros 0IOZ03
y pasa por el centro de la circunferencia inscrita en el triangulo
LACS"+LS"CB > LACS'+LS'CB. (4)
° °°
1 2 3 , Designemos esta recta por L 4 •
Anatogamente se demuestra que los pIanos P23 , P 24 Y P34
Pero, para el angulo triedrico CASS" tenemos:
determinan la recta comun para elIos LI perpendicular al plano
LACS + LSCS"> LACS". (5) del triangulo 020304' que pas a por el centro de la
Sustituyendo en la desigualdad (4) al LACS" por una circunferencia inscrita en este triangulo, y as! sucesivamente.
magnitud mayor, de acuerdo con (5), obtendremos: Como resultado \legamos al siguiente problema (fig. 223):
°
en cada cara de la piramide triangular I 0 0_0,04 hay inscrita
J

LACS + (LSCS"+LS"CB) > LACS'+LS'CB, una circunferencia por cuyo centro pasa la perpendicular ala
GEOMETRiA/Estereometria (Resoluciones y Soluciones) 323

cara. Es necesario demostrar que las cuatro perpendiculares tres esferas que hacen contacto entre sf de dos en dos. En efecto,
L I , L z' L3 y L4 tienen un punto comun, si se sabe que los si Pes el punto de interseccion de las bisectrices del Ll ABC,
puntos de contacto de dos circunferencias con una misma YPI' P2 YP 3 son los pies de las perpendiculares bajadas desde
arista coinciden. P respectivamente a los lados AB, BC y CA, entonces,
D
A~ =Ap"

y las esferas de centros A, B y C, cuyos radios son respect i-


vamente iguales a

hacen contacto entre sf de dos en dos.


Sea ABCD la pinimide dada (fig. 224). Examinemos las
°4 B tres esferas de radios rA , rR y rC' cuyos centros son A, B y C y
Fig. 223 Fig. 224 que hac en contacto entre sf de dos en dos. Designemos por AI'
BI YC I los puntos en los cuales las superficies de las esferas se
Este hecho es casi evidente. Designemos por 0 el punto cruzan con las aristas AD, BD y CD, y demostremos que
de intersecci6n de las rectas LI YL 4 ; estas ultimas se intersec-
tan, puesto que se encuentran en un mismo plano P Z3 Y no
son paralelas. Demostremos, ahora, que las rectas L3 Y L2
pasan tambien por el punto O. En efecto, el punto 0 se Segun la condici6n del problema
encuentra en la linea de interseccion de los pIanos P I2 Y P24 ,
AD+ BC = BD+ AC.
puesto que la recta L4 pertenece al plano P 12 , Y la recta LI' al
plano P 24 • Pero, la linea de intersecci6n de P I2 con P 24 es la De acuerdo con la construcci6n tenemos que
recta L3 • Por consiguiente, L3 pasa por el punto O. Ami-
logamente demostramos que la recta L2 pasa por el punto O. AD =rA +A1D; BC=rs+rc ,
BD=rR +B1D; AC = rA + rc-
511. Si se conocen tres puntos A, B y C no pertenecientes a
una misma recta, entonces, estos puntos son los centros de
324 PROBLEMAS DE MATEMAnCAS ELEMENTALES
- - - - - - - - - - - - - -- - - - - - - - - -- - - - - - - - - - -
Colocando las ultimas cuatro expresiones en la igualdad
anterior, obtendremos:

AID = BID.
o bien
Analogamente, valiendonos de la igualdad
Fig. 225
BD+AC=CD+AB,

hallaremos:

De esta ecuaci6n, hallaremos:


Por consiguiente, la esfera de centro D y de radio 'Yz
rD=A1D=B1D=C1D
Por consiguiente los radios de las esferas deberan satisfacer
hace contacto con cada una de las tres primeras esferas y,
la relaci6n
por 10 tanto, las cuatro esferas construidas hacen contacto
entre sf de dos en dos. ~>(
- -)2
Ft+~
512. Designemos los radios de las esferas por rl' r z y r 3 , y sea
513. Supongamos que el numero de caras laterales de la
r1 ~ r 2 ~ r 3 . Tracemos un plano tangente a las dos primeras
piramide sea igual an. Unamos el punto arbitrario 0, tornado
esferas. Ademas, tracemos por los centros de estas esferas
en el plano de su base, con todos los vertices yexaminemos
un plano perpendicular al plano tangente y examinemos la
11 piramides triangulares con un vert ice comun en el punto
circunferencia de radio r que hace contacto con las dos circun-
0. Es evidente que el volumen V de la piramide dada es igual
ferencias grandes obtenidas en la secci6n y con su recta
a la suma de los volumenes de las piramides triangulares
tangente comun (fig. 225). La tercera esfera puede, evidente-
obtenidas. Tenemos:
mente hacer contacto con las dos primeras esferas y con el
plano ~ue tiene contacto con elias, si esta esfera "no es dema-
siado pequefia", a saber, si r3 ~ r. Tenemos (fig. 225) que
GEOMETRiAiEstereometria (Resoluciones y Soluciones) 325

donde r l , r 2, ••• , rr, son las distancias desde el punto 0 hasta 515. Tracemos el plano indicado en el problema por los
las caras laterales, y S el area de la cara lateral. vertices B, D y F (fig. 227) Y otro plano, paralelo al primero,
Por consiguiente, 1j + r2 + ... + rn = 3; es una magnitud por los vertices C, E y G. Ambos pianos forman en su intersec-
constante que no depende de la posicion del punto 0 en el cion con el paralelepfpedo triangulos equilciteros iguales.
plano de la base, 10 que era necesario demostrar. Designemos la longitud de cada lado de estos triangulos por
a. Si, ahora, trazamos por el punto medio de una de las seis
514. Examinemos los dos pIanos sombreados en la fig. 226 aristas que unen los vertices de los dos triangulos mencio-
y el triangulo ADE en el plano P que pasa por los vertices A, nados, por ejemplo, por el punto medio N de la arista Be, un
D, H y E del paralelepfpedo dado. EI plano P corta al plano plano paralelo a los pianos indicados, este dara en la seccion
del f..BCD por la recta KD que pasa por con el paralelepfpedo el hexagono MNPQRS, todos los lados
;..-- ---."lJ del cual, evidentemente, resuItaran iguales a ~. Observemos,
ademas, que MN II DF y NP II BD. Por esta razon, el L MNP
complementa al L BDF hasta 180° y, por consiguiente, el
LMNP = 120°. Analogamente establecemos que los demas
angulos del hexagono son tambien iguales a 120°.

516. Sea SABC el tetraedro dado, P y Q los puntos medios


Fig. 226 Fig. 227 de las aristas opuestas AC y SB, MPNQ cierta seccion del
tetraedro en la que se encuentra el segmento PQ (fig. 228).
el punto K de interseccion de las diagonales del para Ie le-
Examinemos la seccion plana SPB que, evidentemente, divide
pfpedo ABEC, y, por consiguiente, el segmento KD es una
al tetraedro en dos partes equidimensionales. EI problema
mediana del f.. A ED. Es evidente, que AO es tambien una
quedara solucionado si demostramos que las piramides SPQN
mediana del f.. AED. Por esta razon, el punto S que nos
y MPQB son equidimensionales.
interesa, es el punto de interseccion de las medianas del
Bajemos sobre el plano SPB las perpendiculares desde los
f.. AED y, por 10 tanto,
puntos My Ny designemos sus pies respectivamente por K
AS=~AO=.!AH y L. Puesto que los triangulos PQB y SPQ son equidimen-
3 3' sionales, entonces, para resolver el problema es suficiente
con 10 cual el problema queda demostrado. demostrar que LN=MK: Estableceremos esta igualdad al
demostrar que
326 PROBLEMAS DE MATEMATICAS ELEMENTALES

MO = NO. (1) mos, a continuaci6n, por las aristas AB, Be y AC pIanos que
dividan los correspondientes angulos diedros por la mitad.
Estos pIanos se intersectan en el punto 0 equidistante de las
cuatro caras de la pinimide y, por consiguiente, que es el
centro de la esfera inscrita en la pinimide. Es facil ver, que
en el caso que se exam ina, en virtud de la igualdad indicada
de los triangulos, el punto 0 resultara sobre la altura SP de la
piramide. Repitiendo los razonamientos, estableceremos que
todas las alturas de la piramide se intersectan en el punto O.
Fig. 228 Fig. 229 Valiendonos de este hecho, podemos afirmar que, por ejem-
plo, los triangulos APS y SPE se encuentran en un mismo
Examinemos, con este fin, un par de pianos paralelos en los plano y, por consiguiente, los segmentos AP y P E pertenecen
que se encuentran las rectas que se cruzan SC y AB (fig. 229). a una misma recta. Por eso, AE es simultaneamente la bisec-
Dado que el segmento PQ une los puntos medios de los triz y la altura del Ll ABC. Por la misma causa, las demas
segmentos AC y SB, el, evidentemente, esta dispuesto en un bisectrices del Ll ABC son al mismo tiempo sus alturas. Por
plano paralelo a los pianos dados y que equidista de estos. En consiguiente, el Ll ABC es equilatero. Repitiendo los razona-
virtud de esto, el segmento AfN, al intersecarse con el segmento mientos, estableceremos que todas las caras de la piramide
PQ, sera dividido por el punto de intersecci6n por la mitad. representan triangulos equilateros, con 10 cual el problema
s queda resuelto.
517. SeaSABCla piramide dada (fig.
230). Bajemos desde el vertice S la 518. Supongamos que el segmento AB se encuentra en el
altura SP de la caraABC, y las alturas plano Q, y el segmento CD, en el plano P, y sea P IIQ (fig.
SD, SE y SF de las tres caras 231). Tracemos por el punto A una recta paralela a CD y
restantes. Es facil ver, que los tracemos el segmento AA, =CD. A base de los lados AB y AA,
triangulosSPD,SPEy SPF, en virtud construyamos el paralelogramo ABB,A,. Realicemos una
de la igualdad de los angulos SDP, construcci6n analoga en el plano P. Uniendo A con C, B con
SEP y SFP, son iguales entre sl (com- C" A, con D y B, con D, obtendremos el paralelep1pedo
parese con el problema 458). Trace-
B ABB,ApCCp,. Examinando la cara ACB como base de la
Fig. 230 piramide DACB, observamos que el volumen de la piramide
GEOMETRiAIEstereometria (Resoluciones y Soluciones) 327

B SO=RO y, por consi-guiente, QO=PO, como lineas inclinadas


que tienen iguales proyecciones. Teniendo en cuenta, ademas,
D
que segun la
MO=NO, (2)
D,
construcci6n podemos deducir que los triangulos OMQ y
ONP son rectangulos e iguales. De aqui se desprende la
igualdad de los angulos:

Fig. 231 Fig. 232


LMQO=LNPO. (3)

Asf pues, en un sentido la afirmaci6n queda demostrada.


es igual a ~ del volumen del paralelepipedo. Puesto que, sin Si, ahora, al contrario, se cumple la igualdad de los angulos
embargo, el volumen del paralelepipedo no varia al trasladar (3), entonces, en virtud de (2), tiene lugar la igualdad de los
los segmentos (no varian ni el area de la base ABBIA I, ni la triangulos QMO y PNO. Como consecuencia de esto,
altura, es decir, la distancia entre los pianos P y Q), entonces, QO=PO y, por 10 tanto, SO=OR. De aqui se desprende ya la
tampoco varia el volumen de la piramide. igualdad (1).

519. Sean P y Q los puntos de intersecci6n de la recta dada 520. Unamos los puntos B y C, A Y D con segmentos de
con las caras CBA y DBA del angulo diedro (fig. 232). rectas (fig. 233). Tracemos por el punto A una recta paralela
Tracemos por la arista AB el plano bisector ABE y, luego, a MN hasta que se encuentre en el punto K con la recta que
por el punto 0 de su intersecci6n con la recta PQ, tracemos pasa por los puntos By N. Observemos que AK=2MN, puesto
el plano C I B I D I perpendicular a la arista AB. Sea,., a E

continuaci6n, OM ..1 BID], ON ..1 B]C] Y SR la proyecclOn


de PQ sobre el plano DIBIC I de tal modo que
Qs ..1 B D y PR.l B]C]. Si los puntos Py Qequidistan de
] ] " I
la arista, es decir, si entonces, el triangulo BIRS es Isosce es,

(1) c
Fig. 233 Fig. 234
328 PROBLEMAS DE MATEMATICAS ELEMENTALES

queMNes lamediadeltrianguloABK.Luego, !1BNC = !1KND, Es facil comprender que nuestro razonamiento conserva
ya que BN=NK, CN=ND Y LBNC=LKND. Por eso, su vigor tambien para el caso cuando el angulo tetraedrico
DK = BC. Del trianguloADK se desprende: no es convexo, es decir, cuando la arista ED resulta al otro
lado del plano CEB.
DK + AD > AK = 2MN
522. Supongamos que sea dado el angulo tetraedrico convexo
(aqu! tiene importancia que D no se encuentra sobre la recta
con el vertice S (fig. 235). Prolonguemos los pianos BSC y
AK, de 10 contrario, tendrfamos que haber puesto el signo ~).
ASD hasta que se crucen por la recta II' Luego, prolonguemos
As! pues,
'?
los pianos ASB y DSC hasta que se crucen por la recta Las
BC+AD>2MN,
'I '2'
rectas y evidentemente, no coinciden (de 10 contrario,
10 que era necesario demostrar. todas las caras en su prolongaci6n pasarian por una misma
recta). Designemos por P el plano en el que se encuentran
521. Sean A, B, C Y D puntos arbitrarios que se encuentran 'I '2'
las rectas y Valiendonos de la convexidad del angulo
en las aristas del angulo tetraedrico con el vert ice E (fig. tetraedrico, es f<icil demostrar que el plano P tiene solamente
234). Demostremos, por ejemplo, que un punto comun con el angulo dado, el punto de intersecci6n
S, de modo que todo el angulo se encuentra a un lado del
plano P (este hecho es casi evidente). Demostremos, ahora,
LCED < LCEA + L AEB + L BED, (1)
que todo plano que corte al angulo tetraedrico y que sea
Tracemos el plano CEB. Segun la propiedad de los angulos paralelo al plano P, formara en la secci6n con este angulo un
pianos de un angulo triedrico paralelogramo. En efecto, en virtud de 10 expuesto, tal plano
cortara a las cuatro aristas del angulo tetraedrico. Designando
L CED < L CEB + L BED. (2) los respectivos puntos de intersecci6n por A', B', C' Y D',
B'II
observemos que A' B' C', puesto que estos dos segmentos
y por la misma causa son por separado paralelos a 'I; por la misma causa
A'B'liUC'. '
LCEB < LCEA + LAEB. (2)
Por consiguiente, el cuadrangulo A'We'D ' es un paralelo-
De las desigualdades (2) y (3) se desprende (1). De este gramo, 10 que era necesario demostrar.
modo, la desigualdad (I) queda demostrada.
GEOMETRiAiEstereometria (Resoluciones y Soluciones) 329

D KD1C I • Segun la construccion tenemos que KN ..L DC, por


consiguiente, KN ..L DICI y, por 10 tanto, KN es la altura del
Ll KDIC I. Por eso, N es el punto medio del segmento D I C 1 y,
por 10 tanto, tam bien del segmento DC.
De este modo, para las condiciones del problema, el pie
de la perpendicular comun ados aristas que se cruzan de la
B
piramide divide a estas aristas por la mitad.
Fig. 235 Fig. 236
De la fig. 237 es facil observar que LK = KM, puesto
que DD] = CC I . Por eso (vease la fig. 236) AL = BM Y de
la igualdad de los triangulos rectangulos ALD y BMC se
523. Sean DL y CMlas alturas de los triangulosADB y ACB, desprende que
bajadas a su base com un AB (fig. 236). Dado que estos AD=BC.
triangulos son equidimensionales, entonces, DL = CM.
Supongamos, ademas, que en KN la perpendicular com un a Analogamente se demuestra que AC = BD Y que
las aristas que se cruzan AB y DC. AB = DC. Por consiguiente, todas las caras son iguales entre
sl, como triangulos que tienen tres lados iguales.
B

3. Lugar geometrico de los puntos

524. Sea P uno de los pIanos que pasa por el punto dado A, y
Fig. 237
Mia proyeccion del punto dado B sobre el plano P. Supon-
gamos, a continuacion que sea C el punto medio del segmento
Tracemos por el segmento KN el plano P perpendicular a
AB (fig. 238). Puesto que el triangulo AMB es rectangulo,
la arista AB. Proyectemos el cuadrilatero LMCD sobre el
plano P (fig. 237). Puesto que los segmentos DL y CM se entonces, CM = ~ AB. De este modo, todos los puntos M se
proyectan sin que varien sus longitudes (ambos son paralelos encuentran a una misma distancia ~ AB del punto C y, por
al plano P), y el segmento LM tiene como proyeccion un consiguiente, estan dispuestos en una esfera de radio ~ AB
punto, en la proyeccion se obtiene el triangulo isosceles cuyo centro es el punto C. Es f:'lcil ver, ademas, que cualquier
330 PROBLEMAS DE MATEMAnCAS ELEMENTALES

punto de dicha esfera coincide con una de las proyecciones encontrara en el plano PI' Examinemos, ahora, el triangulo
del punto B. Asi pues, el lugar geometrico buscado es una rectangulo OMC. El punto C no depende de laelecci6n del
esfera cuyo diametro es AB. plano secante P, y la hipotenusa OC del triangulo rectangulo
OMC es una magnitud constante. Si D es el punto medio de
B
~C, entonces MD = or· Por consiguiente, si 1 no tiene
puntos comunes con la esfera, el lugar geometrico buscado
es una parte del arco de una circunferencia de radio °2C ,
encerrada dentro de la esfera (el arco se encuentra en el plano
PlY pasa por el centro de la esfera). Si I tiene contacto con la
p esfera, el lugar geometrico buscado sera una circunferencia
de radio 1, donde R es el radio de la esfera. Si, por fin, I
Fig. 238 cruza a la esfera, el lugar geometrico de los puntos M sera
una circunferencia de radio °2C ,
p
526. Ellugar geometrico buscado es una superficie de revoluci6n,
~ _ _-+_~M obtenida como resultado de la rotaci6n del arco de una
circunferencia 0 de toda la circunferencia (vease la resoluci6n
: del problema anterior) alrededor de su diametro oC.

527. Demostremos que el lugar geometrico buscado es una


Fig. 239

525. Sea 0 el centro de la esfera dada. Tracemos por la recta esfera de radio R!f y que el centro de esta esfera coincide
dada 1 cierto plano P que corta a la esfera por una con el centro de la esfera dada.
circunferencia con centro en el punto M(fig. 239). Como es Sea M un punto arbitrario del lugar geometrico buscado;
sabido, OM 1. P. Tracemos, a continuaci6n, por el punto 0 los segmentos MA, MB y MC (fig. 240), por ser segmentos
el plano PI perpendicular a la recta I. Designemos el punto tangentes a la esfera, trazados desde un mismo punto, son
de intersecci6n del plano PI con la recta 1 por C. Dado que iguales entre sl. Por eso, los triangulos rectangulos AMC,
los pianos PI y P son perpendiculares, el segmento OM se CMB y AMB son tambien iguales. Por consiguiente, el
GEOMETRiN'Estereometria (Resoluciones y Soluciones) 331

!1ABC es equiilitero. Es evidente, que el segmento OM bajada desde A al plano por C. Tomemos, a continuaci6n,
cortan! a este triangulo en su centro de gravedad 0 1, Sea cualquier recta que pase por el punto B y bajemos a esta
AM = a, entonces recta la perpendicular AD (fig. 241). Entonces, porel teorema

AC=a-!i y AO, = -
a-/6 . conocido, CD ~ BD.
3 Por consiguiente, el punto D se encuentra sobre una
circunferencia cuyo diametro es el segmento Pc. Es facil
Colocando estos valores en la igualdad
demostrar que tambien, al contrario, cualquier punto de la
OM·AO, =OA·AM circunferencia indicada es el pie de la perpendicular bajada
desde el punto A a cierta recta de la familia examinada. Por
(aqu! nos valemos de que el !10AM es rectangulo y escri- eso, el lugar geometrico buscado es una circunferencia de
bimos su area por dos metodos), obtendremos: diametro BC.

-/6
OM·a-=Ra. 529. Son posibles dos casos. I) La recta AB no es paralela al
3 plano examinado P. Designemos por D el respectivo punto
De aqu!, de intersecci6n de AB con P (fig. 242). Sea Mel punto de
tangencia del plano con una de las esferas del conjunto
OM= -/6 R. examinado. Tracemos LIn plano por las rectas AB y DM. Este
2
plano cortara a la esfera por una circunferencia que hace con-
As! pues, el punto M se encuen-
tra en la esfera mencionada mas
arriba. Girando la esfera dada
junto con las tangentesAM, CMy A
BM alrededor del centro 0, nos
convenceremos de que cualquier
punto de la esfera pertenece al
lugar geometrico examinado. Fig. 240

528. Designemos el punto dado del espacio por A, el punto Fig. 241 Fig. 242
de intersecci6n de las rectas por B, y el pie de la perpendicular
332 PROBLEMAS DE MATEMATICAS ELEMENTALES

tacto con la recta DM en el punto M. Por la conocida radio OM, cuyo centro sea el punto 0, entonces, esta esfera
propiedad de la tangente y la secante trazadas desde un mismo hara contacto con el plano P y pasara por los puntos A y B.
punto a una circunferencia, tenemos: De este modo, al contrario, cualquier punto de la circun-
ferencia C pertenece a nuestro lugar geometrico. Por cons i-
guiente, ellugar geometrico buscado es la circunferencia C.
2) En el caso, cuando la recta AB es paralela al plano, el
Por consiguiente, el segmento DM tiene una longitud
lugar geometrico buscado representa una recta que se
con stante igual a -J BD· DA, que no depende de la eleccion
encuentra en el plano P y que es perpendicular a la proyeccion
de la esfera y, par 10 tanto, todos los puntos M se encuentran
del segmento AB sobre el plano Pyla divide por la mitad.
en una circunferencia de radio r = ~ DB. DA cuyo centro es
el punto D. Designemos esta circunferencia por C.
530. Caso a). Sea Del punto medio del segmento AB (fig.
Supongamos, ahora, 10 contrario, que M es cierto punto de la
244), C el punto movil, Q el centro de graved ad del t.ABC,
circunferencia C; demostremos que este punto pertenece al
y Q' el centro de graved ad del t. ASB. Puesto que el punto Q
lugar geometrica examinado.
divide al segmento DC en la relacion de 1:2, el lugar
Tracemos por los puntos A, B Y M una circunferencia
geometrico de estos puntos es, evidentemente, un rayo
auxiliar y designemos su centro por 0 1 (fig. 243). Puesto
paralelo a la arista SE, que para el punto Q', 0 sea, por el
que en las condiciones del problema DB· DA = DM2,
centro de gravedad del t. ASB.
entonces, la recta DM es tangente a esta circunferencia y, par
s
consiguiente, qM -.l DM. Levantemos, ahora, en el punto
M una perpendicular al plano P, y en el punto 01' una
perpendicular al plano de la circunferencia auxiliar. Estas
dos perpendiculares se encuentran en un plano perpendicular
a la recta DM en el punto M, y no son paralelas (en el caso
contrario, el punto 01' y junto con este los puntos A y B, se
encontrarian en el plano P). Por esta razon, estas
perpendiculares se intersecan en cierto punto O. Es facil ver,
/ 1 B \

que OA = OB = OM, puesto que las proyecciones de estos I'--D _ _ _ --.-J! A

segmentos alA, alB y aiM son iguales entre sf como radios


Fig. 243 Fig. 244
de una circunferencia. Por eso, si se construye una esfera de
GEOMETRiAIEstereometria (Resoluciones y Soluciones) 333

Caso b). Si, ahora, es el punto Bel que se desplaza por e gitud la tiene la perpendicular comun a estas rectas. No es
la arista SG, entonces, los centros de graved ad Q' de los diffcil comprender, que la perpendicular comun a dichas
triangulos ASB se encontraran sobre un rayo paralelo a la rectas es el segmento M 0 que une el punto medio de la
arista SG, que pasa por el punto Q" que divide al segmento arista CE y de la diagonal AB. En efecto, el f."AM'Bes
AS en la relaci6n de 2: 1, contando de A a S, y los rayos is6sceles y por eso M'O 1. AB. Puesto que tambien el
examinados en el caso a), correspondientes a cada posici6n f."COE es is6sceles, M'O 1. CEo Asf pues, la secci6n de
fijada del punto B, lIenaran la secci6n del angulo triedrico menor area es la secci6n que divide a la arista CE por la
con un plano que pasa por el punto Q" paralelamente a la mitad; el area correspondiente S es
arista SG y SE. ~ 2 j,:
S = a.fj . a,J2 =a \/ 6.
2 2
4. Valores nuiximos y minimos Este mismo problema puede ser resuelto de otra manera,
si se emplea el siguiente teorema: el cuadrado del area de un
531. Sin derogar la comunidad, se puede considerar, que el poligono plano es igual a la suma de los cuadrados de las
plano secante se cruza con e la arista CE del cuba (fig. 245). areas de sus proyecciones sobre tres pIanos perpendiculares
Es facil ver, que en la secci6n se obtiene siempre cierto entre sf. Este teorema se demuestra sin dificultad a base de
paralelogramo AMBN. EI area S del paralelogramo puede la f6rmula, por la cual el area de la proyecci6n de un poligono
ser hall ada por la f6rmula plano sobre un plano es igual al area del poligono multiplicada
1/ S=AB·MK, por el coseno del angulo entre los pIanos (v ease la formula
(1) en la resoluci6n del problema 456).
donde por MK se ha designado la Considerando a este teorema demostrado, designemos por
perpendicular bajada desde el punto x la longitud del segmento CM(vease la fig. 245). Las proyec-
M de la arista CE a la diagonal AB. ciones del paralelogramo que nos interesa sobre los pianos
Asf pues, el area S sera minima ACD, ECDB Y BDN estan representadas en el orden corres-
junto con la longitud del segmento pondiente en la fig. 246, a, by C. Las areas de las proyecciones
MK. Pero, entre los segmentos son respectivamente iguales a a 2 , ax y a 2 - ax, asf que, en
que unen los puntos de las rectas que virtud del teorema mencionado,
Fig. 245 se cruzan CE y AB, la menor 1011-
334 PROBLEMAS DE MATEMAnCAS ELEMENTALES

AI :;SJD:.
C D E
~.

~ B
N

0) b) c)
At=::::::::;:=7J.;"--------'-'D
x
Fig. 246
Fig. 247

S-, (,)2
= \a- + ( ax ) 2 + \a
(2 - ax )2 = 2a 2 (x 2 - ax + a- . ?)
Puesto que L LKN es igual al angulo formado por las
? ,
Representando al trinomio cuadrado x- - ax + a- en la rectas que se cruzan AB y CD, el seno de este angulo es una
forma magnitud constante para todas las secciones paralelas
examinadas. De este modo, el area de la seccion depende
solamente de la magnitud del producto KN . KL. Designemos
la longitud del segmento AK por x. Entonces, en virtud de la
semejanza de los triangulos, tenemos:
hallamos (comparese con (1), pagina 42) que SZ tendra su
KN AD-x KL x
valor mlnimo cuando x = 1,
y el valor mlnimo del area es
AB Ad CD AD
_ I,) 22 a 2 -_
Smin - o ! ~a
a2~
.
Multipliquemos estas igualdades entre Sl:
V 4 2 KN . KL = AB ·CD (AD - x)x.
AD2
532. El cuadrilatero MNKL obtenido en la seccion de la
piramide ABCD (fig. 247) es un paralelogramo, puesto que Dado que A:~D es una magnitud con stante, de la formula
LKIICD y MN IICD; por consiguiente, LK I MN y, analo-
anterior se desprende que el producto que nos interesa
gamente, LM !!KN, si L LKN = a, entonces, el area del
KN . KL adquiere su valor maximo junto con el producto
paralelogramo, por la formula conocida, es
(AD -x)x.
S = Klv· KL sen a.
TRlGONoMETRiAIResoluciones y Soluciones 335

Examinando este producto como el trinomio cuadrado Trigonometria


- x 2 + ADx y representandolo en la forma

1. Transformacion de las expresiones que contienen


nos convencemos de que este alcanza su valor maximo funciones trigonometricas
cuando x = ADI2 (comparese con (1), pagina 42).
533. Empleando la formula

a 3 + b 3 = (a + b) (a 2 - ab + b 2 ) = (a + b) l(a + br -3ab ),
obtenemos

534. Designemos la parte izquierda de la identidad por S y


sustituyemos el producto 2 cos a cos ~de la formula (14)
pagina 69, por la suma cos(a + ~)+ cos (a - ~). Entonces S
se escribira en la siguiente forma:

S - cos 2 a - cos (a + ~)cos(a - ~).


Aplicando de nuevo la formula (14), hallamos:

cos (a +~) cos (a -~) = ~ (cos 2a + cos 2~).


2
336 PROBLEMAS DE MATEMAnCAS ELEMENTALES

Por otra parte, empleando por segunda vez la f6rmula para


Si, ahora, sustituimos cos 2 a por 1+C~ 2a definitiva- la tangente de la suma de dos angulos, hallaremos facilmente
mente obtendremos: que
s == 1- cos 2~ == sen2R tg x -3_
tg x (3 -tg 2 X )
2. (2)
2 1--', \- 3 tg x

10 que era necesario demostrar. Comparando (I) Y (2), obtenemos la soluci6n del problema.
Observacion: La f6rmula (2) puede ser tambien deducida
535. De la formula de las f6rmulas (7) y (8) pagina 69

tg (a + I--'R) == _t=
g_a_+----'tg::::....:~'----- 537. Valiendonos de las f6rmulas para la suma y la diferencia
\- tg a· tg ~ de los sen os, representamos la parte izquierda de la identidad
en la forma siguiente:
se desprende que

tg a + tg ~ == tg(a + ~){l- tg a tg ~], 2 sen -a+~ a-~ - 2 cos (a+~l


- cos-- a+~
y + - - ) sen - - ==
2 2 2 2
de donde
== 2 sen-2
~ Lcos-
a +-·
2
r
a - -~ -cos (
y+a + ~)] .
-2-
tg a + tg ~ - tg (a + ~) == -tg a tg ~ tg (a + ~).

Suponiendo en la ultima igualdad a == x, ~ == 2x, obtene- De aqui, empleando la f6rmula para la diferencia de los
mos la f6rmula requerida. cosenos, obtenemos facilmente que la parte izquierda de la
identidad coincide con la derecha.
536. Tenemos:
538. Utilizando la identidad del problema 537, obtendremos:
a+~ ~+y y+a
sen a+sen ~+sen y =4 sen--sen - - sen-- =
222
y a ~
= 4 cos - cos-cos-
2 2 2'
TRIGONoMETRiAIResoluciones y Soluciones, 337

puesto que, 13 = (a +13)-a, 2a + 13 =(a + 13)+ a.


a+13 7t Y 13+y 7t a a+y 7t 13 Colocando estas expresiones para 13 y 2a + 13 en la
2 2 2' 2 2 2' 2 2 2 igualdad inicial
539. Valiendonos de la identidad indicada en el problema 3 sen 13 = sen (2a + 13) (2)
537,obtendremos:
y valiendonos de las formulas para el senos de la suma y la
sen 2na + sen 2n13 + sen 2ny = diferencia de dos angulos, transformemos la igualdad (2) a
(1) la forma siguiente:
= 4- sen n(a + 13)· sen n (13 + y). sen n (y + a).

A continuaci6n, tenemos que sen (a + 13)cos a = 2 cos (a + 13)sen a. (3)

sen n(a + 13) = sen n(7t - y) = (_I)n+lsen ny. Dividiendo ambos miembros de (3) por cos a cos (a + 13),
obtenemos (1).
Transformando amilogamente los otros dos facto res en la
parte derecha de (1), obtenemos la solucion del problema. 542. Todos los valores de a y 13 son admisibles, excepto
aquellos para los cuales cos (a + !3) = 0 y cos!3 = A. Obser-
540. Para la demostracion multiplicamos ambos miembros vando que sen a = sen (a + 13 - 13), escribamos la igualdad
de la igualdad cos (a + 13) = 0 par 2 sen 13 y empleamos la inicial en la forma siguiente:
formula (15) pagina 69.
sen (a + 13)cos 13 - cos(a + 13)sen 13 = A sen (a + 13)· (1)
541. Los valores admisibles de los argumentos se determinan
Dividiendo ambos miembros de (1) por cos (a + 13)* 0,
de la condicion de que cos a cos (a + 13) * o. Observemos
obtendremos que tg(a + 13)cos 13- sen 13 = A tg(a + 13). Ex-
que la igualdad
presando de aqul tg (a + 13), hallaremos la igualdad reque-
tg (a + 13) = 2 tg a, (I)
rida.
que se necesita demostrar, contiene los argumentos
a + 13 y a. Por esta razon, es natural introducir estos mismos 543. Es facil comprobar que, en virtud de las condiciones
argumentos en la igualdad inicial. Tenemos: del problema, sen a cos a cos 13 * 0, puesto que de 10 contra-
338 PROBLEMAS DE MATEMATICAS ELEMENTALES

rio tendriamos que I m I : : : In I· Por esta razon, la igualdad a que era necesario demostrar.
demostrar tiene sentido. Representemos esta igualdad en la
forma tg a + ta n. m + n tg a, 544. Examinemos la identidad
--=____=:0___1-'_ = __
1- tg a tg 13 m - n (1)
COS (x + y + z) = cos {x + y)cos z - sen (x + y)sen z =
de donde
= cos x cos yeas z - cos z sen x sen y - cos y sen x sen z -
m+n
tg (a + 13) = - - tg a. (2) - cos x sen y sen z.
m-n

Sustituyamos en la relacion (2) las tangentes de los fmgulos Puesto que segllll la condicion del problema
a y a + 13 por los senos y cosenos, reduzcamos el quebrado '*
cos x cos y cos z 0, de esta identidad se desprende que
a un denominador comun y eliminemos el denominador
cos (x + y+z)=
comun. Obtendremos:
= cos x cos yeas z (1- tg x tg Y - tg Y tg z - tg z tg x).
m [cos a sen (a + (3)- sen a cos(a + (3)]-
(3) 545. Primera resoluci6n. Segun la condicion del problema
- n [sen a cos(a + (3)+ cos a sen (a + (3)] = 0

o bien o< a < 1t, 0 < 13 < 1t, 0 < Y < 1t Y a + 13 + Y = 1t. (1)

msen l3-nsen(2a+I3)=0. (4) Por eso, de (1) se desprende que

As! pues, la demostracion se reduce a la demostracion de


la relacion (4). Dado que la relacion (4) se cum pie por las
(I3+Y) (1t
tgl-2- = tg 2 - l
a\ 1
2)
=u·
(2)
condiciones del problema, entonces tiene lugar (3) y, por
tg
2
consiguiente, tambien (2). Por otra parte, por la f6rmula de la tangente de la suma,
Pero, de (2) se desprende (1), Y de (I) se deriva la relaci6n tendremos:
1 + tg 13
tg a 1 - tg a tg 13 (3)
m+n /11-n
TRIGONoMETRiAiResoluciones y Soluciones 339

fgualando los segundos miembros de las igualdades (2) y Si k es impar, entonces, la expresi6n examinada es igual a
(3) Y liberandonos de los denominadores, obtenemos la 1 y no depende de x, y y z. Si k es par, entonces depende de x,
igualdad requerida. yyz.
Segunda resolucion. De la f6rmula
547. Primera resolucion. Observemos, primeramente, que
tg [3 tg y *" 1, puesto que, en el caso contrario, tendrfamos
que tg [3 + tg Y = 0, 10 que es incompatible can la igualdad
tg [3 tg Y = 1. Por esta raz6n, de la condici6n del problema
se deduce que

tga=- tg[3+tgy =-tg([3+y)=tg(-[3-y),


demo strada en el problema anterior, hallamos directamente 1- tg [3 tg Y
que a [3 [3 y y a
1 - tg 2 tg 2 - tg 2tg 2 - tg 2tg 2 = 0, de donde hallamos que a=kn-[3-y, es decir, que
a + [3 + y = kn.
puesto que Segunda resolucion. En el problema 544 fue demo strada
a+[3+y 1t
la f6rmula para el coseno de la suma de tres angulos. Analo-
gamente se puede obtener la f6rmula
2 2
546. Por el sentido de la expresi6n que se exam ina en el sen (a +~+y)=
problema, cos x cos y cos z *" 0. Por esta raz6n, de la f6r- = cos a cos ~ cos y(tg a + tg ~ + tg Y- tg a tg ~ tg y),
mula obtenida en el problema 544, hallamos:
suponiendo que cos a cos [3 cos y *" 0. Por esta f6rmula
cos(x+ y+ z) hallamos que, seg(m las condiciones de este problema,
tg x tg Y + tg y tg z + tg z tg x = 1- =
cos x cos y cos z
1[
sen (a + [3 + y) = 0, es decir, 0.+ [3 + Y= kn.
cos - k
=1- 2 548. Designemos la sum a dada por S. Transformemos los
cos x cos y cos z dos primeros sumandos de la forma siguiente:
340 PROBLEMAS DE MATEMATlCAS ELEMENTALES

COS 2 2x 2
2
cotg 2x - tg 2x =
2
~ - sen sen ~ 2x = S = -cos(a + ~)cos(a - ~)-cOS2 y +
sen~ 2x cos- 2x
+ cos (a + ~)cos y + cos (a - ~)cos y.
cos 4 2x - sen 4 2x cos 2 2x - sen 2
2x 4 cos 4x
sen 2 2x cos 2 2x 1 ~ sen 2 4x . Agrupando los sumandos en esta expresion, hallamos que
- sen~ 4x
4
Por consiguiente, s = -[cos (a - [3) - cos y] [cos (a + f3) - cos y] .
4 cos 4x ( ) 4 cos 4x (1- sen 8x ) . Por consiguiente,
S= 2 1- 2 sen 4x cos 4x = ")
sen 4x sen- 4x a-f3+y y-a+ R a+R+y a+R-y
S = 4 sen sen I-' sen I-' sen I-' .
2 2 2 2
Puesto que 1- sen 8x = 2 sen2(~ - 4x), entonces,
obtendremos definitivamente: 550. La expresion que se examina se puede transfonnar de
la manera siguiente (vease (13) pag. xxxx):
s = 8 cos 4x sen 2 (~- 4X) 1-4sen lOOsen 70° 1-2(cos 60°-cos 80°) 2cos80°
- -- - -- -- = = - -- -
sen 2 4x 2 sen 10° 2 sen 10° 2 cos 80°
549. Designemos la expresion que se exam ina por S . As! pues,
Transformemos los dos primeros sumandos por la formula
(16) pag. 69, y sustituyamos el producto cos a cos f3 por la - - - - - 2 sen 70° = 1.
2 sen 10°
suma de la formula (14) pagina 69, y por fin, sustituyamos
sen 2 y por 1- cos 2 y. Entonces, obtendremos: 551. En virtud de la fonnula (12), expuesta en la pagina 69,
la parte izquierda de la identidad es igual a

s = - ~ (cos 2a +cos 2~)-cOS2 y+ 1t 31t


2 2 sen - sen - . (1)
+ [cos (a + ~)+cos(a- ~)]cos y. 10 10

Transfonnando la suma cos 2a + cos 2[3 en un producto Multiplicando y dividiendo (1) por 2 cos l~o cos ~~ y
y abriendo los corchetes, obtenemos: haciendo uso de la formula para sen 2a, obtendremos:
TRlGONoMETRIAiResoluciones y Soluciones 341

7t 37t De aqui se deduce que la suma examinada es igual a --t.


7t 37t sen - sen-
2 sen-sen - = 5 5. 553. Empleando para todos los sumandos de la suma S que
10 10 7t 37t
2 cos - cos- se exam ina, al principio la formula (16), Y a continuacion la
10 10
Pero, (17) pagina 69, hallaremos que

3 1( 7t 37t
S = 2 - 2 cos 8" + cos 8 + cos 8 + cos 8
57t 77t\
j+
y

cos
37t (7t 37t\ 7t
10 = sen" 2 -10) = sen 5·
l(
8
7t + cos -37t + cos -57t + cos -77t) .
+ - cos-
4 4 4 4

Por consiguiente, la parte izquierda de la identidades igual Las sumas entre parentesis son iguales acero, puesto que
1
a 2.
7t 77t 37t 57t
cos - =-cos - cos - =-cos -
552. Multiplicando y dividiendo la parte izquierda de la 88' 8 8 '
identidad por 2 sen ~ y valiendonos de las formulas que 7t 37t 57t 77t
expresan el producto de funciones trigonometricas por las cos- =-cos- cos- = -cos-.
4 4' 4 4
sumas, hallaremos:
Por consiguiente, S = t.
2rr 4rr 6rr
cos - + cos - + cos - =
777 554. Si en la identidad
2rr rr 4rr rr 6rr rr
2 cos - sen - + 2 cos -sen - + 2 cos - sen-
7 7 7 7 7 7 tg a tg(60° - a}tg(60° + a} = tg 3a. (1)
2 sen ~
7 hacemos a = 20° (vease el problema 536), entonces,
3rr rr 5rr 3rr 5rr
sen - - sen - + sen - - sen - + sen rr - sen -
obtendremos directamente que
7 7 7 7 7
2 sen ~ tg 20° tg 40° tg 80° = ·J i (2)
7
342 PROBLEMAS DE MATE MAn CAS ELEMENTALES

Expongamos otra resolucion sin emplear la formula (1). De (3) y (4) se desprende (2).
Transformemos a parte el producto de los senos y los cosenos.
Empleando las formulas (13) y (15) pagina 69, obtendremos:
2. Ecuaciones trigonometricas y sistemas
I de ecuaciones
sen 20 0 sen 40 0 sen 80 0 = - (cos 20 0 - cos 60° )sen 80 0 =
2
= ~(sen 100 + sen 60 0 0
_ ~sen 80 0 ) •
555. La ecuacion puede escribirse asi:
2 2 2
4 sen x cos x (sen 2 x - cos 2 x) = I
Observando que
o bien
sen 100 0 = sen 80 0 ,

- 2 sen 2x cos 2x = -sen 4x = 1.


hallamos:
Respuesta:
sen 20 0 sen 40 0 sen 80 0
J3.
=- (3)
8 x = -~ + k· ~ (k = 0, ± I, ± 2, ... ).
8 n
Luego,

cos 20 0 cos
400
cos
800
= 2 sen 20° cos 20° cos 40° cos 80°
2 sen 20°
=
556.
x = ~ + fat y x = -*
La ecuaclOn pierde el sentido cuando
+ fat, para los demas valores de x es
equivalente a la siguiente:
sen 40° cos 40° cos 80° sen 80° cos 80° sen 160°
2 sen 20° 4 sen 20° 8 sen 20° cos--
x - sen x 1
- -- = + sen 2 x
sen 20° 1 cos x + sen x
8 sen 20° 8
Oespues de simples simplificaciones, obtenemos:
Asi pues,
1 sen x(3 + sen 2x + cos 2x) = o.
cos 20 0 cos 40 0 cos 80 0 = -. (4)
8
TRIGONoMETRiAlResoluciones y Soluciones 343

La ecuaci6n sen 2x + cos 2x + 3 = 0, evidentemente, no


tiene raices, por eso, la ecuaci6n inicial se reduce a la ecuaci6n x, = 2: (_I)k+1 + 1m
~ 6 '
sen x =0.
Respuesta: x = 1m. 559. Escribamos la ecuaci6n dada en la forma

557. La ecuaci6n se puede escribir en la forma siguiente:


[-sen
I \/3
~ ,2 1 n 5
2x + -cos 2x - -COS(2X - - ) - - = 0
(sen x + cos x) 2 + (sen x + cos x) + (cos 2 X - sen 2 x ) = 0 2 2 ) 4 6 4

o bien o bien

(sen x + cos x) (I + 2 cos x) = o.


Igualando cada uno de los parentesis acero, hallamos todas
las raices. n 2n Resolviendo la ecuaci6n cuadrada (1), hallamos:
Respuesta: XI = -"4 + 1m, x 2 = ±3 + 21m.

558. Escribamos la ecuaci6n dada de la siguiente manera: cos( 2x - ~ ) = -1, x=-+Im.


12
7n

sen x + I - cos 2x = cos x - cos 3x + sen 2x.


La segunda rafz de la ecuaci6n cuadrada (1), igual a ~, no
Despues de las simplificaciones comprensibles, obtene-
da soluci6n, puesto que I cos a i::::; 1.
mos:
560. Dividiendo ambos miembros de la ecuaci6n por 2, la
sen x + 2 sen 2 x =2 sen 2x· sen x + sen 2x
reduciremos a la forma
y, por consiguiente, n\
sen 17 x + sen ( 5x + - 1=0,
sen x{1 + 2 sen x)(l- 2 cos x)= o. 3)

Respuesta: de donde
344 PROBLEMAS DE MATEMAnCAS ELEMENTALES

El primer parentesis no se reduce a cero. Por eso, es


2 sen (IIX + ~ ) cos ( 6x - ~ ) = o. suficiente examinar la ecuaci6n sen x + cos x-I = O. Esta
ultima ecuacion se reduce a la forma
7t 1m 7t (2k + 1)7t
Respuesta: xI = - 66 + 11' x2 =-+
36 12
.
7t
561. La ecuacion dada pierde el sentido cuando cos x = 0; Respuesta; XI = 27tk, x~ = - + 27tk.
*
por eso, se puede considerar que cos x O. Observando que - 2
el segundo miembro de la ecuacion es igual a 563. Empleando las formulas conocidas, escribamos la ecua-
3 sen x cos x + 3 cos 2 x, y dividiendo ambos miembros por cion dada en la forma siguiente:
cos 2 x, obtendremos:
~
cosec-x - sec 2 x - cotg 2 x - tg-x
?
- cos 2 x - sen-x
?
= - 3. (1)

Puesto que
o bien
cosec 2 x = 1+ cotg 2 x ~
y sec- x = 1+ tg 2x,
7t 7t 7t la ecuacion (1) se reduce a la forma
Respuesta: Xl = -"4 + 1m, x2 = -::;- + 1m,
.)
X3 =--+1m.
3
562. Valiendonos de la formula para la suma de los cubos de
dos numeros transformemos el primer miembro de la ecua- 7t 7t
Respuesta: x = - + k -.
cion de la siguiente manera: 4 2

(sen x + cos x) (I - sen x cos x) = (I - ~ sen 2X) (sen x + cos x). 564. Utilizando la identidad

Por consiguiente, la ecuacion inicial toma la forma

( 1- ~ sen 2X) (sen x + cos x-I) = O.


~ 2
TRlGONoMETRiAiResoluciones y Soluciones 345

transformemos la ecuacion a la forma


la ecuacion (I) toma la forma
22x 3
sen - =-.
3 4 k[cos(x - a)- cos (x + a)] = cos(x - a)- cos(3x - a)
3n± I o bien
Respuesta: x = -2-n (n = 0, ± 1, ± 2, ... ).

565. Utilizando la identidad, que figura en la resolucion del k sen x sen a = sen (2x - a) sen x. (2)
problema anterior, obtendremos la ecuacion
La ecuacion (2) se descompone en dos:
sen 2 2x+sen2x-I=0. a) sen x = 0; entonces x = In;
Resolviendo esta ecuacion, hallamos: b) sen (2x -a) = k sen a;

-IS -1
sen2x=--.
entonces
2

k 1 -IS -I kn
Respuesta: x = ( -1 ) - arcsen - - + - . Para que la ultima expresion tenga sentido, k y a debenin
2 2 2 estar enlazadas por la condicion

566. Escribamos la ecuacion dada en la forma !k sen a! ~ 1.


(1 +k) cosx co~2x-a)=cos(x-a)+k cos2x co~x-a). (1) 567. Dado que los numeros a, b, c y d son los terminos
sucesivos de una progresion aritmetica, entonces se puede
Dado que hacer b = a + r, c = a + 2r, d = a + 3r (r es la diferencia de
la progresion). Empleando la formula
cos x cos(2x - a) = ~[cos(3x - a)+ cos (x - a)],
2
sen a sen f3 - ~ [cos (a - (3)- cos (a + (3)].
cos (x - a) cos 2x = ~[cos(3x - a)+ cos(x - a)]. 2
2
346 PROBLEMAS DE MATEMAnCAS ELEMENTALES

representemos la ecuaci6n en la forma


2X ,x X x
3 cos - + 2 sen - - + sen - cos - =0
cos(2a + r)x -cos(2a + 5r)x = 0 2 2 2 2

o bien es equivalente a la siguiente:


,x x
sen(2a + 3r)x· sen 2r x = 0, 2 tg~ - + tg - + 3 = 0
2 2
de donde
y no tiene soluciones reales.
1m 1m
Xl = 2a + 3r ' x 2 = 2r . 1t
Respuesta: x = "2 + 21m.
Las f6rmulas escritas tienen sentido, puesto que
569. Primera resoluci6n. La ecuaci6n pierde el sentido cuando
2a + 3r = b + c > 0 y r =1= O. x = 1m, y para los demas valores de x es equivalente a la
siguiente:
568. Escribamos la ecuaci6n dada en la forma siguiente:
= 2 sen 2x· sen x. (1)

l
cos x - sen x

2 X 2 X
sen -X
2
I Sustituyendo el producto que figura en el segundo miembro
cos "2 -sen "2 = 2 ---
x -I
cos -
2
j de (I) por la suma, segun la f6rmula (13) pagina 69,
obtenemos:

cos x - sen x = cos x - cos 3x, sen x = cos 3x,


0, despues de simples transformaciones, en la forma
de donde

= sen (~ - 3x )
l, x
2
X
cos- - sen-
2
)(3 cos-,x-2 + 2 sen-?x-2 + sen -cos-
2
x
2
J= O.
X sen x

y, por consiguiente,
La ecuaci6n
TRlGONoMETRiAiResoiuciones y Soiuciones 347

Observacion: Las dos ultimas series de soluciones se


2 sen ( 2x - ~ ) cos( x - ~ J= 0. pueden escribir mediante una sola formula (2).

Respuesta: 570. Aplicando el primer miembro de la ecuacion la formula


1t 1m (14), pagina 69, obtenemos:
x =-+ -
I 8 2'
(2)
cos(2x -(3)+ cos 13 = cos 13,
31t
x2 = - + 1m.
4 de donde

Segunda resoluci6n. Empleando la formula (20), pagina cos(2x -(3) = 0.


69, y suponiendo que sea tg x = t, obtenemos la ecuaci6n
Por consiguiente,
t3 + 3t 2 +t - 1 = 0.
Descomponiendo el primer miembro en factores, obtene-
mos:
571. La ecuaci6n inicial se puede escribir en la forma
(t + l)~ + I--J2)V + 1 + -J2)= 0,
sen a + [sen(2<p + a) - sen{2<p - a)] = sen(<p + a) - sen(<p - a),
de donde,
0, despues de simples transformaciones, en la forma
(tgx)l=-I, (tgx),=-J2-I, (tgx)3=-I--J2.
sen a + 2 sen a cos 2<p = 2 sen a . cos <po
Respuesta:
°
Suponiendo que sen a -::f::- (en el caso contrario cos <p es
Xl = 31t + 1m; x 2 = arctg (.)2 - I) + k1t; indeterminable), obtenemos:
4
X3 = -arctg (I + -J2) + 1m. 1 + 2 cos 2<p - 2 cos <p = 0, 4 cos 2 <p - 2 cos <p - I = 0,
1±J5
cos co = ---- .
, 4
348 PROBLEMAS DE MATEMAnCAS ELEMENTALES

Puesto que el angulo <p se encuentra en el tercer cuadrante, 573. Utilizando las f6rmulas (18) Y(19), pagina 69, reducimos
entonces, cos <p < O. Por consiguiente,
la relaci6n dada sen a + cos a = -:; a la forma
1-15
cos<p = - -- .
4 (2 +.J7) tg 2 a - 4tg a - (2 -.J7) = O.
2 2
572. Empleando la f6rmula cos 2 <p = l+C~S 2q> , escribamos la Resolviendo esta ecuaci6n respecto a tgt, obtendremos:
ecuaci6n en la forma

cos 2(a + x)+ cos 2(a - x) = 2a - 2 (tg a) =


2 1
3.J7 =.J7 - 2
2+ 7
y
o bien

cos 2a cos 2x = a-I,


.J7 -2
3
de donde
a-I Comprobemos si satisfacen los valores hallados de tg t
cos 2x = . (1) las condiciones del problema.
cos 2a
Puesto que 0 < t < t,
entonces,
Puesto que, por otra parte,
o< tg -a 7t
< tg -
r;;
= -v 2 - 1.
cotg x =± II+ cos 2x , 2 8
~ 1- cos 2x EI valor
entonces, de (1) hallamos que ) 7 -2
3
r- - ---
ra-l + cos 2a
cotg x = ± 1-- -- -- satisface la condici6n del problema, ya que ,173-2 < -Ii -1 .
'v1- a + cos 2a La raiz .J7 - 2 debe suprimirse, puesto que
De la f6rmula (1) se desprende que el problema tiene
sentido cuando cos 2a * 0 y ! cos 2a ~ I a-I ! .I .J7-2>-li-1.
TRlGONoMETRiAiResoluciones y Soluciones 349

574. Haciendo sen x - cos x = I Y valiendonos de la identi- Esta ecuacion es del mismo tipo que la del problema 574.
dad (sen x - cos x)2 = 1- 2 sen x cos x, escribamos la ecua- Sustituyendo sen x + cos x = I, la ecuacion (l) se lleva a la
cion inicial en la forma ecuacion cuadrada 12 + 41 + 3 = 0, cuyas ralces son
I, = -1 Y 12 = -3. Puesto que I sen x + cos xl:s;; J2, a la
12 + 121 -13 = O. ecuacion inicialla pueden satisfacer solamente las rakes de
la ecuacion
Esta ecuacion tiene las rakes I, = -13 Y 12 = 1. Pero,
sen x + cos x = -1 (2)
l=senx-cosx=J2sen(x-~), dedonde I 11:s;; J2, por
Resolviendo la ecuacion (2), obtenemos:
consiguiente, la raiz I, = -13 puede no examinarse. Por esta
n:
razon, la ecuacion inicial se reduce a la siguiente: Xl = -- + 21m,
2
x 2 = (2k + 1)n:.

La segunda serie de rakes se debe despreciar, puesto que


sen x2 = 0 y la ecuacion inicial pierde el sentido.
n: n:
Respuesta: x, = n: + 21m, x2 = 2" + 21m. Respuesta x = - -2 + 2kn:.
.
575. Transformemos la ecuacion dada a la forma 576. La ecuacion dada pierde el senti do cuando x = 1m, y
cuando x"* 1m puede ser escrita en la forma
2 cos 2 ~(2 + sen x)+ sen x = O.
2
Utilizando la formula 2 cos 2 ~ = I + cos x y abriendo los
. . 0 btenemos: 2 Pasando todos los terminos de la ecuacion a la parte
parentesls,
izquierda y descomponiendolos en factores, obtenemos:
2 + 2 {sen x + cos x)+ sen X· cos x = O. (1)
(cosx-sen x) (sen 2 x+cos2 x+sen x cosx+sen x+cosx)= o.
350 PROBLEMAS DE MATEMATlCAS ELEMENTALES

De aqui se desprenden dos posibilidades:


a) sen x - cos x = 0, entonces,
2 (sen 3x cos2x - cos 3x sen 2x) cos 2x =
x
1
= ~4rr + 1m.' (I)
= sen 2x (sen 2x sen 3x + cos 2x cos 3x).

de donde x = krr.
b) sen 2 x + cos 2 X + sen x cos x + sen x + cos x = 0. (2)
578. La ecuaci6n dada se puede escribir en la forma
La ecuaci6n (2) es analoga ala examinada en el problema
574 y tiene las soluciones
3(COS 2x _ cos 3X) = sen 2x + cos 2x
rr sen 2x sen 3x cos 2x sen 2x
x2 = - ~ + 21m (3)
2
y o bien
X3 = (2k + I)rr (4) 3 sen x
sen 2x sen 3x sen 2x cos 2x
Pero, los val ores de x que figuran en la f6rmula (4), no son
ralces de la ecuaci6n inicial, puesto que, siendo x = nrr la Observemos que esta ecuaci6n tiene sentido si
ecuaci6n in icial pierde el sentido. Por consiguiente, la ecuaci6n
tiene las rakes determinadas por las f6rmulas (I) y (3). sen 2x;;t:. 0, sen 3x;;t:. 0, cos 2x;;t:. 0.

577. Escribamos la ecuaci6n de la manera siguiente: Para los val ores de x, para los cuales la ecuaci6n (1) tiene
sentido,
3 sen x cos 2x = sen 3x,

o bien
sen x (3 - 4 sen 2 x- 3cos 2x) = 0,
Reduciendo los quebrados a un comun denominador y o bien
liberandonos de este ultimo, obtendremos la ecuaci6n 2 sen 3 x = 0.
TRlGONoMETRiAiResoluciones y Soluciones 351

Puesto que la ultima ecuaci6n es equivalente a la ecuaci6n Durante la resoluci6n se realiz61a multiplicaci6n de ambos
sen x = 0, entonces, en virtud de la observaci6n hecha mas miembros de la ecuaci6n por cos x cos 2x sen 3x. Pero, es
arriba, la ecuaci6n inicial no tiene rakes. facil ver, que para ninguno de los valores hallados de x este
producto se reduce a cero. Por consiguiente, todos los valores
579. Escribimos la ecuacion en la forma hall ados de x son rakes de la ecuaci6n inicial.

6(tg x + cotg 3x) = tg 2x + cotg 3x, 580. Reduciendo los quebrados que figuran en el segundo
miembro de la ecuaci6n a un comun denominador y
despues de 10 cualla transformamos de la siguiente manera:
empleando la f6rmula

6 (sen~ + cos 3X) = sen 2x + cos 3x as _b 5 =(a-b)(a 4 +a 3b+a 2 b 2 +ab 3 +b 4 ),


cos x sen 3x cos 2x sen 3x
obtenemos:
o bien
sen x cos x{sen x - cos x) (sen 4 x + sen 3 x cos x +
6cos2x cosx
+sen-" x cos- x+sen x coso'x+cos
4 x ) =sen x-cosx.
cos x sen 3x cos 2x sen 3x '
6 cos 2 2x = cos 2 x; De aqul se desprende, que a bien
12 cos 2 2x - cos 2x -1 = 0.
sen x - cos x = 0. (I)
Resolviendo la ultima ecuaci6n, hallamos:
o bien
1± 7 {4 3 '
cos2x=--, sen x cos x\sen x + sen x cos x + sen x coso x +
24
cos 4 X + sen 2 x cos 2 x )-1 = 0, (2)
de donde
I 1 1 Transformemos, ahora, la ecuaci6n (2), aprovechando que
1) cos 2x =-.., , x = ±--arccos- + 1m'
-' 2 3 '
sen 4x + cos 4x = (2
sen x + cos 2x )2 - 2 sen 2x cos-' x,
2) cos 2x = - --,
4
1
x = ±larccos( -l) + 1m.
352 PROBLEMAS DE MATEMATICAS ELEMENTALES

yque, 2 X 2X
sen --cos -
x x 2 2_ 2 cos x
sen 3 X COS X + COS 3 X sen X = sen X COS X. tg- - cotg - =
2 2 x x sen x
sen-cos-
Haciendo, ademas, en la ecuaci6n (2) sen X cos X = y, 2 2
escribamos la ecuaci6n (2) en la forma
3 2
i
Por consiguiente, si x"* krt y x"* + m1t (k y m son
Y -y -y+l=O (3) numeros enteros arbitrarios), el segundo miembro de la
ecuaci6n es igual a - 2 sen x cos x.
o (despues de descomponer el primer miembro en factores)
en la forma
*
El primer miembro de la ecuaci6n no tiene sentido cuando
i i
x = + krt y x = + I· (i = 0, ± 1, ± 2, ... ), Y para los
demas val ores de x, es igual a la - tg x, puesto que

Si Y = I, es decir, si sen X cos X = I, entonces, sen 2x = 2,


10 que es imposible. Si y = -I, entonces, sen 2x = -2, 10
tg (x - ~ Jtg ( x + ~ J= tg ( x - ~ Jcotg [ %- (x + ~ J]=
que es tambien imposible.
Asi pues, la ecuaci6n (2) no tiene rakes. Por consiguiente, = tg ( x - ~ Jcotg ( x - ~ J= -1.
(1), es decir, x *
las raices de la ecuaci6n inicial son las rakes de la ecuaci6n
= + 1tn. i
Asi pues, si x"* + nm y x"*
ecuaci6n inicial tiene la forma
* i,+ entonces la

581. EI primer miembro de la ecuaci6n pierde el sentido


i
cuando x = k1t Y cuando x = + m1t, puesto que siendo
tg x = 2 sen x cos x.
x = 2lJr la funci6n cotg~ es indeterminada, siendo Esta ecuaci6n tiene las rakes
x = (21 + 1)1t es indeterminada la funci6n tg y siendo 1, 1t 1t
i
x = + m1t, el denominador del segundo miembro se hace x=krt y x=-+I-.
4 2
igual a cero. Si x"* krt tenemos:
De aqui se desprende que la ecuaci6n inicial no tiene rakes.
TRlGONoMETRiAiResoluciones y Soluciones 353

582. Multiplicando el segundo miembro de la ecuaci6n por En este caso, la ecuaci6n (1) se escribe en la forma
sen 2 x + cos 2 X = 1, la llevamos a la forma

(I-a)sen 2 x -sen x cos x-(a + 2)cos 2 x = 0. (1)


cos x (sen x + 3 cos x) = °
y tiene las soluciones siguientes:
"*
Supongamos, al principio, que a l. Entonces, de (1) se
rr
"*
desprende que cos x 0, puesto que, en el caso contrario, xl = - + 1m, x2 = -arctg 3 + 1m.
tendrfamos que sen x = cos x = 0, 10 cual es imposible. 2
Dividiendo ambos miembros de (1) por cos 2 x y haciendo 583. Empleando las f6rmulas
tg x = t, obtendremos la ecuaci6n

(I-a)t 2 -t-(a+2)=0. (2) 4 _ ( 1 - cos 2X) 2 2 1 + cos 2x


sen x -I ' cos x = - - - -
2
La ecuaci6n (1) tiene soluci6n cuando, y s6lo cuando, las " 2
rakes de la ecuaci6n (2) son reales, es decir, cuando su y haciendo cos 2x = t, escribamos la ecuaci6n dada en la
discriminante es forma siguiente:

D = 4a 2 - 4a + 9 ~ 0. (3) t2 - 6t + 4a 2 - 3 = 0. (1)

Resolviendo la desigualdad (3), hallamos: La ecuaci6n inicial tendni soluci6n solamente para tales
valores de a, para los cuales las rakes t, y t2 de la ecuaci6n
(1) sean reales y, por 10 men os, una de estas rakes, por su
-
-JiO + I -::;a-::; -JiO - 1. valor absoluto, no sea mayor que la unidad.
(4)
2 2 Resolviendo la ecuaci6n (I), hallamos:
Sean t, Y t2 las rakes de la ecuaci6n (2). Entonces, las
correspondientes soluciones de la ecuaci6n (1) tienen la forma tl =3-2'J3-a-,
r:::-J
t2 =3+2')3-a .
r-2

Xl = arctg tl + 1m, X2 = arctg t2 + krr. Por consiguiente, las rakes de la ecuaci6n (1) son reales si

Examinemos ahora el caso en que a = 1. (2)


354 PROBLEMAS DE MATEMAnCAS ELEM ENIALES

Si se cumple la condicion (2), entonces 12 > I, y por esta 584. Transformemos la ecuacion dada, multiplicando ambos
razon, esta raiz puede ser despreciada. De este modo, el miembros por 32 sen ~. Empleando unas cuantas veces la
problema se reduce a la determinacion de aquellos valores 1
formula sen a cos a = sen 2a, obtendremos:
de a que satisfacen la condicion (2), para los cuales III I:::; 1,
es decir, para los cuales 32 1tX
sen - 1tX = sen-
31 31
o bien
- 1 :::; 3 - 2 -J 3 - a 2 :::; 1. (3)
1tX 33
De (3) hallamos que sen - COS - 1tX = 0. (1)
2 62
-4:::;-2 -J3-a 2 :::;-2, De aqui hallamos dos series de raices:

x2 = -31 (2n + I)
de donde
(n = 0, ± 1, ± 2, ... ) .
33
2 ?-J3 - a 2 ? 1. (4) Puesto que durante la resolucion se realizo la multi plica-
cion de ambos miembros de la ecuacion dada por el factor
32 sen ~~, que puede reducirse acero, la ecuacion (1) puede
Puesto que la desigualdad 2? ~3 - a 2 se cumple siendo
tener raices ajenas para la ecuacion inicial. Seran raices ajenas
Ia 1:::;J"3, entonces, el sistema de desigualdades (4) se reduce unicamente las raices de la ecuacion
a la desigualdad
1tX
sen - =0. (2)
-J3-a 2 ?I, 31
que no satisfagan a la ecuacion inicial.
de donde hallamos que I a I :::;-fi.
Las raices de la ecuacion (2) se dan por la formula
Asi pues, la ecuacion inicial es soluble si I a I :::; -fi, y tiene
la solucion x = 31k (k = 0, ± I, ± 2, . .. ) (3)

y, como es f£lcil ver, no satisfacen a la ecuacion inicial. Por


esta razon, de la serie hallada de raices de la ecuacion (1),
TRiGONoMETRiAIResoluciones y Soluciones 355

deben ser excluidas todas aquellas que tienen la forma (3).


Para las rakes de la primera serie esto conduce a la ecuaci6n
2n = 31k, que es posible solamente para los valores pares
dek, es decir, para k = 21 y n = 311 (I = 0, ± I, ± 2, ... ). Para Pero, sen a= sen 13 cuando, y s610 cuando, 0 bien
las rakes de la segunda serie amilogamente obtenemos la a -13 = 2kn, bien a + 13 = (2m + 1)1t (k, m = 0, ± 1, ± 2, ...) .
0
igualdad ~j (2n + 1) = 31k 0 bien 2n + 1 = 33k, que es posible Por consiguiente,
unicamente cuando k es impar, 0 sea, cuando
k=2/+1 y n=33/+16(/=0,±I,±2,... ). Para las rakes 1t 1t
-+ 7x- - -5x = 2kn
de la segunda serie amilogamente obtenemos la igualdad 6 3
o bien
*(2n + 1) = 31k 0 bien 2n + 1 = 33k, que es posible
unicamente cuando k es impar, 0 sea, cuando k = 21 + 1 y
2: + 7 x + 2: + 5x = (2m + 1)1t.
n=33/+16(/;t:O, ±I, ±2, ... ). 6 3

As! pues, las rakes de la ecuaci6n inicial son: As! pues, las rakes de la ecuaci6n serlin:

Xl = 2n, donde n ;t: 31/, } X = ~(12k + 1),1


31 I = 0, ± 1, ± 2, ... 12
(k, m = 0, ± I, ± 2, ... ).
x2 = -(2n + 1), donde n;t: 331 + 16.
33 x =..2:.(4m+
24
I).J
585. Escribamos la ecuaci6n de la manera siguiente:
586. Puesto que el primer miembro de la ecuaci6n es igual a
1 -J3 -J3 1
-cos 7x+-sen 7x= - cos5x+-sen 5x,
2 2 2 2 2 - (7 + sen 2x) (sen 2 X - sen 4 x) =
o bien
= 2 - (7 + sen 2x) sen 2 X cos 2 X = 2 - (7 + sen 2x )3. sen 2 2x,
1t 1t 1t 1t 4
sen - cos 7x + cos-sen 7x = sen - -cos 5x + cos - sen 5x,
haciendo t = sen 2x, escribamos la ecuaci6n en la forma
6 6 3 3

es decir, (I)
356 PROBLEMAS DE MATE~I."TICAS ELEMENTALES

La ecuaci6n (1) tiene la raiz evidente II = 1. Sus otras dos Puesto que siempre Isen (x + <p )1
~ 1, esta ecuaci6n tiene
rakes se hallan de la ecuaci6n
soluci6n si, y s610 si, lei
~ ,j a 2 + b 2 0 si c 2 ~ a 2 + b 2. Esta
t 2 + 8t + 8 = O. (2) es precisamente la condici6n de resolubilidad del problema.
Estas rakes son iguales a A continuaci6n, hallamos:

-4+2-J2 Y -4-2-fi.
Estos valores se pueden despreciar, puesto que en valor
absoluto son mayores que la unidad. Por consiguiente, las
Observando que
rakes de la ecuaci6n inicial son las rakes de la ecuaci6n
sen 2x = 1.
sen x = sen (x + <p - <p) = sen (x + <p )cos <p - cos (x + <p )sen <p,
11
Respuesta: x = 4" + 1m. cos x = cos (x + <p - <p) = cos (x + <p )cos <p + sen (x + <p )sen <po

y colocando (2) y (3) en la parte derecha de la expresi6n (1),


"*
587. Se puede considerar que a 2 + b 2 0, puesto que, en el
definitivamente obtendremos dos soluciones:
caso contrario la ecuaci6n toma la forma c = 0 y no permite
hallar sen x y cos x. Es conocido, que si a 2 + b 2 0, "* ~--

entonces, siempre existe un angulo <p, 0 ~ <p < 211, tal, que ) _bc-a'ia2+b2-c2
a sen x - 2 2 '
a b a +b
(1)

Dividiendo esta ecuaci6n miembro a miembro por


~a2 + II y utilizando (1), obtenemos la siguiente ecuaci6n
equivalente:
(2)
TRlGONoMETRiAlResoluciones y Soluciones 357

588. Observando que (b cos x + a) (b sen x + a) 7= 0 (en el 7(

caso contrario, la ecuacion pierde el senti do ), nos liberamos Respuesta: x = - + 1m.


4
de los denominadores. Como resultado obtenemos:
589. Valiendonos de la identidad

ab sen 2 x + (a 2 + b 2 )sen x + ab = 6
cos x = l I + cos2 2X)
( 3 I( 2
= 8" 1 + 3 cos 2x + 3 cos 2x + cos 2x,
3)

= ab cos 2 x + (a 2 + b 2 ) cos x + ab,


y de la formula
de donde
cos 6x = 4 cos 3 2x - 3 cos 2x (vease (8), pagina 69),

lIevamos la ecuaci6n a la forma


y la ecuacion se descompone en dos:
4 cos 2 2x + 5 cos 2x + 1 = O.
7(
la. sen x = cos x, de donde x = - + 1m De (1) hallamos q-=-
ue=---_ _ _ _ _ __
4
y
1
(cos 2x)\ = -1, (cos 2x), = - - .
a a2 + b2 - 4
2 . sen x + cos x = - - -
ab Respuesta:
Pero la ultima ecuacion no tiene soluciones, puesto que ( 1 ')
a 2 +b 2 •
~ ?: 2, mlentras que
x\ = lk + 2 ' - )7('

x2 = ±~arccoi -~) + 1m.


2 \ 4
590. Empleando las formulas

, 1- cos 2a 2
sen - a = - y cos 2a = 2 cos a-I,
2
358 PROBLEMAS DE MATEMAnCAS ELEMENTALES

representemos la ecuacion en la forma la ecuacion (I) toma la forma

(I - cos 2x)3 + 3 cos 2x + 2 (2 cos 2 2x -1)+ 1 = 0, 4 cos 2 2x = 0.


o bien de don de
2 ~
7 cos 2x - cos" 2x = 0, 1t 1t
cos 2x = 0, x=-+ - n.
de donde 4 2
1t 1t
cos 2x = 0,
x= - +k -.
4 2
x
592. Utilizando la identidad (sen 2 + cos 2 x)2 = 1, obten-
dremos:
591. De las formulas para sen 3x y cos 3x, hallaremos:
4 4 1 2
sen x + cos x = 1- - sen 2x,
3 cos 3x + 3 cos x 3 3 sen x-sen 3x 2
cos x = , sen x = . de dOllde
4 4
2
Por consiguiente, la ecuacion se puede presentar en la 8 8 1 2 1 4 17
sen + cos x = ( 1 - - sen 2x ) - -- sen 2x = -
forma 2 8 32'

cos 3x(cos 3x + 3 cos x)+ sen 3x(3 sen x - sen 3x) = 0,


2 I 4 17 4 2 15
I-sen 2x+ - sen 2x=-,sen 2x-8sen 2x+ - =0.
8 32 4
o bien
Resolviendo la ecuacion bicuadrada obtenida, hallaremos:
3 (cos 3x cos x + sen 3x sen x)+ cos 2 3x -sen 2 3x = 0, ~ 1
2 7 1t 1t
sen 2x = 4± - , sen.) 2x =-, 2x=-+k - .
o bien 2 2 4 2

3 cos 2x + cos 6x = 0. (1) de donde


2k+ 1
x= - -1t.
Pero, puesto que 8
3 2 cos 6x + 3 cos 2x
cos x = -,
4
TRlGONoMETRiAlResoJuciones y Soluciones 359

1- cos 2x 0,despues de descomponer las sumas de los senos en facto res,


593. Sustituyendo se11 2 x y cos 2 x por 2 y en la forma
3x 5x 2 x
1+ cos 2 x , 'b I ', I sen - sen 2x sen - cos x cos - = 0,
--2- - respectlvamente, escn amos a ecuaClOn en a 2 2 2

forma siguiente: Igualando cada uno de los factores acero, obtendremos


cinco series de soluciones
(
I - cos 2X) 5
+
(1 + cos 2x'\I
5 29 42
=-cos x,
2 2) 16
2111 2113
o bien l)x= - 11:' 2) x = 112 11:' 3)x= - 11:'
3 ' 2 ' 5 '

Haciendo cos 2x = y , despues de simples transforma-


ciones, obtenemos la siguiente ecuaci6n bicuadrada respecto donde 11 1, 11 2 , 11 3 , 114 Y 115 son numeros enteros arbitrarios,
ay: Observando, que las soluciones de las series 4) y 5) se
contienen en la serie 2), definitivamente obtendremos las
siguientes tres series de soluciones:
Esta ecuaci6n tiene solamente dos rakes reales: '1 11 11) 2113
I) X = ~ 11:' 2) x = -= 11:' 3)x= - 11:
YI 2 = ± 1, Por consiguiente, cos 2x = ± 1, de don de 3 ' 2 ' 5 '
donde 111' 112 Y 113 son numeros enteros cualesquiera,
x = ~ (2k + 1), donde k = 0, ± 1, ± 2, ' . ,
595. Primera resoluci6n. Cuando 11 = 1 la ecuaci6n se
594. Valiendonos de la identidad obtenida en el problema transforma en una identidad, Si 11 > 1, entonces, de la
261, escribamos la ecuaci6n inicial en la forma identidad

(sen x + sen 2x) (sen 2x + sen 3x) (sen x + sen 3x) = 0,


360 PROBLEMAS DE MATEMAnCAS ELEMENTALES

Con ayuda de la f6rmula (7), pagina 69, la ultima ecuaci6n


1 = (~en2 x + COS 2 X Y+ sen 2n x + (; )sen 2(n-l) x COS 2 X + ... se puede escribir as!:
... + C:Jsen 2 x COS 2(1I-1) x + COS 2n X,
sen y (4 sen 2 y -1)= O. (1)
en virtud de la ecuaci6n dada, obtenemos:
La ecuaci6n (1) tiene las siguientes soluciones:
(~)sen2(n-l)x cos 2 x + ... + (n:l)sen 2 x cos 2(n-l) x = o. k 1t k+I1t
YI == krr., Y2 == ( - 1) 6" + d, Y3 == ( - 1) - + rck.
6

b
Dado que ninguno de los sumandos es negativo, entonces,
·
o len sen 2 x = 0, 0 b·len cos 2 x = 0 y x ="2
11 k
.
Volviendo al argumento x, por la f6rmula x == 2y, 3: -
definitivamente obtendremos tres series de soluciones de la
ecuaci6n inicial
Segunda resoluci6n. Es evidente, que la ecuaci6n se
satisfuce six toma valores multiplos de l'
es decir, si x = k 1
(k es un numero entero). Demostremos que la ecuaci6n 3rc
Xl == - - 2krc
5 '

no tiene otras raices. Sea Xo 7= k ·1;


entonces sen 2 Xo < -1 Y
cos 2 Xo < 1, de donde se desprende que para n > 1 tendremos
que sen 2n Xo < sen 2 Xo y COS 2n Xo < COS 2 Xo y, por 10 tanto,
597. Puesto que i cos al:s; 1 y sen a ~ -I, entonces

sen2nxo + cos 2n Xo < sen2xo + cos 2 Xo = 1. Con esto el pro- i cos 4x - cos 2x I:s; 2, sen 3x + 5 ~ 4.
blema queda resuelto.
Asf pues, el primer miembro de la ecuaci6n no sobrepasa
3x
596. Hagamos ~~ -1 = y, entonces,
TC
--+-- =
10 2
de 4, el segundo miembro no es menor de 4. Por consiguiente,
I cos 4x - cos 2x I == +2 (y, entonces, 0 bien cos 4x = -I Y
= 7! - 3 (~~ -~) = 7! - 3Y la ecuaci6n toma la forma cos2x=l, o bien cos4x==1 y cos2x==-I) y sen3x==-1.
Examinemos los casos posibles.
sen 3 Y = 2 sen y.
TRlGONoMETRIAIResoluciones y Soluciones 361

a)cos4x=-1, x=(%+~)1t; ~
sen (x + ) sen 2x = 1. (1)
cos 2x = 1, x = 1tk;
Puesto que Isen a.1 ~ 1, entonces, (1) tiene lugar, si, 0 bien
1t 21t
sen 3x = -1, x=--+-I.
6 3
No hay rakes comunes. sen ( x + ~ ) = -1 Y sen 2x = -1,
1tn o bien
b) cos 4x = 1, X=_·

~
2 '
sen ( x + ) = 1 Y sen 2x = 1.
cos 2x = -1, x =(k + ~)1t;
Pero, las dos primeras ecuaciones no tienen rakes comu-
1t 2 4/-1 nes, y las dos segundas ecuaciones tienen las rakes comunes
sen 3x = -1, x = -- + -1t1 = - -1t.
6 3 6 x = ~ + 2kn. Por consiguiente, la ecuaci6n dada tiene las
Las rakes comunes son: rakes:

x= (2m + ~)1t, m= 0, ± 1, ± 2, ... x = 2: + 2kn.


4
598. Transformemos la ecuaci6n a la forma 599. Dividiendo la ecuaci6n dada miembro a miembro por 2
y observando que
I 1 I
- sen x + --- cos x = .
.fi .fi 2 sen x cos x
1t I
sen ( x+ - )=---
1
" 4 sen 2x obtendremos la ecuaci6n equivalente

o bien ~
sen( x + )sen 4x = 1.
362 PROBLEMAS DE MATEMATICAS ELEMENTALES

La ultima igualdad es posible solamente en el caso cuando A2 = (sen x + cosec x )2 =- 1 +


- 2-
2 + - -12- =
cos x sen x cos x sen x
sen(x+~)=±l y sen4x=±1.
- -::-- -c:-- +
2 4 4
+ - --
sen 2 x cos 2 x sen x cos x sen 2 2x sen 2x'
de donde

x =- ~ ± ~ + 2mr y x= 1( ~± + 2mn) ,
Cada uno de los sumandos que figuran en la parte derecha,
al aumentar x desde cero hasta -I se porta de la siguiente

donde n y m son numeros enteros. 19ualando ambos valores manera: al principio disminuye desde + 00 hasta
entre sf, despues de simplificar por re, obtenemos la ecuaci6n 4 (cuando 0 < x S; *), y luego aumenta desde 4 hasta + 00
1 11m
- - ± - + 2n = ± - + -
3 2 8 2'
(cuando* s; x < -I) ; para x = *' ambos sumandos adquieren
simultaneamente sus valores minimos, por consiguiente,
o despues de la multiplicaci6n por 24
tambien la suma tendra sus valores mfnimos cuando x = *.
12m - 48n = -8 ± 9.
Para cualesquiera valores enteros de m y n, la parte AI mismo tiempo, ')..2 = 8. Por esta raz6n, si 0 < x < -I,
izquierda es un numero entero par, y la parte derecha, un
entonces, ')..2 ~ 8, Y puesto que sec x y cosec x en el primer
numero impar (10 -17). La ultima igualdad, para valores
enteros de m y n, es imposible, 10 que habia que demostrar. cuadrante son positivos, entonces, ').. ~ 2-fi. La grafica de la
funci6n ').. (x) se muestra en la fig. 248.
600. Primera resolucion. El problema es equivalente al
siguiente: (.que valores puede tomar la funci6n
Segunda resolucion. Observemos en seguida que debe-
').. = sec x + cosec x, si el argumento x varia en los !imites de
O<x < 1!.? mos limitarnos a examinar solamente los val ores positivos
2 '
Examinemos la funci6n de ').., puesto que siendo 0 < x < -I, las funciones sec x y
cosec x son positivas. Transformando la ecuaci6n a la forma
TRlGONoMETRlAlResoluciones y Soluciones 363

sen x + cos x = A sen x cos x, entonces, la ecuaci6n

elevemos ambos miembros de esta ecuaci6n al cuadrado,


como resultado obtendremos:

1 + sen x cos x = 1..2 sen 2 x cos 2 x. tendra una soluci6n x tal, que 0 < x < ~. Esto ultimo satisfani
tambien a la ecuaci6n inicial, de 10 cual es facil convencerse.
Haciendo, ahora, sen 2x = Z, tendremos: Si no se satisface la desigualdad (2), no existe la soluci6n
A necesaria. As! pues, el problema se ha reducido a la resoluci6n
A2Z 2 -4z-4=0 , de la desigualdad (2). Liberando esta desigualdad del

de donde
U denominador, obtenemos facilmente que A ~ 2-fi.

601. Del sistema dado obtenemos directamente que


2 ±~4+ 41..2 x + y = 1m, x- y = 11[.
z\,2 = (1)
1..2
De aqu!
0 ~ ~ x
4 2 k+ 1 k-1
X=--1[ y =--1[.
Puesto que por la condici6n del Fig. 248 2 ' 2
problema 0 < x < ~, entonces
Segun lacondici6ndel problema O~k+/~2, 0~k-l~2.
Z = sen 2x > 0, y debemos tomar en la igualdad (1) el signa
A estas desigualdades las satisfacen los siguientes 5 pares
mas, es decir,
de val ores de k y I:
1) k = 0, 1=0; 2) k = 1, 1=0;
3) k = 1, I = -1; 4) k = 1, 1=1;
Si ahora conseguimos satisfacer la desigualdad
5) k = 2, 1=0.

Respuesta:
364 PROBLEMAS DE MATEMATICAS ELEMENTALES

x3 = 0, Y3 = re; X4 = re, Y 4 = 0; Xs = re, Ys = re. 2k + 2n+ 1 2k - 6n -3


X= re y = re.
602. Transformemos el sistema a la forma 4 ' 4

sen 2 x = 1+ sen x sen y,l 603. Elevemos ambas ecuaciones al cuadrado, sumemoslas
miembro a miembro y hagamos uso de la identidad
x = I + cos x cos yJ
(1)
cos 2
sen 6 x + cos 6 x = 1- i sen 2 2x
Sumando y sustrayendo las ecuaciones del sistema (I), 4
obtenemos el sistema
(vease el problema 533). Obtendremos: sen 2 2x = l. Si
cos 2x - cos (x + y) = a,} (2)
sen 2x = 1, entonces, 0 bien x = 1I4 + 21m ' o bien
1 + cos (x - y) = 0.
x = ~ + (2k + l)re. En el primer caso, del sistema inicial
La primera ecuaci6n del sistema (2) se puede escribir asf:
hallaremos que sen y = cos y = 12 y en el segundo caso,
~. Analogamente se examina el caso
sen y = cos y = ,,2
cos 2x - cos (x + y) = 2 sen ( 3X; y ) sen (y - x) = o.
cuando sen 2x = -1.
Respuesta:
Si sen(x- y)=O, entonces, x- y=lm. Pero, de la
segunda ecuaci6n del sistema (2) hallamos: re re
xI = - + 21m YI = -4 + 2/re·'
4 '
cos(x-y)=-I, x-y=(2n+l)re.
X2 = ~ + (2k + I )re, Y2 = ~ + (2/ + l)re;
Por consiguiente, en este caso, tenemos una cantidad 4 4
innumerable de soluciones: x - y = (2n + 1)re.
3re 3re
x3 = - + 21m, Y3 = -
4
+2Ire·
'
4
Si sen-(3X+V)
2 = , ° entonces, 3x + y = 21m. Pero 3
x4 = 4re + (2k + l)re, Y4 = ire + (2/ + l)re.
4
x- y = (2n + l)re. Por consiguiente,
TRIGONoMETRiAlResoluciones y Soluciones 365

604. La primera ecuacion se puede escribir en la forma


1t
X- Y = ±- + 2/1t.
4
sen (x+ y) = 1,
cos x cos y
de don de, en virtud de la segunda ecuacion, obtenemos: Si tiene lugar (2), entonces cos{x+y)=-1 y

.fi cos (x - y) = 3f, 10 cual, es imposible .


sen (x + y) = cos x cos y = -
2 As! pues, para la determinacion de x e y hemos obtenido
Por consiguiente, 0 bien el sistema de ecuacion
1t
x+ y=-+21m, (1) x+ y =-+21m,
4

~ ~+
o bien 1t }
(4)
x- y ± 21n.
x+y = - 7r + (2k + 1)7r. (2)
4
En concordancia con la eleccion del signa en la segunda
Transformemos la segunda ecuacion del sistema inicial ecuacion del sistema (4) obtenemos dos series de soluciones:
de la siguiente manera:

cos {x + y)+cos{x- y)=.fi. y


De aquf X2 = {k + /)1t,

cos{x- y)=.fi -cos{x+ y). (3) 605. Dividiendo miembro a miembro la primera ecuacion
por la segunda, obtendremos:

Si tiene lugar (1), entonces cos (x + y) = 1, y de la for- (1)


mula (3) hallamos:
366 PROBLEMAS DE MATEMATICAS ELEMENTALES

Sumando esta ecuaci6n a la primera y sustrayendo de (1) 1 1 J[

la primera ecuaci6n, obtendremos un sistema equivalente al


c) X3 = (k + I ) J[ - - arccos / + -,
2 2" 2 8
inicial:
Y = (/-k)J[ -~arccos -l- _ J[.
~,}
3 2 2-12 8'
cos(X- Y)=
d) x = (k + I)J[ _ ~ arccos-1- _ J[
cos(x+ Y)= 21-. Y
4

= (I -
2
1
k ) J[ - - arccos-- + -.
2-12
1
8'
J[

4 2 2-12 8
de donde 1t I
x - Y = ± - + 21m, ~

x +y = ± arccos 2~ 21~.J+
(2) 606. Transformemos la segunda ecuaci6n a la forma

~ [ cos (x + y) + cos (x - y) ] = a.
2
En concordancia con la elecci6n de los signos en la Pero puesto que x+ y =<p, entonces, co~x- y)=2a-cos<p.
ecuaci6n (2) obtenemos las siguientes cuatro series de Obtenemos el sistema de ecuaciones
soluciones:
1 1 J[ x + Y = <p, }
a) Xl = (k + I ) J[ + - arccos ~ + -,
2 2'\j2 8 x - Y = ± arccos (2a - cos <p)+ 1m.
Respuesta:
1 1 J[
Yl = (I - k ) J[ + 2. arccos 2-12 -"8; x =.! ± ~arccos(2a - cos <p)+ 1m,
2 2
1 1 J[
b) x? = (k + 1) J[ + - arccos ~ - - ,
Y =.! =+= ~arccos(2a - cos <p )-Im,
- 2 2'\j2 8
2 2
Y = (/- k)J[ + ~arccos --l- + J[.
2 2 2-12 8 ' con la particularidad de que a y <p debenin estar enlazados
par la condici6n 12a - cos <p i:::::: 1.
TRlGONOMETRiAIResoiuciones y Soiuciones 367

607. Puesto que el primer miembro de la primera ecuaci6n cos (x - 2y)= cos (x -n) = -cos x = 0,
del sistema no supera a la unidad, el sistema puede tener
soluci6n solamente cuando a = 0. Suponiendo que sea a = 0, sen (x - 2y) = sen (x -n) = -sen x = 0.
obtenemos el sistema Pero sen x y cos x no pueden ser al mismo tiempo iguales

sen x·cos 2y = I,} acero, puesto que sen 2 x + cos 2 x = 1. Es eVI'dente, que a
(en el caso contrario, cos (x - 2 y ) = sen (x - 2 y ) = 0).
"* °
(I)
cos x . sen 2 y = 0, Dividiendo la segunda ecuaci6n miembro a miembro por
la primera (en virtud de la observaci6n hecha mas arriba, tal
De la segunda ecuaci6n del sistema (I) se desprende que, division es posible), obtendremos:
o bien cos x = 0, 0 bien sen 2y = 0. Si cos x = 0, entonces, n
tg(x - 2y) = I, x-2y=-+k7t. (1)
n 4
para xl = 2 + 2mn, de la primera ecuaci6n haHamos que
Examinemos dos casos:
Yl =n:rr, Y para X2 =-f+2krc, obtenemos que a) k es un numero par. En este caso

Y2 = V+ 1)n. EI caso sen 2y = ° no da nuevas soluciones. cos(x-2y)= ~ =acos3 y, COSY=~a~ =Ie;


As! pues, el sistema de ecuaciones es soluble solamente en
el caso en que a = 0, y tiene las dos series de soluciones y = ± arccos Ie + 2mn.

siguientes:
Colocando este valor de yen (1), obtenemos:
n x = ± 2 arccos Ie + (4m + k)n + ~.
xl = - + 2mn, Yl = nn;
4
2

x2 = -~ + 2kn, Y2 = (I + lJn. b) k es un numero impar. Entonces, cos {x - 2y) = -


=a cos3 y,
Jz =

608. Observemos que cos Y no puede ser igual a cero. En y = ± arccos (- Ie) + 2mn.
efecto, si cos Y = 0, entonces Y = + k7t y 1
368 PROBLEMAS DE MATEMAnCAS ELEMENTALES

De (1) haHamos:
4 cos {x - y )cos (x + y) = 1 + 4 cos 2 (x - y) .
x = ±2 arccos{- 'A) + {4m + k)7t + 2:.
4 EI sistema inicial puede ser sustituido por el siguiente
EI sistema es soluble cuando a> ~. equivalente:

(1)
609. Elevando las relaciones dadas al cuadrado, obtendremos:
(2)

sen 2 x + 2 sen x sen y + sen 2 y = a 2 , (1 )


cos 2 x + 2 cos x cos y + cos 2 Y = b2 . (2) Comparemos ambos miembros de la ecuaci6n (1).
Tenemos:
Sumando y sustrayendo (1) y (2) miembro a miembro,
haHaremos: 14 cos a . cos (x + y) 1~ 41cos a 1.

2 + 2 cos (x - y) = a2 + b 2 , (3) Por otra parte, de la desigualdad (I ± 2 cos a)2 ;::: 0 se


cos 2 2x + cos 2y + 2 cos {x + y) = b 2 - a2 . (4) desprende que

La ecuaci6n (4) puede ser transformada a la forma 41 cos a 1~ 1 + 4 cos 2 a,


can la particularidad de que el signa de igualdad tiene lugar
2 cos(x + y)[cos{x - y)+ 1]= b 2 - a2 . (5) solamente en el caso en que 2 1cos a 1= 1. Por consiguiente,
De (3) y (5) hallaremos: el sistema (1) - (2) puede tener soluci6n solamente con la
b 2 _a 2 condici6n de que cos a =
1 1 -t.
cos (x + y)= 2 2·
a +b
Examinemos dos posibilidades:
610. Valiendonos de la f6rmula
I
a) cos a =-.
cos 2x + cos 2y = 2 cos{x + y)cos(x - y), 2
escribamos la segunda ecuaci6n del sistema en la forma De (1) haHamos que cos (x + y) = 1, es decir,
TRlGONOMETRiAlResoiuciones y Soiuciones 369

x+ y = 2k1t. (3) Examinemos dos casos posibles: cos a = 1 Y cos a = -1.


Si cos a = 1, entonces, de (1) se desprende que
Resolviendo el sistema (2) - (3), obtenemos:
a a
t = cos {x - y)= _!.
xI = - + krt, YI = krt - - . 2
2 2 Obtenemos el sistema:
1
b) cos a = - - .
2 x- y = ±11t + 2krt'}
Actuando analogamente, hallamos: x+ y = a,

del cual hallamos que


1t a _1t 1_ a
XI =± - +krt+- YI = +- - lUL + -.
3 2' 3 2
611. Este problema es analogo al anterior. Expongamos, sin Si cos a = -1, entonces, actuando amilogamente, obten-
embargo, una resoluci6n un poco diferente. Empleando la
dremos:
f6rmula (14), pagina 69, representemos la primera ecuaci6n
a 1t
del sistema en la forma
= -a - 1_-1t
2 2
x =krt+ - ±- y IlIL + -.
2 6' 2 6
4 cos 2 (x - y)+ 4 cos {x + y)cos{x - y)+ 1 = o.
612. Examinemos la primera ecuaci6n del sistema.
Haciendo cos{x - y) = t y valiendonos de que x + y = a, En virtud de la desigualdad (1), pagina 69, tenemos que
obtendremos la ecuaci6n
Itg x + _1_1 ~ 2,
4t 2 + 4t cos a + 1 = o. (1)
1 tg x I
Esta ecuaci6n tiene rakes solamente con la condici6n de
que D=16(cos2a-l)~O, es decir, cuando Icosal=l. con la particularidad de que el signa de igualdad tiene lugar
solamente en el caso en que tg x = 1 Y tg x = -1. Puesto
370 PROBLEMAS DE MATEMATICAS ELEMENTALES

que el segundo miembro de la primera ecuaci6n satisface la 3


condici6n
x = -TC + 2krc,
4
3
Y = --TC + 2nTC.
4

la primera ecuaci6n del sistema puede satisfacerse solamente °


613. Observemos que cos x ::F Y cos Y ::F 0, puesto que,
en el caso contrario, la tercera ecuaci6n del sistema no tiene
en los casos siguientes:
sentido. Por esta raz6n, las dos primeras ecuaciones pueden
ser transformadas a la forma
a) tg x = 1, } b)tgx=-I, }

sen(y+~ J= 1; (1) sen (y + ~ J= -1. (2) (a - 1)tg 2 x = 1- b,


(b - 1)tg 2 Y = 1 - a.
(1)
(2)

EI sistema (I) tiene las soluciones siguientes: Pero, a::F 1, puesto que si a = I, entonces, de (1) tendremos
que b = I, 10 que contradice a la condici6n a ::F b. Analogamente,
TC TC si b = 1, entonces, tambien a = I. Por consiguiente, (1) puede
xl = - + krc, YI = -4 + 21TC,. (3)
4 ser dividida miembro a miembro por (2).
Dividiendolas, obtendremos:
y el sistema (2), las soluciones:
Jr 3Jr
x,=--+mJr,
- 4
Y2 = - - + 2nJr. (4)
4

Es facil comprobar que las soluciones, determinadas por


las f6rmulas (3), no satisfacen a la segunda ecuaci6n del Convencemosnos, ademas, de que a::F o. En efecto, si
sistema inicial, y las soluciones obtenidas por las f6rmulas a = 0, entonces, de la segunda ecuaci6n tendremos que
(4), satisfacen a la segunda ecuaci6n (y, por 10 tanto, a todo sen y ::F 0, Y de la tercera tendremos que b = 0, es decir, que
el sistema) solamente en el caso en que sea m un numero a = b = 0, 10 cual es imposible.
impar. Suponiendo en (4) que sea m = 2k + 1, escribamos En virtud de esta observaci6n. de la tercera ecuaci6n
las soluciones del sistema inicial en la forma podemos hallar que
TRiGONoMETRiAlResoluciones y Solucioncs 371

sen a - A sen ~,}


(1)
A cos ~ = B cos a.
Asi pues,
Elevando cada una de estas ecuaciones al cuadrado y
haciendo las sustituciones por las formulas sen 2 a = 1- cos 2 a
y cos 2 fJ -1 = sen 2 fJ obtendremos el sistema

Si 12a = \\-b,
-a entonces a = b, 10 cual es imposible.
2 2 2
cos a + A sen ~ = 1,
~
l
(2)
S1· b
-;;
\- b
= -\-a' entonces a + b = 2ab. B2 cos 2 a + A2 sen 2 ~ = A 2.J

Respuesta: a + b = 2ab. Para a:f::. b, esta condicion es De aqui, cos 2 a y sen 2 )3 se hall an de la unica manera
suficiente para la solubilidad del sistema. cuando, y solo cuando, A2 V- B2):f::. 0; en este caso

614. La segunda relacion, en virtud de la primera, se puede II _A2


escribir asi: cos a = ±i/-- ''
\1- B-
A sen ~ B sen ~ Examinemos los casos particulares, cuando
cos a cos ~ A 2 (1-B 2 )=0. Si A=O, entonces, de (I) obtenemos que
cos a = ±1 y B = 0; en este caso cos a = ± 1, sen ~ queda
o bien
indeterminado. Si B2 = 1, entonces, de (2) obtenemos que
sen ~(A cos ~-B cos a)= O. A2 = 1, Y las ecuaciones dadas no contienen los panimetros
A y B; por eso, la tarea de expresar cos a y sen ~ en funcion
Esta relacion puede ser cumplida cuando sen ~ = 0 y, de A y B pierde el sentido.
entonces, tambien sen a = 0, cos ~ = ± 1, cos a = ± 1, 0 bien
cuando Acos~-Bcosa=O. En este ultimo caso, 615. De la segunda ecuacion deducimos que
obtenemos el sistema
372 PROBLEMAS DE MATEMATICAS ELEMENTALES

y
sen x = sen ( ~ - 2 y ) , 1 1t I
Y2 ::: -arctg -fi + n1t, x2 ::: - + 21m - 2 arctg r;:; - 21tn
2 \/2
y, por consiguiente, 0 bien
o bien
1t
X = - - 2y+ 21m (1)
2 '
o bien

(2)
y
Dirigiendonos a la primera ecuaci6n del sistema dado, en 1t 1
X2 ::: - + arctg r;:; + 2m1t,
el caso (1) hallamos: 2 \/2
2 1
cotg 2y = tg 3 y 0 1- tg Y = tg 3 y. Y2 = -arctg ----r= + 2n1t,
oJ2
2 tg Y

Resolviendo la ecuaci6n bicuadrada, obtenemos que donde m y 11 son numeros enteros cualesquiera.

tg y =± f. En el segundo caso, introduciendo x de la 616. Transformando ambos miembros de la primera ecuaci6n,


f6rmula (2) en la ecuaci6n tg x = tg 3 y, nos convencemos obtendremos:
de que no existen rakes reales. Asi pues, x + y ( cos -2- x + y) = 0.
x - Y - cos -2-
2 sen 2-
I 1t
tgy=±~ y x= -2 -2y+21m
'\/2 . ,
Esta ecuaci6n se satisface en los casos siguientes:
de donde
1°. x=-y+2k1t (k=O,±l, ... ).
Jr 1
x ::: - + 2kJr - 2 arct u -
J 2 e>.J2 - 21lll 2°. Y = 2/1t, x es un numero cllalesqlliera (t::: 0, ± 1, ... ).
3°. x = 2m1t, Y es un numero cualesquiera (m ::: 0, ± 1, ... ).
TRIGONOMETRlAiResoiuciones y Soiuciones 373

La relaci6n 1° es compatible con la segunda ecuaci6n del


I I+I I=
sistema x y 1 unicamente con la condici6n de que
k = 0; en efecto, de 1° se desprende la desigualdad es decir,
1
sen 6 x + cos 6 x =-. (1)
4
la cual, con la condici6n de que sea x I I+Iy I=1, es posible Examinemos la identidad
solamente en el caso en que sea k = 0.
Resolviendo el sistema
sen 6 x + cos 6 = l- l sen 2 2x, (2)
4
x=-y, ixl+!YI=l,
demostrada en el problema 533.
haHamos dos soluciones Comparando (1) y (2), haHamos:
1
Y2 = - .
2 sen 2 2x = 1, sen 2x = ±I,
Razonando amilogamente en los caos 2° y 3°, haHamos x=2:(2n+l) (n=O,±I,±2, ... ).
dos pares mas de soluciones: 4
x3 = 1, Y3 0; =
x4 -I, Y4 0,= = Multiplicando entre Sl las ecuaciones del sistema dado,
tendremos:
y tambien
sen (y - 3x)cos(y - 3x) = 4 sen 3 x cos 3 x,
Xs =0, Ys =I; x6 =0, Y6 =-1. es decir,
Asi pues, el sistema examinado en el problema tiene las sen 2(y-3x)=sen 3 2x.
seis soluciones indicadas.
Pero, sen 2x = ± 1, por eso,
617. Elevemos ambos miembros de cada ecuaci6n del sistema
sen 2(y -3x)= ±l,
al cuadrado y, sumando las igualdades obtenidas, tendremos:
y - 3x = 2: (2m + 1) (m = 0, ± I, ± 2, ... ).
4
374 PROBLEMAS DE MATEMAnCAS ELEMENTALES

Por consiguiente,
(
1tI'fl
sen-+cos- = sen -
1tm 1tn
+cos- 1,
1tn
3

Y = 31t (2n + 1)+ 2: (2m + 1). 2 2 \. 2 2)


4 4
3
cos 2 -sen 2 = (cos 2 -sen 2 J
1tm 1tmnn 1tn
Durante la resoluci6n del sistema se realiz6 la multipli-
caci6n de ambos miembros de la ecuaci6n por expresiones
que contenian inc6gnitas, por eso es posible que se hayan Puesto que la base de la potencia en la parte derecha de
obtenido soluciones ajenas. Comprobemos si todos los pares estas nuevas igualdades puede to mar para los valores enteros
de soluciones obtenidos de x e y son soluciones. Debeni ser: de 11 solamente los valores 0, + 1, -1 Yestos val ores no varian
1t 1t
al ser elevados a la tercera potencia, entonces,
sen ~- (2m+l)=2sen3 - (2n+l),
4 4 3
= (sen - + cos - J
1tn 1t1l 1t1l 1tn
sen ~-- + cos -
cos2:(2m + 1) = 2 cos 3 2:(2n + 1), 2 2 2 2'
4 4
cos -
1t1l 1t1l (
- sen - = cos - - sen -
nil
.
1t1l
J3

0, suponiendo que 22\ 2 2'

sen -1t (2m+ 1) = -- 1~ sen -


1tm I 1tm
+ -~ cos- de aqui obtenemos:
4 --) 2 2 ",'2 2 7t 7t
y sen --m - sen - n = cos -7t n - cos -m.
7t

2 2 2 2 '
cos -1t (2m+ 1) = - 1 cos -1tm - - I-sen -1tIn-. 7t 7t 7t 7t
4 -Ii 2 -Ii 2 - sen -- m + sen - n = cos - -n - cos - m .
2 2 2 2
y haciendo una sustituci6n analoga en la parte derecha,
despues de simplificar por el factor con stante, obtendremos: Sumando y sustrayendo estas ultimas relaciones, obten-
dremos:
TRIGONOMETRlAiResoiuciones y Soiuciones 375

1t
° 1t
sen - m - sen - n
22'
=
dan las soluciones del sistema cuando, y s610 cuando, los
numeros enteros n y m estan enlazados por la relaci6n (4).
(3) Por consiguiente,
1t 1t
cos - n + cos - m = 0,
2 2 x=~(2n+l),
o bien 4

sen~(m - n) cos~(m + n) = 0, y = ~[3(2n + 1)+ 2(n + 4k)+ 1]= 1t [2 (n + k)+ 1].


4 4 4

sen~(m - n) sen~(m + n) = 0, Pero, n + k es un numero entero arbitrario. Designandolo


4 4 por p, tendremos definitivamente que
Puesto que
x = ~(2n + J), y = 1t(2p + J) (n, p = 0, ± 1, ± 2, ... ).
4
618. Elevando ambos miembros de la primera y segunda
ecuaciones al cuadrado y escribiendo la tercera ecuaci6n en
no se reducen a cero simultaneamente, el sistema obtenido
la forma inicial, obtendremos el sistema:
es equivalente a la ecuaci6n
(sen x + sen y) 2 = 4a 2 , I
(cos x + cos y)2 = 4b Z, ~
(1)
de donde tg x·tg y = c.J
m - n = 4k (k = 0, ± I, ± 2, ... ). (4)

Asi pues, los pares de los valores de x e y obtenidos Buscaremos las condiciones que deberan satisfacer los
numeros a, b, c, para que el sistema (1) tenga por 10 menos
y = 31t (2n + 1)+ ~(2n + 1) una soluci6n. Puesto que el sistema dado 10 sustituimos por
4 4 el sistema (1) no equivalente, hace falta demostrar que, para
unas mismas condiciones impuestas a los numeros a, b, c,
ambos sistemas tienen por 10 menos llna soluci6n.
376 PROBLEMAS DE MATEMATICAS ELEMENTALES

Si el sistema dado tiene soluci6n para ciertos valores de a, 10 menos una soluci6n cuando, y s610 cuando, tiene por 10
by c, entonces, es evidente, que tambien el sistema (1) tendni menos una soluci6n el sistema (1).
soluci6n para los mismos val ores de a, bye. Es justa tambien 2,Cuando tiene soluci6n el sistema (1)7 Sumando y sustra-
la afirmaci6n inversa: si el sistema (1) tiene soluci6n para yendo la primera y la segunda ecuaciones del sistema (I),
ciertos valores de a, by c, entonces tambien el sistema dado hallaremos:
tendra soluci6n para los mismos valores de a, by c. cos{X- y) = 2(a 2 + b2 )-1,
En efecto sean XI e Y I las soluciones del sistema (1);
entonces se cumple una de las cuatro posibilidades: cos 2x + cos 2y + 2 cos{x + y)= 4(b 2 _a 2 ).
o bien o bien
cos {x - y)= 2(a 2 + b2 )-1,
sen xI + sen YI = 2a, cos Xl + cos YI = 2b,
cos{x + y)cos (x - y)+ cos {x + y) = 2(b 2 - a 2 ),
o bien de donde
sen Xl + sen YI = -2a, cos XI + cos YI = 2b, cos{x- Y)=2(a 2 +b 2 )_I,
(a 2 +b 2 )cos{x+ y)=b 2 _a 2 .
o bien

sen xl + sen YI = -2a, cos Xl + cos YI = -2b, Hemos obtenido el sistema

o bien cos (x - y) = 2 (a 2 + b2 )- I,}


sen xl + sen YI = 2a, cos xl + cos Yl = -2b, (a 2 + b2 )cos{x + y)= b 2 _ a 2 ,
tgx·tgy=c,
Si tiene lugar el primer caso, entonces XI e Y\ son las
soluciones del sistema dado; en el segundo caso, el sistema
equivalente al sistema (1).
dato tiene, por ejemplo, la soluci6n - Xl> - Yl; en el tercer
Si a 2 + b 2 = 0, la segunda ecuaClOn se satisface
caso, la soluci6n 1t + xl> 1t + Yl; en el cuarto, la soluci6n
cualesquiera que sean los valores de x e y. De la primera
1t - Xl' 1t - YI. Por consiguiente, el sistema dado tiene por
ecuaci6n obtenemos:
TRIGONoMETRiAlResoluciones y Soluciones 377

evidentemente, es justa, y la desigualdad (3) es equivalente


x- Y = 1t + 21m (k = 0, ± 1, ± 2, ... ). a lasiguiente:
la tercera ecuaci6n nos da:

tg(y + 1t + 21m}tg y =c Representemos el primer miembro de la tercera ecuaci6n


del sistema (I) en la forma siguiente:
o bien
tg 2 Y = c.
~Icos{x- y)-cos{x+ y)1
Esta ultima ecuaci6n tiene soluci6n para cualquier valor tg x tg Y = sen x sen Y = ~--------
2
de c ~ 0. Si a 2 + b 2 7= 0, tenemos: 1 (5)
cosxcosy -Icos{x- y)+cos(x+ y)1
2

y sustituyamos en (5) cos (x + y) y cos (x - y) por sus valo-


res de (2). Como resultado obtendremos que la soluci6n del
sistema (2) satisfani ala tercera ecuaci6n del sistema inicial,
S)

Este sistema tiene soluci6n cuando, y s610 cuando, (2 2) b2 - a2 2


_ 2 ~a + b - 1- a 2 + b2 _ (a2 + b2) _ b 2
c - b 2 - a 2 + 2 (a2 + b 2)_ 1 - (a 2 + b 2 Y_a 2 .
a 2 +b 2

Hemos lIegado al siguiente resultado: el sistema dado tiene


por 10 menos una soluci6n en dos casos:

La desigualdad (4), con la condici6n de que


( 2 2
a + b2 ) b2
1) 0< a 2 + b 2 ::::; 1 y C = 2 - ;
(a 2 + b 2 ) _ a 2
2) a = b = 0 y c es cualquier numero no negativo.
378 PROBLEMAS DE MATEMATICAS ELEMENTALES

3. Funciones trigonometricas inversas Por consiguiente,

619. De la definicion de los valores principales de las


funciones trigonometricas inversas se desprende que arcsen ( cos 3: 1t ) = arcsen [ sen ( - 1~) 1= - 1~ .
arccos{cos x) = x, si 0::::; x::::; 1t.
621. Supongamos que sea arctgt=u" arctgt=u 2 ,
Con el fin de utilizar esta formula, sustituyamos sen (- ~)
con ayuda de las formulas de reduccion al coseno del angulo TC
inc1uido entre 0 y 1t. Escribamos las iguaJdades siguientes: arctg* = u 3 , arctgt = u 4 . Es evidente, que 0 < U i < 4' i =

= I, 2, 3, 4. Por eso,

En resumen obtenemos: Para la demostracion de la identidad es suficiente establecer


que

arccos[ sen ( - ~ ) ] = arccos (cos ~: ) = ~: .


Puesto que

620. Por analogia con la resolucion del problema anterior


tenemos:

622. Haciendo arcsen x = a, arccos x = 13, tendremos:

x = sen a y x = cos 13 = sen ( ~ -13) .


TRlGONoMETRiAiResoiuciones y Soiuciones 379

Segun la definicion de los valores principales tenemos que


a) Si O:s: x :s: I, entonces O:s: a :s: ~ y O:s: f3:S: ~ (puesto

que O:s: -11- x 2 :s: 1 ). Queda solamente convencerse de que

De la ultima desigualdad se deriva la desigualdad sen a = sen f3. Pero, en virtud de la desigualdad O:s: a :s: ~,
1t 1t
--:S: - -f3:S:-.
1t
tenemos que sen a = +~1 - x 2 .
2 2 2
Por otra parte, para todos los valores de y(lyi:S:1)
Por 10 tanto, a=i - f3, puesto que los angulos a y ~ - f3
tenemos que sen arcsen y = y, en particular, sen ~ =
estan incluidos entre - ~ y ~ y los senos de estos angulos
1- -2 ~
son iguales entre sl. La formula queda demostrada. = sen arcsen \/1 - x = \/1 - x- . Por consiguiente, para
O:s: x :s: 1, tiene lugar la formula
623. Aprovechando que arcsen x + arccos x = ~ (vease la
resolucion del problema 622), transformemos la ecuacion a
arccos x = arcsen ~ 1 - x 2 .
la forma
(1)
a) Si -1 :s: x :s: 0, entonces
donde t = arcsen x. Siendo a < 312' el discriminante de esta
ecuacion sera "i S 1t -!3 ~ 7t. Puesto que, ademas, sen a =: ~1- x 2 y

D = 361t 4 - 481t 4 (1- 8a) < O. sen (1t - f3) = sen f3 = ~1- x 2 , entonces a = 1t - f3, es decir,
para - 1 :s: x :s: 0 tiene lugar la formula
Por consiguiente, las raices de la ecuacion (I) son irreales
y por eso, para a < 312' la ecuacion inicial no tiene soluciones. arccos x = 1t - arcsen ,j1 - x 2 .

625. Demostremos que arcsen (- x) = -arcsen x. Hagamos


624. Hagamos arccos x = a, arcsen ,J 1 - x2 = f3. arcsen (- x) =: a; entonces - x = sen a y, segun la definicion
de los valores principales,
380 PROBLEMAS DE MATEMAnCAS ELEMENTALES

1t 1t en virtud de (I), obtendremos:


--:5:a:5:-. (1)
2 2 I-x 2
sen a =--2 '
Puesto que sen (- a) = -sen a = x y, puesto que de la l+x

desigualdad (I) se deriva la desigualdad - ~ :5: -a :5: }-, de donde

entonces - a = arcsen x de donde a = -arcsen x, sea, 1 x2


Y = arcsen (sen a) = arcsen ~ =
0
13. (2)
arcsen (- x) = -arcsen x. I +x

Analogamente se demuestra la f6rmula arccos(- x) = Puesto que 0 < x < 1, entonces


= 7r - arccos x.
1t l+x 1t 1t
- < arctg - - < - y - < a < 1t.
626. De la definici6n de los valores principales de las 4 I-x 2 2
funciones trigonometricas inversas se desprende que
De aqu, se desprende que
arcsen (sen a) = a, SI Si -f +21m:5: 7r
- - <a-7r:5:0
2
:5: x :5: f + 2krr, entonces -}-:5: x - 21m :5: ~. Pero, entonces
y
arcsen (sen x) = arcsen [sen (x - 2k1t)] = X - 21m.
arcsen [sen (a - 1t)] = arcsen (- sen a) = arcsen (sen a) = - y.
627. Seg(m la condici6n del problema
Pero el angulo a - 1t se encuentra en los limites del valor
I+x a principal arcsen x.
tg-= - . (1)
2 I-x Por consiguiente,
Utilizando la f6rmula
y = arcsen (sen a) = 1t - a. (3)
a
2tg-
sena= - ~, De (2) Y (3) obtenemos que a + 13 = 1t.
1+ t a2 _
eo 2
TRlGONoMETRiAiResoluciones y Soluciones 381

628. En la formulas arcsen cos arcsen x y arccos sen arccos Descomponiendo el trinomio cuadrado, que figura en la
x se toman los valores principales de las funciones parte izquierda de (1), en factores, obtendremos:
trigonometricas inversas. Examinemos cos arcsen x. Esto es
el coseno de un arco, el seno del cual es igual a x. Por
consiguiente, [sen x + -1+-15)[ -15-1) > O.
-2 - sen x - --2- (2)

cos arcsen x = +~ 1 - x 2 , donde - I :::; x :::; 1.


Pero,

Aqu!, claro esta, es esencial que - ~ :::; arcsen x:::;~. Ana- 1+-15 >1
logamente 2
y por eso,

sen arccos x = +Ft- x 2 , donde - 1 :::; x :::; 1. 1


sen x+ - - >0.
+-J5
2

Designemos y = +~1- x 2 ; entonces 0:::; y :::; 1. Por consiguiente, la desigualdad inicial es equivalente a
As! pues, hay que hallar la relacion entre arcsen y y arccos la siguiente:
y siendo 0:::; y :::; 1. Estos dos angulos complementan uno al -J5 -1
otro hasta ~ (vease la resolucion del problema 622). As! senx> - -
2
pues,
y tiene la solucion
1t
arcsen cos arcsen x + arccos sen arccos x = -.
2
21m + <p < x < 1t - <p + 21m,
donde
4. Desigualdades trigonometricas
<p = arcsen -J5 -1 (k = 0, ± 1, ± 2, ., .).
629. La desigualdad dada es equivalente a la siguiente 2

630. Cuando x = ~ + 1tn, la expresion que se exam ina no


sen 2 x + sen x-I > o. (1) tiene sentido. Para los demas valores de x multipliquemos
382 PROBLEMAS DE MATEMATlCAS ELEMENTALES

ambas partes de la desigualdad por cos 2x. Obtendremos la Respuesta: 0 < x < 3- y %n < x < 1·
desigualdad equivalente
632. La expresion que figura en el denominador de la parte
(sen 2x)2 + lsen 2x - 2> O. izquierda de la desigualdad es positiva, puesto que
2

Resolviendo la desigualdad cuadrada obtenida hallaremos Isen x+ cos xi = I12 sen (x + 1)1 ~ 12.
que 0 bien
Por esta razon, la desigualdad es equivalente a la siguiente:
-3 --J4i
sen 2x<-- - -
4 I
sen 2 x > -1 0 b·len sen x > -1 . I
4 2
o bien n 5
Respuesta: "6 + 1m < x < "6 n + 1m.
sen 2x >
-J4l-3 .
4 633. Escribamos la desigualdad en la forma
La primera de estas desigualdades no puede ser cumplida.
Por consiguiente, (cos x - sen x) [1- (cos x + sen x) ]=
= 2 sen ~(sen ~ - cos~) (cos x - sen x) > o. (1)
1 -J4i-3 n I -J4i-3 2 2 2
1m + - arcsen < x < - - - arcsen + 1m.
2 4 2 2 4
Pero sen 1> 0, puesto que 0 < x < 2n. Examinemos dos
631. Transformando el producto de los senos en una suma, casos posibles, en los cuales se cumple la desigualdad (1).
sustituyamos la desigualdad dada por la siguiente equivalente:
Caso 1.
cos 3x > cos 7x 0 bien sen 5x sen 2). > o.
cos x - sen x > 0,1
Pero, cuando 0 < x < 1
tenemos que sen 2x > 0 y, por x x ~ (2)
consiguiente, la desigualdad inicial es equivalente a la sen "2 - cOS"2 > O.J
siguiente: sen 5x > o.
TRlGONOMETRiAiResoiuciones y Soiuciones 383

Segun la condicion del problema, 0 < x < 27t. Teniendo 1-1


en cuenta este hecho, de (2) hallamos que la primera 2 > o. (2)
desigualdad se cumple cuando 0 < x < ~ 0 bien I -1-1
%7t < x < 27t, y la segunda, cuando ~ < x < 27t. Por EI trinomio t 2 - I -1 tiene las rakes 1-.J5
2 y 1+.J5

. .
conslgmente, en este caso, 4"5 7t < X < 2 7t. Resolviendo (2); hallaremos que, 0 bien

Caso 2. x 1 +-JS
tg~>--

cos x - sen x < a,} 2 2'

x x (3) o bien,
sen ~ - cos~ < o.
2 2 I-J5 x
--<tg~ <l.
EI sistema (3), teniendo en cuenta que 0< x < 2n, se
2 2
cumple cuando ~ < x < ~.
7t 7t 5
I+J5
a) 21m + 2 arctg- - < x < 7t + 21m.
Respuesta: 4" < x < 2 y 4"7t < X < 27t. 2
Respuesta: J5 - 1 7t
634. Hagamos tg1 = t. Entonces, la desigualdad dada tom a b) 21m - 2 arctg - - < x < ~ + 21m.
2 2
la forma
2t - 2 + 212 635. De las formulas para sen 3x y cos 3x (vease la pagina
1> - - - --0- 69) hallamos:
21 +2- 212

o bien 3 cos 3x + 3 cos x 3 3 sen x - sen 3x


cos x = , sen x = .
(t - 1) {(2 + I +~ 0 4 4
2 > . (1)
t -1-1 Valiendonos de estas formulas, escribamos la desigualdad
Puesto que, 12 + t + 1 > 0 para todos los val ores reales de dada en la forma
t, Ja desigualdad (J) es equivalente a la desigualdad 5
(cos 3x + 3 cos x )cos 3x - (3 sen x - sen 3x )sen 3x > ~
2
384 PROBLEMAS DE MATEMATICAS ELEMENTALES

o bien 637. Haciendo tg x = t, obtendremos:

sen 2 3x + cos 2 3x + 3 (cos 3x cos x - sen 3x sen x) > ~, 2t


2 tg2x=--2 '
I-t
o bien
1
cos 4x >-, tg 3x = tg x + tg 2x = 3( - t2
2 1- tg 2x tg x 1 - 3t 2 .
de donde
1t 1t
- - + 21tn < 4x < - + 21tn. La parte izquierda pierde el sentido para aquellos valores
o bien
3 3 t.
de x, con los cuales (2 = 1, (2 = Para los demas val ores de
x, la parte izquierda de la desigualdad es igual a t 4 + 2t 2 + 1
1t 1 1t 1 y, por consiguiente, adquiere valores positivos.
- - + -1tn < x < - + -1tn (n = 0, ± 1, ± 2, ... ).
12 2 12 2
638. En virtud de que
636. La desigualdad a demostrar se puede escribir en la forma
COS 2x 3 cos 2 x - sen 2 x
2
cotg x - l = - - 3 cotg-? x- 1 =
cos 2 <p - sen 2 ~ + sen <p sen 2 x' sen x
2

cotg~ > 2 2 (1)


cos 3x sen 2x - sen 3x cos 2x
2 sen <p cotg 3 X· tg 2 x - 1 = - - -- - - -- -- -
sen 3x cos 2x
Pero, sen <p > 0 cuando 0 < <p < ~, por eso, despues de sen x
multiplicar ambas partes de la desigualdad (1) por sen <p , sen 3x cos 2x
obtendremos la desigualdad equivalente
La parte izquierda de la desigualdad puede ser escrita en
2 cos 2 ~ > cos 2 ~ - sen 2 ~ + sen <p la forma
222

o bien 1 > sen <po La ultima desigualdad se cumple cuando _ sen x(3 cos 2 x - sen 2 x)
0< <p < ~, por consiguiente, es justa tambien la desigualdad sen 4 x sen 3x
inicial.
TRlGONoMETRiAlResoluciones y Soluciones 385

Pero,
(1 + n tg 2 <p Y24n tg 2 <po
sen 3x = sen (x + 2x) = sen x cos 2x +
Obtenemos, pues, la desigualdad evidente
+ cos x sen 2x = sen x (3 cos 2 x - sen 2 x),
(I - n tg 2 <p)2 20.
por eso, la desigualdad dada se reduce a la desigualdad
evidente 640. La desigualdad dada se puede escribir en la forma
1
- - -4< -
sen x
- 1.
~ + 1 - sen x _ 2 - sen x 2
2 2 - sen x 3 - sen x
°
639. Utilizando la formula
y, multiplicandola por 2 (2 - sen x) (3 - sen x) > 0, puede ser
tg (8 -<p ) ==-t ,g==---8_----'tg==---<-'p--- sustituida por la siguiente equivalente:
1 + tg 8 tg <p
y la condie ion o bien
sen 2 x == 5 sen x + 4 2 °
tg 8 == n tg <p, (4 - sen x)(1 - sen x) 20, (1)

obtendremos: De (1) desprendemos que la ultima desigualdad, y junto


con esta tambien la inicial, se cumple para todos los valores
de x, con la particularidad de que cuando x == ~ + 21m, tiene
lugar el signo de igualdad.

Hace falta demostrar que 641. Establezcamos primeramente que

(cotg <p + n tg <p)2 24n


o bien Examinemos un circulo trigonometrico de radio 1 y
supongamos que x denota el valor en radianes de cierto angulo
386 PRORL EMAS DE MATEMATICAS E LEM ENTALES

ADM positivo 0 negativo (fig. 249). Para cllalquier posicion Demostremos que tg(n + l)a >(n + I)tg a, si 0 < a < 4nn'
del punto M Empleemos la formula
-
AM =1I X 104 I I
I ' • =1xi, tgna+tga
tg (n + I) a = . (3)
I BM I = I sen X I . I-tg a tg na

Puesto que la desigualdad (1) se cumple al cumplirse la


Puesto que IBM i:s; AM, entonces I sen xl:s; I xl (Ia condicion (2), entonces, se cumplini, ademas, cuando
igualdad tiene lugar solamente cuando x = 0 ). En virtud de o< a < 4nn' Pero,
esto deducimos que 0 :S; <p :S; ~, es decir, si 0 :S; cos <p :S; 1 < ~,
- n
entonces sen cos <p < cos <po Pero, O:s; sen <p :S; 2 Y por eso 0< tg a < I, (4)
cos <p:S; cos sen <po Definitivamente tenemos que
cos sen <p ~ cos (j) > sen cos <po y, puesto que 0 < na < {, entonces
La desigualdad queda demostrada.
642. Utilicemos el metodo de in- 0 < tg na < I. (5)
duccion completa. Sea n = 2, en-
De (4) Y (5) obtendremos:
tonces 0 < a < { . Por consiguien-
te, ' - ----lL-L--'-----JA o< 1- tg a tg na < I . (6)
2 tg a
tg 2a = ') > 2 tg a, De (6) Y (3) se desprende que tg(n + I)a > (n + I)tg a, 10
1- tg~ a
que era necesario demostrar.

puesto que 0 < 1 - tg 2 a < I. Su- Fig. 249 643. Puesto que a mayor angulo del primer cuadrante Ie
pongamos que sea corresponde mayor valor de la tangente, entonces,
tg na > 11 tg a (1) (1)
con la condicion de que
para i = 1, 2, . .. ,11. Ademas, cos a i > O(i = I, 2, . .. , n). Por
1[ esta razon, la desigualdad (I) se puede escribir en la forma
O<a <----- . (2)
4(n -1)
TRIGONoMETRiAIResoluciones y Soluciones 387

(2) despues de las transformaciones evidentes, obtendremos:

Demos en la desigualdad (2) a i los valores 1, 2, ... , n y


sumemos todas las desigualdades obtenidas. Hallaremos:

tg (cos aj + ... + cos a n)< sen


at at + ... + sen an <
(3)
< tg an (cos at + ... + cos an ) .

Dividiendo todas las partes de la desigualdad (3) por Por consiguiente,


cos aj + ... + cos an (10 cual es posible, puesto que 1 2A-B 1
cos at + .. . + cos an > 0), tendremos: t ~ - cos - - ~ - .
8 2 8
sen aj + ... +sen an 645. Transformemos la parte izquierda de la desigualdad dada
tg al < < tg an"
cos al + ... + cos an de la manera siguiente:

644. Designemos la parte izquierda de la desigualdad que se cos X cos 2 x


exam ina por t. Entonces
sen 2 x(cosx-senx) sen 2 x{!-tgx) tg 2 x(l-tgx)
l( A-B A+B\ A+B
t = - cos - - - cos-- Icos - -
1 + tg 2 x I
2\ 2 2) 2' tg x tg x (I - tg x)"

puesto que
Para simplificar la escritura, hagamos tg x = t. Puesto que
C A+B
sen-- = cos- -- . 0< x <~, entonces
2 2
Haciendo O<t<1. (I)

A+B De este modo, el problema se reduce a la demostraci6n de


cos--=x,
2 la desigualdad
388 PROBLEMAS DE MATEMATICAS ELEMENTALES

l+t 2 ._1_ > 8 I 2 2 tg a 5


Pero, dado que tg a = 5"' entonces tg a =: l-tg2 a = 12·
t t(l-t)
Sustituyendo tg 2a y tg ~ en la f6rmula (1), hallamos que
con la condici6n de que 0 < t < 1. Pero, en virtud de la
desigualdad (1), pagina 20, tenemos que
tg(2a - ~)=: o.
1+t 2 Entonces
-->2.
t
sen (2a -~) = sen (2 arctg~5 - arctg~
12)
1= o.
Ademas,

t(l-t)="4- 1 (12- t J2 ~"4.1 647. Demostremos que tg(a + 2~)= 1. Para hallar
tg(a + 2~) empleemos la f6rmula
Por consiguiente,
tg (a + 2 I-'
R) =: tg a + tg 2~
.
I+ t2 1 1 (1)
- - · -- >2·-=8 1- tg a tg 2~
t t(l-t) 4 '
Calculemos previamente el valor de tg 2~ por la f6rmula
10 que era necesario demostrar.
tg 2~ =: sen 2~ =: 2 sen ~ cos ~ .
cos 2~ cos 2~
5. Problemas diferentes
Hace falta hallar cos ~ y cos 2~. Pero,

646. Hagamos arctg~ = a, arctg l~ = ~ y examinemos


I 2 3
tg(2a - ~). Valiendonos de la f6rmula para la tangente de la cos ~ =: +'111- sen i3 =: r;-;;.
,, 10
diferencia de dos angulos, obtenemos:
(puesto que ~ es un angulo del primer cuadrante; y
tg (2 a n.) _ tg 2a -
-I-' - _.
tg ~ .
2 2 4
(1) cos 2~ cos ~- sen ~
1+ tg 2a tg ~ =: =: - .
5
TRlGONoMETRiAIResoluciones y Soluciones 389

Por consiguiente, tg213 = ~. Colocando el valor halJado De (I) se desprende que y > 0, puesto que, siendo
de tg 213 en (1), obtendremos: x*- kn12,

tg(a+ 213)= 1. cos x < 1 y sen x < I .

Demostremos, ahora, que a + 213 = %. 649. Transformando el producto sen a . sen 2a· sen 3a en
una suma, por la formula (13), pagina 69, obtendremos:
Puesto que tg a = ~ entonces
1
sen a . sen 2a . sen 3a = -- sen 2a (cos 2a -- cos 4a) =
4
1 I 1 I 4
= - sen 4a - - sen 2a . cos 4a :::; - + - < -.
4 2 425
y, ademas, segtin la condicion del problema, a y 13 son
650. Puesto que sen 5x = sen 3x cos 2x + cos 3x sen 2x, con
angulos del primer cuadrante, por 10 tanto,
ayuda de las formulas (5) y (8), pagina 69, despues de simples
O<a<~~ y 0<13<%. De aqui hallamos que calculos, hallaremos que
0< a + 213 < ~ n. Pero, el tinico angulo comprendido entre
o y ~ n, cuya tangente es igtlal ales el angulo %. As! sen5x=5senx-20sen 3 x+16sensx. (1)
pues, a + 213 = %.
Suponiendo en la formula (1) x = 36°, obtendremos la
ecuacion 16t S _ 20t 3 + 5t = 0 para la determinacion de
648. Es necesario que cos x *- 0, sen x *- 0, sen x *- -I, de
sen 36°. Esta ecuacion tiene las siguientes raices:
donde x*- knl2 (k es un numero entero). Para todos los
valores de x, excepto x = kn12, y tiene sentido y es igual a

sen X[I + __ 1_)


cos x sen 2 x (I + cos x)
y - ---':---~ -

- cosx [1 + --1 )-COS2x(l+senx)" (I)


sen x
390 PROBLEMAS DE MATEMATICAS ELEMENTALES

De estas raices son positivas las raices t2 Y (4. Pero, 653. Si n es un numero entero que satisface a la condicion
del problema, entonces, para todos los valores de x, tenemos:
sen 36° :t: (2, puesto que 5+f > 1 y, por consiguiente,
5 5
cos n (x + 3TC)· sen ~ (x + 3TC) = cos nx . sen ~ x.
t2 > -h. ASI pues, n n
(l)

sen 36 ° = t4 1 15-~
= ~\I--.
Suponiendo, en particular, que x = 0, de (1) desprendemos
2 V 2 que n debera satisfacer a la ecuacion sen 1~1t = 0. A esta
651. Utilizando la identidad demostrada en el problema 533, ecuacion la satisfacen solamente aquellos numeros enteros
obtendremos: que son divisores del numero 15.

n=±1,±3,±5,±15. (2)

Por medio de la comprobacion directa nos convencemos


de donde se desprende que el valor maximo de cp (x) es igual
de que, para cada uno de estos valores, la funci6n
aI, y el mlnimo, a 1/4.
cos nx· sen ~ x tiene el periodo 3TC. Con la formula (2) se
652. Como resultado de simples transformaciones obtenemos agotan todos los valores buscados de n.
que
654. Puesto que la suma que se exam ina, siendo x = Xl' es
y = I-cos 2x+ 2(1 + cos2x)+ 3 sen 2x = 3 + 3 sen 2x + cos 2x. igual acero, entonces
Introduciendo el angulo auxiliar cp = arctgt, tendremos a l cos(a l +xl)+···+an cos(a n +xJ=
que = (a l COS a l + ... + an COS an )cos XI - (I)
y =3+ JI<>( 10 sen 2x + Jo cos 2X) = 3 + .!l6 sen{2x + cp).
- (a l sen a l + ... + an sen aJsen XI = O.

Pero, segun la condicion del problema,


Por consiguiente, el valor maximo de yes 3 + M, yel
mlnimo, es igual a 3 --flO. (2)
TRIGONoMETRWResoluciones y Soluciones 391

Ademas, sen xI "* 0, puesto que xI "* 1m. De (1) y (2) Luego, coloquemos en (1) el valor x = T. Entonces,
obtenemos que evidentemente, tendremos que, de acuerdo con (1) y (2),
cos ffi = cos JT = 1, de donde
(3)

Sea, ahora, x un numero cualquiera. Entonces,


=
-v2T = 21n.

Puesto que por suposicion T"* 0, dividiendo (4) por (3),


a l cos(a 1 +x)+ ... +an cos(a n +x)=
=(a] cos a l +... + an cos aJcos x- obtendremos que .,fi = f, donde I y k son numeros enteros.
-(at sen at + . .. +a n sen an)sen x = 0, Esto ultimo, como es sabido, es imposible.

puesto que, en virtud de (2) y (3), las sumas que figuran entre 656. Primera resolucion. Examinemos la suma
parentesis son iguales a cero.
s = (cos x + i sen x) + (cos 2x + i sen 2x) + ...
655. Supongamos 10 contrario, es decir, admitamos que existe
°
T "* tal, que para todos los val ores de x ~ sera ° ... +(cos nx+ i sen nx)

,--- I
y, empleando la formu la de Moivre, (cos x + i sen x = y
cos -V x + T = cos -V x (1) = cos nx + i sen nx, calculemos la suma S como la suma de
una progresion geometrica. Obtendremos:
(1a limitacion x ~ 0 es necesaria por el hecho de que siendo
x < 0, el radical Fx sera Imaginario. Hagamos
primeramente en la formula (1) x = 0; entonces
S = (cos x + i sen x y+ I - (cos x + i sen x)
cos x + i sen x-I
cos 1i = cos 0 = 1 (2) La suma sen x + sen 2x -r ... + sen nx es igual a la parte
imaginaria de S.
y, por 10 tanto
Segunda resolucion. Multiplicando la parte izquierda por
JT = 2kn. (3) 2 sen1 y empleando la formula (13), pagina 69,
obtendremos:
392 PROBLEMAS DE MATEMAnCAS ELEMENTALES

( COS ~2 - COS l2 X) + (COS l2 X- COS ~2 X) + ... .l(


2 4 4 2 n
l(
+ I sen -TCl) + ...- - cos -TC.
A + BI = - cos -TC.
4
+ I sen -TC)n
4
2n - 2n +1 X 2n + 1)
... + ( cos - 2-x-cos-2- x =cos2"-cos-2- x =
1
1( 1- -
l( TC
cos - + i sen - I
TC,n
TC . TC ) 2n 4 4)
= - lcos- + I sen--
nx n+ 1 2 4 4 1 (TC TC )
= 2 sen -
2
. sen - - X
2' 1 - 2" cos + i sen 4 4
de donde, precisamente, se deduce la formula necesaria.
En la ultima expresion se ha usado la formula para la suma
657. Designemos la suma buscada por A y agreguemosle la de los terminos de una progresion geometrica. La sum a
segunda suma buscadaA puede ser hallada como la parte real de la expresion
obtenida. Observando que
n 2n nn
sen -- sen -- sen - n n 1
B= _ _ 4 + 4 + + _ _4_ cos - = sen - = -
2 22 . ., 2n ' 4 4-li'
hallamos sucesivamente que
multiplicandola previamente por i. Obtendremos:

A + Bl. = -1 ( cos --n + I. sen -n'1) + - 1 (cos


2 -n + i sen -2n) + ... ·
I(
11:. 11:
) l-~(Cos~+isen~)n
2 n 4 4
2 4 4 22 \. 4 4 A + B 1= - 1 cos - +Isen - ----+----~~

2l 4 4 1 _ .l (cos ~ + i sen ~
1
(n
.. . + - 1 cos n - + i sen n - .
2n
n) 2 4 4)

~ (cos n ~ + i sen n ~ I
4 4
1-
= __ I_(I+i) 2n 4 4)=
Empleando la formula de Moivre, hallamos: 212 1_ _ I_ _ _ i_
2-12 2-12
TRJGONoMETRiAlResoiuciones y Soiuciones 393

A B
donde sen q> = ~ , cos q> = I .
A2 + B2 -V A2 + B2

Sean, ahora, x I y x 2 los dos valores del argumento indicados

~ ~~[i~~~I); ::l [( -oos n~)-;m~1~


2"
en el problema; entonces, j{Xl) = j{X2) = 0, y puesto que

~ A2 + B2 * 0, sen {Xl + q» = sen{X2 + q» = 0. De aqu!


Xl + q> = m1t, x2 + <p = n1t y, por consiguiente,
Xl - X2 = 1m
[(2F2 -2)+2iF2l! (2n -cos n~ )-i sen n~ I para cierto valor entero de k. Esta igualdad conduce a una
= 2n(10 -4F2) contradicci6n, puesto que segun la condici6n XI - x2 * 1m.
Por consiguiente, A2 + B2 = 0, de donde A = B = 0.
Separando la parte real, obtenemos:

658. La afirmaci6n quedanl demostrada si establecemos que


A = B = 0. Sea A2 + B2 * 0, es decir, por 10 menos uno de
los numeros A, B difiere de cero. Entonces

( -=
A B 'I ,------;:;-- 2
j{X)=-, cos x+ I sen x HA- +B =
\j A2 + B2 \j A2 + B2 )
=I A2 + 8 2 sen {x + tp)'
394 PROBLEMAS DE MATEMATICAS ELEMENTALES

INDICE

Presentaci6n 7

ALGEBRA
Problemas Soluciones

I. Progresiones aritmetica y geometrica 1-23 9 81


2. Ecuaciones algebraicas y sistemas de ecuaciones 24-95 11 90
3. Desigualdades algebraicas 96-123 20 133
4. Ecuaciones logaritmicas y exponenciales, identidades y desigualdades 124-169 24 142
5. Combinatoria y binomio de Newton 170-188 29 158
6. Planteamiento de ecuaciones 189-228 31 165
7. Problemas diferentes 229-291 37 184

GEOMETRiA

A. Planimetria

1. Problemas de calculo 292-324 45 210


2. Problemas de construcci6n 325-338 48 224
3. Problemas de demostraci6n 339-408 49 230
4 .. Lugar geometrico de los puntos 409-420 56 261
5. Determinaci6n de los vaiores maximos y minimos 421-430 57 269
INDlCE 395

B. Estereometria
Problemas Soluciones
1. Problemas de caIculo 431-500 58 274
2. Problemas de demostraci6n 501-523 66 317
3. Lugar geometrico de los puntos 524-530 68 329
4. Valores maximos y mfnimos 531-532 68 333

TRIGONOMETRiA

1. Transformaci6n de las expresiones que contienen funciones trigonometricas 533-554 70 335


2. Ecuaciones trigonometricas y sistemas de ecuaciones 555-618 72 340
3. Funciones trigonometricas inversas 619-628 77 377
4. Desigualdades trigonometricas 629-645 77 380
5. Problemas diferentes 646-658 79 387
1m preso en los Talleres GnHicos
de la Direcci6n de Publicaciones del
Instituto Politecnico Nacional,
Tresguerras 27, Centro Hist6rico, Mexico, D.F.
Mayo 2008. Edici6n: 1 000 ejemplares.

CUIDADO EDITORIAL: Vania B. Castellanos Contreras


DISENO Y FORMACION: Patricia Camargo Higareda
DISENO DE PORTADA: Surabi Dione Calette Daniel
PREPRENSA: Sergio Mujica Ramos
ACABADOS EDITORIALES: Roberto L6pez Moreno
PRODUCCION EDITORIAL: Vania B. Castellanos Contreras
PROCESOS EDITORIALES: Manuel Toral Azuela
DIVISION EDITORIAL: Hector Bello Rios
DIRECTOR: Arturo Salcido Beltran

También podría gustarte